Sie sind auf Seite 1von 118

INSURANCE Page 1

ABOITIZ SHIPPING CORPORATION V NEW - Petitioner elevated the case to the Court of presumed to be negligent since it is tasked with the
INDIA ASSURANCE COMPANY, LTD. Appeals, which in turn, affirmed in toto the trial maintenance of its vessel. Though this duty can be
[CITATION] court’s decision. delegated, still, the shipowner must exercise close
QUISUMBING; May 2, 2006 - Petitioner moved for reconsideration but the same supervision over its men.
was denied. - In the present case, petitioner has the burden of
NATURE - Hence, this petition for review showing that it exercised extraordinary diligence in
Petition for review on certiorari Petitioner’s Claim the transport of the goods it had on board in order to
> Petitioner contends that respondent’s claim for invoke the limited liability doctrine. Differently put,
FACTS damages should only be against the insurance to limit its liability to the amount of the insurance
- Societe Francaise Des Colloides loaded a cargo of proceeds and limited to its pro-rata share in view of proceeds, petitioner has the burden of proving that
textiles and auxiliary chemicals from France on board the doctrine of limited liability the unseaworthiness of its vessel was not due to its
a vessel owned by Franco-Belgian Services, Inc. Respondent’s Comments fault or negligence.
- The cargo was consigned to General Textile, Inc., in > Respondent counters that the doctrine of real and - Considering the evidence presented and the
Manila and insured by respondent New India hypothecary nature of maritime law is not applicable circumstances obtaining in this case, we find that
Assurance Company, Ltd. in the present case because petitioner was found to petitioner failed to discharge this burden. Both the
- While in Hongkong, the cargo was transferred to have been negligent. Hence, according to trial and the appellate courts, in this case, found that
M/V P. Aboitiz for transshipment to Manila. respondent, petitioner should be held liable for the the sinking was not due to the typhoon but to its
- Before departing, the vessel was advised that it total value of the lost cargo unseaworthiness. Evidence on record showed that
was safe to travel to its destination, but while at sea, the weather was moderate when the vessel sank.
the vessel received a report of a typhoon moving ISSUE These factual findings of the Court of Appeals,
within its path. WON the limited liability doctrine applies in this case affirming those of the trial court are not to be
- To avoid the typhoon, the vessel changed its disturbed on appeal, but must be accorded great
course. However, it was still at the fringe of the HELD weight. These findings are conclusive not only on
typhoon when its hull leaked. NO the parties but on this Court as well.
- On October 31, 1980, the vessel sank, but the Ratio Where the shipowner fails to overcome the Disposition Petition is denied for lack of merit.
captain and his crew were saved. presumption of negligence, the doctrine of limited
- On November 3, 1980, the captain of M/V P. liability cannot be applied. PHILAMCARE HEALTH SYSTEMS, INC. V CA
Aboitiz filed his “Marine Protest”, stating that the (TRINOS)
wind force was at 10 to 15 knots at the time the ship Reasoning 379 SCRA 357
foundered and described the weather as “moderate - From the nature of their business and for reasons YNARES-SANTIAGO; March 18, 2002
breeze, small waves, becoming longer, fairly of public policy, common carriers are bound to
frequent white horse observe extraordinary diligence over the goods they NATURE
- Petitioner notified the consignee of the total loss of transport according to all the circumstances of each Petition for review of CA decision
the vessel and all of its cargoes. case. In the event of loss, destruction or
- General Textile, lodged a claim with respondent for deterioration of the insured goods, common carriers FACTS
the amount of its loss. are responsible, unless they can prove that the loss, - Ernani TRINOS, deceased husband of respondent
- Respondent paid General Textile and was destruction or deterioration was brought about by Julita, applied for a health care coverage with
subrogated to the rights of the latter. the causes specified in Article 17341 of the Civil Philamcare Health Systems, Inc. In the standard
- Respondent hired a surveyor to investigate, and Code. In all other cases, common carriers are application form, he answered no to the question:
the same concluded that the cause was the flooding presumed to have been at fault or to have acted “Have you or any of your family members ever
of the holds brought about by the vessel’s negligently, unless they prove that they observed consulted or been treated for high blood pressure,
questionable seaworthiness. extraordinary diligence. Moreover, where the vessel heart trouble, diabetes, cancer, liver disease, asthma
- Respondent filed a complaint for damages against is found unseaworthy, the shipowner is also or peptic ulcer? (If Yes, give details).”
petitioner Aboitiz, Franco-Belgian Services and the - The application was approved for period of one
latter’s local agent, F.E. Zuellig, Inc. (Zuellig) 1 year; upon termination, it was extended for another
Art. 1734. Common carriers are responsible for the loss, destruction,
- On November 20, 1989, the trial court ruled in 2 years. Amount of coverage was increased to a
or deterioration of the goods, unless the same is due to any of the
favor of respondent and held petitioner Aboitiz liable following causes only: maximum sum of P75T per disability.
for the total value of the lost cargo plus legal interest (1) Flood, storm, earthquake, lightning, or other natural disaster or - During this period, Ernani suffered a HEART
calamity;
- The complaint with respect to Franco and Zuellig (2) Act of the public enemy in war, whether international or civil;
ATTACK and was confined at the Manila Medical
was dismissed (3) Act of omission of the shipper or owner of the goods; Center (MMC) for one month. While her husband was
(4) The character of the goods or defects in the packing or in the in the hospital, Julita tried to claim the
containers;
(5) Order or act of competent public authority. hospitalization benefits.
INSURANCE Page 2

- Petitioner treated the Health Care Agreement Ratio Every person has an insurable interest in the (b) Notice must be based on the occurrence after
(HCA) as void since there was a concealment life and health of himself2. The health care effective date of the policy of one or more of the
regarding Ernani’s medical history. Doctors at agreement was in the nature of non-life insurance, grounds mentioned;
the MMC allegedly discovered at the time of his which is primarily a contract of indemnity. Once the (c) Must be in writing, mailed or delivered to the
confinement, he was hypertensive, diabetic and member incurs hospital, medical or any other insured at the address shown in the policy;
asthmatic. Julita then paid the hospitalization expense arising from sickness, injury or other (d) Must state the grounds relied upon provided in
expenses herself, amounting to about P76T. stipulated contingent, the health care provider must Section 64 of the Insurance Code and upon request
- After her husband died, Julita instituted action for pay for the same to the extent agreed upon under of insured, to furnish facts on which cancellation is
damages against Philamcare and its Pres. After the contract. based.
trial, the lower court ruled in her favor and ordered Reasoning - These conditions have not been met. When the
Philamcare to reimburse medical and hospital - A contract of insurance3 is an agreement whereby terms of insurance contract contain limitations on
coverage amounting to P76T plus interest, until fully one undertakes for a consideration to indemnify liability, courts should construe them in such a way
paid; pay moral damages of P10T; pay exemplary another against loss, damage or liability arising from as to preclude insurer from non-compliance of
damages of P10T; atty’s fees of P20T. an unknown or contingent event. obligation. Being a contract of adhesion, terms of an
- CA affirmed the decision of the trial court but - An insurance contract exists where the following insurance contract are to be construed strictly
deleted all awards for damages and absolved elements concur: against the party which prepared it – the insurer.
petitioner Reverente. (a) The insured has an insurable interest; - Also, Philamcare had 12 months from the date of
Petitioner’s Claims (b) The insured is subject to a risk of loss by the issuance of the Agreement within which to contest
(1) Agreement grants “living benefits” such as happening of the peril; the membership of the patient if he had previous
medical check-ups and hospitalization which a (c) The insurer assumes the risk; ailment of asthma, and six months from the issuance
member may immediately enjoy so long as he is (d) Such assumption of risk is part of a general of the agreement if the patient was sick of diabetes
alive upon effectivity of the agreement until its scheme to distribute actual losses among a large or hypertension.
expiration. group of persons bearing a similar risk; and * The health care agreement is in the nature of a
(2) Only medical and hospitalization benefits are (e) In consideration of the insurer’s promise, the contract of indemnity. Hence, payment should be
given under the agreement without any insured pays a premium. made to the party who incurred the expenses. It is
indemnification, unlike in an insurance contract 2. NO clear that respondent paid all the hospital and
where the insured is indemnified for his loss. Ratio Where matters of opinion or judgment are medical bills; thus, she is entitled to reimbursement.
(3) HCAs are only for a period of one year; called for, answers made in good faith and without Disposition Petition DENIED.
therefore, incontestability clause does not apply, as intent to deceive will not avoid a policy even though
it requires effectivity period of at least 2 yrs. they are untrue; since in such case the insurer is not PINEDA V CA (INSULAR LIFE INSURANCE
(4) It is not an insurance company, governed by justified in relying upon such statement, but is COMPANY)
Insurance Commission, but a Health Maintenance obligated to make further inquiry. 226 SCRA 755
Organization under the authority of DOH. Reasoning DAVIDE; September 27, 1993
(5) Trinos concealed a material fact in his - The fraudulent intent on the part of the insured
application. must be established to warrant rescission of the NATURE
(6) Julita was not the legal wife since at the time insurance contract. The right to rescind should be Appeal by certiorari for review and set aside the
of their marriage, the deceased was previously exercised previous to the commencement of an Decision of the public respondent Court of Appeals
married to another woman who was still alive.* action on the contract. No rescission was made. and its Resolution denying the petitioners' motion for
Besides, the cancellation of health care agreements reconsideration
ISSUES as in insurance policies requires:
1. WON a health care agreement is an insurance (a) Prior notice of cancellation to insured; FACTS
contract (If so, “incontestability clause” under the - In 1983, Prime Marine Services, Inc. (PMSI)
Insurance Code is applicable) procured a group policy from Insular Life to provide
2. WON the HCA can be invalidated on the basis of 2 life insurance coverage to its sea-based employees
alleged concealment Sec.10. Every person has an insurable interest in the life and enrolled under the plan. During the effectivity of the
health:
(1) of himself, of his spouse and of his children;
policy, 6 covered employees perished at sea. They
HELD (2) of any person on whom he depends wholly or in part for education were survived by complainants-appellees, the
YES or support, or in whom he has a pecuniary interest; beneficiaries under the policy.
(3) of any person under a legal obligation to him for the payment of
money, respecting property or service, of which death or illness might - complainants-appellees sought to claim death
delay or prevent the performance; and benefits due them and approached Capt. Roberto
(4) of any person upon whose life any estate or interest vested in him
depends.
Nuval, President and GM of PMSI, then executed
3 special powers of attorney authorizing Capt. Nuval to
Section 2 (1) of the Insurance Code
INSURANCE Page 3

“follow up, ask, demand, collect and receive” for of Ayo and LOntok, even if no bond was posted as - the labor of the employees is the true source
their benefit indemnities of sums of money due required of the benefits, which are a form of additional
them…” compensation to them.
- Unknown to the complainants, PMSI filed with HELD - the employer is the agent of the insurer in
Insular Life claims for and in behalf of them through 1. YES performing the duties of administering group
Capt. Nuval, even using the 5 special powers of Ratio Third persons deal with agents at their peril insurance policies. It cannot be said that the
attorney that they executed as documents. Insular and are bound to inquire as to the extent of the employer acts entirely for its own benefit or for the
Life then released 6 checks, payable to the order of power of the agent with whom they contract. The benefit of its employees in undertaking
the complainant-appellees, to the treasurer of PMSI person dealing with an agent must also act administrative functions. While a reduced premium
(who happened to be Capt. Nuval’s son-in-law). with ordinary prudence and reasonable may result if the employer relieves the insurer of
Capt. Nuval then endorsed and deposited these diligence. Obviously, if he knows or has good these tasks, and this, of course, is advantageous to
checks (which were for the complainants) in his bank reason to believe that the agent is exceeding his both the employer and the employees, the insurer
account. authority, he cannot claim protection. So if the also enjoys significant advantages from the
- 3 years after, the complainants-appellees found out suggestions of probable limitations be of such a clear arrangement. The reduction in the premium which
that they were entitled, as beneficiaries, to life and reasonable quality, or if the character assumed results from employer-administration permits the
insular benefits under a group policy with by the agent is of such a suspicious or unreasonable insurer to realize a larger volume of sales, insurer to
respondent-appellant so they sought to recover nature, or if the authority which he seeks to exercise realize a larger volume of sales, and at the same
these benefits from Insular Life. Insular Life denied is of such an unusual or improbable character, as time the insurer's own administrative costs are
the claim, saying that their liability to complainants would suffice to put an ordinarily prudent man upon markedly reduced.
was already extinguished upon delivery to and his guard, the party dealing with him may not shut - the employee has no knowledge of or control over
receipt by PMSI of the 6 checks issued in the his eyes to the real state of the case, but should the employer's actions in handling the policy or its
complainants’ names. Complainants filed case with either refuse to deal with the agent at all, or should administration. An agency relationship is based upon
the Insurance Commission which decided in their ascertain from the principal the true condition of consent by one person that another shall act in his
favor. affairs. behalf and be subject to his control. It is clear from
- Insurance Commission held that the special powers Reasoning the evidence regarding procedural techniques here
of attorney executed by complainants in favor of the - The execution by the principals of special powers of that the insurer-employer relationship meets this
complainants do not contain in unequivocal and clear attorney, which clearly appeared to be in prepared agency test with regard to the administration of the
terms authority to Capt. Nuval to obtain, receive, forms and only had to be filled up with their names, policy, whereas that between the employer and its
receipt from respondent company insurance residences, dates of execution, dates of employees fails to reflect true agency. The insurer
proceeds arising from the death of the seaman- acknowledgement and others, excludes any intent to directs the performance of the employer's
insured; also, that Insular Life did not convincingly grant a general power of attorney or to constitute a administrative acts, and if these duties are not
refuted the claim of Mrs. Alarcon that neither she nor universal agency. Being special powers of attorney, undertaken properly the insurer is in a position to
her husband executed a special power of authority in they must be strictly construed. Insular Life knew exercise more constricted control over the
favor of Capt. Nuval (and therefore, the company that a power of attorney in favor of Capt. Nuval for employer's conduct.
should have not released the check to Capt. Nuval- the collection and receipt of such proceeds was a - ON GROUP INSURANCE: Group insurance is
PMSI); and that it did not observe Sec 180(3), as deviation from its practice with respect to group essentially a single insurance contract that
repealed by Art. 225 of the Family Code, when it policies (that the employer-policyholder is the agent provides coverage for many individuals. In its
released the benefits due to the minor children of of the insurer). original and most common form, group insurance
Ayo and Lontok, when the said complainants did not - The employer acts as a functionary in the collection provides life or health insurance coverage for
post a bond as required and payment of premiums and in performing related the employees of one employer. The coverage
- Insular Life appealed to the CA; CA modified the duties. Likewise falling within the ambit of terms for group insurance are usually stated in a
decision of the Insurance Commission, eliminating administration of a group policy is the disbursement master agreement or policy that is issued by the
the award to the Lontoks and Ayo of insurance payments by the employer to the insurer to a representative of the group or to an
employees. Most policies, such as the one in this administrator of the insurance program, such as an
ISSUES case, require an employee to pay a portion of the employer. Although the employer may be the titular
1. WON Insular Life should be liable to the premium, which the employer deducts from wages or named insured, the insurance is actually
complainants when they relied on the special powers while the remainder is paid by the employer. This is related to the life and health of the employee.
of attorney, which Capt. Nuval presented as known as a contributory plan as compared to a Indeed, the employee is in the position of a real
documents, when they released the checks to the non-contributory plan where the premiums are solely party to the master policy, and even in a non-
latter paid by the employer. contributory plan, the payment by the employer of
2. WON Insular Life should be liable to the the entire premium is a part of the total
complainants when they released the check in favor compensation paid for the services of the employee.
INSURANCE Page 4

2. YES CEBU SHIPYARD ENGINEERING WORKS, INC. V subject to the following overriding limitations and
Ratio Regardless of the value of the unemancipated WILLIAM LINES, INC. and PRUDENTIAL exceptions, namely:
common child's property, the father and mother ipso GUARANTEE and ASSURANCE COMPANY, INC. (a) The total liability of the Contractor to the
jure become the legal guardian of the child's [CITATION] Customer (over and above the liability to replace
property. However, if the market value of the PURISIMA; May 5, 1999 under Clause 10) or of any sub-contractor shall be
property or the annual income of the child exceeds limited in respect of any defect or event (and a
P50,000,00, a bond has to be posted by the parents NATURE series of accidents arising out of the same defect or
concerned to guarantee the performance of the Petition for review on certiorari event shall constitute one defect or event) to the
obligations of a general guardian. sum of Pesos Philippine Currency One Million only.
Reasoning FACTS x x x
- Sec 180, Insurance Code: 'In the absence of a - Cebu Shipyard and Engineering Works, Inc. 20. The insurance on the vessel should be
judicial guardian, the father, or in the latter's (CSEW) is engaged in the business of dry-docking maintained by the customer and/or owner of the
absence or incapacity, the mother of any minor, who and repairing of marine vessels while the Prudential vessel during the period the contract is in effect.”
is an insured or a beneficiary under a contract of life, Guarantee and Assurance, Inc. (Prudential) is in the - While the M/V Manila City was undergoing dry-
health or accident insurance, may exercise, in behalf non-life insurance business. docking and repairs within the premises of CSEW,
of said minor, any right under the policy, without - William Lines, Inc. is in the shipping business. It the master, officers and crew of M/V Manila City
necessity of court authority or the giving of a bond was the owner of M/V Manila City, a luxury stayed in the vessel, using their cabins as living
where the interest of the minor in the particular act passenger-cargo vessel, which caught fire and sank quarters. Other employees hired by William Lines to
involved does not exceed twenty thousand pesos " on Feb. 16, 1991. At the time of the unfortunate do repairs and maintenance work on the vessel were
- …repealed by Art. 225, Family Code: "ART. 225. occurrence sued upon, subject vessel was insured also present during the dry-docking.
The father and the mother shall jointly exercise legal with Prudential for P45M for hull and machinery. The - On February 16, 1991, after subject vessel was
guardianship over the property of their Hull Policy included an “Additional Perils transferred to the docking quay, it caught fire and
unemancipated common child without the necessity (INCHMAREE)” Clause covering loss of or damage to sank, resulting to its eventual total loss.
of a court appointment. In case of disagreement, the the vessel through the negligence of, among others, - On February 21, 1991, William Lines, Inc. filed a
father's decision shall prevail, unless there is judicial ship repairmen complaint for damages against CSEW, alleging that
order to the contrary. - Petitioner CSEW was also insured by Prudential for the fire which broke out in M/V Manila City was
Where the market value of the property or the third party liability under a Shiprepairer’s Legal caused by CSEW’s negligence and lack of care.
annual income of the child exceeds P50,000, the Liability Insurance Policy. The policy was for P10 - On July 15, 1991 was filed an Amended Complaint
parent concerned shall be required to furnish a bond million only, under the limited liability clause, to wit: impleading Prudential as co-plaintiff, after the latter
in such amount as the court may determine, but not - On Feb. 5, 1991, William Lines, Inc. brought its had paid William Lines, Inc. the value of the hull and
less than ten per centum (10%) of the value of the vessel, M/V Manila City, to the Cebu Shipyard in machinery insurance on the M/V Manila City. As a
property or annual income, to guarantee the Lapulapu City for annual dry-docking and repair. result of such payment Prudential was subrogated to
performance of the obligations prescribed for general - On Feb. 6, 1991, an arrival conference was held the claim of P45 million, representing the value of
guardians." between representatives of William Lines, Inc. and the said insurance it paid.
-"market value of the property or the annual CSEW to discuss the work to be undertaken on the On June 10, 1994, the trial court a quo came out
income of the child": the aggregate of the child's M/V Manila City. The contracts, denominated as with a judgment against CSEW:
property or annual income; if this exceeds Work Orders, were signed thereafter., with the 1. To pay unto plaintiff Prudential Guarantee and
P50,000.00, a bond is required - There is no following stipulations: Assurance, Inc., the subrogee, the amount of P45M,
evidence that the share of each of the minors in the “10. The Contractor shall replace at its own work and with interest at the legal rate until full payment is
proceeds of the group policy in question is the at its own cost any work or material which can be made; the amount of P56,715,000 representing loss
minor's only property. Without such evidence, it shown to be defective and which is communicated in of income of M/V MANILA CITY, with interest at the
would not be safe to conclude that, indeed, that is writing within one (1) month of redelivery of the legal rate until full payment is made;
his only property. vessel or if the vessel was not in the Contractor’s 2. To pay unto plaintiff, William Lines, Inc. the
Disposition the instant petition is GRANTED. The Possession, the withdrawal of the Contractor’s amount of P11M as payment, in addition to what it
Decision of 10 October 1991 and the Resolution of 19 workmen, or at its option to pay a sum equal to the received from the insurance company to fully cover
May 1992 of the public respondent in CA-G.R. SP No. cost of such replacement at its own works. These the injury or loss, in order to replace the M/V
22950 are SET ASIDE and the Decision of the conditions shall apply to any such replacements. MANILA CITY, with interest at the legal rate until full
Insurance Commission in IC Case No. RD-058 is 11. Save as provided in Clause 10, the Contractor payment is made; the sum of P927,039 for the loss
REINSTATED. Costs against the private respondent. shall not be under any liability to the Customer either of fuel and lub oil on board the vessel when she was
SO ORDERED. in contract or for delict or quasi-delict or otherwise completely gutted by fire at defendant, Cebu
except for negligence and such liability shall itself be Shipyard’s quay, with interest at the legal rate until
full payment is made; the sum of P3,054,677.95 as
INSURANCE Page 5

payment for the spare parts and materials used in - When M/V Manila City capsized, steel and angle - The that factual findings by the CA are conclusive
the M/V MANILA CITY during dry-docking with bars were noticed to have been newly welded along on the parties and are not reviewable by this Court.
interest at the legal rate until full payment is made; the port side of the hull of the vessel, at the level of They are entitled to great weight and respect, even
P500,000 in moral damages;the amount of P10Min the crew cabins. William Lines did not previously finality, especially when, as in this case, the CA
attorney’s fees; and to pay the costs of this suit. apply for a permit to do hotworks on the said portion affirmed the factual findings arrived at by the trial
- On September 3, 1997, the Court of Appeals of the ship as it should have done pursuant to its court. When supported by sufficient evidence,
affirmed the appealed decision of the trial court, work order with CSEW. findings of fact by the CA affirming those of the trial
ordering CSEW to pay Prudential, the subrogee, the Prudential’s version court, are not to be disturbed on appeal. The
sum of P45 Million, with interest at the legal rate > At around 7AM of Feb. 16, 1991, the Chief Mate of rationale behind this doctrine is that review of the
until full payment is made. M/V Manila City was inspecting the various works findings of fact of the CA is not a function that the
CSEW’s version: being done by CSEW on the vessel, when he saw Supreme Court normally undertakes.
On Feb. 13, 1991, the CSEW completed the that some workers of CSEW were cropping out steel - The CA and the Cebu RTC are agreed that the fire
drydocking of M/V Manila City at its grave dock. It plates on Tank Top No. 12 using acetylene, oxygen which caused the total loss of subject M/V Manila
was then transferred to the docking quay of CSEW and welding torch. He also observed that the rubber City was due to the negligence of the employees and
where the remaining repair to be done was the insulation wire coming out of the air-conditioning workers of CSEW. Both courts found that the M/V
replating of the top of Water Ballast Tank No. 12 unit was already burning, prompting him to scold the Manila City was under the custody and control of
which was subcontracted by CSEW to JNB General workers. petitioner CSEW, when the ill-fated vessel caught
Services. Tank Top No. 12 was at the rear section of > At 2:45 PM of the same day, witnesses saw smoke fire. The decisions of both the lower court and the CA
the vessel, on level with the flooring of the crew coming from Tank No. 12. The vessel’s reeferman set forth clearly the evidence sustaining their finding
cabins located on the vessel’s second deck. reported such occurence to the Chief Mate who of actionable negligence on the part of CSEW. This
At around 7AM of Feb. 16, 1991, the JNB workers immediately assembled the crew members to put factual finding is accorded great weight and is
trimmed and cleaned the tank top framing which out the fire. When it was too hot for them to stay on conclusive on the parties. The court discerns no basis
involved minor hotworks (welding/cutting works). board and seeing that the fire cannot be controlled, for disturbing such finding firmly anchored on
The said work was completed at about 10AM. The the vessel’s crew were forced to withdraw from enough evidence.
JNB workers then proceeded to rig the steel plates, CSEW’s docking quay. - Furthermore, in petitions for review on certiorari,
after which they had their lunch break. The rigging - In the morning of Feb. 17, 1991, M/V Manila City only questions of law may be put into issue.
was resumed at 1PM sank. As the vessel was insured with Prudential Questions of fact cannot be entertained. The finding
While in the process of rigging the second steel Guarantee, William Lines filed a claim for of negligence by the CA is a question which this
plate, the JNB workers noticed smoke coming from constructive total loss, and after a thorough Court cannot look into as it would entail going into
the passageway along the crew cabins. When one of investigation of the surrounding circumstances of the factual matters on which the finding of negligence
the workers, Mr. Casas, proceeded to the tragedy, Prudential found the said insurance claim to was based. Such an approach cannot be allowed by
passageway to ascertain the origin of the smoke, he be meritorious and issued a check in favor of this Court in the absence of clear showing that the
noticed that smoke was gathering on the ceiling of William Lines in the amount of P45 million pesos case falls under any of the exceptions to the well-
the passageway but did not see any fire as the crew representing the total value of M/V Manila City’s hull established principle.
cabins on either side of the passageway were locked. and machinery insurance. The finding by the trial court and the Court of
He immediately sought out the proprietor of JNB, Appeals that M/V Manila City caught fire and sank by
Mr. Buenavista, and the Safety Officer of CSEW, Mr. ISSUES reason of the negligence of the workers of CSEW,
Aves, who sounded the fire alarm. CSEW’s fire 1. WON CSEW had “management and supervisory when the said vessel was under the exclusive
brigade immediately responded as well as the other control“ of the m/v manila city at the time the fire custody and control of CSEW is accordingly upheld.
fire fighting units in Metro Cebu. However, there broke out 2. YES
were no WLI representative, officer or crew to guide 2. WON the doctrine of res ipsa loquitur applies - For the doctrine of res ipsa loquitur to apply to a
the firemen inside the vessel. against the crew given situation, the following conditions must concur:
- Despite the combined efforts of the firemen of the 3. WON CSEW’S expert evidence is admissible or of (1) the accident was of a kind which does not
Lapulapu City Fire Dept., Mandaue Fire Dept., probative value ordinarily occur unless someone is negligent; and (2)
Cordova Fire Dept. Emergency Rescue Unit 4. WON Prudential has the right of subrogation that the instrumentality or agency which caused the
Foundation, and fire brigade of CSEW, the fire was against its own insured THE CONTRACTUAL 5. 5. 5. injury was under the exclusive control of the person
not controlled until 2AM of the following day. 5. WON the provisions limiting csew’s liability for charged with negligence.
- On the early morning of Feb. 17, 1991, gusty winds negligence to a maximum of p1 million are valid The facts and evidence on record reveal the
rekindled the flames on the vessel and fire again concurrence of said conditions in the case under
broke out. Then the huge amounts of water pumped HELD scrutiny. First, the fire that occurred and consumed
into the vessel, coupled with the strong current, 1. YES M/V Manila City would not have happened in the
caused the vessel to tilt until it capsized and sank ordinary course of things if reasonable care and
INSURANCE Page 6

diligence had been exercised. In other words, some - According to petitioner, under the aforecited clause,
negligence must have occurred. Second, the agency William Lines, Inc., agreed to assume the risk of loss
charged with negligence, as found by the trial court of the vessel while under drydock or repair and to
and the CA and as shown by the records, is the 4. YES such extent, it is benefited and effectively
herein petitioner, CSEW, which had control over - Petitioner contends that Prudential is not entitled to constituted as a co-assured under the policy.
subject vessel when it was docked for annual repairs. be subrogated to the rights of William Lines, Inc., - This theory of petitioner is devoid of sustainable
So also, as found by the RTC, “other responsible theorizing that (1) the fire which gutted M/V Manila merit. Clause 20 of the Work Order in question is
causes, including the conduct of the plaintiff, and City was an excluded risk and (2) it is a co-assured clear in the sense that it requires William Lines to
third persons, are sufficiently eliminated by the under the Marine Hull Insurance Policy. maintain insurance on the vessel during the period of
evidence.” - It is petitioner’s submission that the loss of M/V dry-docking or repair. Concededly, such a stipulation
What is more, in the present case the trial court Manila City or damage thereto is expressly excluded works to the benefit of CSEW as the shiprepairer.
found direct evidence to prove that the workers from the coverage of the insurance because the However, the fact that CSEW benefits from the said
and/or employees of CSEW were remiss in their duty same resulted from “want of due diligence by the stipulation does not automatically make it as a co-
of exercising due diligence in the care of subject Assured, Owners or Managers” which is not included assured of William Lines. The intention of the parties
vessel. The direct evidence substantiates the in the risks insured against. Again, this theory of to make each other a co-assured under an insurance
conclusion that CSEW was really negligent. Thus, petitioner is bereft of any factual or legal basis. It policy is to be gleaned principally from the insurance
even without applying the doctrine of res ipsa proceeds from a wrong premise that the fire which contract or policy itself and not from any other
loquitur, in light of the direct evidence on record, the gutted subject vessel was caused by the negligence contract or agreement because the insurance policy
ineluctable conclusion is that CSEW was negligent of the employees of William Lines, Inc. To repeat, denominates the assured and the beneficiaries of the
and consequently liable for damages to the the issue of who between the parties was negligent insurance. The hull and machinery insurance
respondent, William Lines, Inc. has already been resolved against CSEW. Upon proof procured by William Lines, Inc. from Prudential
3. NO of payment by Prudential to William Lines, Inc., the named only “William Lines, Inc.” as the assured.
- Petitioner maintains that the CA erred in former was subrogated to the right of the latter to There was no manifestation of any intention of
disregarding the testimonies of the fire experts, indemnification from CSEW. As aptly ruled by the William Lines, Inc. to constitute CSEW as a co-
Messrs. David Grey and Gregory Michael Southeard, Court of Appeals, the law on the matter is succinct assured under subject policy. It is axiomatic that
who testified on the probable origin of the fire in M/V and clear, to wit: when the terms of a contract are clear its stipulations
Manila City. Petitioner avers that since the said fire - Art. 2207. If the plaintiff’s property has been control.i] Thus, when the insurance policy involved
experts were one in their opinion that the fire did not insured, and he has received indemnity from the named only William Lines, Inc. as the assured
originate in the area of Tank Top No. 12 where the insurance company for the injury or loss arising out thereunder, the claim of CSEW that it is a co-assured
JNB workers were doing hotworks but on the crew of the wrong or breach of contract complained of, is unfounded.
accommodation cabins on the portside No. 2 deck, the insurance company shall be subrogated to the - Then too, in the Additional Perils Clause of the
the RTC and the CA should have given weight to rights of the insured against the wrongdoer or the same Marine Insurance Policy, it is provided that:
such finding based on the testimonies of fire experts; person who has violated the contract. If the amount Subject to the conditions of this Policy, this
petitioner argues. paid by the insurance company does not fully cover insurance also covers loss of or damage to vessel
But courts are not bound by the testimonies of the injury or loss, the aggrieved party shall be directly caused by the following:
expert witnesses. Although they may have probative entitled to recover the deficiency from the person xxx
value, reception in evidence of expert testimonies is causing the loss or injury. Negligence of Charterers and/or Repairers, provided
within the discretion of the court, under Section 49, - Thus, when Prudential, after due verification of the such Charterers and/or Repairers are not an
Rule 130 of the Revised Rules of Court. It is never merit and validity of the insurance claim of William Assured hereunder.
mandatory for judges to give substantial weight to Lines, Inc., paid the latter the total amount covered - As correctly pointed out by respondent Prudential,
expert testimonies. If from the facts and evidence on by its insurance policy, it was subrogated to the right if CSEW were deemed a co-assured under the policy,
record, a conclusion is readily ascertainable, there is of the latter to recover the insured loss from the it would nullify any claim of William Lines, Inc. from
no need for the judge to resort to expert opinion liable party, CSEW. Prudential for any loss or damage caused by the
evidence. In the case under consideration, the - Petitioner theorizes further that there can be no negligence of CSEW. Certainly, no shipowner would
testimonies of the fire experts were not the only right of subrogation as it is deemed a co-assured agree to make a shiprepairer a co-assured under
available evidence on the probable cause and origin under the subject insurance policy. To buttress its such insurance policy; otherwise, any claim for loss
of the fire. There were witnesses who were actually stance that it is a co-assured, petitioner placed or damage under the policy would be invalidated.
on board the vessel when the fire occurred. Between reliance on Clause 20 of the Work Order which Such result could not have been intended by William
the testimonies of the fire experts who merely based states: Lines, Inc.
their findings and opinions on interviews and the 20. The insurance on the vessel should be 5. NO
testimonies of those present during the fire, the maintained by the customer and/or owner of the - Although in this jurisdiction, contracts of adhesion
latter are of more probative value. vessel during the period the contract is in effect. have been consistently upheld as valid per se; as
INSURANCE Page 7

binding as an ordinary contract, the Court recognizes partnership in the City of Lucena. Under the business Reliance Insurance purveyed the same message as
instances when reliance on such contracts cannot be name of New Life Enterprises, the partnership well as Equitable Insurance Corporation.
favored especially where the facts and circumstances engaged in the sale of construction materials at its - The said policy in question follows:
warrant that subject stipulations be disregarded. place of business, a two storey building situated at "The insured shall give notice to the Company of any
Thus, in ruling on the validity and applicability of the Iyam, Lucena City. The facts show that Julian Sy insurance or insurances already effected, or which
stipulation limiting the liability of CSEW for insured the stocks in trade of New Life Enterprises may subsequently be effected, covering any of the
negligence to P1M only, the facts and circumstances with Western Guaranty Corporation, Reliance Surety property or properties consisting of stocks in trade,
vis-a-vis the nature of the provision sought to be and Insurance Co. Inc., and Equitable Insurance goods in process and/or inventories only hereby
enforced should be considered, bearing in mind the Corporation. insured, and unless such notice be given and the
principles of equity and fair play. - On May 15, 1981, Western Guaranty Corporation particulars of such insurance or insurances be stated
- It is worthy to note that M/V Manila City was issued Fire Insurance Policy No. 37201 in the amount therein or endorsed on this policy pursuant to
insured with Prudential for P45M. To determine the of P350,000.00. This policy was renewed on May 13, Section 50 of the Insurance Code, by or on behalf of
validity and sustainability of the claim of William 1982. the Company before the occurrence of any loss or
Lines, Inc., for a total loss, Prudential conducted its - On July 30, 1981, Reliance Surety and Insurance damage, all benefits under this policy shall be
own inquiry. Upon thorough investigation by its hull Co., Inc. issued Fire Insurance Policy No. 69135 in deemed forfeited, provided however, that this
surveyor, M/V Manila City was found to be beyond the amount of P300,000.00 (Renewed under condition shall not apply when the total insurance or
economical salvage and repair. The evaluation of the Renewal Certificate No. 41997). An additional insurances in force at the time of loss or damage is
average adjuster also reported a constructive total insurance was issued by the same company on not more than P200,000.00."
loss. The said claim of William Lines, Inc., was then November 12, 1981 under Fire Insurance Policy No. Petitioner’s comments
found to be valid and compensable such that 71547 in the amount of P700,000.00. > Petitioners contend that they are not to be blamed
Prudential paid the latter the total value of its - On February 8, 1982, Equitable Insurance for the omissions, alleging that insurance agent Leon
insurance claim. Furthermore, it was ascertained that Corporation issued Fire Insurance Policy No. 39328 Alvarez (for Western) and Yap Kam Chuan (for
the replacement cost of the vessel (the price of a in the amount of P200,000.00. Reliance and Equitable) knew about the existence of
vessel similar to M/V Manila City), amounts to P55M. - Thus when the building occupied by the New Life the additional insurance coverage and that they were
- Considering the aforestated circumstances, let Enterprises was gutted by fire at about 2:00 o'clock not informed about the requirement that such other
alone the fact that negligence on the part of in the morning of October 19, 1982, the stocks in or additional insurance should be stated in the policy,
petitioner has been sufficiently proven, it would trade inside said building were insured against fire in as they have not even read said policies.
indeed be unfair and inequitable to limit the liability the total amount of P1,550,000.00. According to the
of petitioner to One Million Pesos only. As aptly held certification issued by the Headquarters, Philippine ISSUE
by the trial court, “it is rather unconscionable if not Constabulary/Integrated National Police, Camp WON New Life Enterprises’ claim for payment be
overstrained.” To allow CSEW to limit its liability to Crame, the cause of fire was electrical in nature. denied
P1M notwithstanding the fact that the total loss According to the plaintiffs, the building and the
suffered by the assured and paid for by Prudential stocks inside were burned. After the fire, Julian Sy HELD
amounted to P45M would sanction the exercise of a went to the agent of Reliance Insurance whom he YES
degree of diligence short of what is ordinarily asked to accompany him to the office of the Ratio Furthermore, when the words and language of
required because, then, it would not be difficult for company so that he can file his claim. He averred documents are clear and plain or readily
petitioner to escape liability by the simple expedient that in support of his claim, he submitted the fire understandable by an ordinary reader thereof, there
of paying an amount very much lower than the clearance, the insurance policies and inventory of is absolutely no room for interpretation or
actual damage or loss suffered by William Lines, Inc. stocks. construction anymore. Courts are not allowed to
Disposition Petition is DENIED. Resolution of the He further testified that the three insurance make contracts for the parties; rather, they will
CA is AFFIRMED. companies are sister companies, and as a matter of intervene only when the terms of the policy are
fact when he was following-up his claim with ambiguous, equivocal, or uncertain. The parties
NEW LIFE ENTERPRISES V CA Equitable Insurance, the Claims Manager told him to must abide by the terms of the contract because
207 SCRA 669 go first to Reliance Insurance and if said company such terms constitute the measure of the insurer's
REGALADO; March 31, 1992 agrees to pay, they would also pay. The same liability and compliance therewith is a condition
treatment was given him by the other insurance precedent to the insured's right of recovery from the
NATURE companies. Ultimately, the three insurance insurer.
Appeal by certiorari companies denied plaintiffs' claim for payment. - While it is a cardinal principle of insurance law that
Respondent’s comments a policy or contract of insurance is to be construed
FACTS > Western Guaranty Corporation through Claims liberally in favor of the insured and strictly against
- The antecedents of this case show that Julian Sy Manager Bernard S. Razon told the plaintiff that his the insurer company, yet contracts of insurance, like
and Jose Sy Bang have formed a business claim 'is denied for breach of policy conditions.' other contracts, are to be construed according to the
INSURANCE Page 8

sense and meaning of the terms which the parties at a high speed, as a result of which, the plaintiff lost ISSUE
themselves have used. If such terms are clear and balance and fell from the bus. As plaintiff clung WON the CA erred in the interpretation of the
unambiguous, they must be taken and understood in instinctively to the handle bar, he was dragged by insurance contract on the limit of the insurer’s
their plain, ordinary and popular sense. Moreover, the bus along the asphalted road. The bus driver, Gil liability
obligations arising from contracts have the force of Agpalo, abruptly stopped the bus. Then fled from the
law between the contracting parties and should be scene, leaving the bus and the injured plaintiff HELD
complied with in good faith. behind. YES
Reasoning - The plaintiff was brought to the Manila Sanitarium - The insurance policy clearly placed the maximum
a. The terms of the contract are clear and and Hospital where the doctors performed 2 major limit of the petitioner's liability for damages arising
unambiguous. The insured is specifically required to surgical operations on plaintiffs right leg. from death or bodily injury at P12,000.00 per
disclose to the insurer any other insurance and its - Plaintiff was confined at the hospital for (40) days, passenger and its maximum liability per accident at
particulars which he may have effected on the same from June 10, 1984 to August 26, 1984. Medical (P50,000.00. Since only one passenger was injured
subject matter. The knowledge of such insurance by expenses totaled the amount of P69,444.41. in the accident, the insurer's liability for the damages
the insurer's agents, even assuming the acquisition Plaintiff’s medical expenses were advanced by his suffered by said passenger is pegged to the amount
thereof by the former, is not the "notice" that would employer Maglines but he was required to reimburse of P12,000.00 only.
stop the insurers from denying the claim. Besides, Maglines on a staggered basis by way of salary - The limit of P50,000.00 per accident means that
the so-called theory of imputed knowledge, that is, deductions. After his release from the hospital, he the insurer's maximum liability for any single
knowledge of the agent is knowledge of the principal, returned to the hospital for further treatment and accident will not exceed P50,000.00 regardless of the
aside from being of dubious applicability here has checkup. The injuries had left plaintiff with a huge number of passengers killed or injured therein.
likewise been roundly refuted by respondent court scar on his right leg. Also, the plaintiff incurred lost The bus company may not recover from the
whose factual findings we find acceptable. earning by way of unearned salaries amounting to insurance company more than P12,000.00 per
b. Petitioners should be aware of the fact that a P7,500.00 due to said physical injuries and the passenger killed or injured, or (P50,000.00) per
party is not relieved of the duty to exercise the consequent hospital confinement. accident even if under the judgment of the court, the
ordinary care and prudence that would be exacted in - Plaintiff filed on June 26, 1985 the complaint erring bus operator will have to pay more than
relation to other contracts. The conformity of the against DMTC and its driver. Agpalo was later P12,000.00 to each injured passenger. The trial
insured to the terms of the policy is implied from his dropped as a party defendant because he could not court's interpretation of the insurance contract was
failure to express any disagreement with what is be served with summons. Upon filing its answer, the correct interpretation.
provided for. defendant DMTC filed a thirdparty complaint against Disposition petition for review is GRANTED. The
First Quezon City Insurance Co., Inc. September 17, decision promulgated by the CA, ordering the third
FIRST QUEZON CITY INSURANCE CO. v. CA (DE 1985, third-party defendant filed its answer to the party defendent, First Quezon City Insurance Co.,
DIOS MARIKINA TRANSPORT CO) third-party complaint. Inc. to indemnify theI private respondent, (DMTC),
218 SCRA 526 - TC held DMTC complaint dismissed for lack of merit the sum of P50,000.00 for the damages of the
GRINO-AQUINO; February 28, 1993 and as regards the third-party complaint First passenger, Jose V. Del Rosario, is hereby modified
Quezon City Insurance Co., Inc. was to indemnify by reducing the award to 12,000.00 only. Costs
NATURE third-party plaintiff DMTC in the sum of P12,000.00 against the private respondent De Dios Marikina
PETITION for review of the decision of the Court of with interest. There being no satisfactory warrant the Transportation Co., Inc.
Appeals. FQCIC seeks to limit to P12000, the amount court dismissed the rest of the claims in the
specified in the insurance contract, it’s liability to complaint and third-party complaint. TY V FIRST NATIONAL SURETY
indemnify the respomdemt DMTC, for the damages - The bus company appealed to the CA, which 1 SCRA 1324
suffered by a passenger, who accidentally fell off the modified the dispositive as regards the third-party LABRADOR; April 29, 1961
bug. complaint, that the third-party defendant First
Quezon City Insurance Co., Inc. be ordered to FACTS
FACTS indemnify third-party plaintiff DMTC the SUM of - At different times within a period of two months
- After sending off certain seamen at the departure P50,000.00 with legal interest. Insurance company prior to 24 December 1953, Diosdado C. Ty,
area of MIA, Jose V. del Rosario proceeded to the filed a MFR which was denied. employed as operator mechanic foreman in the
public utility bus stop. While at the bus stop, the Hence, this petition for review, assailing the Broadway Cotton Factory insured himself in 18 local
plaintiff saw a DMTC bus. While moving at a crawling appellate courts' interpretation of the provision of the insurance companies, among which being the 8
pace, it was taking several passengers, all of whom insurance contract on the limit of the insurer's above-named defendants, which issued to him
managed to board the bus while it was already at the liability. personal accident policies. Plaintiff’s beneficiary was
bus stop; plaintiff was the last one to board the bus. his employer, Broadway Cotton Factory, which paid
While the plaintiff was still on the bus with his hand the insurance premiums. On 24 December 1953, a
on the bus door, the slowly moving bus sped forward fire broke out which totally destroyed the Broadway
INSURANCE Page 9

Cotton Factory. Fighting his way out of the factory, - The Supreme Court affirmed the appealed decision, alongside the water hole. The car was towed and
plaintiff was injured on the left hand by a heavy with costs against the plaintiff-appellant. repaired by Morosi Motors at a total cost of P302.27.
object. He was brought to the Manila Central - When the repairs on the car had already been
University hospital, and after receiving first-aid, he MISAMIS LUMBER V CAPITAL INSURANCE made, Misamis made a report of the accident to
went to the National Orthopedic Hospital for 17 SCRA 288 Capital Insurance.
treatment of his injuries (fractures in index, middle, REYES; May 20, 1966 - Since Capital refused to pay for the total cost of to
fourth, and fifth fingers of left hand). From 26 wage and repairs, suit was filed in the municipal
December 1953 to 8 February 1954, he underwent NATURE court originally.
medical treatment in the hospital. The above- Direct appeal on a point of law from the judgment of - The defendant-appellant admits liability in the
described physical injuries have caused temporary the Court of First Instance of Manila amount of P150, but not for any excess thereof. The
total disability of plaintiff’s left hand. Plaintiff filed the lower court did not exonerate the said appellant for
corresponding notice of accident and notice of claim FACTS the excess because the company's absolution would
with all of the above-named defendants to recover - Misamis Lumber Corporation, under its former render the insurance contract one-sided and that the
indemnity. Defendants rejected plaintiff’s claim for name, Lanao Timber Mills, Inc., insured its Ford said insurer had not shown that the cost of repairs in
indemnity for the reason that there being no Falcon motor car for the amount of P14,000 with the sum of P302.27 is unreasonable, excessive or
severance of amputation of the left hand, the Capital Insurance & Surety Company, Inc. The padded, nor had it shown that it could have
disability suffered by him was not covered by his pertinent provisions of the policy provided, as undertaken the repairs itself at less expense.
policy. follows:
- Plaintiff sued the defendants in the Municipality 1. The Company will subject to the Limits of ISSUE
Court of this City, which dismissed his complaints. Liability indemnify the Insured against loss or WON Capital Insurance can be made to pay more
Thereafter, the plaintiff appealed to the Court of First damage to the Motor Vehicle and its accessories than P150
Instance Manila, presided by Judge Gregorio S. and spare parts whilst thereon.
Narvasa, which absolved the defendants from the 2. (a) by accidental collision or overturning or HELD
complaints. Hence, the appeal. collision or overturning consequent when NO
mechanical breakdown or consequent upon wear - The insurance policy stipulated in paragraph 4 that
ISSUE and tear. if the insured authorizes the repair the liability of the
WON Diosdado Ty is entitled to indemnity under the 3. At its option, the Company may pay in cash the insurer, per its sub-paragraph (a), is limited to
insurance policy for the disability of his left hand amount of the loss or damage or may repair, P150.00. The literal meaning of this stipulation must
reinstate or replace the Motor Vehicle or any part control, it being the actual contract, expressly and
HELD thereof or its accessories or spare parts. The plainly provided for in the policy.
- The agreement contained in the insurance policies liability of the Company shall not exceed the value - Recourse to legal hermeneutics is not called for
is the law between the parties. As the terms of the of the parts lost or damaged and the reasonable because paragraph 4 of the policy is clear and
policies are clear, express and specific that only cost of fitting such parts or the value of the Motor specific and leaves no room for interpretation.
amputation of the left hand should be considered as Vehicle at the time of the loss or damage - The option to undertake the repairs is accorded to
a loss thereof, an interpretation that would include whichever is the loss. The Insured's estimate of the insurance company per paragraph 2. The said
the mere fracture or other temporary disability not value stated in the schedule shall be the maximum company was deprived of the option because the
covered by the policies would certainly be amount payable by the Company in respect of any insured took it upon itself to have the repairs made,
unwarranted. In the case at bar, due to the clarity of claim for loss or damage. and only notified the insurer when the repairs were
the stipulation, distinction between “temporary 4. The Insured may authorize the repair of the done. As a consequence, paragraph 4, which limits
disability” and “total disability” need not be made in Motor Vehicle necessitated by damage for which the company's liability to P150.00, applies.
relation to one’s occupation means that the condition the Company may be liable under this policy - The insurance contract may be rather onerous
of the insurance is such that common prudence provided that: ("one-sided", as the lower court put it), but that in
requires him to desist from transacting his business (a) the estimated cost of such repair does not itself does not justify the abrogation of its express
or renders him incapable of working. While the Court exceed the authorized Repair Limit. terms, terms which the insured accepted or adhered
sympathizes with the plaintiff or his employer, for (b) a detailed estimate of the cost is forwarded to and which is the law between the contracting
whose benefit the policies were issued, it can not go to the Company without delay and providing also parties.
beyond the clear and express conditions of the that the authorized repair limit is P150.00. - To require the insurer to prove that the cost of the
insurance policies, all of which define partial - One night, the insured car, while traveling along in repairs ordered by the insured is unreasonable, when
disability as loss of either hand by a amputation Aurora Boulevard, passed over a water hole which the insurer was not given an opportunity to inspect
through the bones of the wrist.” There was no such the driver did not see because an oncoming car did and assess the damage before the repairs were
amputation in the case at bar. not dim its light. The crankcase and flywheel housing made, is contrary to elementary justice and equity.
of the car broke when it hit a hollow block lying
INSURANCE Page
10

SUN INSURANCE OFFICE LTD. V CA (TAN) ambiguity. It has to be taken in its plain, ordinary, with Batigue , Aquino and John Doe, with violation of
195 SCRA 193 and popular sense. The rejection letter of February P.D. 532 (Anti-Highway Robbery Law)
PARAS; March 13, 1991 29, 1984 was clear and plain. The Court noted that - Demands were made by the Producers upon the
the one year period is likewise in accord with Section Fortune to pay the amount of the loss of
NATURE 23 of the Insurance Code which states that any P725,000.00 but the latter refused to pay as the loss
Petition for certiorari to review the decision of the CA condition which limits the time for commencing an is excluded from the coverage of the insurance policy
action to a period of less than one year when the specifically under "General Exceptions"
FACTS cause of action accrues is void. The right of action, > The company shall not be liable under this
- Private respondent Emilio Tan took from the according to the SC, accrues at the time that the policy in respect of x x x (b) any loss caused by
petitioner a Peso 300,000 property insurance policy claim is rejected at the first instance. A request for any dishonest, fraudulent or criminal act of the
to cover his interest in the electrical insurance store reconsideration of the denial cannot suspend the insured or any officer, employee, partner,
of his brother housed in a building in Iloilo City on running of the prescriptive period. The Court noted director, trustee or authorized representative of
August 15, 1983. Four days after the issuance of the that the rationale for the one year period is to ensure the Insured whether acting alone or in
policy, the building including the insured store that the evidence as to the origin and cause of the conjunction with others.
burned. destruction have not yet disappeared. - Fortune opposes the contention of Producers that
- On August 20, 1983, Tan filed his claim for fire 2. NO Atiga and Magalong are not its "officer, employee, x
loss. Sun Insurance, on February 29, 1984, wrote - The Court clarified its ruling in Eagle Star Insurance x x trustee or authorized representative x x x at the
the private respondent denying the claim. On April 3, Co. vs Chia Yu where it ruled that “the cause of time of the robbery
1984, private respondent wrote another letter to the action in an insurance contract does not accrue until - Trial Court
insurance company requesting reconsideration of the the Insured’s claim is finally rejected by the Insurer” > On being “EMPLOYEES”
denial. Tan’s lawyer wrote another letter to the by stating the use of the word “finally” cannot be Magalong and Atiga were not employees or
insurance company inquiring about the April 3 letter construed to mean the rejection of a petition for representatives of Producers as their services as
which sought for a reconsideration of the denial. In reconsideration. What the court referred to in effect armored car driver and as security guard having
its reply to the lawyer’s letter, Sun Insurance is the rejection in the first instance as claimed by been merely offered by PRC Management and by
reiterated its denial of the claim and enclosed therein Sun Insurance Unicorn Security and which latter firms assigned
copies of the two previous denials dated February Disposition The decision of the CA is reversed and them to plaintiff. The wages and salaries of both
29, 1984 and May 17, 1985. set aside. The case is dismissed Magalong and Atiga are presumably paid by their
- On November 20, 1985, Tan filed a civil case with respective firms, which alone wields the power to
the RTC. Petition filed a motion to dismiss on the FORTUNE INSURANCE AND SURETY CO. INC.V dismiss them
alleged ground that the action has already prescribed CA (PRODUCERS BANK OF THE PHILIPPINES) > On being “AUTHORIZED REPRESENTATIVE”
based on Condition 27 of the Insurance Policy which 244 SCRA 308 They were merely an assigned armored car driver
stated that the window to file the appropriate action DAVIDE; May 23, 1995 and security guard for the money transfer. It was
with either the Insurance Commission or in any court teller Maribeth Alampay who had "custody" of the
of competent jurisdiction is twelve months from the NATURE P725,000.00 cash being transferred along a specified
rejection of the claim. RTC denied the motion and Petition for Review on certiorari of CA decision money route
the subsequent motion for reconsideration. The CA - Court of Appeals
likewise denied the petition of Sun Insurance. FACTS > affirmed in toto
- Producers Bank of the Philippines filed a complaint > A policy or contract of insurance is to be construed
ISSUE against Fortune Insurance and Surety Co., Inc. for liberally in favor of the insured and strictly against
1. WON the court the filing of a motion for recovery of P725,000.00 under the policy issued by the insurance company (New Life Enterprises vs.
reconsideration interrupts the 12 months prescription Fortune. The sum was allegedly lost on June 29, Court of Appeals; Sun Insurance Office, Ltd. vs.
period to contest the denial of the insurance claim 1987 during a robbery of Producer's armored vehicle Court of Appeals). Contracts of insurance, like other
2. WON the rejection of the claim shall be deemed while it was in transit to transfer the money from its contracts, are to be construed according to the sense
final only if it contains words to the effect that the Pasay City Branch to its head office in Makati under and meaning of the terms which the parties
denial is final the custody of its teller, Maribeth Alampay. The themselves have used. If such terms are clear and
armored car was driven by Benjamin Magalong Y de unambiguous, they must be taken and understood in
Vera, escorted by Security Guard Saturnino Atiga Y their plain, ordinary and popular sense (New Life
HELD Rosete. Driver Magalong was assigned by PRC Enterprises Case; Sun Insurance Office).
1. NO Management Systems. > The language used by Fortune in the policy is
- The SC held that Condition 27 of the Insurance - After an investigation by the Pasay police, driver plain, ordinary and simple. No other interpretation is
policy is very clear and free from any doubt or Magalong and guard Atiga were charged, together necessary. The word "employee" should be taken to
INSURANCE Page
11

mean in the ordinary sense. The Labor Code is a Art. 106. Contractor or subcontractor. - There is WON Fortune Insurance and Surety Co. Inc. is liable
special law specifically dealing with/and specifically "labor-only" contracting where the person under the Money, Security, and Payroll Robbery
designed to protect labor and therefore its definition supplying workers to an employer does not have policy it issued to Producers Bank of the Philippines
as to employer-employee relationships insofar as the substantial capital or investment in the form of or WON recovery is precluded under the general
application/enforcement of said Code is concerned tools, equipment, machineries, work premises, exceptions clause of the policy
must necessarily be inapplicable to an insurance among others, and the workers recruited and
contract. Had it intended to apply the Labor Code in placed by such persons are performing activities HELD
defining what the word "employee" refers to, it which are directly related to the principal business NO
must/ should have so stated expressly in the of such employer. In such cases, the person or Ratio A contract of insurance is a contract of
insurance policy. Said driver and security guard intermediary shall be considered merely as an adhesion, thus any ambiguity therein should be
cannot be considered as employees of Producers agent of the employer who shall be responsible to resolved against the insurer, or it should be
bank because it has no power to hire or to dismiss the workers in the same manner and extent as if construed liberally in favor of the insured and strictly
said driver and security guard under the contracts the latter were directly employed by him. against the insurer. Limitations of liability should be
except only to ask for their replacements from the > International Timber Corp. vs. NLRC - a "labor- regarded with extreme jealousy and must be
contractors. only" contractor is equivalent to a finding that there construed in such a way as to preclude the insurer
- Fortune’s Contention is an employer-employee relationship between the from non-compliance with its obligation. It goes
> when Producers commissioned a guard and a owner of the project and the employee of the "labor- without saying then that if the terms of the contract
driver to transfer its funds from one branch to only" contractor are clear and unambiguous, there is no room for
another, they effectively and necessarily became its - Producer’s Contention construction and such terms cannot be enlarged or
authorized representatives in the care and custody of > Magalong and Atiga were not its employees since diminished by judicial construction.
the money. Assuming that they could not be it had nothing to do with their selection and - An insurance contract is a contract of indemnity
considered authorized representatives, they were, engagement, the payment of their wages, their upon the terms and conditions specified therein. It is
nevertheless, employees of Producers. It asserts that dismissal, and the control of their conduct. settled that the terms of the policy constitute the
the existence of an employer-employee relationship > International Timber Corp. is not applicable to all measure of the insurer's liability. In the absence of
"is determined by law and being such, it cannot be cases but only when it becomes necessary to prevent statutory prohibition to the contrary, insurance
the subject of agreement." Thus, if there was in any violation or circumvention of the Labor Code, a companies have the same rights as individuals to
reality an employer-employee relationship between social legislation whose provisions may set aside limit their liability and to impose whatever conditions
Producers, on the one hand, and Magalong and contracts entered into by parties in order to give they deem best upon their obligations not
Atiga, on the other, the provisions in the contracts of protection to the working man. inconsistent with public policy.
Producers with PRC Management System for > American President Lines vs. Clave should be Reasoning
Magalong and with Unicorn Security Services for applied which stated - It should be noted that the insurance policy entered
Atiga which state that Producers is not their In determining the existence of employer- into by the parties is a theft or robbery insurance
employer and that it is absolved from any liability as employee relationship, the following elements are policy which is a form of casualty insurance. Section
an employer, would not obliterate the relationship. generally considered, namely: (1) the selection 174 of the Insurance Code provides:
> an employer-employee relationship depends upon and engagement of the employee; (2) the Sec. 174. Casualty insurance is insurance covering
four standards: payment of wages; (3) the power of dismissal; and loss or liability arising from accident or mishap,
(1) the manner of selection and engagement of the (4) the power to control the employee's conduct. excluding certain types of loss which by law or
putative employee - Since under Producers' contract with PRC custom are considered as failing exclusively within
(2) the mode of payment of wages Management Systems it is the latter which assigned the scope of insurance such as fire or marine. It
(3) the presence or absence of a power to dismiss Magalong as the driver of Producers' armored car includes, but is not limited to, employer's liability
and and was responsible for his faithful discharge of his insurance, public liability insurance, motor vehicle
(4) the presence and absence of a power to control duties and responsibilities, and since Producers paid liability insurance, plate glass insurance, burglary
the putative employee's conduct. the monthly compensation of P1,400.00 per driver to and theft insurance, personal accident and health
> Of the four, the right-of-control test has been held PRC Management Systems and not to Magalong, it is insurance as written by non-life insurance
to be the decisive factor. It asserts that the power of clear that Magalong was not Producers' employee. As companies, and other substantially similar kinds of
control over Magalong and Atiga was vested in and to Atiga, Producers relies on the provision of its insurance. (italics supplied)
exercised by Producers. Fortune further insists that contract with Unicorn Security Services which - Except with respect to compulsory motor vehicle
PRC Management System and Unicorn Security provides that the guards of the latter "are in no liability insurance, the Insurance Code contains no
Services are but "labor-only" contractors under sense employees of the CLIENT." other provisions applicable to casualty insurance or
Article 106 of the Labor Code which provides: to robbery insurance in particular. These contracts
ISSUE are, therefore, governed by the general provisions
INSURANCE Page
12

applicable to all types of insurance. Outside of these, branch to its head office in Makati, its "authorized reconsideration, where the court said that although it
the rights and obligations of the parties must be representatives" who served as such with its teller now prohibits filing of such motion, the instant
determined by the terms of their contract, taking Maribeth Alampay. Howsoever viewed, Producers motion was filed before the effectivity of this rule,
into consideration its purpose and always in entrusted the three with the specific duty to safely thus allowing the adjudication of the case)
accordance with the general principles of insurance transfer the money to its head office, with Alampay WON Fidelity was liable to pay Verendia considering
law. to be responsible for its custody in transit; Magalong the circumstances
- With the foregoing principles in mind, it may now to drive the armored vehicle which would carry the
be asked whether Magalong and Atiga qualify as money; and Atiga to provide the needed security for HELD
employees or authorized representatives has been the money, the vehicle, and his two other 1. NO
aptly observed that in burglary, robbery, and theft companions. In short, for these particular tasks, the Ratio As the insurance contract is the law between
insurance, "the opportunity to defraud the insurer - three acted as agents of Producers. A the parties, Verendia is deemed to have forfeited his
the moral hazard - is so great that insurers have "representative" is defined as one who represents or right to claim by the misrepresentation he made.
found it necessary to fill up their policies with stands in the place of another; one who represents Reasoning
countless restrictions, many designed to reduce this others or another in a special capacity, as an agent, - the court reviewed the factual findings of the courts
hazard. Seldom does the insurer assume the risk of and is interchangeable with "agent." below, since it appears that there was a
all losses due to the hazards insured against." Disposition instant petition is hereby GRANTED. CA misapprehension of the facts by the CA.
Persons frequently excluded under such provisions decision and RTC Makati decision are REVERSED and - Verendia is found to have concocted the lease
are those in the insured's service and employment. SET ASIDE. Civil Case is DISMISSED. contract to deflect responsibility for the fire towards
The purpose of the exception is to guard against an alleged lessee, even making it appear that the
liability should the theft be committed by one having VERENDIA V CA (FIDELITY & SURETY CO. OF alleged lessee had disappeared, inflated the value of
unrestricted access to the property. In such cases, THE PHILS) the property, and insured same property with two
the terms specifying the excluded classes are to be 217 SCRA 417 other companies.
given their meaning as understood in common MELO; January 22, 1993 - An insurance contract is the law between the
speech. The terms "service" and "employment" are parties, its terms and conditions constitute the
generally associated with the idea of selection, NATURE measure of the insurer’s liability and compliance
control, and compensation. Petition to review decision of the CA therewith is a condition precedent to the insured’s
- There is marked disagreement between the parties right to recovery from the insurer.
on the correct meaning of the terms "employee" FACTS - As it is also a contract of adhesion, an insurance
and "authorized representatives." - Fidelity Co. issued a Fire Insurance Policy covering contract should be liberally construed in favor of the
It is clear to us that insofar as Fortune is concerned, Verendia’s residential building in the amount of insured and strictly against the insurer company
it was its intention to exclude and exempt from P385k. Verendia also insured the same building with which usually prepares it.
protection and coverage losses arising from two other companies (Country Bankers Insurance for - Considering, however, the fact that Verendia used
dishonest, fraudulent, or criminal acts of persons P56k, and Development Insurance for P400k). a false lease contract to support his claim, the terms
granted or having unrestricted access to Producers' - While all 3 policies were in force, the insured of the policy should be strictly construed against the
money or payroll. When it used then the term property was completely destroyed by fire. Verendia insured. Verendia failed to live by the terms of the
"employee," it must have had in mind any person filed a claim against Fidelity, but the latter refused policy, specifically Section 13 thereof which is
who qualifies as such as generally and universally payment, thus a complaint was filed in the RTC. expressed in terms that are clear and unambiguous,
understood, or jurisprudentially established in the Fidelity’s reason for refusal: the policy was avoided that all benefits under the policy shall be forfeited “If
light of the four standards in the determination of by reason of over-insurance, and that Verendia the claim be in any respect fraudulent, or if any false
the employer-employee relationship or as statutorily maliciously represented that the building was under declaration be made or used in support thereof, or if
declared even in a limited sense as in the case of lease to a Roberto Garcia, when it was actually a any fraudulent means or devises are used by the
Article 106 of the Labor Code which considers the Marcelo Garcia who was the lessee. Insured or anyone acting in his behalf to obtain any
employees under a "labor-only" contract as - RTC: policy was violated by Verendia when it failed benefit under the policy”. Verendia, having presented
employees of the party employing them and not of to inform Fidelity of his other insurance coverages, a false declaration to support his claim for benefits in
the party who supplied them to the employer. thus no need to pay. the form of a fraudulent lease contract, he forfeited
- But even granting for the sake of argument that - CA: reversed decision all benefits therein by virtue of Section 13 of the
these contracts were not "labor-only" contracts, and policy in the absence of proof that Fidelity waived
PRC Management Systems and Unicorn Security ISSUE such provision. Worse yet, by presenting a false
Services were truly independent contractors, we are (There is a procedural issue involved here, but is lease contract, Verendia reprehensibly disregarded
satisfied that Magalong and Atiga were, in respect of irrelevant to our discussion. It concerns the filing of a the principle that insurance contracts are uberrimae
the transfer of Producer's money from its Pasay City motion for extension of time to file a motion for fidae and demand the most abundant good faith.
INSURANCE Page
13

Disposition Decision of CA reversed, and that of mishap Federico Songco (father) and Rodolfo Songco on the doctrine of good faith and the avoidance of
RTC is reinstated. (son) died, Carlos Songco (another son), the latter's harm that will befall the innocent party due to its
FIELDMEN'S INSURANCE CO. INC V VDA. DE wife, Angelita Songco, and a family friend by the injurious reliance, the failure to apply it in this case
SONGCO name of Jose Manuel sustained physical injuries of would result in a gross travesty of justice.
25 SCRA 20 varying degree." 1 - That is all that needs be said insofar as the first
FERNANDO; 1968 - It was further shown according to the decision of alleged error of respondent Court of Appeals is
respondent Court of Appeals: "Amor Songco, 42- concerned, petitioner being adamant in its far-from-
FACTS year-old son of deceased Federico Songco, testifying reasonable plea that estoppel could not be invoked
- An insurance firm, petitioner Fieldmen's Insurance as witness, declared that when insurance agent by the heirs of the insured as a bar to the alleged
Co., Inc., was not allowed to escape liability under a Benjamin Sambat was inducing his father to insure breach of warranty and condition in the policy. lt
common carrier insurance policy on the pretext that his vehicle, he butted in saying: 'That cannot be, Mr. would now rely on the fact that the insured owned a
what was insured, not once but twice, was a private Sambat, because our vehicle is an "owner" private private vehicle, not a common carrier, something
vehicle and not a common carrier, the policy being vehicle and not for passengers,' to which agent which it knew all along when not once but twice its
issued upon the insistence of its agent who Sambat replied: 'whether our vehicle was an "owner" agent, no doubt without any objection in its part,
discounted fears of the insured that his privately type or for passengers it could be insured because exerted the utmost pressure on the insured, a man
owned vehicle might not fall within its terms, the their company is not owned by the Government and of scant education, to enter into such a contract.
insured moreover being "a man of scant education," the Government has nothing to do with their - Nor is there any merit to the second alleged error
finishing only the first grade. So it was held in a company. So they could do what they please of respondent Court that no legal liability was
decision of the lower court thereafter affirmed by whenever they believe a vehicle is insurable' ... In incurred under the policy by petitioner. Why liability
respondent Court of Appeals. Petitioner in seeking spite of the fact that the present case was filed and under the terms of the policy 5 was inescapable was
the review of the above decision of respondent Court tried in the CFI of Pampanga, the defendant set forth in the decision of respondent Court of
of Appeals cannot be so sanguine as to entertain the company did not even care to rebut Amor Songco's Appeals. Thus: "Since some of the conditions
belief that a different outcome could be expected. To testimony by calling on the witness-stand agent contained in the policy issued by the defendant-
be more explicit, we sustain the Court of Appeals. Benjamin Sambat, its Pampanga Field appellant were impossible to comply with under the
- The facts as found by respondent Court of Appeals, Representative." 2 existing conditions at the time and 'inconsistent with
binding upon us, follow: "This is a peculiar case. - The plaintiffs in the lower court, likewise the known facts,' the insurer 'is estopped from
Federico Songco of Floridablanca, Pampanga, a man respondents here, were the surviving widow and asserting breach of such conditions.' From this
of scant education being only a first grader ..., children of the deceased Federico Songco as well as jurisprudence, we find no valid reason to deviate and
owned a private jeepney with Plate No. 41-289 for the injured passenger Jose Manuel. On the above consequently hold that the decision appealed from
the year 1960. On September 15, 1960, as such facts they prevailed, as had been mentioned, in the should be affirmed. The injured parties, to wit, Carlos
private vehicle owner, he was induced by Fieldmen's lower court and in the respondent Court of Songco, Angelito Songco and Jose Manuel, for whose
Insurance Company Pampanga agent Benjamin Appeals.1awphîl.nèt hospital and medical expenses the defendant
Sambat to apply for a Common Carrier's Liability - The basis for the favorable judgment is the doctrine company was being made liable, were passengers of
Insurance Policy covering his motor vehicle ... Upon announced in Qua Chee Gan v. Law Union and Rock the jeepney at the time of the occurrence, and
paying an annual premium of P16.50, defendant Insurance Co., Ltd., 3 with Justice J. B. L. Reyes Rodolfo Songco, for whose burial expenses the
Fieldmen's Insurance Company, Inc. issued on speaking for the Court. It is now beyond question defendant company was also being made liable was
September 19, 1960, Common Carriers Accident that where inequitable conduct is shown by an the driver of the vehicle in question. Except for the
Insurance Policy No. 45-HO- 4254 ... the duration of insurance firm, it is "estopped from enforcing fact, that they were not fare paying passengers, their
which will be for one (1) year, effective September forfeitures in its favor, in order to forestall fraud or status as beneficiaries under the policy is recognized
15, 1960 to September 15, 1961. On September 22, imposition on the insured." 4 therein." 6
1961, the defendant company, upon payment of the - As much, if not much more so than the Qua Chee - Even if it be assumed that there was an ambiguity,
corresponding premium, renewed the policy by Gan decision, this is a case where the doctrine of an excerpt from the Qua Chee Gan decision would
extending the coverage from October 15, 1961 to estoppel undeniably calls for application. After reveal anew the weakness of petitioner's contention.
October 15, 1962. This time Federico Songco's petitioner Fieldmen's Insurance Co., Inc. had led the Thus: "Moreover, taking into account the well known
private jeepney carried Plate No. J-68136- insured Federico Songco to believe that he could rule that ambiguities or obscurities must be strictly
Pampanga-1961. ... On October 29, 1961, during the qualify under the common carrier liability insurance interpreted against the party that caused them, the
effectivity of the renewed policy, the insured vehicle policy, and to enter into contract of insurance paying 'memo of warranty' invoked by appellant bars the
while being driven by Rodolfo Songco, a duly the premiums due, it could not, thereafter, in any latter from questioning the existence of the
licensed driver and son of Federico (the vehicle litigation arising out of such representation, be appliances called for in the insured premises, since
owner) collided with a car in the municipality of permitted to change its stand to the detriment of the its initial expression, 'the undernoted appliances for
Calumpit, province of Bulacan, as a result of which heirs of the insured. As estoppel is primarily based the extinction of fire being kept on the premises
INSURANCE Page
14

insured hereby, ... it is hereby warranted ...,' admits complying with its obligation in the lower court and question of ownership was an excepted risk under
of interpretation as an admission of the existence of the Court of Appeals. Much less should it find any the marine insurance policies.
such appliances which appellant cannot now receptivity from us for its unwarranted and Petitioners Claim
contradict, should the parol evidence rule apply." 7 unjustified plea to escape from its liability. - an arrest by civil authority is not compensable
- To the same effect is the following citation from the since the term "arrest" refers to "political or
same leading case: "This rigid application of the rule MALAYAN INSURANCE CORP. V CA (TKC executive acts" and does not include a loss caused
on ambiguities has become necessary in view of MARKETING CORP.) by riot or by ordinary judicial process as in this case
current business practices. The courts cannot ignore 270 SCRA 242 - the deletion of the Free from Capture or Seizure
that nowadays monopolies, cartels and concentration ROMERO; March 20, 1997 Clause would leave the assured covered solely for
of capital, endowed with overwhelming economic the perils specified by the wording of the policy itself
power, manage to impose upon parties dealing with NATURE - the rationale for the exclusion of an arrest pursuant
them cunningly prepared 'agreements' that the Petition for review on certiorari to judicial authorities is to eliminate collusion
weaker party may not change one whit, his between unscrupulous assured and civil authorities.
participation in the 'agreement' being reduced to the FACTS - any loss which private respondent may have
alternative to 'take it or leave it' labelled since - TKC Marketing Corp. was the owner/consignee of incurred was in the nature and form of unrecovered
Raymond Saleilles 'contracts by adherence' (contrats some 3,189.171 metric tons of soya bean meal acquisition value brought about by a voluntary
d'adhesion), in contrast to those entered into by which was loaded on board the ship MV Al Kaziemah sacrifice sale and not by arrest, detention or seizure
parties bargaining on an equal footing, such for carriage from the port of Rio del Grande, Brazil, of the ship.
contracts (of which policies of insurance and to the port of Manila. Said cargo was insured against - its act of rejecting the claim was a result of its
international bills of lading are prime examples) the risk of loss by petitioner Malayan Insurance honest belief that the arrest of the vessel was not a
obviously call for greater strictness and vigilance on Corporation for which it issued two (2) Marine Cargo compensable risk under the policies issued
the part of courts of justice with a view to protecting Policies. Respondents Comments
the weaker party from abuses and imposition, and - While the vessel was docked in Durban, South - petitioner, being the sole author of the policies,
prevent their becoming traps for the unwary (New Africa the civil authorities arrested and detained it "arrests" should be strictly interpreted against it
Civil Code. Article 24; Sent. of Supreme Court of because of a lawsuit on a question of ownership and because the rule is that any ambiguity is to be taken
Spain, 13 Dec. 1934, 27 February 1942)." 8 possession. TKC Marketing notified Malayan of the contra proferentum. Risk policies should be
- The last error assigned which would find fault with arrest of the vessel and made a formal claim for the construed reasonably and in a manner as to make
the decision of respondent Court of Appeals insofar dollar equivalent on the policies (US$916,886.66) for effective the intentions and expectations of the
as it affirmed the lower court award for exemplary non-delivery of the cargo. It likewise sought the parties.
damages as well as attorney's fees is, on its face, of assistance of Malayan on what to do with the cargo. - the policies clearly stipulate that they cover the
no persuasive force at all. - Malayan replied that the arrest of the vessel by civil risks of non-delivery of an entire package and that it
- The conclusion that inescapably emerges from the authority was not a peril covered by the policies. TKC was petitioner itself that invited and granted the
above is the correctness of the decision of advised Malayan that it might tranship the cargo and extensions and collected premiums thereon.
respondent Court of Appeals sought to be reviewed. requested an extension of the insurance coverage
For, to borrow once again from the language of the until actual transhipment, which extension was ISSUES
Qua Chee Gan opinion: "The contract of insurance is approved upon payment of additional premium. The 1. WON the arrest of the vessel was a risk covered
one of perfect good faith (uberima fides) not for the insurance coverage was extended under the same under the subject insurance policies
insured alone,but equally so for the insurer; in fact, terms and conditions embodied in the original 2. WON insurance policies should be strictly
it is more so for the latter, since its dominant policies while in the process of making arrangements construed against the insurer
bargaining position carries with it stricter for the transhipment of the cargo from Durban to
responsibility." 9 Manila. However the cargo was sold in Durban, HELD
- This is merely to stress that while the morality of South Africa, for US$154.40 per metric ton or a total 1.YES
the business world is not the morality of institutions of P10,304,231.75 due to its perishable nature which - With the incorporation of subsection 1.1 of Section
of rectitude like the pulpit and the academe, it could no longer stand a voyage of twenty days to 1 of the Institute War Clauses, "arrest" caused by
cannot descend so low as to be another name for Manila and another twenty days for the discharge ordinary judicial process is deemed included among
guile or deception. Moreover, should it happen thus, thereof. It reduced its claim to US$448,806.09 (or its the covered risks. This interpretation becomes
no court of justice should allow itself to lend its peso equivalent of P9,879,928.89 at the exchange inevitable when subsection 1.1 of Section 1 of the
approval and support.1awphîl.nèt rate of P22.0138 per $1.00) representing its loss Institute War Clauses provided that "this insurance
- We have no choice but to recognize the monetary after the proceeds of the sale were deducted from covers the risks excluded from the Standard Form of
responsibility of petitioner Fieldmen's Insurance Co., the original claim.Malayan maintained its position English Marine Policy by the clause 'Warranted free
Inc. It did not succeed in its persistent effort to avoid that the arrest of the vessel by civil authorities on a of capture, seizure, arrest, etc. x x x'" or the F.C. &
INSURANCE Page
15

S. Clause. Jurisprudentially, "arrests" caused by be construed against the underwriters by whom the moral damages and attorney's fees because these
ordinary judicial process is also a risk excluded from policy is framed, and for whose benefit the exception items are not among those included in the Schedule
the Standard Form of English Marine Policy by the is introduced. Indemnities set forth in the insurance policy.
F.C. & S. Clause. - Petitioner Western in effect contends before this
- Petitioner cannot adopt the argument that the WESTERN GUARANTY CORPORATION V CA Court, as it did before the Court of Appeals, that
"arrest" caused by ordinary judicial process is not (RODRIGUEZ, and DE DIOS TRANSPORTATION because the Schedule of Indemnities limits the
included in the covered risk simply because the F.C. CO) amount payable for certain kinds of expenses
& S. Clause under the Institute War Clauses can only 187 SCRA 652 "hospital room", "surgical expenses", "an
be operative in case of hostilities or warlike FELICIANO; July 20, 1990 aesthesiologists' fee", "operating room" and "medical
operations on account of its heading "Institute War expenses" that Schedule should be read as
Clauses." FACTS excluding liability for any other type of expense or
2. YES - At around 4:30 in the afternoon of 27 March 1982, damage or loss even though actually sustained or
Ratio Insurance Policies should be construed while crossing Airport Road on a pedestrian lane on incurred by the third party victim. We are not
liberally in favor of the insured and strictly against her way to work, respondent Priscilla E. Rodriguez persuaded by Western's contention.
the insurer. was struck by a De Dios passenger bus owned by
Reasoning respondent De Dios Transportation Co., Inc., then ISSUE
- An insurance contract should be so interpreted as driven by one Walter Saga y Aspero. The bus driver WON the Schedule of indemnities as stated in the
to carry out the purpose for which the parties disregarded the stop signal given by a traffic insurance policy should be construed strictly to
entered into the contract which is, to insure against policeman to allow pedestrians to cross the road. exclude all others not explicitly stated therein
risks of loss or damage to the goods. Such Priscilla was thrown to the ground, hitting her
interpretation should result from the natural and forehead. She was treated at the Protacio Emergency
reasonable meaning of language in the policy. Where Hospital and later on hospitalized at the San Juan De
restrictive provisions are open to two interpretations, Dios Hospital. Her face was permanently disfigured, HELD
that which is most favorable to the insured is causing her serious anxiety and moral distress. NO
adopted. - Respondent bus company was insured with Ratio An insurance policy being in the nature of an
Indemnity and liability insurance policies are petitioner Western Guaranty Corporation ("Western") adhesion contract is to be strictly construed against
construed in accordance with the general rule of under its Master Policy which enumerated specific the insurer and liberally in favor of the insured.
resolving any ambiguity therein in favor of the liabilities of the insurance company and ended with a Reasoning
insured, where the contract or policy is prepared by clause to clarify the limitations of the amount which - Firstly, the Schedule of Indemnities does not
the insurer. A contract of insurance, being a contract could be granted as indemnity. purport to restrict the kinds of damages that may be
of adhesion, par excellence, any ambiguity therein - Respondent Priscilla Rodriguez filed a complaint for awarded against Western once liability has arisen.
should be resolved against the insurer.Limitations of damages before the Regional Trial Court of Makati Section 1, quoted above, does refer to certain "Limits
liability should be regarded with extreme jealousy against De Dios Transportation Co. and Walter A. of Liability" which in the case of the third party
and must be construed in such a way as to preclude Saga. Respondent De Dios Transportation Co., in liability section of the Master Policy, is apparently
the insurer from noncompliance with its obligations turn, filed a third-party complaint against its P50,000.00 per person per accident. Within this
- It must be borne in mind that such contracts are insurance carrier, petitioner Western. over-all quantitative limit, all kinds of damages
invariably prepared by the companies and must be - On 6 August 1985, the trial court rendered a allowable by law "actual or compensatory
accepted by the insured in the form in which they decision in favor of respondent Priscilla E. Rodriguez, damages"; "moral damages"; "nominal damages";
are written. Any construction of a marine policy - On appeal, the Court of Appeals affirmed in toto "temperate or moderate damages"; "liquidated
rendering it void should be avoided. Such policies the decision of the trial court. Petitioner moved for damages"; and "exemplary damages" may be
will, therefore, be construed strictly against the the reconsideration of the appellate court's decision. awarded by a competent court against the insurer
company in order to avoid a forfeiture, unless no In a Resolution dated 10 January 1990, the Court of once liability is shown to have arisen, and the
other result is possible from the language used. Appeals denied the motion for reconsideration for essential requisites or conditions for grant of each
- If a marine insurance company desires to limit or lack of merit. Petitioner Western is now before us on species of damages are present. It appears to us
restrict the operation of the general provisions of its a Petition for Review alleging that the Court of self-evident that the Schedule of Indemnities was not
contract by special proviso, exception, or exemption, Appeals erred in holding petitioner liable to pay intended to be an enumeration, much less a closed
it should express such limitation in clear and beyond the limits set forth in the Schedule enumeration, of the specific kinds of damages which
unmistakable language. Indemnities and in finding Western liable for loss of may be awarded under the Master Policy Western
Be that as it may, exceptions to the general earnings, moral damages and attorney's fees. has issued.
coverage are construed most strongly against the Succinctly stated, it is petitioner Western's position - Secondly, the reading urged by Western of the
company. Even an express exception in a policy is to that it cannot be held liable for loss of earnings, Schedule of Indemnities comes too close to working
INSURANCE Page
16

fraud upon both the insured and the third party and loose, in which the appellee dealt extensively. pasted (with other similar riders) on the face of the
beneficiary of Section 1, quoted above. For Western's They had been, with their contents, insured with the policies.4
reading would drastically and without warning limit defendant Company since 1937, and the loose made - It is argued that since the bodegas insured had an
the otherwise unlimited (save for the over-all payable to the Philippine National Bank as mortgage external wall perimeter of 500 meters or 1,640 feet,
quantitative limit of liability of P50,000.00 per person of the hemp and crops, to the extent of its interest. the appellee should have 11 fire hydrants in the
per accident) and comprehensive scope of liability - Fire of undetermined origin that broke out in the compound, and that he actually had only 2, with a
assumed by the insurer Western under Section 1: early morning of July 21, 1940, and lasted almost further pair nearby, belonging to the municipality of
"all sums necessary to discharge liability of the one week, gutted and completely destroyed Bodegas Tabaco.
insured in respect of [bodily injury to a third party]". Nos. 1, 2 and 4, with the merchandise stored
This result which is not essentially different from therein. Plaintiff-appellee informed the insurer by ISSUES
taking away with the left hand what had been given telegram on the same date; and on the next day, the 1. WON the defendant-appellant can claim the
with the right hand we must avoid as obviously fire adjusters engaged by appellant insurance policies it had issued as void ab initio
repugnant to public policy. If what Western now company arrived and proceeded to examine and 2. WON the insured violated the "Hemp Warranty"
urges is what Western intended to achieve by its photograph the premises, pored over the books of provisions of Policy No. 2637165 against the storage
Schedule of Indemnities, it was incumbent upon the insured and conducted an extensive of gasoline
Western to use language far more specific and investigation. The plaintiff having submitted the 3. WON the insured connived at the loss and
precise than that used in fact by Western, so that the corresponding fire claims, totalling P398,562.81 (but fraudulently inflated the quantity of the insured stock
insured, and potential purchasers of its Master reduced to the full amount of the insurance, in the burnt bodegas
Policy, and the Office of the Insurance P370,000), the Insurance Company resisted
Commissioner, may be properly informed and act payment, claiming violation of warranties and HELD
accordingly. conditions, filing of fraudulent claims, and that the 1. NO
- Petitioner Western would have us construe the fire had been deliberately caused by the insured or Ratio It is usually held that where the insurer, at the
Schedule of Indemnities as comprising contractual by other persons in connivance with him. time of the issuance of a policy of insurance, has
limitations of liability which, as already noted, is - Que Chee Gan, with his brother, Qua Chee Pao, knowledge of existing facts which, if insisted on,
comprehensively defined in Section 1 "Liability to and some employees of his, were indicted and tried would invalidate the contract from its very inception,
the Public" of the Master Policy. It is well-settled, in 1940 for the crime of arson, it being claimed that such knowledge constitutes a waiver of conditions in
however, that contractual limitations of liability found they had set fire to the destroyed warehouses to the contract inconsistent with the facts, and the
in insurance contracts should be regarded by courts collect the insurance. They were, however, acquitted insurer is stopped thereafter from asserting the
with a jaundiced eye and extreme care and should by the trial court. breach of such conditions. The law is charitable
be so construed as to preclude the insurer from - the civil suit to collect the insurance money enough to assume, in the absence of any showing to
evading compliance with its just obligations. proceeded to its trial with the CFI holding that: the contrary, that an insurance company intends to
- Finally, an insurance contract is a contract of judgment is rendered for the plaintiff and against the executed a valid contract in return for the premium
adhesion. The rule is well entrenched in our defendant condemning the latter to pay the former received; and when the policy contains a condition
jurisprudence that the terms of such contract are to — (a) Under the first cause of action, the sum of which renders it voidable at its inception, and this
be construed strictly against the party which P146,394.48; (b) Under the second cause of action, result is known to the insurer, it will be presumed to
prepared the contract, which in this case happens to the sum of P150,000; (c) Under the third cause of have intended to waive the conditions and to execute
be petitioner Western. action, the sum of P5,000; (d) Under the fourth a binding contract, rather than to have deceived the
cause of action, the sum of P15,000; and (e) Under
QUA CHEE GAN V LAW UNION AND ROCK the fifth cause of action, the sum of P40,000; all of 4
Memo. of Warranty. — The undernoted Appliances for the extinction
INSURANCE CO., LTD. which shall bear interest at the rate of 8% per of fire being kept on the premises insured hereby, and it being
96 PHIL 85 annum in accordance with Section 91 (b) of the declared and understood that there is an ample and constant water
supply with sufficient pressure available at all seasons for the same, it
REYES; December 17, 1955 Insurance Act from September 26, 1940, until each is hereby warranted that the said appliances shall be maintained in
is paid, with costs against the defendant. efficient working order during the currency of this policy, by reason
NATURE - In its first assignment of error, the insurance whereof a discount of 2 1/2 per cent is allowed on the premium
chargeable under this policy.
An appeal by defendant insurance company from the company alleges that the trial Court should have Hydrants in the compound, not less in number than one for each 150
decision of CFI in favor of the plaintiff held that the policies were avoided for breach of feet of external wall measurement of building, protected, with not less
than 100 feet of hose piping and nozzles for every two hydrants kept
warranty, specifically the one appearing on a rider under cover in convenient places, the hydrants being supplied with
FACTS water pressure by a pumping engine, or from some other source,
- before the last war, plaintiff-appellee owned 4 capable of discharging at the rate of not less than 200 gallons of water
per minute into the upper story of the highest building protected, and a
warehouses or bodegas in Tabaco, Albay, used for trained brigade of not less than 20 men to work the same.'
the storage of stocks of copra and of hemp, baled
INSURANCE Page
17

insured into thinking he is insured when in fact he is purpose of defense to an action to recover for a loss point below 300 Fahrenheit)", and is decidedly
not, and to have taken his money without thereafter occurring and at the same time treat it as ambiguous and uncertain; for in ordinary parlance,
consideration. valid for the purpose of earning and collecting further "Oils" mean "lubricants" and not gasoline or
Reasoning premiums. kerosene. And how many insured, it may well be
- The appellant is barred estoppel to claim violation - Moreover, taking into account the well known rule wondered, are in a position to understand or
of the so-called fire hydrants warranty, for the that ambiguities or obscurities must be strictly determine "flash point below 300 Fahrenheit.
reason that knowing fully all that the number of interpreted against the party that caused them, the - If the company intended to rely upon a condition of
hydrants demanded therein never existed from the "memo of warranty" invoked by appellant bars the that character, it ought to have been plainly
very beginning, the appellant neverthless issued the latter from questioning the existence of the expressed in the policy.
policies in question subject to such warranty, and appliances called for in the insured premises - The contract of insurance is one of perfect good
received the corresponding premiums. The insurance On the alleged violations of the plaintiff The faith not for the insured alone, but equally so for the
company was aware, even before the policies were alleged violation of the warranty of 100 feet of fire insurer; in fact, it is mere so for the latter, since its
issued, that in the premises insured there were only hose for every two hydrants, must be equally dominant bargaining position carries with it stricter
two fire hydrants installed by Qua Chee Gan and two rejected, since the appellant's argument thereon is responsibility.
others nearby, owned by the municipality of Tabaco, based on the assumption that the insured was bound - Another point that is in favor of the insured is that
contrary to the requirements of the warranty in to maintain no less than eleven hydrants, which the gasoline kept in Bodega No. 2 was only incidental
question requirement appellant is estopped from enforcing. to his business, being no more than a customary 2
- The plain, human justice of this doctrine is - As to maintenance of a trained fire brigade of 20 day's supply for the five or six motor vehicles used
perfectly apparent. To allow a company to men, the record is preponderant that the same was for transporting of the stored merchandise. "It is well
accept one's money for a policy of insurance organized, and drilled, from time to give, although settled that the keeping of inflammable oils on the
which it then knows to be void and of no effect, not maintained as a permanently separate unit, premises though prohibited by the policy does not
though it knows as it must, that the assured which the warranty did not require. void it if such keeping is incidental to the business."
believes it to be valid and binding, is so 2. NO On the submission of books, voucbers, etc. The
contrary to the dictates of honesty and fair Ratio Here, again, by reason of the exclusive control charge that the insured failed or refused to submit to
dealing, and so closely related to positive fraud, of the insurance company over the terms and the examiners of the insurer the books, vouchers,
as to the abhorrent to fair-minded men. It phraseology of the contract, the ambiguity must be etc. demanded by them was found unsubstantiated
would be to allow the company to treat the held strictly against the insurer and liberally in favor by the trial Court, and no reason has been shown to
policy as valid long enough to get the premium of the insured, specially to avoid a forfeiture. alter this finding. The insured gave the insurance
on it, and leave it at liberty to repudiate it the Insurance is, in its nature, complex and difficult examiner all the date he asked for, and the examiner
next moment. This cannot be deemed to be the for the layman to understand. Policies are even kept and photographed some of the examined
real intention of the parties. To hold that a prepared by experts who know and can books in his possession. What does appear to have
literal construction of the policy expressed the anticipate the hearing and possible been rejected by the insured was the demand that
true intention of the company would be to complications of every contingency. So long as he should submit "a list of all books, vouchers,
indict it, for fraudulent purposes and designs insurance companies insist upon the use of receipts and other records", but the refusal of the
which we cannot believe it to be guilty of. ambiguous, intricate and technical provisions, insured in this instance was well justified, since the
- The appellant company so worded the policies that which conceal rather than frankly disclose, demand for a list of all the vouchers (which were not
while exacting the greater number of fire hydrants their own intentions, the courts must, in in use by the insured) and receipts was positively
and appliances, it kept the premium discount at the fairness to those who purchase insurance, unreasonable, considering that such listing was
minimum of 2 1/2%, thereby giving the insurance construe every ambiguity in favor of the superfluous because the insurer was not denied
company a double benefit. Such abnormal treatment insured. An insurer should not be allowed, by access to the records, that the volume of Qua Chee
of the insured strongly points at an abuse of the the use of obscure phrases and exceptions, to Gan's business ran into millions, and that the
insurance company's selection of the words and defeat the very purpose for which the policy demand was made just after the fire when
terms of the contract, over which it had absolute was procured. everything was in turmoil. That the representatives
control. Reasoning of the insurance company were able to secure all the
- Receipt of Premiums or Assessments after Cause - Appellee admitted that there were 36 cans of date they needed is proved by the fact that the
for Forfeiture Other than Nonpayment. — It is a well gasoline in the building designed. It However, adjuster Alexander Stewart was able to prepare his
settled rule of law that an insurer which with gasoline is not specifically mentioned among the own balance sheet that did not differ from that
knowledge of facts entitling it to treat a policy as no prohibited articles listed in the so-called "hemp submitted by the insured except for the valuation of
longer in force, receives and accepts a premium on warranty." The cause relied upon by the insurer the merchandise, as expressly found by the Court in
the policy, estopped to take advantage of the speaks of "oils (animal and/or vegetable and/or the criminal case for arson.
forfeiture. It cannot treat the policy as void for the mineral and/or their liquid products having a flash 3. NO
INSURANCE Page
18

Ratio Both defenses are predicted on the - Under the policy: - Feb 24, 1957, Francisco del Rosario while on board
assumption that the insured was in financial Part I. Indemnity for Death the motor launch ISLAMA, with his beneficiary to the
difficulties and set the fire to defraud the insurance If the insured sustains any bodily injury policy, Remedios Jayme, were forced to jump off said
company, presumably in order to pay off the which is effected solely through violent, launch on account of fire which broke out on said
Philippine National Bank, to which most of the external, visible and accidental means, vessel, resulting in the death by drowning of the
insured hemp and copra was pledged. Both defenses and which shall result, independently of insured and his beneficiary.
are fatally undermined by the established fact that, all other causes and within sixty days - Simeon del Rosario, the insured’s father, filed a
notwithstanding the insurer's refusal to pay the value from the occurrence thereof, in the Death claim for payment with the company. The company
of the policies the extensive resources of the insured of the Insured, the Company shall pay the paid him P1000 pursuant to section 1 Part I of the
enabled him to pay off the National Bank in a short amount set opposite such injury: policy.
time; and if he was able to do so, no motive appears - On the same date, Atty. Francisco wrote to the
for attempt to defraud the insurer. While the Section 1. Injury sustained other than company acknowledging receipt by his client of the
acquittal of the insured in the arson case is not res those specified below unless excepted P1000 but informing said company that said amount
judicata on the present civil action, the insurer's hereinafter P1000 was not the correct one. He claimed that the amount
evidence, to judge from the decision in the criminal Section 2. Injury sustained by the payable should be P1500 under the provision of
case, is practically identical in both cases and must wrecking or disablement of a railroad Section 2 Part I, based on the rule of pari materia.
lead to the same result, since the proof to establish passenger car or street railway car in or - The company referred the matter to the Insurance
the defense of connivance at the fire in order to on which the Insured is traveling as a Coomissioner, who was of the opinion that the
defraud the insurer "cannot be materially less farepaying passenger P1500 liability of the company was only P1000. thus the
convincing than that required in order to convict the Section 3. Injury sustained by the burning company refused to pay more that P1000. Atty.
insured of the crime of arson." of a church, theatre, public library or Francisco wrote a subsequent letter to company
- As to the defense that the burned bodegas could municipal administration building while asking for p3000, which the company refused to pay.
not possibly have contained the quantities of copra the Insured is therein at the - A complaint for recovery of the balance of P2000
and hemp stated in the fire claims, the insurer's case commencement of the fire P2000 was instituted with the CFI Rizal, praying for a
rests almost exclusively on the estimates, inferences Section 4. Injury sustained by the further sum of P10000 as attorney’s fees, expenses
and conclusions of its adjuster investigator who wrecking or disablement of a regular of litigation and costs.
examined the premises during and after the fire. His passenger elevator car in which the - CFI ruled in favor of petitioner, ordering the
testimony, however, was based on inferences from Insured is being conveyed as a passenger company to pay P2000 to del Rosario.
the photographs and traces found after the fire, and (Elevator in mines exluded) P2500
must yield to the contradictory testimony of those Section 5. Injury sustained by a stroke of ISSUE
who actually saw the contents of the bodegas shortly lightning or by a cycloneP3000 How much should the indemnity be
before the fire, while inspecting them for the xxxx xxxx
mortgagee Bank. xxxx HELD
Disposition We find no reversible error in the Part VI. Exceptions - All the parties agree that indemnity has to be paid,
judgment appealed from, wherefore the same is This policy shall not cover disappearance but the conflict centers on how much it should be.
hereby affirmed. of the Insured nor shall it cover Death, - Where there is ambiguity with respect to the terms
Disability, Hospital fees, or Loss of time, and conditions of the policy, the same will be
DEL ROSARIO V EQUITABLE INSURANCE & caused to the insured: resolved against the one responsible thereof.
CASUALTY CO., INC x x x (h) By drowning except as a Generally, the insured has little, if any, participation
8 SCRA 343 consequence of the wrecking or in the preparation of the policy, together with the
PAREDES; June 29, 1963 disablement in the Philippine waters of a drafting of its terms and conditions. The
passenger steam or motor vessel in interpretation of obscure stipulations in a contract
NATURE which the Insured is traveling as a should not favor the party who caused the obscurity.
Appeal from judgment of CFI Rizal farepaying passenger; x x x - SC agreed with the ruling of the lower court:
- A rider to the Policy contained the following; x x x death by drowning is a ground for recovery
FACTS IV. DROWNING apart from the bodily injury because death by
- Francisco del Rosario was insured by Equitable It is hereby declared and agreed that bodily injury is covered by Part I of the policy
Insurance and Casualty Co. Inc under Personal exemption clause Letter (h) in PART VI of while death by drowning is covered by Part VI
Accident Policy no. 7136. The Company bound itself the policy is hereby waived by the thereof. But while the policy mentions specific
to pay P1000 to P3000 as indemnity for the death company, and to form a part of the amounts that may be recovered for death for
of the insured. provision covered by the policy. bodily injury, yet, there is not specific amount
INSURANCE Page
19

mentioned in the policy for death thru drowning claim under the policy. On 28 Dec 1990, the private respondent, thereby, for not disclosing such fact,
although the latter is, under Part VI of the policy, respondent denied the claim because it found that at violating Condition 3 of the policy
a ground for recovery thereunder. Since the the time of the loss the petitioner's stocks-in-trade 2. if he had, WON he is precluded from recovering
defendant has bound itself to pay P1000 to were likewise covered by two fire insurance policies therefrom
P3000 as indemnity for the death of the insured for P100,000.00 each, issued by the Cebu Branch of
but the policy does not positively state any the Philippines First Insurance Co., Inc. (PFIC). HELD
definite amount that may be recovered in case of -The basis of the private respondent's denial was the 1. YES
death by drowning, there is an ambiguity in this petitioner's alleged violation of Condition 3 of the - We agree w/ the CA that the petitioner knew of the
respect in the policy, which ambiguity must be policy. prior policies issued by the PFIC. His letter of 18
interpreted in favor of the insured and strictly - Geagonia then filed a complaint against the private January 1991 to the private respondent conclusively
against the insurer so as to allow a greater respondent w/ the Insurance Commission for the proves this knowledge. His testimony to the contrary
indemnity. x x x plaintiff is therefore entitled to recovery of P100,000.00 under fire insurance policy, before the Insurance Commissioner and which the
recover P3000. for attorney's fees, and costs of litigation. He claims latter relied upon cannot prevail over a written
Disposition Judgment appealed from is affirmed. that the time he obtained the private respondent's admission made ante litem motam. It was, indeed,
fire insurance policy he knew that the two policies incredible that he did not know about the prior
GEAGONIA v. CA (COUNTRY BANKERS issued by the PFIC were already in existence; policies since these policies were not new or original.
INSURANCE) however, he had no knowledge of the provision in 2. NO
8 SCRA 343 the private respondent's policy requiring him to - It must, however, be underscored that unlike the
DAVIDE; February 6 1995 inform it of the prior policies; this requirement was "other insurance" clauses involved in General
not mentioned to him by the private respondent's Insurance and Surety Corp. vs. Ng Hua or in
FACTS agent; and had it been so mentioned, he would not Pioneer Insurance & Surety Corp. vs. Yap, which
-Geagonia is the owner of Norman's Mart located in have withheld such information. He further asserted read:
the public market of San Francisco, Agusan del Sur. that the total of the amounts claimed under the "The insured shall give notice to the company of any
On 22 Dec 1989, he obtained from the private three policies was below the actual value of his insurance or insurances already effected, or which
respondent fire insurance policy for P100,000.00. stocks at the time of loss, w/c was P1M. may subsequently be effected covering any of the
The period of the policy was from 22 Dec 1989 to 22 - The Insurance Commission found that the property hereby insured, and unless such notice be
Dec 1990 and covered the ff: "Stock-in-trade petitioner did not violate Condition 3 as he had no given and the particulars of such insurance or
consisting principally of dry goods such as RTW's for knowledge of the existence of the two fire insurance insurances be stated in or endorsed on this Policy by
men and women wear and other usual to assured's policies obtained from the PFIC; that it was Cebu or on behalf of the Company before the occurrence
business. Tesing Textiles w/c procured the PFIC policies w/o of any loss or damage, all benefits under this Policy
-The policy contained the following condition: informing him or securing his consent; and that Cebu shall be forfeited." or in the 1930 case of Santa Ana
"3. The insured shall give notice to the Company of Tesing Textile, as his creditor, had insurable interest vs. Commercial Union Assurance Co. which
any insurance or insurances already effected, or on the stocks. These findings were based on the provided "that any outstanding insurance upon the
which may subsequently be effected, covering any of petitioner's testimony that he came to know of the whole or a portion of the objects thereby assured
the property or properties consisting of stocks in PFIC policies only when he filed his claim with the must be declared by the insured in writing and he
trade, goods in process and/or inventories only private respondent and that Cebu Tesing Textile must cause the company to add or insert it in the
hereby insured, and unless notice be given and the obtained them and paid for their premiums w/o policy, without which such policy shall be null and
particulars of such insurance or insurances be stated informing him. The Insurance Commission then void, and the insured will not be entitled to
therein or endorsed in this policy pursuant to Section ordered the respondent company to pay complainant indemnity in case of loss," Condition 3 in the
50 of the Insurance Code, by or on behalf of the the sum of P100,000.00 with legal interest from the private respondent's policy No. F-14622 does
Company before the occurrence of any loss or time the complaint was filed until fully satisfied plus not absolutely declare void any violation
damage, all benefits under this policy shall be the amount of P10,000.00 as attorney's fees. thereof. It expressly provides that the
deemed forfeited, provided however, that this -CA reversed the decision of the Insurance condition "shall not apply when the total
condition shall not apply when the total insurance or Commission because it found that the petitioner insurance or insurances in force at the time of
insurances in force at the time of the loss or damage knew of the existence of the two other policies issued the loss or damage is not more than
is not more than P200,000.00." by the PFIC P200,000.00."
-On 27 May 1990, fire of accidental origin broke out - Interpretation: It is a cardinal rule on insurance
at around 7:30 p.m. at the public market of San ISSUES that a policy or insurance contract is to be
Francisco, Agusan del Sur. The petitioner's insured 1. WON the petitioner had prior knowledge of the interpreted liberally in favor of the insured and
stocks-in-trade were completely destroyed two insurance policies issued by the PFIC when he strictly against the company, the reason being,
prompting him to file w/ the private respondent a obtained the fire insurance policy from the private undoubtedly, to afford the greatest protection which
INSURANCE Page
20

the insured was endeavoring to secure when he ASIDE and the decision of the Insurance Commission ISSUES
applied for insurance. It is also a cardinal principle of in Case No. 3340 is REINSTATED. 1. WON Felix Lim’s death was an accident, thus
law that forfeitures are not favored and that any making his widow Nerissa liable to claim the accident
construction which would result in the forfeiture of SUN INSURANCE OFFICE, LTD. V CA (LIM) insurance
the policy benefits for the person claiming 211 SCRA 554 2. WON the award of damages to Nerissa Lim was
thereunder, will be avoided, if it is possible to CRUZ; July 17, 1992 justified
construe the policy in a manner which would permit
recovery, as, for example, by finding a waiver for NATURE HELD
such forfeiture. Stated differently, provisions, Petition for review from the decision of the Court of 1. YES, Felix Lim’s death was an accident.
conditions or exceptions in policies which tend Appeals Ratio There is no accident when a deliberate act is
to work a forfeiture of insurance policies should performed unless some additional, unexpected,
be construed most strictly against those for FACTS independent and unforeseen happening occurs which
whose benefits they are inserted, and most - Felix Lim was issued a Personal Accident Policy produces or brings bout their injury or death.
favorably toward those against whom they are insurance with petitioner company with a face value Reasoning
intended to operate. The reason for this is that, of P200,000. His beneficiary was his wife Nerissa. - An accident has been defined to be that which
except for riders which may later be inserted, the - October 6, 1982 – Felix accidentally shot himself in happens by chance or fortuitously without intention
insured sees the contract already in its final form and the head with his own gun.  or design and which is unexpected, unusual and
has had no voice in the selection or arrangement of - He was playing with the handgun after he had unforeseen. It an event that takes pace without
the words employed therein. On the other hand, the removed the gun’s magazine (kasi naman…). one’s foresight or expectastion – an event that
language of the contract was carefully chosen and - He pointed the gun at his secretary and only proceeds from an unknown cause or is an unusual
deliberated upon by experts and legal advisers who witness Pilar Nalagon as a joke and assured her effect of a known case and therefore not expected.
had acted exclusively in the interest of the insurers that the gun was not loaded (are you sure…). It happens without any human agency, an event
and the technical language employed therein is - He then put the gun to his temple and fired it which, under the circumstances, is unusual to and
rarely understood by ordinary laymen. (haaay, sabi ko na nga ba). not expected by the person to whom it happens.
- With these principles in mind, we are of the - Both parties are in agreement that there was no - The firing of the gun was deemed to be the
opinion that Condition 3 of the subject policy is suicide. unexpected and independent and unforeseen
not totally free from ambiguity and must be - Nerissa claimed as Felix’s beneficiary but Sun occurrence that led to the insured person’s death.
meticulously analyzed. Such analysis leads us Insurance would not grant her claim, saying that her - There was no willful exposure to needless peril for
to conclude that (a) the prohibition applies only husband’s death was not an accident. the part of Felix. Suicide and exposure to needless
to double insurance, and (b) the nullity of the - Nerissa sued Sun Insurance and won the case. peril are similar in the sense that both signify
policy shall only be to the extent exceeding Sun Insurance was ordered to pay her P200,000 disregard for one’s life. Suicide imparts a positive
P200,000.00 of the total policies obtained. representing the face value of the claim along with act of ending one’s life whereas the latter indicates
- Furthermore, by stating within Condition 3 itself moral, exemplary and compensatory damages and recklessness that is almost suicidal in intent.
that such condition shall not apply if the total attorney’s fees. The decision was affirmed by the - Accident insurance policies were never meant to
insurance in force at the time of loss does not exceed CA. reward the insured for his tendency to show off or
P200,000.00, the private respondent was amenable Petitioners’ Claim for his miscalculations. They were intended to
to assume a co-insurer's liability up to a loss not - Sun Insurance cites one of the four exceptions in provide for contingencies.
exceeding P200,000.00. What it had in mind was to the contract of insurance which includes bodily injury - Lim was unquestionably negligent but it should not
discourage over-insurance. Indeed, the rationale consequent upon the insured person attempting to prevent his widow from recovering from the
behind the incorporation of "other insurance" clause commit suicide or willfully exposing himself to insurance policy he obtained precisely against
in fire policies is to prevent over-insurance and thus needless peril in an attempt to save a human life. accident.
avert the perpetration of fraud. When a property - There mere act of pointing the gun to his temple - Insurance contracts are, as a rule, supposed to be
owner obtains insurance policies from two or more showed that Felix willfully exposed himself to danger interpreted liberally in favor of the assured.
insurers in a total amount that exceeds the because a gun should always be handled with 2. NO, the claim for damages should not be granted
property's value, the insured may have an caution. for being unjust.
inducement to destroy the property for the purpose Respondents’ Comments Ratio A person may be made liable to the payment
of collecting the insurance. The public as well as the - Felix believed the gun to be safe because he had of moral damages if his act is wrongful. The adverse
insurer is interested in preventing a situation in removed the magazine. result of an action does not per se make the act
which a fire would be profitable to the insured. - He repeatedly assured his secretary that the gun wrongful and subject the act or to the payment of
Disposition Petition granted. The decision of the was not loaded. moral damages.
Court of Appeals in CA-G.R. SP No. 31916 is SET Reasoning
INSURANCE Page
21

- Petitioner was acting in good faith when it resisted of two and partly one storey constructed of reconsidered its decision of July 15, 1993, as
the private respondent’s claim on the ground that concrete below, timber above undergalvanized regards the imposition of interest.
the death of the insured was covered by the iron roof occupied as garage and quarters and - Undaunted, petitioner Rizal Surety & Insurance
exception. partly by open space and/or tracking/ Company found its way to the Court.
- The issue was debatable and was clearly not raised packing, beyond which is the aforementioned
only for the purpose of evading a legitimate Magdalo Street; on its right and left by ISSUE
obligation. driveway, thence open spaces, and at the rear WON the fire insurance policy litigated upon
by open spaces.'" protected only the contents of the main building
RIZAL SURETY & INSURANCE COMPANY V CA - The same pieces of property insured with the (four-span), and did not include those stored in the
(TRANSWORLD KNITTING MILLS, INC.) petitioner were also insured with New India two-storey annex building
336 SCRA 12 Assurance Company, Ltd., (New India).
PURISIMA; July 18, 2000 - Fire broke out in the compound of Transworld, HELD
razing the middle portion of its four-span building NO
NATURE and partly gutting the left and right sections - Resolution of the issue posited hinges on the
Petition for Review on Certiorari under Rule 45 of the thereof. A two-storey building (behind said four- proper interpretation of the stipulation in subject fire
Rules of Court span building) where fun and amusement insurance policy regarding its coverage, which reads:
machines and spare parts were stored, was also "xxx contained and/or stored during the currency of
FACTS destroyed by the fire. this Policy in the premises occupied by them forming
- Rizal Surety & Insurance Company (Rizal - Transworld filed its insurance claims with Rizal part of the buildings situate (sic) within own
Insurance) issued Fire Insurance Policy No. 45727 in Surety & Insurance Company and New India Compound xxx"
favor of Transworld Knitting Mills, Inc. (Transworld). Assurance Company but to no avail. - It can be gleaned unerringly that the fire insurance
- Pertinent portions of subject policy on the buildings - Private respondent brought against the said policy in question did not limit its coverage to what
insured, and location thereof, read: insurance companies an action for collection of were stored in the four-span building. As opined by
"‘On stocks of finished and/or unfinished sum of money and damages. the trial court of origin, two requirements must
products, raw materials and supplies of every - Petitioner Rizal Insurance countered that its fire concur in order that the said fun and amusement
kind and description, the properties of the insurance policy sued upon covered only the machines and spare parts would be deemed
Insureds and/or held by them in trust, on contents of the four-span building, which was protected by the fire insurance policy under scrutiny,
commission or on joint account with others partly burned, and not the damage caused by the to wit:
and/or for which they (sic) responsible in case fire on the two-storey annex building. "First, said properties must be contained and/or
of loss whilst contained and/or stored during - The trial court dismissed the case as against The stored in the areas occupied by Transworld and
the currency of this Policy in the premises New India Assurance Co., Ltd. but ordered second, said areas must form part of the building
occupied by them forming part of the buildings defendant Rizal Surety And Insurance Company to described in the policy xxx"
situate (sic) within own Compound at pay Transwrold (sic) Knitting Mills, Inc. - Said building of four-span lofty one storey in height
MAGDALO STREET, BARRIO UGONG, PASIG, - Both the petitioner, Rizal Insurance Company, with mezzanine portions is constructed of reinforced
METRO MANILA, PHILIPPINES, BLOCK NO. and private respondent, Transworld Knitting Mills, concrete and hollow blocks and/or concrete under
601.’ Inc., went to the Court of Appeals, which required galvanized iron roof and occupied as hosiery mills,
x New India Assurance Company to pay plaintiff- garment and lingerie factory, transistor-stereo
xx............ appellant the amount of P1,818,604.19 while the assembly plant, offices, ware house and caretaker's
...xxx....... Rizal Surety has to pay the plaintiff-appellant quarter.
........xxx P470,328.67. - The Court is mindful of the well-entrenched
‘Said building of four-span lofty one storey in - New India appealed to the Court theorizing inter doctrine that factual findings by the Court of Appeals
height with mezzanine portions is constructed alia that the private respondent could not be are conclusive on the parties and not reviewable by
of reinforced concrete and hollow blocks compensated for the loss of the fun and this Court, and the same carry even more weight
and/or concrete under galvanized iron roof amusement machines and spare parts stored at when the Court of Appeals has affirmed the findings
and occupied as hosiery mills, garment and the two-storey building because it (Transworld) of fact arrived at by the lower court.
lingerie factory, transistor-stereo assembly had no insurable interest in said goods or items. - In the case under consideration, both the trial court
plant, offices, warehouse and caretaker's - The Court denied the appeal with finality. and the Court of Appeals found that the so called
quarters. - Petitioner Rizal Insurance and private respondent "annex " was not an annex building but an integral
'Bounds in front partly by one-storey concrete Transworld, interposed a Motion for and inseparable part of the four-span building
building under galvanized iron roof occupied Reconsideration before the Court of Appeals, which described in the policy and consequently, the
as canteen and guardhouse, partly by building
INSURANCE Page
22

machines and spare parts stored therein were acting exclusively in the interest of, the PAN MALAYAN INSURANCE CORPORATION vs.
covered by the fire insurance in dispute. insurance company.' " COURT OF APPEALS (ERLINDA FABIE & HER
- Verily, the two-storey building involved, a - Equally relevant is the following disquisition of the UNKNOWN DRIVER)
permanent structure which adjoins and Court in Fieldmen's Insurance Company, Inc. vs. 184 SCRA 55; G.R. No. 81026
intercommunicates with the "first right span of the Vda. De Songco, to wit: CORTES; April 3, 1990
lofty storey building", formed part thereof, and "'This rigid application of the rule on ambiguities
meets the requisites for compensability under the has become necessary in view of current FACTS
fire insurance policy sued upon. business practices. The courts cannot ignore - December 10, 1985: PANMALAY filed a complaint
- So also, considering that the two-storey building that nowadays monopolies, cartels and for damages with the RTC of Makati against private
aforementioned was already existing when subject concentration of capital, endowed with respondents Erlinda Fabie and her driver. PANMALAY
fire insurance policy contract was entered into, overwhelming economic power, manage to averred the following: that it insured a Mitsubishi
petitioner should have specifically excluded the said impose upon parties dealing with them Colt Lancer car registered in the name of Canlubang
two-storey building from the coverage of the fire cunningly prepared 'agreements' that the Automotive Resources Corporation [CANLUBANG];
insurance if minded to exclude the same but if did weaker party may not change one whit, his that on May 26, 1985, due to the "carelessness,
not, and instead, went on to provide that such fire participation in the 'agreement' being reduced to recklessness, and imprudence" of the unknown
insurance policy covers the products, raw materials the alternative to 'take it or leave it' labelled driver of a pick-up, the insured car was hit and
and supplies stored within the premises of since Raymond Saleilles 'contracts by suffered damages in the amount of P42,052.00; that
respondent Transworld which was an integral part of adherence' (contrats [sic] d'adhesion), in PANMALAY defrayed the cost of repair of the insured
the four-span building occupied by Transworld, contrast to these entered into by parties car and, therefore, was subrogated to the rights of
knowing fully well the existence of such building bargaining on an equal footing, such contracts CANLUBANG against the driver of the pick-up and his
adjoining and intercommunicating with the right (of which policies of insurance and international employer, Erlinda Fabie; and that, despite repeated
section of the four-span building. bills of lading are prime example) obviously call demands, defendants, failed and refused to pay the
- Indeed, the stipulation as to the coverage of the for greater strictness and vigilance on the part claim of PANMALAY. PANMALAY clarified that the
fire insurance policy under controversy has created a of courts of justice with a view to protecting the damage caused to the insured car was settled under
doubt regarding the portions of the building insured weaker party from abuses and imposition, and the "own damage", coverage of the insurance policy.
thereby. Article 1377 of the New Civil Code provides: prevent their becoming traps for the unwary.'" - Private respondents filed a Motion to Dismiss
"Art.1377. The interpretation of obscure words - The issue of whether or not Transworld has an alleging that PANMALAY had no cause of action
or stipulations in a contract shall not favor the insurable interest in the fun and amusement against them. RTC dismissed PANMALAY's complaint
party who caused the obscurity" machines and spare parts, which entitles it to be for no cause of action and denied PANMALAY's
- Conformably, it stands to reason that the doubt indemnified for the loss thereof, had been settled in motion for reconsideration. CA affirmed. Hence, this
should be resolved against the petitioner, Rizal G.R. No. L-111118, entitled New India Assurance petition for review.
Surety Insurance Company, whose lawyer or Company, Ltd., vs. Court of Appeals, where the
managers drafted the fire insurance policy contract appeal of New India from the decision of the Court of ISSUE
under scrutiny. Citing the aforecited provision of law Appeals under review, was denied with finality by WON the insurer PANMALAY may institute an action
in point, the Court in Landicho vs. Government this Court on February 2, 1994. to recover the amount it had paid its assured in
Service Insurance System, ruled: - The rule on conclusiveness of judgment, which settlement of an insurance claim against private
"This is particularly true as regards insurance obtains under the premises, precludes the relitigation respondents as the parties allegedly responsible for
policies, in respect of which it is settled that the of a particular fact or issue in another action between the damage caused to the insured vehicle
'terms in an insurance policy, which are the same parties based on a different claim or cause
ambiguous, equivocal, or uncertain x x x are to of action. "xxx the judgment in the prior action HELD
be construed strictly and most strongly against operates as estoppel only as to those matters in YES
the insurer, and liberally in favor of the insured issue or points controverted, upon the determination - Article 2207 of the Civil Code is founded on the
so as to effect the dominant purpose of of which the finding or judgment was rendered. In well-settled principle of subrogation. If the insured
indemnity or payment to the insured, especially fine, the previous judgment is conclusive in the property is destroyed or damaged through the fault
where forfeiture is involved' and the reason for second case, only as those matters actually and or negligence of a party other than the assured, then
this is that the 'insured usually has no voice in directly controverted and determined and not as to the insurer, upon payment to the assured, will be
the selection or arrangement of the words matters merely involved therein." subrogated to the rights of the assured to recover
employed and that the language of the contract Disposition Decision, and the Resolution of the CA from the wrongdoer to the extent that the insurer
is selected with great care and deliberation by WERE AFFIRMED in toto. No pronouncement as to has been obligated to pay.
experts and legal advisers employed by, and costs. - General Rule: Payment by the insurer to the
assured operates as an equitable assignment to the
INSURANCE Page
23

former of all remedies which the latter may have technical meaning, the Court has on several Front: by a driveway thence at 18 meters distance
against the third party whose negligence or wrongful occasions defined these terms to mean that which by Bldg. No. 2.
act caused the loss. The right of subrogation is not takes place "without one's foresight or expectation, Right: by an open space thence by Bldg. No. 4.
dependent upon, nor does it grow out of, any privity an event that proceeds from an unknown cause, or is Left: Adjoining thence an imperfect wall by Bldg.
of contract or upon written assignment of claim. It an unusual effect of a known cause and, therefore, No. 4.
accrues simply upon payment of the insurance claim not expected." The concept "accident" is not Rear: by an open space thence at 8 meters
by the insurer. necessarily synonymous with the concept of "no distance.”
- Exceptions: fault". It may be utilized simply to distinguish - A fire that broke out in the early morning of
a.) if the assured by his own act releases the intentional or malicious acts from negligent or September 30,1991 gutted and consumed the new
wrongdoer or third party liable for the loss or careless acts of man. oil mill. Petitioner rejected respondent’s claim for
damage, from liability - Obiter Dicta: Even if under the above the insurance proceeds on the ground that no policy
b.) where the insurer pays the assured the value of circumstances PANMALAY could not be deemed was issued by it covering the burned oil mill. It
the lost goods without notifying the carrier who has subrogated to the rights of its assured under Article stated that the description of the insured
in good faith settled the assured's claim for loss 2207 of the Civil Code, PANMALAY would still have a establishment referred to another building.
c.) where the insurer pays the assured for a loss cause of action against private respondents. The Petitioner’s Claim
which is not a risk covered by the policy, thereby insurer who may have no rights of subrogation due The policies referred to the old mill, as stated in the
effecting "voluntary payment" to "voluntary" payment may nevertheless recover description contained in the policy.
- None of the exceptions are availing in the present from the third party responsible for the damage to
case. the insured property under Article 1236 of the Civil ISSUE
- When PANMALAY utilized the phrase "own damage" Code. WON new oil mill is insured by fire insurance policy
— a phrase which, incidentally, is not found in the Disposition Petition is GRANTED. Petitioner's
insurance policy — to define the basis for its complaint for damages against private respondents HELD
settlement of CANLUBANG's claim under the policy, it is REINSTATED. Case remanded to the lower court YES, new oil mill is insured.
simply meant that it had assumed to reimburse the for trial on the merits. Ratio In construing the words used descriptive of a
costs for repairing the damage to the insured building insured, the greatest liberality is shown by
vehicle. AMERICAN HOME ASSURANCE COMPANY V the courts in giving effect to the insurance. In view
- It is a basic rule in the interpretation of contracts TANTUCO ENTERPRISES of the custom of insurance agents to examine
that the terms of a contract are to be construed 366 SCRA 740 buildings before writing policies upon them, and
according to the sense and meaning of the terms PUNO; October 8, 2001 since a mistake as to the identity and character of
which the parties thereto have used. In the case of the building is extremely unlikely, the courts are
property insurance policies, the evident intention of NATURE inclined to consider that the policy of insurance
the contracting parties, i.e., the insurer and the Petition for Review on Certiorari assailing the covers any building which the parties manifestly
assured, determine the import of the various terms Decision of the Court of Appeals. intended to insure, however inaccurate the
and provisions embodied in the policy. It is only description may be.
when the terms of the policy are ambiguous, FACTS Reasoning
equivocal or uncertain, such that the parties - Respondent Tantuco Enterprises, Inc. is engaged in - The parties manifestly intended to insure the new
themselves disagree about the meaning of particular the coconut oil milling and refining industry. It owns oil mill. “On machineries and equipment with
provisions, that the courts will intervene. In such an two oil mills. Both are located at its factory complete accessories usual to a coconut oil mill
event, the policy will be construed by the courts compound at Iyam, Lucena City. Respondent including stocks of copra, copra cake and copra mills
liberally in favor of the assured and strictly against commenced its business operations with only one oil whilst contained in the new oil mill building, situate
the insurer. mill. In 1988, it started operating its second oil mill (sic) at UNNO. ALONG NATIONAL HIGH WAY, BO.
- Considering that the very parties to the policy were ( the new oil mill). IYAM, LUCENA CITY UNBLOCKED.”
not shown to be in disagreement regarding the - The two oil mills were separately covered by fire - If the parties really intended to protect the first oil
meaning and coverage of Section III-1, specifically insurance policies issued by petitioner American mill, then there is no need to specify it as new. It
sub-paragraph (a) thereof, it was improper for the Home Assurance Co. The first oil mill was insured would be absurd to assume that respondent would
appellate court to indulge in contract construction, to Policy No. 306-7432324-3 for the period March 1, protect its first oil mill for different amounts and
apply the ejusdem generis rule, and to ascribe 1991 to 1992.The new oil mill was insured under leave uncovered its second one.
meaning contrary to the clear intention and Policy No. 306-7432321-9 for the same term. Official - As may be gleaned from the testimony of the
understanding of these parties. receipts indicating payment for the full amount of the petitioner’s employee, the source of the discrepancy
- Although the terms "accident" or "accidental" as premium were issued by the petitioner's agent. happened during the preparation of the written
used in insurance contracts have not acquired a - Policy description: contract.
INSURANCE Page
24

- Respondent is not estopped from claiming that the award of exemplary and moral damages, as well as therewith is a condition precedent to the insured's
policy description is wrong. Evidence on record attorney’s fees. ( P32,000 total) right of recovery from the insurer.
reveals that respondent’s operating manager, Mr. - On November 11, 1981, Milagros Cayas filed a - In the case at bar, the insurance policy clearly and
Edison Tantuco, notified the petitioner’s agent with complaint for a sum of money and damages against categorically placed petitioner's liability for all
whom respondent negotiated for the contract about PCSI in the Court of First Instance of Cavite. damages arising out of death or bodily injury
the inaccurate description in the policy. However, - In view of Milagros Cayas' failure to prosecute the sustained by one person as a result of any one
Mr. Borja assured Mr. Tantuco that the use of the case, the court motu propio ordered its dismissal accident at P12,000.00.
adjective new will distinguish the insured property. without prejudice. - Said amount complied with the minimum fixed by
- Regarding policy requirements that fire - Alleging that she had not received a copy of the the law then prevailing, Section 377 of Presidential
extinguishment appliances should be available and in answer to the complaint, and that "out of Decree No. 612, which provided that the liability of
good working condition, warranty did not require sportsmanship", she did not file a motion to hold land transportation vehicle operators for bodily
respondent to provide for all the fire extinguishing PCSI in default, Milagros Cayas moved for the injuries sustained by a passenger arising out of the
appliances enumerated therein. Neither did it reconsideration of the dismissal order. Said motion use of their vehicles shall not be less than P12,000.
require that the appliances are restricted to those for reconsideration was acted upon favorably by the - In other words, under the law, the minimum
mentioned in the warranty. (Within the vicinity of the court. liability is P12,000 per passenger. Petitioner's liability
new oil mill can be found the following devices: - About two months later, Milagros Cayas filed a under the insurance contract not being less than
numerous portable fire extinguishers, two fire hoses, motion to declare PCSI in default for its failure to file P12,000.00, and therefore not contrary to law,
fire hydrant, and an emergency fire engine.) an answer. morals, good customs, public order or public policy,
- The object of the court in construing a contract is - The motion was granted and plaintiff was allowed said stipulation must be upheld as effective, valid
to ascertain the intent of the parties to the contract to adduce evidence ex-parte. and binding as between the parties.
and to enforce the agreement which the parties have - On July 13, 1982, the court rendered judgment by - In like manner, we rule as valid and binding upon
entered into. In determining what the parties default ordering PCSI to pay Milagros Cayas P50,000 private respondent the condition requiring her to
intended, the courts will read and construe the policy as compensation for the injured passengers, P5,000 secure the written permission of petitioner before
as a whole and if possible, give effect to all the parts as moral damages and P5,000 as attorney's fees. effecting any payment in settlement of any claim
of the contract. - Said decision was set aside after the PCSI filed a against her.
Disposition Petition is dismissed. motion therefor. Trial of the case ensued. - There is nothing unreasonable, arbitrary or
- In due course, the court promulgated a decision objectionable in this stipulation as would warrant its
PERLA COMPANIA DE SEGUROS, INC. v, CA ordering defendant Perla Compania de Seguros, Inc. nullification. The same was obviously designed to
(MILAGROS CAYAS) to pay plaintiff Milagros Cayas the sum of safeguard the insurer's interest against collusion
185 SCRA 741 P50,000.00 under its maximum liability as provided between the insured and the claimants.
FERNAN; May 28, 1990 for in the insurance policy; and the sum of P5,000.00 - It being specifically required that petitioner's
as reasonable attorney's fee written consent be first secured before any payment
NATURE - PCSI appealed to the Court of Appeals, which in settlement of any claim could be made, private
Petition for review on certiorari of a decision of the affirmed in toto the lower court's decision. respondent is precluded from seeking reimbursement
Court of Appeals - Its motion for reconsideration having been denied of the payments made to the three other passangers
by said appellate court, PCSI filed this petition in view of her failure to comply with the condition
FACTS contained in the insurance policy.
- Private respondent Milagros Cayas was the ISSUE - Clearly, the fundamental principle that contracts
registered owner of a Mazda bus, insured with Perla WON, as maintained by petitioner, its liability is are respected as the law between the contracting
Compania de Seguros, Inc. (PCSI) under a policy limited only to the payment made by private parties finds application in the present case.
issued on February 3, 1978. respondent to Perea and only up to the amount of - It was error on the part of the trial and appellate
- On December 17, 1978, the bus figured in an P12,000.00 courts to have disregarded the stipulations of the
accident in Naic, Cavite injuring several of its parties and to have substituted their own
passengers. HELD interpretation of the insurance policy.
- One of them, 19-year old Edgardo Perea, sued YES - In Phil. American General Insurance Co., Inc vs.
Milagros Cayas for damages in the CFI of Cavite, - The insurance policy involved explicitly limits Mutuc, we ruled that contracts which are the private
while three others agreed to a settlement of petitioner's liability to P12,000.00 per person and to laws of the contracting parties should be fulfilled
P4,000.00 each. P50,000.00 per accident. according to the literal sense of their stipulations, if
- After trial, the court rendered a decision in favor of - In Stokes vs. Malayan Insurance Co., Inc., the their terms are clear and leave no room for doubt as
Perea, ordering Cayas to compensate him, with an Court held that the terms of the contract constitute to the intention of the contracting parties, for
the measure of the insurer's liability and compliance contracts are obligatory, no matter what form they
INSURANCE Page
25

may be, whenever the essential requisites for their - Petitioners claimed that BPI required them to insurance policy over the mortgaged chattel is for
validity are present. obtain a motor vehicle insurance policy from FGU the benefit of BPI, this will result in partial or full
- In Pacific Oxygen & Acetylene Co. vs. Central Insurance Corporation (FGU Insurance). This is a satisfaction of the obligation only if the insurer pays
Bank," it was stated that the first and fundamental sister company of BPI. They had been paying the the mortgagee, BPI, or if the insurance proceeds
duty of the courts is the application of the law monthly installments on the vehicle until it figured in were paid to BPI. In this case, upon the loss of the
according to its express terms, interpretation being an accident where it became a total wreck. Under the vehicle due to total wreck, the petitioners filed a
called for only when such literal application is terms of the insurance policy from FGU Insurance, claim under the insurance policy, collected and
impossible. the vehicle had to be replaced or its value paid to received the proceeds thereof, but did not settle their
- We observe that although Milagros Cayas was able them. Due to the failure and refusal of FGU obligation with BPI which remained outstanding
to prove a total loss of only P44,000.00, petitioner Insurance to replace the vehicle or pay its value, despite the loss of the vehicle.
was made liable for the amount of P50,000.00, the they stopped payment of the monthly installments. Disposition CA decision AFFIRMED with the
maximum liability per accident stipulated in the - RTC ordered POLTANS to pay BPI the said amount. modification that the interest rate be reduced to 12%
policy. This is patent error. An insurance indemnity, - CA reversed and remanded case to RTC for trial on per annum from 24 May 1994 until fully paid, and
being merely an assistance or restitution insofar as the merits. the award of attorney’s fees be reduced to P50T.
can be fairly ascertained, cannot be availed of by any - RTC again ruled in favor of BPI. CA affirmed.
accident victim or claimant as an instrument of FILIPINO MERCHANTS INS. v. CA (CHOA TIEK
enrichment by reason of an accident. ISSUES SENG)
Disposition Petition granted. The decision of the 1. WON contracts presented in evidence by BPI were 179 SCRA 638
Court of Appeals is modified in that petitioner shall unjust and unacceptable contracts of adhesion REGALADO; November 28, 1989
pay Milagros Cayas the amount of Twelve Thousand 2. WON the terms and conditions of the
Pesos (P12,000. 00) plus legal interest from the comprehensive car insurance policy issued by FGU NATURE
promulgation of the decision of the lower court until should be deemed as having automatically operated Review of the decision of the CA
it is fully paid and attorney's fees in the amount of in favor of BPI as the assured mortgagee, and if so,
P5,000.00. it should be deemed as resulting in the FACTS
extinguishment of petitioner’s obligation - Plaintiff insured said shipment with defendant
POLTAN v. BPI & JOHN DOE insurance company under said cargo for the goods
G.R. No. 164307 HELD described as 600 metric tons of fishmeal in new
CHICO-NAZARIO; March 5, 2007 1. NO gunny bags of 90 kilos each from Bangkok, Thailand
Ratio A contract of adhesion is one in which one of to Manila against all risks under warehouse to
NATURE the parties imposes a ready-made form of contract, warehouse terms.
Petition for review of CA decision which the other party may accept or reject, but - Some of the goods arrived in bad condition. Plaintiff
which the latter cannot modify. It is just as binding made a claim against Filipino Merchants Insurance
FACTS as ordinary contracts. Company. The latter refused to pay. Plaintiff brought
- Petitioners POLTAN obtained a loan evidenced by a Reasoning an action against them. The defendant insurance
promissory note from the MANTRADE Dev’t Corp. - Petitioners failed to show that they were under company presented a third party complaint against
This was secured by a chattel mortgage over a 1-unit duress or forced to sign the loan documents. The the vessel and the arrastre contractor.
Nissan Sentra vehicle. natural presumption is that one does not sign a - Judgment was rendered against the insurance
- With notice to petitioners, MANTRADE assigned to document without first informing himself of its company. On the third party complaint, the third
BPI, by way of a Deed of Assignment, all its rights, contents and consequences. party defendants were ordered to pay the third party
title and interest to the promissory note and chattel - Contracts of adhesion are not entirely prohibited plaintiffs. The CA affirmed, but modified the same
mortgage. even as the courts remain careful in scrutinizing the with regard to the adjudication of the third-party
- Petitioners defaulted and so BPI demanded the factual circumstances underlying each case to complaint
whole balance of P286.5T including accrued interest, determine the respective claims of contending
or to return to BPI the possession of the motor parties on their efficacy. ISSUES
vehicle for foreclosure. It is specifically provided in 2. NO 1. WON some fortuity, casualty or accidental cause is
the promissory note and chattel mortgage that Reasoning needed to be proved despite the “all risks” policy (as
failure to pay any installment when due shall make - Petitioners failed to show any provision in the asserted by the insurance company)
subsequent installments and the entire balance of insurance policy or mortgage contract providing that 2. WON the respondent has an insurable interest
the obligation due and payable. the loss of the mortgaged vehicle extinguishes their
- After they refused to do so, BPI then filed principal obligation to BPI. HELD
complaint. - While it is true that the proceeds from the 1. NO
INSURANCE Page
26

- The very nature of the term "all risks" must be buyer, delivery of the goods to a carrier, whether books of accounts as unpaid and thus become
given a broad and comprehensive meaning as named by the buyer or not, for, the purpose of receivable item from their customers and dealers. x
covering any loss other than a willful and fraudulent transmission to the buyer is deemed to be a delivery xx
act of the insured. 7 This is pursuant to the very of the goods to the buyer, the exceptions to said rule - Petitioner is a customer and dealer of the products
purpose of an "all risks" insurance to give protection not obtaining in the present case. The Court has of IMC and LSPI. On February 25, 1991, the Gaisano
to the insured in those cases where difficulties of heretofore ruled that the delivery of the goods on Superstore Complex in Cagayan de Oro City, owned
logical explanation or some mystery surround the board the carrying vessels partake of the nature of by petitioner, was consumed by fire. Included in the
loss or damage to property. actual delivery since, from that time, the foreign items lost or destroyed in the fire were stocks of
- Generally, the burden of proof is upon the insured buyers assumed the risks of loss of the goods and ready-made clothing materials sold and delivered by
to show that a loss arose from a covered peril, but paid the insurance premium covering them IMC and LSPI. On February 4, 1992, respondent
under an "all risks" policy the burden is not on the - Moreover, the issue of lack of insurable interest filed a complaint for damages against petitioner. It
insured to prove the precise cause of loss or damage was not raised in petitioner’s answer. alleges that IMC and LSPI filed with respondent their
for which it seeks compensation. The insured under Disposition Petition denied claims under their respective fire insurance policies
an "all risks insurance policy" has the initial burden with book debt endorsements; that as of February
of proving that the cargo was in good condition when GAISANO CAGAYAN v. INSURANCE Co. OF 25, 1991, the unpaid accounts of petitioner on the
the policy attached and that the cargo was damaged NORTH AMERICA sale and delivery of ready-made clothing materials
when unloaded from the vessel; thereafter, the 490 SCRA 296 with IMC was P2,119,205.00 while with LSPI it was
burden then shifts to the insurer to show the Austria-Martinez; June 8, 2006 P535,613.00; that respondent paid the claims of IMC
exception to the coverage. As we held in Paris- and LSPI and, by virtue thereof, respondent was
Manila Perfumery Co. vs. Phoenix Assurance Co., NATURE subrogated to their rights against petitioner; that
Ltd. the basic rule is that the insurance company has Petition for review on certiorari of the Decision of the respondent made several demands for payment upon
the burden of proving that the loss is caused by the Court of Appeals petitioner but these went unheeded. In its Answer
risk excepted and for want of such proof, the with Counter Claim dated July 4, 1995, petitioner
company is liable. In the present case, there being FACTS contends that it could not be held liable because the
no showing that the loss was caused by any of the - Intercapitol Marketing Corporation (IMC) is the property covered by the insurance policies were
excepted perils, the insurer is liable under the policy. maker of Wrangler Blue Jeans. Levi Strauss (Phils.) destroyed due to fortuities event or force majeure;
2. YES Inc. (LSPI) is the local distributor of products bearing that respondent’s right of subrogation has no basis
- Section 13 of the Insurance Code defines insurable trademarks owned by Levi Strauss & Co.. IMC and inasmuch as there was no breach of contract
interest in property as every interest in property, LSPI separately obtained from respondent fire committed by it since the loss was due to fire which
whether real or personal, or any relation thereto, or insurance policies with book debt endorsements. it could not prevent or foresee; that IMC and LSPI
liability in respect thereof, of such nature that a The insurance policies provide for coverage on “book never communicated to it that they insured their
contemplated peril might directly damnify the debts in connection with ready-made clothing properties; that it never consented to paying the
insured. In principle, anyone has an insurable materials which have been sold or delivered to claim of the insured.
interest in property who derives a benefit from its various customers and dealers of the Insured - At the pre-trial conference the parties failed to
existence or would suffer loss from its destruction anywhere in the Philippines.” The policies defined arrive at an amicable settlement. Thus, trial on the
whether he has or has not any title in, or lien upon book debts as the “unpaid account still appearing in merits ensued. On August 31, 1998, the RTC
or possession of the property y. 16 Insurable interest the Book of Account of the Insured 45 days after the rendered its decision dismissing respondent’s
in property may consist in (a) an existing interest; time of the loss covered under this Policy.” The complaint. It held that the fire was purely accidental;
(b) an inchoate interest founded on an existing policies also provide for the following conditions: that the cause of the fire was not attributable to the
interest; or (c) an expectancy, coupled with an 1. Warranted that the Company shall not negligence of the petitioner; that it has not been
existing interest in that out of which the expectancy be liable for any unpaid account in established that petitioner is the debtor of IMC and
arises. respect of the merchandise sold and LSPI; that since the sales invoices state that “it is
- Respondent’s interest over the goods is based on delivered by the Insured which are further agreed that merely for purpose of securing
the perfected contract of sale. The perfected contract outstanding at the date of loss for a the payment of purchase price, the above-described
of sale between him and the shipper of the goods period in excess of six (6) months from merchandise remains the property of the vendor
operates to vest in him an equitable title even before the date of the covering invoice or until the purchase price is fully paid”, IMC and LSPI
delivery or before be performed the conditions of the actual delivery of the merchandise retained ownership of the delivered goods and must
sale. whichever shall first occur. bear the loss. Dissatisfied, petitioner appealed to the
- Further, Article 1523 of the Civil Code provides that 2. Warranted that the Insured shall submit to the CA. On October 11, 2000, the CA rendered its
where, in pursuance of a contract of sale, the seller Company within twelve (12) days after the close of decision setting aside the decision of the RTC. The
is authorized or required to send the goods to the every calendar month all amount shown in their CA held that the sales invoices are proofs of sale,
INSURANCE Page
27

being detailed statements of the nature, quantity and delivered to the customers and dealers of the with an existing interest in that out of which the
cost of the thing sold; that loss of the goods in the insured. expectancy arises.
fire must be borne by petitioner since the proviso - Indeed, when the terms of the agreement are clear - Therefore, an insurable interest in property does
contained in the sales invoices is an exception under and explicit that they do not justify an attempt to not necessarily imply a property interest in, or a lien
Article 1504 (1) of the Civil Code, to the general rule read into it any alleged intention of the parties, the upon, or possession of, the subject matter of the
that if the thing is lost by a fortuitous event, the risk terms are to be understood literally just as they insurance, and neither the title nor a beneficial
is borne by the owner of the thing at the time the appear on the face of the contract. Thus, what were interest is requisite to the existence of such an
loss under the principle of res perit domino; that insured against were the accounts of IMC and LSPI interest, it is sufficient that the insured is so situated
petitioner’s obligation to IMC and LSPI is not the with petitioner which remained unpaid 45 days after with reference to the property that he would be liable
delivery of the lost goods but the payment of its the loss through fire, and not the loss or destruction to loss should it be injured or destroyed by the peril
unpaid account and as such the obligation to pay is of the goods delivered. against which it is insured. Anyone has an insurable
not extinguished, even if the fire is considered a 2. NO interest in property who derives a benefit from its
fortuitous event; that by subrogation, the insurer has - The present case clearly falls under paragraph (1), existence or would suffer loss from its destruction.
the right to go against petitioner; that, being a fire Article 1504 of the Civil Code: Indeed, a vendor or seller retains an insurable
insurance with book debt endorsements, what was ART. 1504. Unless otherwise agreed, the goods interest in the property sold so long as he has any
insured was the vendor’s interest as a creditor. remain at the seller’s risk until the ownership interest therein, in other words, so long as he would
Petitioner filed a motion for reconsideration but it therein is transferred to the buyer, but when the suffer by its destruction, as where he has a vendor’s
was denied by the CA in its Resolution dated April ownership therein is transferred to the buyer the lien. In this case, the insurable interest of IMC and
11, 2001. goods are at the buyer’s risk whether actual LSPI pertain to the unpaid accounts appearing in
delivery has been made or not, except that: their Books of Account 45 days after the time of the
ISSUES (1) Where delivery of the goods has been made to loss covered by the policies.
1. WON the CA erred in construing a fire insurance the buyer or to a bailee for the buyer, in 3. YES
policy on book debts as one covering the unpaid pursuance of the contract and the ownership in - Petitioner’s argument that it is not liable because
accounts of IMC and LSPI since such insurance the goods has been retained by the seller the fire is a fortuitous event under Article 1174 of the
applies to loss of the ready-made clothing materials merely to secure performance by the buyer of Civil Code is misplaced. As held earlier, petitioner
sold and delivered to petitioner. his obligations under the contract, the goods bears the loss under Article 1504 (1) of the Civil
2. WON IMC bears the risk of loss because it are at the buyer’s risk from the time of such Code.
expressly reserved ownership of the goods by delivery; (Emphasis supplied) - Moreover, it must be stressed that the insurance in
stipulating in the sales invoices that “[i]t is further - Thus, when the seller retains ownership only to this case is not for loss of goods by fire but for
agreed that merely for purpose of securing the insure that the buyer will pay its debt, the risk of loss petitioner’s accounts with IMC and LSPI that
payment of the purchase price the above described is borne by the buyer. Accordingly, petitioner bears remained unpaid 45 days after the fire. Accordingly,
merchandise remains the property of the vendor the risk of loss of the goods delivered. petitioner’s obligation is for the payment of money.
until the purchase price thereof is fully paid.” - IMC and LSPI did not lose complete interest over Where the obligation consists in the payment of
3. WON the petitioner liable for the unpaid accounts the goods. They have an insurable interest until full money, the failure of the debtor to make the
payment of the value of the delivered goods. Unlike payment even by reason of a fortuitous event shall
HELD the civil law concept of res perit domino, where not relieve him of his liability. The rationale for this is
1. NO ownership is the basis for consideration of who bears that the rule that an obligor should be held exempt
- It is well-settled that when the words of a contract the risk of loss, in property insurance, one’s from liability when the loss occurs thru a fortuitous
are plain and readily understood, there is no room interest is not determined by concept of title, event only holds true when the obligation consists in
for construction. In this case, the questioned but whether insured has substantial economic the delivery of a determinate thing and there is no
insurance policies provide coverage for “book debts interest in the property. stipulation holding him liable even in case of
in connection with ready-made clothing materials - Section 13 of our Insurance Code defines insurable fortuitous event. It does not apply when the
which have been sold or delivered to various interest as “every interest in property, whether real obligation is pecuniary in nature.
customers and dealers of the Insured anywhere in or personal, or any relation thereto, or liability in - Under Article 1263 of the Civil Code, “[i]n an
the Philippines.”; and defined book debts as the respect thereof, of such nature that a contemplated obligation to deliver a generic thing, the loss or
“unpaid account still appearing in the Book of peril might directly damnify the insured.” destruction of anything of the same kind does not
Account of the Insured 45 days after the time of the Parenthetically, under Section 14 of the same Code, extinguish the obligation.” If the obligation is generic
loss covered under this Policy.” Nowhere is it an insurable interest in property may consist in: (a) in the sense that the object thereof is designated
provided in the questioned insurance policies that the an existing interest; (b) an inchoate interest founded merely by its class or genus without any particular
subject of the insurance is the goods sold and on existing interest; or (c) an expectancy, coupled designation or physical segregation from all others of
the same class, the loss or destruction of anything of
INSURANCE Page
28

the same kind even without the debtor’s fault and LSPI may have against petitioner. Failure to was made from respondent Travellers for its share in
before he has incurred in delay will not have the substantiate the claim of subrogation is fatal to the loss but was refused. Hence, complainants
effect of extinguishing the obligation. This rule is petitioner’s case for recovery of the amount of demanded from the other 3 respondents the balance
based on the principle that the genus of a thing can P535,613.00. of each share in the loss based on the computation
never perish. Genus nunquan perit. An obligation Disposition Petition is partly GRANTED. The excluding Travellers Multi-Indemnity in the amount
to pay money is generic; therefore, it is not assailed Decision dated October 11, 2000 and of P30,894.31 (P5,732.79-ZIC: P22,294.62, PBAC:
excused by fortuitous loss of any specific Resolution dated April 11, 2001 of the Court of and P2,866.90, SSS) but was refused, hence, this
property of the debtor. Appeals in CA-G.R. CV No. 61848 are AFFIRMED action.
- Thus, whether fire is a fortuitous event or petitioner with the MODIFICATION that the order to pay the
was negligent are matters immaterial to this case. amount of P535,613.00 to respondent is DELETED ISSUE
What is relevant here is whether it has been for lack of factual basis. WON petitioner Tai Tong has insurable interest in the
established that petitioner has outstanding accounts said policy
with IMC and LSPI. TAI TONG CHUACHE & CO v. INSURANCE
- With respect to IMC, the respondent has COMMISSION and TRAVELLERS MULTI- HELD
adequately established its claim. Petitioner has an INDEMNITY CORPORATION YES
outstanding account with IMC in the amount of 158 SCRA 366 - First, respondent insurance commission based its
P2,119,205.00, check voucher evidencing payment GANCAYCO; February 29, 1988 findings on mere inference. Respondent Insurance
to IMC, subrogation receipt executed by IMC in favor Commission absolved respondent insurance company
of respondent upon receipt of the insurance NATURE from liability on the basis of the certification issued
proceeds. All these documents have been properly Petition for review on certiorari of the decision of the by the then CFI, that in a certain civil action against
identified, presented and marked as exhibits in court. Insurance Commission the Palomos, Arsenio Lopez Chua stands as the
The subrogation receipt, by itself, is sufficient to complainant and not Tai Tong Chuache. From said
establish not only the relationship of respondent as FACTS evidence respondent commission inferred that the
insurer and IMC as the insured, but also the amount - Complainants Palomo acquired a parcel of land and credit extended by herein petitioner to the Palomos
paid to settle the insurance claim. The right of a building located in Davao City. They assumed the secured by the insured property must have been
subrogation accrues simply upon payment by the mortgage of the building in favor of SSS, which paid. Such is a glaring error which this Court cannot
insurance company of the insurance claim. building was insured with respondent SSS Accredited sanction.
Respondent’s action against petitioner is squarely Group of Insurers for P25K. - Second, it has been held in a long line of cases that
sanctioned by Article 2207 of the Civil Code which - On April 19, 1975, Azucena Palomo obtained a when the creditor is in possession of the document of
provides: P100K loan from Tai Tong Chuache Inc. (TTCC) and credit, he need not prove non-payment for it is
Art. 2207. If the plaintiff’s property has been executed a mortgage over the land and the building presumed. The validity of the insurance policy taken
insured, and he has received indemnity from the in favor of Tai Tong Chuache & Co. as security of b petitioner was not assailed by private respondent.
insurance company for the injury or loss arising payment .On April 25, 1975, Arsenio Chua, Moreover, petitioner's claim that the loan extended
out of the wrong or breach of contract complained representative of TTCC insured the latter's interest to the Palomos has not yet been paid was
of, the insurance company shall be subrogated to with Travellers Multi-Indemnity Corporation corroborated by Azucena Palomo who testified that
the rights of the insured against the wrongdoer or (Travellers) for P100K (P70K for bldg and P30K for they are still indebted to herein petitioner. So at the
the person who has violated the contract. x x x the contents thereof) time of the fire, petitioner as mortgagee still had
- Petitioner failed to refute respondent’s evidence. - On June 11, 1975, Pedro Palomo secured a Fire insurable interest therein.
- As to LSPI, respondent failed to present sufficient Insurance Policy, covering the building for P50K with - And third, petitioner's declaration that Arsenio
evidence to prove its cause of action. No evidentiary respondent Zenith Insurance Corporation (ZIC). Lopez Chua acts as the managing partner of the
weight can be given to Exhibit “F Levi Strauss”, a Another Fire Insurance Policy was later procured partnership was corroborated by respondent
letter dated April 23, 1991 from petitioner’s General from respondent Philippine British Assurance insurance company. Thus Chua as the managing
Manager, Stephen S. Gaisano, Jr., since it is not an Company (PBAC), covering the same building for partner of the partnership may execute all acts of
admission of petitioner’s unpaid account with LSPI. P50K and contents thereof for P70K. On July 31, administration including the right to sue debtors of
It only confirms the loss of Levi’s products in the 1975, the building and the contents were totally the partnership in case of their failure to pay their
amount of P535,613.00 in the fire that razed razed by fire. obligations when it became due and demandable. Or
petitioner’s building on February 25, 1991. - Based on the computation of the loss, including the at the least, Chua being a partner of petitioner Tai
- Moreover, there is no proof of full settlement of the Travellers, respondents, ZIC, PBAC, and SSS paid Tong Chuache & Company is an agent of the
insurance claim of LSPI; no subrogation receipt was their corresponding shares of the loss. Complainants partnership. Being an agent, it is understood that he
offered in evidence. Thus, there is no evidence that were paid the following: P41,546.79 by PBAC, acted for and in behalf of the firm.
respondent has been subrogated to any right which P11,877.14 by ZIC, and P5,936.57 by SSS. Demand
INSURANCE Page
29

Disposition Appealed decision SET ASIDE and aunt. Felicidad Estrada, with whom Sindayen left the
ANOTHER judgment is rendered order private HELD sum of P26.06 to complete the payment of the first
respondent Travellers to pay petitioner the face NO annual premium of P40.06.
value of Fire Insurance Policy in the amount of - An essential requisite of a valid contract is consent. - On January 1, 1933, Sindayen, who was then
P100K. Costs against said private respondent. Consent must be manifested by the meeting of the twenty-nine years of age, was examined by the
offer and the acceptance upon the thing and the company's doctor who made a favorable report, to
cause which are to constitute the contract. the company. On January 11, 1933, The company
PEREZ v. CA (BF LIFEMAN INSURANCE CORP.) - The offer must be certain and the acceptance accepted the risk and issued policy No. 47710 dated
323 SCRA 613 absolute. When Perez filed the application, it was back to December 1, 1932, and mailed the same to
YNARES-SANTIAGO; January 28, 2000 subject to the acceptance of BF. The perfection was its agent, Cristobal Mendoza, in Camiling, Tarlac, for
also further conditioned upon (1) the issuance of the delivery to the insured. -On January 11, 1933,
NATURE policy, Sindayen was at work in the Bureau of Printing. On
Petition for review on certiorari (2) the payment of the premium, and (3) the January 12, he complained of a severe headache and
delivery to and acceptance by the applicant in good remained at home. On January 15, he called a
FACTS health. physician who found that he was suffering from
- Primitivo Perez has been insured with the BF - The delivery and acceptance by the applicant was a acute nephritis and uremia and on January 19, 1933,
Lifeman Insurance Corporation (BF hereafter) since suspensive condition which was not fulfilled he died.
1980 for Php20,000. inasmuch as the applicant was already dead at the - On January 18, 1933, the agent, in accordance with
Sometime in 1987, Rodolfo Lalog (agent of BF) time the policy was issued. The non-fulfillment of the his agreement with the insured, delivered the policy
convinced him to apply for additional insurance condition resulted to Felicidad Estrada upon her payment of the balance
coverage of Php50,000. in the non-perfection of the contract. of the first year's annual premium. The agent asked
Perez accomplished the application form and passed - An application for insurance is merely an offer Felicidad Estrada if her nephew was in good health
the required medical examination. He also paid which requires the overt act of the insurer for it to and she replied that she believed so because she had
Php2,075 premium) to Lalog. ripen to a contract. Delay in acting on the application no information that he was sick and he thereupon
- On November 25, 1987, Perez died while riding a does not constitute acceptance even though the delivered to her the policy.
banca which capsized during a storm. During this insured has forwarded - On January 20, 1933, the agent learned of the
time his application papers for the additional his first premium with his application. Delay, in this death of Arturo Sindayen and called on Felicidad
insurance coverage was still with the Gumaca, case, does not constitute gross negligence because Estrada and asked her to return the policy. But he
Quezon the application was granted within the normal did not return or offer to return the premium paid.
office of BF. processing time. Felicidad Estrada on his aforesaid statement gave
- Without knowing that Perez died on November 25, Disposition Decision of CA affirmed in so far as it him the policy.
1987, BF approved Perez's application and issued the declared the insurance policy for Php50,000 issued - On February 4, 1933 Insular Life obtained from the
corresponding policy for the Php50,000 on December by BF null and void (no recission because it beneficiary, Sindayen’s wife, her signature to a legal
2, 1987. presupposes a valid contract) document entitled "ACCORD, SATISFACTION AND
- Virginia Perez (wife of the deceased) claimed the RELEASE" whereby in consideration of the sum of
benefits under the insurance policies of the VDA. DE SINDAYEN v. INSULAR P40.06 paid to her by a check of the company, she
deceased, but she was only able to receive 62 Phil 51 "assigns, releases and forever discharges said Isular
Php40,000 under the first insurance policy. BUTTE; September 4, 1935 Life Assurance Co., Ltd., its successors and assigns,
BF refused to pay the proceeds amounting to of all claims, obligation in or indebtedness. The said
Php150,000 under the additional policy coverage of FACTS check for P40.06 was never cashed but returned to
Php50,000 because they maintain that such policy - Arturo Sindayen, up to the time of his death on the company and appears in the record of this case
had not been perfected. January 19, 1933, was employed as a linotype as Exhibit D. Thereupon this action was brought to
- On September 21, 1990, BF filed a complaint operator in the Bureau of Printing at Manila and had enforce payment of the policy.
against Mrs. Perez seeking recission and declaration been such for eleven years prior thereto. While there By the terms of the policy, an annual premium of
of nullity of the insurance contract in question. Mrs. he made a written application on December 26, P40.06 is due on the first day of December of each
Perez filed a conterclaim 1932, to the defendant Insular Life Assurance Co., year, the first premium already paid by the insured
for the collection of Php150,000 plus damages. Ltd., through its agent, Cristobal Mendoza, for a covering the period from December 1, 1932. It is to
policy of insurance on his life in the sum of P1,000 December 1, 1933. It is to be noted that the policy
ISSUE and he paid to the agent P15 cash as part of the first was not issued and the company assumed no actual
WON there was a consummated contract of premium. It was agreed with the agent that the risk prior to January 11, 1933.The application which
insurance between Perez and BF policy, when and if issued, should be delivered to his the insured signed in Camiling, Tarlac, on December
INSURANCE Page
30

26, 1932, contained among others the following - Mendoza was duly licensed by the Insurance - "A local agent of an insurance company, whose
provisions: Commissioner to act as the agent of the defendant only power is to solicit applications for insurance, and
“3 That the said policy shall not take effect until the insurance company. The well known custom of the forward them to the company for approval, when, if
first premium has been paid and the policy has been insurance business and the evidence in this case approved to the insured, has no power to waive any
delivered to and accepted by me, while I am in good prove that Mendoza was not regarded by the of the provision of the policy so delivered."
health.” company as a mere conduit or automaton for the - It is clear, therefore, that the delivery of the policy
-Main defense of the company in this case, namely, performance of the physical act of placing the policy by Mendoza does not bind the defendant, nor is the
that the said policy never took effect because of in the hands of the insured defendant estopped from alleging its defense, for the
paragraph 3 of the application above quoted, for at - Granted that Mendoza's decision that the condition simple reason that Mendoza was not an agent with
the time of its delivery by the agent as aforesaid the had been met by the insured and that it was proper authority to issue policies or to accept risks in the
insured was not in good health to make a delivery of the policy to him is just as name of his principle.
binding on the company as if the decision had been -There is another ground upon which the majority
ISSUE made by its board of directors. Granted that opinion is based, namely, that the defendant waived
WON the insurance policy is valid Mendoza made a mistake of judgement because he the defense it now invokes, by reason of the delivery
acted on insufficient evidence as to the state of of the policy by its invokes, by reason of the delivery
HELD health of the insured. But it is not charged that the of the policy by its agent. It is admitted that if the
YES mistake was induced by any misconduct or omission delivery of the policy was due to fraud, legally there
- There is one line of cases which holds that the of duty of the insured. could have been no waiver. In view of the facts
stipulation contained in paragraph 3 is in the nature - It is the interest not only the applicant but of all established and admitted, there is no doubt, as to
of a condition precedent, that is to say, that there insurance companies as well that there should be the existence of the fraud. -Estrada, as a
can be no valid delivery to the insured unless he is in some act which gives the applicant the definite representative of the insured was not only bound to
good health at the time; that this condition assurance that the contract has been consummated. give a truthful information on the state of health of
precedent goes to the very essence of the contract This sense of security and of peace of mind that the insured, but it was her duty to find out it his true
and cannot be waived by the agent making delivery one's defendants are provided for without risk either state of health in order to give true and correct
of the policy of loss or of litigation is the bedrock of life insurance. information. When she gave Mendoza an incorrect
- On the other hand, a number of American decisions When the policy is issued and delivered, in the information tending to create the impression that the
hold that an agent to whom a life insurance policy absence of fraud or other grounds for rescission, it is insured was well when in fact he was seriously ill,
similar to the one here involved was sent with plainly not within the intention of the parties that there is no doubt that she committed fraud and
instructions to deliver it to the insured has authority there should be any questions held in abeyance or imparted a deceitful information to the defendant
to bind the company by making such delivery, reserved for future determination that leave the very agent
although the insured was not in good health at the existence of the contract in suspense and doubt.
time of delivery, on the theory that the delivery of - It is therefore in the public interest, for the public is ENRIQUEZ v. SUN LIFE OF CANADA
the policy being the final act to the consummation of profoundly and generally interested in life insurance, 41 PHIL 269
the contract, the condition as to the insurer's good as well as in the interest of the insurance companies MALCOLM; November 29, 1920
health was waived by the company. themselves by giving certainly and security to their
- we are inclined to the view that it is more policies, that we are constrained to hold, as we, do, NATURE
consonant with the well known practice of life that the delivery of the policy to the insured by an Appeal from judgment of trial court denying
insurance companies and the evidence in the present agent of the company who is authorized to make plaintiff’s (administrator of the estate of the late
case to rest our decision on the proposition that delivery or without delivery is the final act which Joaquin Ma. Herrer) action to recover from the
Mendoza was authorized by the company to make binds the company (and the insured as well) in the defendant life insurance company the sum of pesos
the delivery of the policy when he received the absence of fraud or other legal ground for rescission 6,000 paid by the deceased for a life annuity.
payment of the first premium and he was satisfied - The company therefore having decided that all the
that the insured was in good health. As was well said conditions precedent to the taking effect of the policy FACTS
in the case of MeLaurin vs. Mutual Life Insurance Co. had been complied with and having accepted the - On September 24, 1917, Joaquin Herrer made
“It is plain, therefore, that upon the facts it is not premium and delivered the policy thereafter to the application to the Sun Life Assurance Company of
necessarily a case of waiver or of estoppel, but a insured, the company is now estopped to assert that Canada through its office in Manila for a life annuity.
case where the local agents, in the exercise of the it never intended that the policy should take effect. Two days later he paid the sum of P6,000 to the
powers lodged in them, accepted the premium and manager of the company's Manila office and was
delivered the policy. That act binds their principal, SEPARATE OPINION given a receipt.
the defendant.” - The application was immediately forwarded to the
IMPERIAL [dissent] head office of the company at Montreal, Canada. On
INSURANCE Page
31

November 26, 1917, the head office gave notice of - Until quite recently, all of the provisions concerning Disposition Judgment is reversed, and the plaintiff
acceptance by cable to Manila. (Whether on the life insurance in the Philippines were found in the shall have and recover from the defendant the sum
same day the cable was received, notice was sent by Code of Commerce and the Civil Code. After July 1, of P6,000 with legal interest from November 20,
the Manila office of Herrera that the application had 1915, there was, however, in force the Insurance 1918, until paid, without special finding as to costs in
been accepted, is a disputed point, which will be Act. No. 2427. Chapter IV of this Act concerns life either instance.
discussed later.) On December 4, 1917, the policy and health insurance. The Act expressly repealed
was issued at Montreal. On December 18, 1917, Title VIII of Book II and Section III of Title III of
attorney Aurelio A. Torres wrote to the Manila office Book III of the code of Commerce. The law of
of the company stating that Herrer desired to insurance is consequently now found in the
withdraw his application. The following day the local Insurance Act and the Civil Code. VELASCO and ACOSTA v. APOSTOL and
office replied to Mr. Torres, stating that the policy - While, as just noticed, the Insurance Act deals with MAHARLIKA INSURANCE CO., INC.
had been issued, and called attention to the life insurance, it is silent as to the methods to be 173 SCRA 228
notification of November 26, 1917. This letter was followed in order that there may be a contract of REGALADO.; May 9, 1989
received by Mr. Torres on the morning of December insurance. On the other hand, the Civil Code, in
21, 1917. Mr. Herrer died on December 20, 1917. article 1802, not only describes a contact of life NATURE
- The chief clerk of the Manila office of Sun Life annuity markedly similar to the one we are Petition for review on certiorari
testified that he prepared the letter and handed it to considering, but in two other articles, gives strong
the local manager, Mr. E. E. White, for signature. clues as to the proper disposition of the case. For FACTS
The local manager, Mr. White, testified to having instance, article 16 of the Civil Code provides that - Petitioners were plaintiffs in a civil case of which
received the cablegram accepting the application of "In matters which are governed by special laws, any public respondent Hon. Apostol was the judge.
Mr. Herrer from the home office on November 26, deficiency of the latter shall be supplied by the - The case was an offshoot of an incident: plaintiffs
1917. He said that on the same day he signed a provisions of this Code." On the supposition, were riding in their car, when a taxicab crossed a
letter notifying Mr. Herrer of this acceptance. The therefore, which is incontestable, that the special law center island in the road and collided with their car.
witness further said that letters, after being signed, on the subject of insurance is deficient in enunciating Private respondent Maharlika was eventually
were sent to the chief clerk and placed on the the principles governing acceptance, the subject- impleaded as a defendant in this case, with an
mailing desk for transmission. Mr. Tuason, who was matter of the Civil code, if there be any, would be allegation that the taxicab involved was insured
the chief clerk on November 26, 1917, was not controlling. In the Civil Code is found article 1262 against third party liability for P20,000.00 with
called as a witness. providing that "Consent is shown by the concurrence private respondent at the time of the accident
- For the defense, attorney Manuel Torres testified to of offer and acceptance with respect to the thing and - Maharlika claimed there was no cause of action
having prepared the will of Joaquin Ma. Herrer. That the consideration which are to constitute the against it because at the time of the accident, the
on this occasion, Mr. Herrer mentioned his contract. An acceptance made by letter shall not bind alleged insurance policy was not in force due to the
application for a life annuity, and that he said that the person making the offer except from the time it non-payment of the premium thereon. Also, even if
the only document relating to the transaction in his came to his knowledge. the cab had been insured, the complaint would be
possession was the provisional receipt. Rafael - According to the provisional receipt, three things premature since the policy provides that the insurer
Enriquez, the administrator of the estate, testified had to be accomplished by the insurance company would be liable only when the insured becomes
that he had gone through the effects of the deceased before there was a contract: (1) There had to be a legally liable.
and had found no letter of notification from the medical examination of the applicant; (2) there had - Trial court ruled in favor of the plaintiff, holding the
insurance company to Mr. Herrer. to be approval of the application by the head office of defendants liable for repair of the car, medical
the company; and (3) this approval had in some way expenses, etc. BUT Maharlike was exonerated on the
ISSUE to be communicated by the company to the gnd that the policy was not in force.
WON there exists a contract for life annuity between applicant. The further admitted facts are that the - Petitioners elevated this case to this court, faulting
Herrer and defendant head office in Montreal did accept the application, did the respondent judge for considering the defense of
cable the Manila office to that effect, did actually late payment of premium when “the same was
HELD issue the policy and did, through its agent in Manila, waived at the pre-trial”, hence the evidence of late
NO actually write the letter of notification and place it in payment should be disregarded supposedly because
Ratio The law applicable to the case is found to be the usual channels for transmission to the addressee. the private respondent had admitted that such fact
the second paragraph of article 1262 of the Civil - The contract for a life annuity in the case at bar was not in issue.
Code providing that an acceptance made by letter was not perfected because it has not been proved - (More pertinent to this class: ) petitioners assert
shall not bind the person making the offer except satisfactorily that the acceptance of the application that the private respondent had agreed to grant the
from the time it came to his knowledge. ever came to the knowledge of the applicant. then prospective insured a credit extension of the
Reasoning premium due.
INSURANCE Page
32

- This controversy arose under the old insurance law,


Act No. 2427. TIBAY v. CA (FORTUNE LIFE & GENERAL HELD
- The accident occurred in 1973. The complaint was INSURANCE) NO
filed on July 20, 1974.  both before the 257 SCRA 126 Ratio Where the insurer and the insured expressly
effectivity of Presidential Decree no. 612, the BELLOSILLO; May 24, 1996 stipulated that the policy is not in force until the
subsequent insurance law which repealed its premium has been fully paid the payment of partial
predecessor FACTS premium by the assured in this particular instance
- The former insurance law, which applies to the case - On 22 January 1987 Fortune Life and General should not be considered the payment required by
here, provided: An insurer is entitled to the payment Insurance Co., Inc. (FORTUNE) issued Fire Insurance the law and the stipulation of the parties. Rather, it
of premium as soon as the thing insured is exposed Policy No. 136171 in favor of Violeta R. Tibay and/or must be taken in the concept of a deposit to be held
to the peril insured against, unless there is clear Nicolas Roraldo on their two-storey residential in trust by the insurer until such time that the full
agreement to grant the insured credit extension of building located at 5855 Zobel Street, Makati City, amount has been tendered and duly receipted for.
the premium due. No policy issued by an insurance together with all their personal effects therein. The Reasoning
company is valid and binding unless and until the insurance was for P600,000 covering the period from - As expressly agreed upon in the contract, full
premium thereof has been paid. 23 January 1987 to 23 January 1988. On 23 January payment must be made before the risk occurs for the
1987, of the total premium of P2,983.50, Violeta policy to be considered effective and in force. Thus,
ISSUE Tibay only paid P600 thus leaving a considerable no vinculum juris whereby the insurer bound itself to
WON the insurance policy would be valid and binding balance unpaid. indemnify the assured according to law ever resulted
notwithstanding the non-payment of the premium - On 8 March 1987 the insured building was from the fractional payment of premium. The
completely destroyed by fire. Two days later, Violeta insurance contract itself expressly provided that the
HELD Tibay paid the balance of the premium. On the same policy would be effective only when the premium was
NO day, she filed with FORTUNE a claim on the fire paid in full. It would have been altogether different
Ratio Act No. 2427: an insurance policy would be insurance policy. Her claim was accordingly referred were it not so stipulated. Ergo, petitioners had
valid and binding notwithstanding the non-payment to its adjuster, Goodwill Adjustment Services, Inc. absolute freedom of choice whether or not to be
of the premium if there was a clear agreement to (GASI), which immediately wrote Violeta requesting insured by FORTUNE under the terms of its policy
grant to the insured credit extension. Such her to furnish it with the necessary documents for and they freely opted to adhere thereto.
agreement may be express or implied. the investigation and processing of her claim. - Indeed, and far more importantly, the cardinal
Reasoning Petitioner forthwith complied. On 28 March 1987 she polestar in the construction of an insurance contract
- Petitioners maintain that in spite of their late signed a nonwaiver agreement with GASI to the is the intention of the parties as expressed in the
payment, the policy is binding because there was an effect that any action taken by the companies shall policy. Courts have no other function but to enforce
implied agreement to grant a credit extension so as not be, or be claimed to be, an admission of liability. the same. The rule that contracts of insurance will be
to make the policy effective. To them, the - FORTUNE denied the claim of Violeta for violation of construed in favor of the insured and most strongly
subsequent acceptance of the premium and delivery Policy Condition No. 2♪ and of Sec. 77 of the against the insurer should not be permitted to have
of the policy estops the respondent company from Insurance Code. Efforts to settle the case before the the effect of making a plain agreement ambiguous
asserting that the policy is ineffective. Insurance Commission proved futile. On 3 March and then construe it in favor of the insured. Verily, it
The court however sees no proof of any such implied 1988 Violeta and the other petitioners sued is elemental law that the payment of premium is
agreement. The purported nexus between the FORTUNE for damages in the amount of P600,000 requisite to keep the policy of insurance in force. If
delivery of the policy and the grant of credit representing the total coverage of the fire insurance the premium is not paid in the manner prescribed in
extension is too tenuous to support the conclusion policy plus 12% interest per annum, P100,000 the policy as intended by the parties the policy is
for which petitioners contend. moral damages, and attorney's fees equivalent to ineffective. Partial payment even when accepted as a
 Parenthetically mention: in the present law, 20% of the total claim. The trial court ruled for partial payment will not keep the policy alive even
Section 77 of the Insurance Code of 1978 has petitioners. CA reversed. for such fractional part of the year as the part
deleted the clause "unless there is clear agreement payment bears to the whole payment.
to grant the insured credit extension of the premium ISSUE Disposition Petition is DENIED. Decision of the CA
due" which was then involved in this controversy. WON a fire insurance policy is valid, binding and is AFFIRMED.
Disposition Fnding no reversible error, the enforceable upon mere partial payment of premium
judgment appealed from is hereby AFFIRMED.

SEPARATE OPINION

This policy including any renewal thereof and/or any endorsement
thereon is not in force until the premium has been fully paid to and VITUG [dissent]
duly receipted by the Company in the manner provided herein.
INSURANCE Page
33

- The law neither requires, nor measures the contain a credit clause in its favor and the receipts paid, notwithstanding any agreement to the
strength of the vinculum juris by, any specific for the installment payments covering the policy for contrary.
amount of premium payment. It should thus be 1984-85, as well as the two (2) previous policies,
enough that payment on the premium, partly or in stated the following reservations: ISSUE
full, is made by the insured which the insurer 2. Acceptance of this payment shall not waive WON payment by installment of the premiums due
accepts. In fine, it is either that a juridical tie exists any of the company rights to deny liability on any on an insurance policy invalidates the contract of
(by such payment) or that it is not extant at all (by claim under the policy arising before such insurance
an absence thereof). Once the juridical relation payments or after the expiration of the credit
comes into being, the full efficacy, not merely pro clause of the policy; and HELD
tanto, of the insurance contract naturally follows. 3. Subject to no loss prior to premium payment. Ratio Where the risk is entire and the contract is
Verily, not only is there an insurance perfected but If there be any loss such is not covered. indivisible, the insured is not entitled to a refund of
also a partially performed contract. In case of loss, - Petitioner further claimed that the policy was never the premiums paid if the insurer was exposed to the
recovery on the basis of the full contract value, less binding and valid, and no risk attached to the policy. risk insured for any period, however brief or
the unpaid premium can accordingly be had; It then pleaded a counterclaim for P152k for the momentary.
conversely, if no loss occurs, the insurer can demand premiums already paid for 1984-85, and in its Reasoning
the payment of the unpaid balance of the premium. answer with amended counterclaim, sought the - The obligation to pay premiums when due is
The insured, on the one hand, cannot avoid the refund of P924,206.10 representing the premium ordinarily as indivisible obligation to pay the entire
obligation of paying the balance of the premium payments for 1982-85. premium. Here, the parties herein agreed to make
while the insurer, upon the other hand, cannot treat - Trial court dismissed the complaint and the the premiums payable in installments, and there is
the contract as valid only for the purpose of counterclaim upon the following findings: (1) no pretense that the parties never envisioned to
collecting premiums and as invalid for the purpose of payment of the premiums of the three policies were make the insurance contract binding between them.
indemnity. made during the term of said policies, hence, it could And the insured never informed the insurer that it
not be said, inspite of the reservations, that no risk was terminating the policy because the terms were
MAKATI TUSCANY v. CA ( AMERICAN HOME attached under the policies; (2) as regards the unacceptable.
ASSURANCE CO.) unpaid premiums, in view of the reservation in the - There is nothing in Section 77 which suggests that
215 SCRA 462 receipts ordinarily issued by AHAC on premium the parties may not agree to allow payment of the
BELLOSILLO; November 6, 1992 payments the only plausible conclusion is that AHAC premiums in installment, or to consider the contract
has no right to demand their payment after the lapse as valid and binding upon payment of the first
NATURE of the term of said policy on March 1, 1985. premium.
Appeal from decision of the CA Therefore, Tuscany was justified in refusing to pay - The records clearly show that petitioner and private
the same. respondent intended subject insurance policies to be
FACTS - CA modified the decision by ordering Tuscany to binding and effective notwithstanding the staggered
- American Home Assurance Co. (AHAC), pay the balance of the premiums due on the third payment of the premiums. Acceptance of payments
represented by American International Underwriters policy plus legal interest until fully paid, and speaks loudly of the insurer's intention to honor the
(Phils.), Inc., issued in favor of petitioner Makati affirming the denial of the counterclaim. policies it issued to petitioner.
Tuscany Condominium Corporation an insurance Petitioner’s Claims - Section 78 of the Insurance Code in effect allows
policy on the latter's building and premises, for the Petitioner argues that where the premiums is not waiver by the insurer of the condition of prepayment
period 1 March 1982 to1 March 1983. The premium actually paid in full, the policy would only be by making an acknowledgment in the insurance
was paid on installments all of which were accepted effective if there is an acknowledgment in the policy policy of receipt of premium as conclusive evidence
by AHAC. of the receipt of premium pursuant to Sec. 78 of the of payment so far as to make the policy binding
- A second policy was issued to renew the first one, Insurance Code. The absence of an express despite the fact that premium is actually unpaid.
this time covering the period 1 March 1983 to 1 acknowledgment in the policies of such receipt of the Section 77 merely precludes the parties from
March 1984. This was also pain in installment basis. corresponding premium payments, and petitioner's stipulating that the policy is valid even if premiums
- A third policy was again issued for the period 1 failure to pay said premiums on or before the are not paid, but does not expressly prohibit an
March 1984 to 1 March 1985. For this, petitioner effective dates of said policies rendered them invalid. agreement granting credit extension, and such an
made two installment payments, both accepted by Petitioner thus concludes that there cannot be a agreement is not contrary to morals, good customs,
AHAC. Thereafter, petitioner refused to pay the perfected contract of insurance upon mere partial public order or public policy.
balance of the premium. AHAC filed an action to payment of the premiums because under Sec. 77 of - At the very least, both parties should be deemed in
recover the unpaid balance of P314,103.05. the Insurance Code, no contract of insurance is valid estoppel to question the arrangement they have
- Petitioner explained that it discontinued the and binding unless the premium thereof has been voluntarily accepted.
payment of premiums because the policy did not
INSURANCE Page
34

Disposition Judgment affirmed. Costs against company records revealed that Areola failed to pay complaint, Areola is left without a cause of action on
petitioner. his premiums. which to predicate his claim for damages.
o Under the terms of the statement of account - Reinstatement effectively restored Areola to all his
SOUTH SEA SURETY AND INSURANCE v. CA issued by Prudential, Areola was supposed to rights under the policy.
(VALENZUELA HARDWOOD) pay the total amount of P1,609.65 which
244 SCRA 744 included the premium of P1,470.00, ISSUES
VITUG; June 2, 1995 documentary stamp of P110.25 and 2% 1. WON the erroneous act of canceling subject
premium tax of P29.40. insurance policy entitle petitioner-insured to payment
NATURE o The statement of account stated that it must not of damages
Petition for review on certiorari be considered a receipt as an official receipt will 2. WON the subsequent act of reinstating the
be issued upon payment of the account. And if wrongfully cancelled insurance policy obliterate
FACTS payment was made to a representative, the whatever liability for damages Prudential has
- Hardwood entered into agreement with Seven Bros client must demand for a Provisional Receipt and
Shipping, where latter undertook to load the former’s if Official Receipts aren’t received within 7 days, HELD
logs on vessel. Hardwood insured the logs with Prudential should be notified. If payment is 1. YES
South Sea Surety which issued Marine Cargo made to their office, clients should demand for 2. NO
Insurance Policy. The vessel sank Jan 25, 1984. an OR. Reasoning
- Hardwood filed claim with South Sea and Seven - August 3, 1985- Prudential offered to reinstate - Malapit's fraudulent act of misappropriating the
Bros. Trial Court favored Hardwood. CA decided same policy it had previously cancelled and even premiums paid by petitioner-insured is beyond doubt
against South Sea, but absolved Seven Bros. South proposed to extend its lifetime to December 17, directly imputable to Prudential.
Sea filed this instant petition. 1985, upon a finding that the cancellation was - A corporation, such as respondent insurance
erroneous and that the premiums were paid in full by company, acts solely thru its employees. The latter’s
ISSUES Areola but were not remitted by Teofilo M. Malapit, acts are considered as its own for which it can be
WON the insurance contract was already in effect Prudential's branch manager. held to account.
when the vessel sank Petitioners’ Claims - The facts are clear as to the relationship between
- The fraudulent act of in misappropriating Areola’s private respondent insurance company and Malapit.
HELD premium payments is the proximate cause of the His act of receiving the premiums collected is well
YES cancellation of the insurance policy. within the province of his authority as manager.
- It is already in effect because Hardwood has - Areola theorized that Malapit's act of signing and Thus, his receipt of said premiums is receipt by
already paid the insurance premium. even sending the notice of cancellation himself, private respondent insurance company who, by
It delivered the check to Victorio Chua before the notwithstanding his personal knowledge of provision of law, particularly under Article 1910 of
vessel sank, but Victorio Chua was only to deliver the petitioner-insured's full payment of premiums, the Civil Code, is bound by the acts of its agent.
check to South Sea five days after the vessel sank. further reinforces the allegation of bad faith. - Article 1910 thus reads:
Appellant argues that Chua was not its broker, but it - Such fraudulent act committed by Malapit is Art. 1910. The principal must comply with all the
was found that Chua was authorized by South Sea to attributable to Prudential. obligations which the agent may have contracted
receive the premium on its behalf. - Malapit's actuations are therefore not separate and within the scope of his authority.
distinct from that of Prudential’s. It must, therefore, As for any obligation wherein the agent has
bear the consequences of the erroneous cancellation exceeded his power, the principal is not bound
AREOLA v. CA (PRUDENTIAL GUARANTEE AND of subject insurance policy caused by the non- except when he ratifies it expressly or tacitly.
ASSURANCE, INC.) remittance by its own employee of the premiums - Malapit's failure to remit the premiums he received
236 SCRA 643 paid. cannot constitute a defense for private respondent
ROMERO; September 22, 1994 - Subsequent reinstatement could not possibly insurance company; no exoneration from liability
absolve respondent insurance company from liability, could result therefrom.
NATURE CERTIORARI there being an obvious breach of contract. After all - Prudential’s earlier act of reinstating the insurance
damage had already been inflicted on him and no policy can not obliterate the injury inflicted on
FACTS amount of rectification could remedy the same. petitioner-insured.
- June 29, 1985- 7 months after the issuance of Respondent’s Argument - Respondent company should be reminded that a
Santos Areola's Personal Accident Insurance Policy - Prudential argues that where reinstatement, the contract of insurance creates reciprocal obligations
No. PA-20015 (covering a period of one year), equitable relief sought by Areola was granted at an for both insurer and insured.
Prudential unilaterally cancelled the same since opportune moment, i.e. prior to the filing of the - Reciprocal obligations are those which arise from
the same cause and in which each party is both a
INSURANCE Page
35

debtor and a creditor of the other, such that the After due trial, on March 10, 1993, the Regional Trial
obligation of one is dependent upon the obligation of NATURE Court, Branch 58, Makati, rendered decision, the
the other. Petition for review on certiorari of a decision of the dispositive portion of which reads:
- Under the circumstances of instant case, the Court of Appeals. "WHEREFORE, premises considered, judgment is
relationship as creditor and debtor between the hereby rendered in favor of the plaintiff and against
parties arose from a common cause: i.e., by reason FACTS the defendant, as follows.
of their agreement to enter into a contract of - On April 15, 1991, petitioner issued five (5) "(1) Authorizing and allowing the plaintiff to
insurance under whose terms, Prudential promised to insurance policies covering respondent's various consign/deposit with this Court the sum of
extend protection to Areola against the risk insured property described therein against fire, for the period P225,753.95 (refused by the defendant) as full
for a consideration in the form of premiums to be from May 22, 1991 to May 22, 1992. payment of the corresponding premiums for the
paid by the latter. - In March 1992, petitioner evaluated the policies replacement-renewal policies for Exhibits A, B, C, D
- Under the law governing reciprocal obligations, and decided not to renew them upon expiration of and E; "(2) Declaring plaintiff to have fully complied
particularly the second paragraph of Article 1191, the their terms on May 22, 1992. Petitioner advised with its obligation to pay the premium thereby
injured party, Areola in this case, is given a choice respondent's broker, Zuellig Insurance Brokers, Inc. rendering the replacement-renewal policy of Exhibits
between fulfillment or rescission of the obligation in of its intention not to renew the policies. A, B, C, D and E effective and binding for the
case one of the obligors, such as respondent - On April 6, 1992, petitioner gave written notice to duration May 22, 1992 until May 22, 1993; and,
insurance company, fails to comply with what is respondent of the non-renewal of the policies at the ordering defendant to deliver forthwith to plaintiff the
incumbent upon him. address stated in the policies. said replacement-renewal policies; "(3) Declaring
- However, said article entitles the injured party to - On June 13, 1992, fire razed respondent's property Exhibits A & B, in force from August 22, 1991 up to
payment of damages, regardless of whether he covered by three of the insurance policies petitioner August 23, 1992 and August 9, 1991 to August 9,
demands fulfillment or rescission of the obligation. issued. 1992, respectively; and "(4) Ordering the defendant
- Untenable then is reinstatement insurance - On July 13, 1992, respondent presented to to pay plaintiff the sums of. (a) P18,645,000.00
company's argument, namely, that reinstatement petitioner's cashier at its head office five (5) representing the latter's claim for indemnity under
being equivalent to fulfillment of its obligation, manager's checks in the total amount of Exhibits A, B & C and/or its replacement-renewal
divests petitioner-insured of a rightful claim for P225,753.95, representing premium for the renewal policies; (b) 25% of the total amount due as and for
payment of damages. Such a claim finds no support of the policies from May 22, 1992 to May 22, 1993. attorney's fees; (c) P25,000.00 as necessary
in our laws on obligations and contracts. No notice of loss was filed by respondent under the litigation expenses; and, (d) the costs of suit.
DAMAGES: policies prior to July 14, 1992. “xxx ”
- The nature of damages to be awarded, however, - On July 14, 1992, respondent filed with petitioner - In due time, petitioner appealed to the Court of
would be in the form of nominal damages its formal claim for indemnification of the insured Appeals (CA). The CA promulgated its decision
- Although the erroneous cancellation of the property razed by fire. On the same day, petitioner affirming that of the Regional Trial Court with the
insurance policy constituted a breach of contract, returned to respondent the five manager's checks modification that item No. 3 of the dispositive portion
Prudential within a reasonable time took steps to that it tendered, and at the same time rejected was deleted, and the award of attorney's fees was
rectify the wrong committed by reinstating the respondent's claim for the reasons (a) that the reduced to 10% of the total amount due.
insurance policy of petitioner. policies had expired and were not renewed, and (b) It held that following previous practice, respondent
- Moreover, no actual or substantial damage or that the fire occurred on June 13, 1992, before was allowed a 60- to 90-day credit term for the
injury was inflicted on petitioner Areola at the time respondent's tender of premium payment. renewal of its policies, and that the acceptance of the
the insurance policy was cancelled. - On July, 21, 1992, respondent filed with the late premium payment suggested an understanding
- Nominal damages are "recoverable where a legal Regional Trial Court, Branch 58, Makati City, a civil that payment could be made later. Hence, this
right is technically violated and must be vindicated complaint against petitioner for recovery, of appeal.
against an invasion that has produced no actual P18.645,000.00, representing the face value of the
present loss of any kind, or where there has been a policies covering respondent's insured property razed ISSUE
breach of contract and no substantial injury or actual by fire, and for attorney's fees. WON the fire insurance policies issued by petitioner
damages whatsoever have been or can be shown. - On October 23, 1992, after its motion to dismiss to the respondent covering the period May 22, 1991
Disposition Petition for review on certiorari is had been denied, petitioner filed an answer to the to May 22, 1992, had expired on the latter date or
hereby GRANTED. RTC’ s DECISION is REINSTATED. complaint. It alleged that the complaint "fails to state had been extended or renewed by an implied credit
a cause of action"; that petitioner was not liable to arrangement though actual payment of premium was
UCPB GENERAL INSURANCE CO., INC. v. -respondent for insurance proceeds under the tendered on a later date after the occurrence of the
MASAGANA TELAMART, INC. policies because at the time of the loss of risk (fire) insured against
308 SCRA 259 respondent's property due to fire, the policies had
PARDO; June 15, 1999 long expired and were not renewed.
INSURANCE Page
36

non-renewal was made within 45 days before 22 May the Court of Appeals, are indeed duly established:
HELD 1992, or before the expiration date of the fire 1. For years, Petitioner had been issuing fire
NO insurance policies. Thus, the policies in question policies to the Respondent, and these policies
- An insurance policy, other than life, issued were renewed by operation of law and were effective were annually renewed.
originally or on renewal, is not valid and binding until and valid on 30 June 1992 when the fire occurred, 2. Petitioner had been granting Respondent a
actual payment of the premium. Any agreement to since the premiums were paid within the 60- to 90- 60- to 90-day credit term within which to pay
the contrary is void. The parties may not agree day credit term. the premiums on the renewed policies.
expressly or impliedly on the extension of credit or - Respondent likewise disagrees with its ruling that 3. There was no valid notice of non-renewal of
time to pay the premium and consider the policy parties may neither agree expressly or impliedly on the policies in question, as there is no proof at
binding before actual payment. the extension of credit or time to pay the premium all that the notice sent by ordinary mail was
Disposition Judgment reversed and set aside nor consider a policy binding before actual payment. received by Respondent, and the copy thereof
It urges the Court to take judicial notice of the fact allegedly sent to Zuellig was ever transmitted
that despite the express provision of Section 77 of to Respondent.
the Insurance Code, extension of credit terms in 4. The premiums for the policies in question in
premium payment has been the prevalent practice in the aggregate amount of P225,753.95 were
the insurance industry. Most insurance companies, paid by Respondent within the 60- to 90-day
including Petitioner, extend credit terms because credit term and were duly accepted and
Section 77 of the Insurance Code is not a prohibitive received by Petitioner’s cashier.
injunction but is merely designed for the protection
of the parties to an insurance contract. The Code ISSUE
itself, in Section 78, authorizes the validity of a WON Sec. 77 of the Insurance Code of 1978 must be
UCPB GENERAL INSURANCE CO., INC. v. policy notwithstanding non-payment of premiums. strictly applied to Petitioner’s advantage despite its
MASAGANA TELAMART, INC. (EN BANC) - Respondent also asserts that the principle of practice of granting a 60- to 90-day credit term for
356 SCRA 307 estoppel applies to Petitioner. Despite its awareness the payment of premiums
DAVIDE; April 4, 2001 of Section 77 Petitioner persuaded and induced HELD
Respondent to believe that payment of premium on NO
NATURE the 60- to 90-day credit term was perfectly alright; - Section 77 of the Insurance Code of 1978 provides:
Motion for reconsideration of the decision of the in fact it accepted payments within 60 to 90 days SEC. 77. An insurer is entitled to payment of the
Supreme Court. after the due dates. By extending credit and premium as soon as the thing insured is exposed
habitually accepting payments 60 to 90 days from to the peril insured against. Notwithstanding any
FACTS the effective dates of the policies, it has implicitly agreement to the contrary, no policy or contract of
- In its decision of 15 June 1999, the SC defined the agreed to modify the tenor of the insurance policy insurance issued by an insurance company is valid
main issue to be “whether the fire insurance policies and in effect waived the provision therein that it and binding unless and until the premium thereof
issued by petitioner to the respondent covering the would pay only for the loss or damage in case the has been paid, except in the case of a life or an
period from May 22, 1991 to May 22, 1992 had been same occurred after payment of the premium. industrial life policy whenever the grace period
extended or renewed by an implied credit - Petitioner filed an opposition to the Respondent’s provision applies.
arrangement though actual payment of premium was motion for reconsideration. It argues that both the - This Section is a reproduction of Section 77 of P.D.
tendered on a later date and after the occurrence of trial court and the Court of Appeals overlooked the No. 612 (The Insurance Code) promulgated on 18
the (fire) risk insured against.” The Court resolved fact that on 6 April 1992 Petitioner sent by ordinary December 1974. In turn, this Section has its source
this issue in the negative in view of Section 77 of the mail to Respondent a notice of non-renewal and sent in Section 72 of Act No. 2427 otherwise known as
Insurance Code and its decisions in Valenzuela v. by personal delivery a copy thereof to Respondent’s the Insurance Act as amended by R.A. No. 3540,
Court of Appeals; South Sea Surety and Insurance broker, Zuellig. Both courts likewise ignored the fact approved on 21 June 1963, which read:
Co., Inc. v. Court of Appeals; and Tibay v. Court of that Respondent was fully aware of the notice of SEC. 72. An insurer is entitled to payment of
Appeals. Accordingly, it reversed and set aside the non-renewal. A reading of Section 66 of the premium as soon as the thing insured is exposed
decision of the Court of Appeals. Insurance Code readily shows that in order for an to the peril insured against, unless there is clear
- Respondent seasonably filed a motion for the insured to be entitled to a renewal of a non-life agreement to grant the insured credit extension of
reconsideration of the adverse verdict. It alleges in policy, payment of the premium due on the effective the premium due. No policy issued by an
the motion that the SC had made in the decision its date of renewal should first be made. Respondent’s insurance company is valid and binding unless and
own findings of facts, which are not in accord with argument that Section 77 is not a prohibitive until the premium thereof has been paid.
those of the trial court and the Court of Appeals. provision finds no authoritative support. (Underscoring supplied)
The courts below correctly found that no notice of - The following facts, as found by the trial court and - It can be seen at once that Section 77 does not
INSURANCE Page
37

restate the portion of Section 72 expressly that of the Court of Appeals affirmed in toto. 14, 1962, ACME continued to insure its properties
permitting an agreement to extend the period to pay with INSURER in the amount of P200,000 for the
the premium. But there are exceptions to Section SEPARATE OPINION period May 15, 1962 up to May 15, 1963.
77. - On May 14, 1963, INSURER issued Renewal Receipt
The first exception is provided by Section 77 itself, VITUG to cover the period May 15, 1963 to May 15, 1964.
and that is, in case of a life or industrial life policy - An essential characteristic of an insurance is its - On January 8, 1964, ACME paid P3,331.26 as
whenever the grace period provision applies. being synallagmatic, a highly reciprocal contract premium. The INSURER applied the payment as
The second is that covered by Section 78 of the where the rights and obligations of the parties renewal premium for the period of May 15, 1963 to
Insurance Code, which provides: correlate and mutually correspond. May 15, 1964.
SEC. 78. Any acknowledgment in a policy or - By weight of authority, estoppel cannot create a - On May 15, 1964, INSURER issued a Renewal
contract of insurance of the receipt of premium is contract of insurance, neither can it be successfully Receipt for the period of May 15, 1964 to May 15,
conclusive evidence of its payment, so far as to invoked to create a primary liability, nor can it give 1965 (for renewal premium of P3,331.26 yet to be
make the policy binding, notwithstanding any validity to what the law so procribes as a matter of paid) with a stamped note that says that the
stipulation therein that it shall not be binding until public policy. insurance will be deemed valid and binding only
premium is actually paid. when the premium and documentary stamps have
- A third exception was laid down in Makati Tuscany PARDO [dissent] actually been paid in full and duly acknowledged in
Condominium Corporation vs. Court of Appeals, - An assured’s failure to give notice of the fire an official receipt. ACME was given 90 days to pay
wherein we ruled that Section 77 may not apply if immediately upon its occurrence blatantly showed otherwise the policy would automatically become
the parties have agreed to the payment in the fraudulent character of its claims. Respondent is void and ineffective. (ACME should pay short period
installments of the premium and partial payment has required by law and by express terms of the policy to premium for 90 days before the period expires. If
been made at the time of loss. Tuscany has give immediate written notice of loss. This must be they are able to pay the whole amount before the
provided a fourth exception to Section 77, namely, complied with in the utmost good faith. 90-day period, the automatic termination won’t apply
that the insurer may grant credit extension for the - Assuming arguendo that the 60- to 90-day credit anymore).
payment of the premium. This simply means that if has been agreed between the parties, respondent - On May 26, 1964, ACME, through its President,
the insurer has granted the insured a credit term for could not still invoke estoppel to back up its claim. signed a promissory note saying that they promise to
the payment of the premium and loss occurs before Estoppel cannot give validity to an act that is pay the premium and documentary stamps and
the expiration of the term, recovery on the policy prohibited by law or against public policy. The actual agreed to the automatic cancellation penalty for not
should be allowed even though the premium is paid payment of premiums is a condition precedent to the complying.
after the loss but within the credit term. validity of an insurance contract other than life - On October 13, 1964, ACME’s properties were
Moreover, there is nothing in Section 77 which insurance policy. Any agreement to the contrary is completely destroyed by fire. ACME filed insurance
prohibits the parties in an insurance contract to void as against law and public policy. claim but the INSURER disclaimed liability on the
provide a credit term within which to pay the ground that as of the date of loss, the properties
premiums. That agreement is not against the law, ACME SHOE RUBBER & PLASTIC CORP. v. CA burned were not covered by insurance.
morals, good customs, public order or public policy. (DOMESTIC INSURANCE COMPANY OF THE - ACME claims that the January 8, 1964 payment
The agreement binds the parties. Article 1306 of the PHILS.) was for the period 1964-1965 and that INSURER had
Civil Code provides: 134 SCRA 155 no right to apply it to the period 1963-1964 because
ART. 1306. The contracting parties may establish MELENCIO-HERRERA; January 17, 1985. under RA 3540, the policy was void and INSURER
such stipulations clauses, terms and conditions as could have validly disclaimed liability for loss had one
they may deem convenient, provided they are not NATURE occurred then.
contrary to law, morals, good customs, public Petition for Review on Certiorari of the Decision of - TC found INSURER liable for P200k and opined that
order, or public policy. the then Court of Appeals (CA-G. R. No. 58917-R), there was a clear intention on the INSURER's part to
- Finally, it would be unjust and inequitable if denying recovery on an insurance policy, thereby grant ACME a credit extension for the payment of the
recovery on the policy would not be permitted reversing the judgment of the Court of First Instance premium due; and that to allow the INSURER to
against Petitioner, which had consistently granted a of Rizal, Branch XII, at Caloocan City, which had apply the premium ACME paid on January 8, 1964.
60- to 90-day credit term for the payment of allowed such recovery. CA reversed TC and dismissed the suit on the ground
premiums despite its full awareness of Section 77. that, as of the moment of loss, ACME's properties
Estoppel bars it from taking refuge under said FACTS were not insured and the INSURER could not be held
Section since Respondent relied in good faith on such - ACME Shoe Rubber and Plastic Corporation (ACME) liable for any indemnity as a result of the loss.
practice. Estoppel then is the fifth exception to had been insuring yearly against fire its building,
Section 77. machines and general merchandise with Domestic ISSUE
Disposition Judgment reconsidered and set aside, Insurance Company (INSURER) since 1946. On May WON the premium payment for 1964-1965 was paid
INSURANCE Page
38

ABAD SANTOS; September 30, 1982. is of the essence in respect of the payment of the
HELD insurance premium so that if it is not paid the
NO NATURE contract does not take effect unless there is still
- Not having paid the 1964-1965 premium within the Appeal from CFI decision on question of law. another stipulation to the contrary. In the instant
extension granted, and pursuant to R.A. No. 3540, case, the INSURED was given a grace period to pay
the policy was automatically cancelled and there was FACTS the premium but the period having expired with no
no insurance coverage to speak of as of the date of - Arce (INSURED) owned a residential house which payment made, he cannot insist that the COMPANY is
the fire on October 13, 1964. was insured with the appellant COMPANY since 1961. nonetheless obligated to him.
- The pertinent provision of Republic Act No. 3540 In November 1965, the COMPANY sent to the - Prior to the amendment (italicized portion above),
reads: INSURED a Renewal Certificate to cover the period an insurance contract was effective even if the
"Sec. 72. An insurer is entitled to payment of the from December 5, 1965 to December 5,1966, and premium had not been paid so that an insurer was
premium as soon as the thing insured is exposed requested payment of the corresponding premium. obligated to pay indemnity in case of loss and
to the peril insured against, unless there is clear Anticipating that the premium could not be paid on correlatively he had also the right to sue for payment
agreement to grant the insured credit extension of time, the INSURED asked for an extension which was of the premium. But the amendment to Sec. 72 has
the premium due. No policy issued by an granted by the COMPANY. After the lapse of the radically changed the legal regime in that unless the
insurance company is valid and binding unless and requested extension, INSURED still failed to pay the premium is paid there is no insurance.
until the premium thereof has been paid." premium. Thereafter, the house of the INSURED was Disposition The decision of the court a quo is
- RA 3540 was approved on June 20, 2963 and was totally destroyed by fire. Upon INSURED's reversed; the appellee's complaint is dismissed. No
put into effect on Oct 1, 1963. It could not be applied presentation of claim for indemnity, he was told that special pronouncement as to costs.
retroactively to the renewal of the policy for the no indemnity was due because the premium was not - Irrelevant facts: The premium costs P38.10. After
1963-1964 period because said policy was renewed paid. The INSURED sued the COMPANY for the fire, the COMPANY issued a check for P300 to
on May 14, 1963. (Laws have no retroactive effect indemnity. Arce as donation. Arce accepted the check, but still
unless the contrary is provided.) Therefore, the Jan - The trial court held the COMPANY liable to sued the company.
8, 1964 payment was properly applied to the 1963- indemnify the INSURED on the ground that since the CAPITAL INC. v. PLASTIC ERA CO.
1964 premium. The Trial Court's opinion that there COMPANY could have demanded payment of the 65 SCRA 134
was a clear agreement to grant ACME credit premium, mutuality of obligation required that it MARTIN; July 18, 1975
extension for 1964-1965 is negated by ACME's should be liable on the policy.
Promissory Note binding itself to pay within ninety NATURE
days from the effective date of this policy, 15th May, ISSUE Petition for review of a decision of the CA affirming
1964. The credit extension was granted for 90 days WON the COMPANY can be held liable on its policy the decision of the CFI of Manila
only. (So wala na by August 16, 1964.)
- If ACME was granted credit extensions in the past, HELD FACTS
the promissory note it signed did away with such NO. - On December 17, 1960, petitioner Capital
credit arrangement. Also, before RA 3540, the - The Court commiserates with the INSURED. They Insurance & Surety Co., Inc. delivered to the
Renewal Receipts issued by INSURER did not contain are well aware that many insurance companies have respondent Plastic Era Manufacturing Co., Inc., its
the auto-cancellation after 90 days note. By 1964, fallen into the condemnable practice of collecting open Fire Policy No. 22760 wherein the former
however, the situation had changed by the passage premiums promptly but resort to all kinds of excuses undertook to insure the latter's building, equipments,
of the RA: no policy could be valid and binding to deny or delay payment of just claims. Unhappily raw materials, products and accessories located at
unless and until the premium thereof had been the instant case is one where the insurer has the law Sheridan Street, Mandaluyong, Rizal. The policy
paid. on its side. expressly provides that if the property insured would
- What became automatically cancelled by R.A. No. - Sec. 72 of the Insurance Act, as amended by R.A. be destroyed or damaged by fire after the payment
3540 was the 1964-1965 policy for ACME's failure to No. 3540 reads: of the premiums, at anytime between the 15th day
pay the premium within the 90-day extension "SEC. 72. An insurer is entitled to payment of of December 1960 and one o'clock in the afternoon
granted, and in accordance with the express terms of premium as soon as the thing insured is exposed of the 15th day of December 1961, the insurance
the Promissory Note that it had signed. to the perils insured against, unless there is clear company shall make good all such loss or damage in
Disposition The judgment under review is hereby agreement to grant credit extension for the an amount not exceeding P100,000.00. When the
affirmed. Without pronouncement as to costs. premium due. No policy issued by an insurance policy was delivered, Plastic Era failed to pay the
company is valid and binding unless and until the corresponding insurance premium. On January 8,
PEDRO ARCE v. THE CAPITAL INSURANCE & premium thereof has been paid." 1961, in partial payment of the insurance premium,
SURETY CO., INC. - It is obvious from both the Insurance Act, as Plastic Era delivered to Capital Insurance, a check for
11 SCRA 63 amended, and the stipulation of the parties that time the amount of P1,000.00 postdated January 16,
INSURANCE Page
39

1961. However, Capital Insurance tried to deposit Disposition The decision of the CA is AFFIRMED in invokes Rule 45 of the Rules of Court for certiorari
the check only on February 20, 1961 and the same toto. but the petition still exceeds the 15 day limit from
was dishonored by the bank for lack of funds. the June 13 notice.
- Two days after the insurance premium became MALAYAN INSURANCE CO., INC. v. ARNALDO -Respondents, on the other hand, invoke Sec. 39 of
due, at about 4:00 to 5:00 o'clock in the morning, and PINCA B.P. 129 which pegs the period for appeal from
the property insured by Plastic Era was destroyed by 154 SCRA 672 decisions of any court in all cases at 15 days from
fire. In less than a month Plastic Era demanded from CRUZ; October 12, 1987 the notice of the decision appealed from. Since the
Capital Insurance the payment of the sum of MFR was filed only 15 days after receiving notice of
P100,000.00 as indemnity for the loss of the insured FACTS the decision, it was already 18 days late by July 2.
property under Policy No. 22760 but the latter - On June 7, 1981, Malayan Insurance Co. (MICO), So whichever is applied, the petition is still late.
refused for the reason that, among others, Plastic issued fire insurance for the amount of P14,000 on Substantive
Era failed to pay the insurance premium. the property of private respondent, Pinca, effective 2. YES
July 1981-1982. MICO later allegedly cancelled the - A valid cancellation requires the following
ISSUES policy for non-payment of the premium and sent a conditions based on Sections 64-65 of the Code:
1. WON a contract of insurance has been duly notice to Pinca. On Dec. 24 Adora, an agent of MICO, prior notice which must be based on the occurrence
perfected between petitioner and respondent received Pinca’s payment, which was remitted to of one or more of the grounds mentioned in Sec 64
2. WON the dishonored check constituted payment MICO. On Jan. 18, 1982, Pinca’s property was (in this case, non-payment of premium), after the
completely burned. On Feb. 5, MICO returned Pinca’s effective date of the policy; the notice must be
HELD payment to Adora on the ground that her policy had written and mailed to the address on the policy; it
1. YES been cancelled; the latter refused to accept it. Her must state the ground(s) for cancellation and the
- Tender of draft or check in order to effect payment demand for payment having been rejected by MICO, insurer must furnish details upon the request of the
that would extinguish the debtor's liability should be Pinca went to the Insurance Commission. Public insured.
actually cashed. If the delivery of the check of Plastic respondent Arnaldo, the Insurance Commissioner, - It is undisputed that payment of premium was
Era to Capital Insurance were to be viewed in the sustained Pinca, hence this petition from MICO. made. Petitioner relies heavily on Sec 77 of the
light of the foregoing, no payment of the premium Records show MICO received Arnaldo’s decision on Insurance Code to contest this, the said provision
had been effected. Significantly, Capital Insurance April 10; MICO filed a MFR on April 25 which was requiring payment of premium as soon as the thing
accepted the promise of Plastic Era to pay the denied on June 4; MICO received notice of this denial is exposed to the peril insured against and that the
insurance premium within 30 days from the effective on June 14; instant petition was filed on July 2. policy is invalid without it. However, this is not
date of policy. By so doing, it has implicitly agreed to applicable in the instant case as payment was
modify the tenor of the insurance policy and in ISSUES eventually made. It is to be noted that the premium
effect, waived the provision therein that it would only Procedural invoice was stamped “Payment Received”, indicating
pay for the loss or damage in case the same occurs 1. WON the petition should be dismissed for late an understanding between the parties that payment
after the payment of the premium. Considering that filing could be made later. This is furthered by the fact
the insurance policy is silent as to the mode of Substantive that Adora had earlier told her to call him anytime
payment, Capital Insurance is deemed to have 2. WON there was a valid insurance contract at the she was ready with her payment. The Court also
accepted the promissory note in payment of the time of the loss finds it strange that MICO only sought to return
premium. This rendered the policy immediately 3. WON Adora was authorized to receive such Pinca’s Jan. 15 payment only on Feb. 5, long after
operative on the date it was delivered. payment her house had burned down—this makes petitioner’s
2. YES 4. WON an adjuster is indispensable in the motives highly suspect.
- Although the check was due for payment on valuation of the loss - MICO claims to have sent a notice to Pinca, who
January 16, 1961 and Plastic Era had sufficient funds flatly denied receiving one. Pinca did not have to
to cover it as of January 19, 1961, Capital Insurance HELD prove this since the strict language of Sec 64
decided to hold the same for thirty-five (35) days Procedural requires that MICO ensure the cancellation was
before presenting it for payment. Having held the 1. YES actually sent to and received by the insured.
check for such an unreasonable period of time, - Petitioner invokes Sec 416 of the Insurance Code - MICO also suggests that Pinca knew the policy had
Capital Insurance was estopped from claiming a which grants it 30 days from notice of the Insurance been cancelled and was paying the premium in order
forfeiture of its policy for non-payment even if the Commission within which to appeal by certiorari with to renew the policy. A close study of the transcripts
check had been dishonored later. Where the check is the Court. MICO filed its MFR on April 25, 15 days show, however, that Pinca only meant to renew the
held for an unreasonable time before presenting it after the notice; the reglementary period began to policy had it been cancelled but not if it was still in
for payment, the insurer may be held estopped from run again after June 13. Since the petition was filed effect—it was conditional. Payment was thus legally
claiming a forfeiture if the check is dishonored. only on July 2, it was tardy by 4 days. Alternatively it made on the original transaction and validly received
INSURANCE Page
40

by Adora, who was not informed of the alleged - From January 1, 1942 to December 31, 1946, collection of premiums, were done in Toronto,
cancellation and thus saw no reason to reject the Plaintiff head office at Toronto applied the provisions Canada
payment. of the automatic premium loan clauses upon the 5. WON the fact that plaintiff-appellant was not doing
3. YES nonpayment of the corresponding premiums by the business in the Philippines during the period from
- Sec. 306 of the Insurance Code provides that any people who subscribed to the insurance. The net January 1, 1942 to September 30, 1945, inclusive,
insurance company that delivers a policy to its agent amount of premiums advanced (by the company) or exempts it from payment of premium taxes
is deemed to have authorized such agent to receive loaned (to the insured) as payment for the premium corresponding to said period
payment of premium on its behalf. It is a well-known due totaled P1,069,254.98.
principle under the law of agency that payment to an - Meer, the Collector of the National Internal
authorized agent is equivalent to payment to the Revenue assessed the net amount of premium at HELD
principal himself. MICO’s acknowledgement of Adora P17,917.12 pursuant to SEC.255, National Internal NOTE (example given by the plaintiff):
as its agent thus defeats its contention that he was Revenue Code6 "Suppose that 'A', 30 years of age, secures a 20-year
not authorized to receive payments on its behalf. - Company protested the assessment, but paid the endowment policy for P5,000 from plaintiff-appellant
4. NO taxes anyway. Then they filed a complaint to recover Company and pays an annual premium of P250. 'A'
- In absence of fraud, the amount of the loss may be money paid under protest for taxes pays the first ten yearly premiums amounting to
determined on the basis of such proof offered by the - CFI: Dissmiss complaint P2,500 and on this amount plaintiff-appellant pays
insured. Here. The certification of the Integrated - PLAINTIFF’s MAIN CONTENTION: when it made the corresponding taxes under section 255 of the
National Police as the extent of the loss should premium loans or premium advances by virtue of the National Internal Revenue Code. Suppose also that
suffice. non-forfeiture clauses, it did not collect premiums the cash value of said policy after the payment of the
Disposition petition is DENIED within the meaning of the above sections of the law, 10th annual premium amounts to P1,000." When on
and therefore it is not amenable to the tax therein the eleventh year the annual premium fell due and
MANUFACTURERS LIFE INSURANCE CO. v. MEER provided. the insured remitted no money within the mouth
89 PHIL 351 grace, the insurer treated the premium then over
BENGZON, June 29, 1951 ISSUES due as paid from the cash value, the amount being
1. WON premium advances made by plaintiff- a loan to the policyholder1 who could discharge it
NATURE appellant under the automatic premium loan clause at any time with interest at 6 per cent. The insurance
APPEAL from a judgment of the Court of First of its policies are premiums collected' by the contract, therefore, continued in force for the
Instance of Manila Company subject to tax eleventh year.
2. WON, in the application of the automatic premium 1. YES
FACTS loan clause of plaintiff-appellant's policies, there is - Based on the example given by the plaintiff, the
(this is a tax case. What’s really important here is 'payment in money, notes, credits, or any substitutes insurer collected the amount of P250 as the annual
the definition of CASH SURRENDER VALUE). for money premium for the eleventh year on the said policy
- Manufacturers Life Insurance Company is a duly 3. WON the collection of the alleged deficiency when it loaned to “A” the sum of P250. The insurer
organized corporation which has its head office at premium taxes constitutes double taxation “became a creditor” of the loan, but not of the
Toronto. It is duly registered and licensed to engage 4. WON the making of premium advances, granting premium that had already been paid (advanced by
in life insurance business in the Philippines, and, for the sake of argument that it amounted to the insurer). The insurer is entitled to collect interest
maintains a branch office in Manila. It was engaged on the loan, not on the premium. "A" paid the
in such business in the Philippines for more than five premium for the eleventh year; but in turn he
years before and including the year 1941. But due to 'When the premium falls due and is not paid in cash within the month's grace, if the Cash Value of this policy and of any became a debtor of the company for the sum of
the exigencies of the war It closed the branch office bonus additions and dividends left on accumulation (after deducting any accumulated indebtedness) be less than the
P250. This debt he could repay either by later
premium then due, the Company will, without further requests, continue this insurance in force for a period * * *.
at Manila during 1942 up to September 1945. '10. Cash and Paid-Up Insurance Values.-At the end of the third policy year or thereafter, upon the legal surrender of this
remitting the money to the insurer or by letting the
- Plaintiff issued a number of life insurance policies in Policy to the Company while there is no default in premium payments or within two months after the due date of the cash value compensate for it. The debt may also be
the Philippines containing stipulations referred to as premium in default, the Company will (1) grant a cash value as specified in Column (A) increased by the cash value of any

bonus additions and dividends left on accumulation, which have been alloted to this Policy, less all indebtedness to the
deducted from the amount of the policy should "A"
NONFORFEITURE CLAUSES5 Company on this Polley an the date of ouch surrender, or (2) endorse this Policy as a Non-Participating Paid-up Polley for the die thereafter during the continuance of the policy.
amount as specified In Column (B) of the Table of Guaranteed Values * * *.
- ON ARGUMENT THAT THE ASSETS OF THE INSURER REMAINED THE
'11. Extended Insurance-After the premiums for three or more full years have been paid hereunder in cash, if any
5"'8. Automatic Premium Loan.-This Policy shall not lapse for non-payment of any premium after it has been three full years SAME AFTER THE APPLICATION OF THE AUTOMATIC PREMIUM LOAN
subsequent premium is not paid when due, and there is no indebtedness to the Company on the written request of the
in force, it, at the due date of such premium, the Cash Value of this Policy and of any bonus additions and dividends left on insured * * *." CLAUSE: there was an increase in assets in the form of
accumulation (after deducting any indebtedness to the company and the interest accrued thereon) shall exceed the amount

of said premium. In which event the company will, without further request, treat the premium then due as paid, and the
6"SEC. 255. Taxes on insurance premiums.-There shall be collected from every person, company, or corporation (except CREDIT for the advances made (in the example, the
amount of such premium, with interest from its actual due date at six per cent per annum, compounded yearly, and one per purely cooperative companies or associations) doing insurance business of any sort in the Philippines a tax of one per P250 for the 11th year).
cent, compounded yearly, for expenses, shall be a first lien on this Policy in the Company's favour in priority to the claim of centum of the total premiums collected * * * whether such permiums are paid in money, notes, credits, or any substitute for
- ON ARGUMENT THAT IF THE CREDIT IS PAID OUT OF THE CASH
any assignee or any other person. The accumulated lien may at any time, while the Policy is in force, be paid in whole or in money but premiums refunded within six months after payment on account of rejection of risk or returned for other reason

part. to person insured shall not be included in the taxable receipts * * *."
SURRENDER VALUE, THERE WERE NO NEW FUNDS ADDED TO THE
INSURANCE Page
41

COMPANY'S ASSETS”: Cash surrender value "as applied Disposition finding no prejudicial error in the - May 15, 1951: Defendant sent a letter with official
to a life insurance policy, is the amount of money the appealed decision, we hereby affirm it with costs. receipt of the P165.15 paid by Rufino as well as a
company agrees to pay to the holder of the policy if Certificate of Reinstatement.
he surrenders it and releases his claims upon it. The - June 7, 1951: Rufino presented a death claim as
more premiums the insured has paid the greater will survivor-beneficiary of his deceased wife. Payment
be the surrender value; but the surrender value is was denied by the defendant.
always a lesser sum than the total amount of - April 1952: Rufino filed a complaint in CFI against
premiums paid." (Cyclopedia Law Dictionary 3d. ed. Crown Life for the recovery of the amount of P5,000
1077.) The cash value or cash surrender value is as the face value of a joint 20-year endowment
therefore an amount which the insurance company insurance policy issued by defendant in favor of
holds In trust for the insured to be delivered to him ANDRES v. CROWN LIFE INSURANCE plaintiff and his wife, on Feb. 13, 1950. In its
upon demand. It is therefore a liability of the 102 Phil. 919 answer, Crown Life disclaimed liability and set forth
company to the insured. Now then, when the REYES, J.B.L., Jan.28, 1958 the special defense that the aforementioned policy
company's credit for advances is paid out of the cash had already lapsed.
value or cash surrender value, that value and the NATURE - Aug. 5, 1954: CFI rendered a decision absolving
company's liability is thereby diminished pro tanto. Appeal from judgment of CFI the defendant company from any liability on the
2. YES ground that the policy had lapsed and it was not
- the insurer agreed to consider the premium paid on FACTS reinstated at the time of the plaintiff’s wife’s death.
the strength of the automatic loan. The premium was - Feb. 13, 1950: For the sum of P5,000, defendant- Plaintiff later appealed to the CA but the same was
therefore paid by means of a "note" or "credit" or appellee Crown Life issued an insurance policy in the certified by the CA to the SC for having no question
"other substitute for money" and the tax is due name of plaintiff-appellant Rufino and his wife, with of fact.
because section 255 above quoted levies taxes the stipulation that the premiums are to be paid
according to the total premiums collected by the semi-annually. ISSUE
insurer "whether such premiums are paid in money, - The premiums for the 1st and 2nd semester of the 1st WON the insurance policy, which has been in a state
notes, credits or any substitute for money. year, in the amount of P165.15 were paid by Rufino of lapse before May 3, 1951, has been validly and
3. NO but the premium for the third semester, in the same completely reinstated after said date (Was there a
- No constitutional prohibition against double amount, was not paid. perfected contract of reinstatement after the policy
taxation. - Jan. 6, 1951, Crown Life, through its branch lapsed due to non-payment of premiums?)
4. NO secretary, wrote to Mr. and Mrs. Andres advising
- The loans are made to policyholders in the them that their insurance policy lapsed on Dec. 26, HELD
Philippines, who in turn pay therewith the premium 1950 and the amount of P165.15 was overdue, NO
to the insurer thru the Manila branch. Approval of giving them 60 days from the date of lapse to file an Ratio The stipulation in a life insurance policy giving
appellant's position will enable foreign insurers to application for reinstatement. Crown Life later sent the insured the privilege to reinstate it upon written
evade the tax by contriving to require that premium another letter telling the spouses Andres that their application does not give the insured absolute right
payments shall be made at their head offices. What insurance policy was no longer in force. to such reinstatement by the mere filing of an
is important, the law does not contemplate - Feb. 1951: Plaintiff and his wife executed a application. The Company has the right to deny the
premiums collected in the Philippines. It is enough Statement of Health and application for reinstatement if it is not satisfied as to the
that the insurer is doing insurance business in the reinstatement of the aforesaid policy. insurability of the insured and if the latter does not
Philippines, irrespective of the place of its - Feb. 20, 1951: Plaintiff wrote a letter to the pay all overdue premiums and all other indebtedness
organization or establishment. defendant, enclosed with a money order for P100. to the Company. After the death of the insured the
5. NO Upon acceptance, defendant advised Rufino that its insurance Company cannot be compelled to entertain
- Although during those years the appellant was not main office had approved the application and that an application for reinstatement of the policy
open for new business because its branch office was the reinstatement of the lapsed policy was subject to because the conditions precedent to reinstatement
closed, still it was practically and legally, operating in the payment of the remaining premium balance of can no longer be determined and satisfied.
this country by collecting premiums on its P65.15. Reasoning
outstanding policies, incurring the risks and/or - May 3, 1951: Severa Andres died of dystocia, - The stipulations of facts render it undisputable that
enjoying the benefits consequent thereto, without contracted pelvis. the original policy lapsed for non-payment of
having previously taken any steps indicating - May 5, 1951: Plaintiff sent a letter enclosed with a premiums on Dec. 26, 1950, upon expiration of the
withdrawal in good faith from this field of economic money order in the amount of P65, for the remaining 31-day grace period.
activity. balance due. - As found by the lower court, the conditions set
forth in the policy for reinstatement as provided in
INSURANCE Page
42

the contract itself are the following: (A) application GUTIERREZ; October 19, 1990 Court has consistently held that termination may be
shall be made within 3 years from the date of lapse; effected even if the principal acts in bad faith,
(B) there should be a production of evidence of the NATURE subject only to the principal's liability for damages.
good health of the insured; (C) if the rate of Petition for review of the decision of theca. (CA ordered Valenzuela to pay Philamgen the
premium depends upon the age of the Beneficiary, amount of One Million Nine Hundred Thirty-Two
there should likewise be a production of evidence of FACTS Thousand Five Hundred Thirty-Two and 17/100
his or her good health; (D) there should be - Petitioner Arturo P. Valenzuela is a General Agent Pesos (P1,932,532.17) with legal interest)
presented such other evidence of insurability at the of private respondent Philippine American General
date of application for reinstatement; (E) there Insurance Company, Inc. (Philamgen for short) since ISSUES
should be no change which has taken place in such 1965. As such, he was authorized to solicit and sell 1. WON whether or not Philamgen and/or its officers
good health and insurability subsequent to the date in behalf of Philamgen all kinds of non-life insurance, can be held liable for damages due to the
of such application and before the policy is and in consideration of services rendered was termination of the General Agency Agreement it
reinstated; and (F) all overdue premiums and other entitled to receive the full agent's commission of entered into with the petitioners
indebtedness in respect of the policy, together with 32.5% from Philamgen under the scheduled 2. WON petitioners are liable to Philamgen for the
interest at 6%, compounded annually, should first be commission rates. unpaid and uncollected premiums
paid. - From 1973 to 1975, Valenzuela solicited marine
- The plaintiff did not comply with the last condition; insurance from one of his clients, the Delta Motors, HELD
for he only paid P100 before his wife’s death; and Inc. (Division of Electronics Airconditioning and 1. YES
despite the Company’s reminders, he only remitted Refrigeration) in the amount of P4.4 Million from - If a principal acts in bad faith and with abuse of
the balance of P65.15 two days after his wife died. which he was entitled to a commission of 32%. right in terminating the agency, then he is liable in
On the face of such facts, the Company had the right However, Valenzuela did not receive his full damages.
to treat the contract as lapsed and refuse payment of commission which amounted to P1.6 Million from the - There is an exception to the principle that an
the policy. P4.4 Million insurance coverage of the Delta Motors. agency is revocable at will and that is when the
- Rufino contends that the condition regarding During the period 1976 to 1978, premium payments agency has been given not only for the interest of
payment of the premium was waived by the amounting to P1,946,886.00 were paid directly to the principal but for the interest of third persons or
insurance Company through its letters, wherein it Philamgen and Valenzuela's commission to which he for the mutual interest of the principal and the agent.
made statements such as: “If you are unable to pay is entitled amounted to P632,737.00. In these cases, it is evident that the agency ceases
the full amount immediately, send as large amount - In 1977, Philamgen started to become interested in to be freely revocable by the sole will of the principal
as possible and advise us how soon you expect to be and expressed its intent to share in the commission (PROCEDURAL: Where the findings of the Court of
able to pay the balance; we will work out an due Valenzuela on a fifty-fifty basis. Valenzuela Appeals and the trial court are contrary to each
adjustment most beneficial to you.” The Court found refused. other, this Court may scrutinize the evidence on
the statements to be too vague and indefinite to - Because of the refusal of Valenzuela, Philamgen record
indicate an intention on the insurer’s part to waive and its officers took drastic action against - After a painstaking review of the entire records of
the full payment as prerequisite to the reinstatement Valenzuela. They: (a) reversed the commission due the case and the findings of facts of both the court a
of the lapsed policy. The Court reiterated the rule him by not crediting in his account the commission quo and respondent appellate court, the Court
that a waiver must be clear and positive, the intent earned from the Delta Motors, Inc. insurance ; (b) affirmed the trial court’s findings.)
to waive shown clearly and convincingly. On the placed agency transactions on a cash-and-carry - The principal cause of the termination of Valenzuela
other hand, It found subsequent letters sent by basis; (c) threatened the cancellation of policies as General Agent of Philamgen arose from his refusal
defendant indicating that they insisted on full issued by his agency; and (d) started to leak out to share his Delta commission. The records sustain
payment of the premium before the policy was news that Valenzuela has a substantial account with the conclusions of the trial court on the apparent bad
reinstated and that defendant did not consider partial Philamgen. All of these acts resulted in the decline of faith of the private respondents in terminating the
payment as sufficient consideration for the his business as insurance agent. General Agency Agreement of petitioners.
reinstatement. Plaintiff-Appellant’s failure to remit - Then on December 27, 1978, Philamgen terminated - It is also evident from the records that the agency
the balance before the death of his wife operated to the General Agency Agreement of Valenzuela. involving petitioner and private respondent is one
deprive him of any right to waive the policy and - Lower court: the termination of Valenzuela as "coupled with an interest," and, therefore, should not
recover the face value thereof. General Agent was improper because the record will be freely revocable at the unilateral will of the latter.
Disposition Judgment appealed from is affirmed. show the principal cause of the termination of the - The private respondents by the simple expedient of
plaintiff as General Agent of defendant Philamgen terminating the General Agency Agreement
VALENZUELA v. CA (PHILIPPINE AMERICAN was his refusal to share his Delta commission. appropriated the entire insurance business of
GENERAL INSURANCE COMPANY, INC.) - CA: In any event the principal's power to revoke an Valenzuela. With the termination of the General
191 SCRA 1 agency at will is so pervasive, that the Supreme Agency Agreement, Valenzuela would no longer be
INSURANCE Page
43

entitled to commission on the renewal of insurance shall be deemed terminated upon the satisfaction of
policies of clients sourced from his agency. Worse, the judgment as modified. ISSUE
despite the termination of the agency, Philamgen WON the contract of insurance between Luis Lim and
continued to hold Valenzuela jointly and severally CHAPTER V – THE POLICY, PARTIES THERETO, Sun Life Assurance Company of Canada was
liable with the insured for unpaid premiums. Under & RIGHTS THEREON perfected
these circumstances, it is clear that Valenzuela had
an interest in the continuation of the agency when it DE LIM v. SUN LIFE ASSURANCE COMPANY OF HELD
was unceremoniously terminated not only because of CANADA NO.
the commissions he should continue to receive from 41 PHIL 263 - The document it is to be a provisional policy "for
the insurance business he has solicited and procured MALCOLM; November 29, 1920 four months only from the date of this application."
but also for the fact that by the very acts of the Immediately following the words fixing the four
respondents, he was made liable to Philamgen in the NATURE months period comes the word "provided" which has
event the insured fail to pay the premiums due. They Appeal from an order of the CFI of Zamboanga the meaning of "if." Otherwise stated, the policy for
are estopped by their own positive averments and sustaining a demurrer to plaintiff's complaint upon four months is expressly made subject to the
claims for damages. the ground that it fails to state a cause of action. affirmative condition that the company shall confirm
- "The principal may not defeat the agent's right to this agreement by issuing a policy on said application
indemnification by a termination of the contract of FACTS when the same shall be submitted to the head office
agency (Erskine v. Chevrolet Motors Co. 185 NC 479, - On July 6, 1917, Luis Lim of Zamboanga made in Montreal. To re-enforce the same there follows the
117 SE 706, 32 ALR 196). application to the Sun Life Assurance Company of negative condition - "Should the company not issue
- For the pivotal factor rendering Philamgen and the Canada for a policy of insurance on his life in the such a policy, then this agreement shall be null and
other private respondents liable in damages is that sum of P5,000. In his application Lim designated his void ab initio, and the company shall be held not to
the termination by them of the General Agency wife, Pilar de Lim, the plaintiff herein, as the have been on the risks." Certainly language could
Agreement was tainted with bad faith. This is in beneficiary. The first premium of P433 was paid by hardly be used which would more clearly stipulate
accordance with the precepts in Human Relations Lim, and upon such payment the company issued that the agreement should not go into effect until the
enshrined in our Civil Code. what was called a ''provisional policy." Luis Lim died home office of the company should confirm it by
2. NO. The respondent court erred in holding on August 23, 1917, after the issuance of the issuing a policy. As we read and understand the so-
Valenzuela liable. There was no factual and legal provisional policy but before approval of the called provisional policy, it amounts to nothing but
basis for the award. Under Section 77 of the application by the home office of the insurance an acknowledgment on behalf of the company, that
Insurance Code, the remedy for the non-payment company. Pilar de Lim brought an action to recover it has received from the person named therein the
of premiums is to put an end to and render the from the Sun Life sum of P5,000, the amount named sum of money agreed upon as the first year's
insurance policy not binding - "Sec. 77 . . . in the provisional policy. premium upon a policy to be issued upon the
[N]otwithstanding any agreement to the contrary, no - The "provisional policy" reads: "Received (subject application, if the application is accepted by the
policy or contract of insurance is valid and binding to the following stipulations and agreements) the company.
unless and until the premiums thereof have been sum of P433, being the amount of the first year's - It is of course a primary rule that a contract of
paid except in the case of a life or industrial life premium for a Life Assurance Policy on the life of Mr. insurance, like other contracts, must be assented to
policy whenever the grace period provision applies Luis D. Lim of Zamboanga for P5,000, for which an by both parties either in person or by their agents.
(P.D. 612, as amended otherwise known as the application dated the 6th day of July, 1917, has been So long as an application for insurance has not been
Insurance Code of 1974) made to the Sun Life Assurance Company of Canada. either accepted or rejected, it is merely an offer or
- This is buttressed by Section 776 of the - The above-mentioned life is to be assured in proposal to make a contract. The contract, to be
Insurance Code (Presidential Decree No. 612, accordance with the terms and conditions contained binding from the date of the application must have
promulgated on December 18, 1974), which now or inserted by the Company in the policy which may been a completed contract, one that leaves nothing
provides that no contract of Insurance by an be granted by it in this particular case for four to be done, nothing to be completed, nothing to be
insurance company is valid and binding unless and months only from the date of the application, passed upon, or determined, before it shall take
until the premium thereof has been paid, provided that the Company shall confirm this effect. There can be no contract of insurance unless
notwithstanding any agreement to the contrary." agreement by issuing a policy on said application the minds of the parties have met in agreement. Our
Disposition Petition is GRANTED. CA decision SET when the same shall be submitted to the Head Office view is, that a contract of insurance was not here
ASIDE. The decision of the TC REINSTATED with the in Montreal. Should the Company not issue such a consummated by the parties.
MODIFICATIONS. And that the contractual policy, then this agreement shall be null and void ab - The trial court committed no error in sustaining the
relationship between Arturo P. Valenzuela and initio, and the Company shall be held not to have demurrer and dismissing the case. It is to be noted,
Philippine American General Insurance Company been on the risk at all, but in such case the amount however that counsel for appellee admits the liability
herein acknowledged shall be returned.
INSURANCE Page
44

of the company for the return of the first premium to might have recovered, the widow of the decedent Dr.
the estate of the deceased. Leuterio may file the suit against the insurer,
Grepalife.
GREAT PACIFIC LIFE v. CA (LEUTERIO) ISSUES 2. NO
316 SCRA 677 1. WON CA erred in holding petitioner liable to DBP Ratio The fraudulent intent on the part of the
QUISUMBING; October 13, 1999 as beneficiary in a group life insurance contract from insured must be established to entitle the insurer to
a complaint filed by the widow of the rescind the contract. Misrepresentation as a defense
NATURE decedent/mortgagor of the insurer to avoid liability is an affirmative
Petition for Review of CA decision 2. WON CA erred in not finding that Dr. Leuterio defense and the duty to establish such defense by
concealed that he had hypertension, which would satisfactory and convincing evidence rests upon the
FACTS vitiate the insurance contract insurer. In the case at bar, the petitioner failed to
- A contract of group life insurance was executed 3. WON CA erred in holding Grepalife liable for clearly and satisfactorily establish its defense, and is
between petitioner Great Pacific Life Assurance P86,200.00 without proof of the actual outstanding therefore liable to pay the proceeds of the insurance.
Corporation (hereinafter Grepalife) and Development mortgage payable by the mortgagor to DBP Reasoning
Bank of the Philippines (hereinafter DBP). Grepalife [a] The insured, Dr. Leuterio, had answered in his
agreed to insure the lives of eligible housing loan HELD insurance application that he was in good health and
mortgagors of DBP. 1. NO that he had not consulted a doctor or any of the
- In Nov. 1983, Dr. Wilfredo Leuterio, a physician Ratio Insured, being the person with whom the enumerated ailments, including hypertension; when
and a housing debtor of DBP applied for membership contract was made, is primarily the proper person to he died the attending physician had certified in the
in the group life insurance plan. In an application bring suit. Subject to some exceptions, insured may death certificate that the former died of cerebral
form, Dr. Leuterio answered Qs concerning his health thus sue, although the policy is taken wholly or in hemorrhage, probably secondary to hypertension.
condition as follows: part for the benefit of another person named or From this report, petitioner Grepalife refused to pay
Q: Have you ever had, or consulted, a physician for unnamed, and although it is expressly made payable the insurance claim. It alleged that the insured had
a heart condition, high blood pressure, cancer, to another as his interest may appear or otherwise. concealed the fact that he had hypertension.
diabetes, lung, kidney or stomach disorder or any Although a policy issued to a mortgagor is taken out [b] Contrary to Grepalife’s allegations, there was no
other physical impairment? No. for the benefit of the mortgagee and is made payable sufficient proof that the insured had suffered from
Q: Are you now, to the best of your knowledge, in to him, yet the mortgagor may sue thereon in his hypertension. Aside from the statement of the
good health? Yes. own name, especially where the mortgagee's interest insured's widow who was not even sure if the
- Grepalife issued an insurance coverage of Dr. is less than the full amount recoverable under the medicines taken by Dr. Leuterio were for
Leuterio, to the extent of his DBP mortgage policy. (See Sec. 8, Insurance Code) hypertension, the appellant had not proven nor
indebtedness of P86,200.00. In Aug. 1984, Dr. Reasoning produced any witness who could attest to Dr.
Leuterio died due to "massive cerebral hemorrhage." [a] The insured private respondent did not cede to Leuterio's medical history.
DBP submitted a death claim to Grepalife. Grepalife the mortgagee all his rights or interests in the [c] Grepalife had failed to establish that there was
denied the claim because Dr. Leuterio was not insurance, the policy stating that: “In the event of concealment made by the insured, hence, it cannot
physically healthy when he applied for an insurance. the debtor's death before his indebtedness with the refuse payment of the claim.
Grepalife insisted that Dr. Leuterio did not disclose Creditor (DBP) shall have been fully paid, an amount 3. NO
he had been suffering from hypertension, which to pay the outstanding indebtedness shall first be - Considering the supervening event that DBP
caused his death. Allegedly, such non-disclosure paid to the creditor and the balance of sum assured, foreclosed in 1995 their residential lot, in satisfaction
constituted concealment that justified the denial of if there is any, shall then be paid to the of mortgagor's outstanding loan, the insurance
the claim. beneficiary/ies designated by the debtor.” When DBP proceeds shall inure to the benefit of the heirs of the
- Herein respondent Medarda Leuterio, widow, filed a submitted the insurance claim against Grepalife, the deceased person or his beneficiaries. Equity dictates
complaint with RTC against Grepalife for "Specific latter denied payment thereof, interposing the that DBP should not unjustly enrich itself at the
Performance with Damages." Dr. Mejia, who issued defense of concealment committed by the insured. expense of another. Hence, it cannot collect the
the death certificate, testified that Dr. Leuterio Thereafter, DBP collected the debt from the insurance proceeds, after it already foreclosed on the
complained of headaches presumably due to high mortgagor and took the necessary action of mortgage. The proceeds now rightly belong to Dr.
blood pressure. The inference was not conclusive foreclosure on the residential lot of private Leuterio's heirs represented by his widow, herein
because Dr. Leuterio was not autopsied, hence, other respondent. private respondent.
causes were not ruled out. [b] Since a policy of insurance upon life or health - The Court ruled this issue based on the clear
- RTC ruled in favor of respondent widow and against may pass by transfer, will or succession to any provisions of the policy. The mortgagor paid the
Grepalife. CA sustained the RTC decision. Hence, the person, whether he has an insurable interest or not, premium according to the coverage of his insurance,
present petition. and such person may recover it whatever the insured which states that: "The policy states that upon
INSURANCE Page
45

receipt of due proof of the Debtor's death during the policies inasmuch as said policies covered the actual consequence, no separate premiums are intended or
terms of this insurance, a death benefit in the number of logs loaded on board. But it is covered by required to be paid on a Cover Note.
amount of P86,200.00 shall be paid… In the event of Cover Note. c. The petitioner paid in full all the premiums as
the debtor's death before his indebtedness with the - On January 13, 1964 - the defendant wrote the called for by the statement issued by private
creditor shall have been fully paid, an amount to pay plaintiff denying the latter's claim, on the ground respondent after the issuance of the two regular
the outstanding indebtedness shall first be paid to that defendant's investigation revealed that the marine insurance policies, thereby leaving no
the Creditor and the balance of the Sum Assured, if entire shipment of logs covered by the two marines account unpaid by petitioner due on the insurance
there is any shall then be paid to the beneficiary/ies policies were received in good order at their point of coverage, which must be deemed to include the
designated by the debtor." From this, it is clear that destination. It was further stated that the said loss Cover Note. If the Note is to be treated as a separate
Grepalife is liable and that Dr. Leuterio’s heirs must may not be considered as covered under cover note policy instead of integrating it to the regular policies
get the proceeds. because the said note had become 'null and void by subsequently issued, the purpose and function of the
Disposition Petition DENIED. CA Decision AFFIRMED virtue of the issuance of two marine policies. Cover Note would be set at naught or rendered
with modification. - The CFI of Manila ruled in favour of the petitioner. meaningless, for it is in a real sense a contract, not a
- The Court of Appeals reversed the decision of the mere application for insurance which is a mere offer.
PACIFIC TIMBER EXPORT CORPORATION v. CA CFI. Had all the logs been lost during the loading
(WORKMEN’S INSURANCE CO) operations, but after the issuance of the Cover Note,
112 SCRA 199 ISSUES liability on the note would have already arisen even
DE CASTRO; February 25, 1982 1. WON the cover note is null and void for lack of before payment of premium. This is how the cover
valuable consideration because no separate note as a "binder" should legally operate; otherwise,
FACTS premiums are collected by private respondent on all it would serve no practical purpose in the realm of
- March 19, 1963 - the plaintiff secured temporary its cover notes commerce, and is supported by the doctrine that
insurance from the defendant for its exportation of 2. WON the court of appeals erred in holding that where a policy is delivered without requiring
1,250,000 board feet of Philippine Lauan and Apitong private respondent was released from liability under payment of the premium, the presumption is that a
logs to be shipped from the Diapitan Bay, Quezon to the cover note due to unreasonable delay in giving credit was intended and policy is valid.
Okinawa and Tokyo, Japan. The defendant issued on notice of loss because the court disregarded the 2. NO
said date Cover Note No. 1010, insuring the said proven fact that private respondent did not promptly - The private respondent company never raised this
cargo of the plaintiff "Subject to the Terms and and specifically object to the claim on the ground of ground in the proceedings. It must be because it did
Conditions of the WORKMEN'S INSURANCE delay in giving notice of loss and, consequently, not find any delay, as this Court fails to find a real
COMPANY, INC. printed Marine Policy form as filed objections on that ground are waived under section and substantial sign thereof. But even on the
with and approved by the Office of the Insurance 84 of the insurance act assumption that there was delay, this Court is
Commissioner. satisfied and convinced that as expressly provided by
- April 2, 1963 - The two (2) regular marine cargo HELD law, waiver can successfully be raised against private
policies were issued by the defendant in favor of the 1. NO respondent. Thus Section 84 of the Insurance Act
plaintiff. The total cargo insured under the two Ratio Cover note is issued with a consideration provides:
marine policies accordingly consisted of 1,395 logs, when, by express stipulation, the cover note is made "Section 84. - Delay in the presentation to an
or the equivalent of 1,195,498 bd. ft. subject to the terms and conditions of the marine insurer of notice or proof of loss is waived if
- After the issuance of cover note but before the policies, and the payment of premiums is one of the caused by any act of his or if he omits to take
issuance of the two marine policies some of the logs terms of the policies. objection promptly and specifically upon that
intended to be exported were lost during loading Reasoning ground."
operations in the Diapitan Bay due to bad weather. a. the cover note in question is subject to the terms - From what has been said, We find duly
- April 4, 1963 - The plaintiff informed the defendant and conditions of the marine policies substantiated petitioner's assignments of error.
about the loss of 'approximately 32 pieces of logs' b. Nature of the Cover Note: The fact that no Disposition The appealed decision is set aside and
during loading through a letter. separate premium was paid on the Cover Note the decision of the Court of First Instance is
- The plaintiff subsequently submitted a 'Claim before the loss insured against occurred, does not reinstated in toto with the affirmance of this Court.
Statement' demanding payment of the loss under the militate against the validity of petitioner's contention,
second marine cargo policy. for no such premium could have been paid, since by DEVELOPMENT INSURANCE v. IAC (PHIL UNION
- July 17, 1963 - the defendant requested the First the nature of the Cover Note, it did not contain, as REALTY DEVELOPMENT CORP)
Philippine Adjustment Corporation to inspect the loss all Cover Notes do not contain particulars of the 143 SCRA 62
and assess the damage. shipment that would serve as basis for the CRUZ; July 16, 1986
- August 23, 1963 - the adjuster reported that 'the computation of the premiums. As a logical
loss of 30 pieces of logs is not covered by the two FACTS
INSURANCE Page
46

- A fire occurred in the building of the private "If the property hereby insured shall, at the ---- or inaction ---- which indeed enabled it to avoid
respondent and it sued for recovery of damages from breaking out of any fire, be collectively of greater payment for more than five years from the filing of
the petitioner on the basis of an insurance contract value than the sum insured thereon then the the claim against it in 1980.
between them. The petitioner allegedly failed to insured shall be considered as being his own Disposition The appealed decision is affirmed in
answer on time and was declared in default by TC. A insurer for the difference, and shall bear a ratable full, with costs against the petitioner.
judgment of default was rendered on the strength of proportion of the loss accordingly. Every item, if
the evidence submitted ex parte by the private more than one, of the policy shall be separately HARDING v. COMMERCIAL UNION ASSURANCE
respondent, which was allowed full recovery of its subject to this condition." 38 PHIL 464
claimed damages. - However, there is no evidence on record that the FISHER; August 10, 1918
- On learning of this decision, the petitioner moved building was worth P5,800,000.00 at the time of the
to lift the order of default, invoking excusable loss. On the contrary, the building was insured at FACTS
neglect, and to vacate the judgment by default. Its P2,500,000.00, and this must be considered, by - Mrs. Harding was the owner of a Studebaker
motion was denied. agreement, the actual value of the property insured automobile; in consideration of the payment to the
- On appeal, IAC affirmed the TC decision in toto. on the day the fire occurred. This valuation becomes defendant of the premium of P150, by said plaintiff,
even more believable if it is remembered that at the Mrs. Henry E. Harding, with the consent of her
ISSUE time the building was burned it was still under husband, the defendant by its duly authorized agent,
1. WON default of petitioner is based on excusable construction and not yet completed. Smith, Bell & Company (limited), made its policy of
neglect - The Court notes that the policy in this case is an insurance in writing upon said automobile was set
2. What is the amount of indemnity due to the open policy and is subject to the express condition forth in said policy to be P3,000 that the value of
private respondent under its insurance contract? that: said automobile was set forth in said policy to be
WON CFI was correct in interpreting the contract "Open Policy. P3,000; that on March 24, 1916, said automobile
This is an open policy as defined in Sec57 of the was totally destroyed by fire; that the loss thereby to
HELD Insurance Act. In the event of loss, whether total plaintiffs was the sum of P3,000.
1. NO or partial, it is understood that the amount of the - The defendant’s version is that by request of Mrs.
- Summons was served through its vice-president. loss shall be subject to appraisal and the liability of Harding, it issued the policy of insurance on an
There were even several extensions to the original the company, if established, shall be limited to the automobile alleged by the said plaintiff to be her
period to answer. As a consequence, the TC, on actual loss, subject to the applicable terms, property. It was made by means of a proposal in
motion of the private respondent filed declared the conditions, warranties and clauses of this Policy, writing signed and delivered by said plaintiff to the
petitioner in default. This was done almost one and in no case shall exceed the amount of the defendant, guaranteeing the truth of the statements
month later. Even so, the petitioner made no move policy." contained therein which said proposal is referred to
at all for two months thereafter. It was only more - As defined in the aforestated provision, which is in the said policy of insurance made a part thereof;
than one month after the judgment of default was now Sec60 of the Insurance Code, "an open policy is that certain of the statements and representations
rendered by the TC that it filed a motion to lift the one in which the value of the thing insured is not contained in said proposal and warranted by said
order of default and vacate the judgment by default. agreed upon but is left to be ascertained in case of plaintiff to be true, to wit: (a) the price paid by the
- There is a pattern of inexcusable neglect. loss.". proposer for the said automobile; (b) the value of
2. The policy is an open policy which means that the - The actual loss has been ascertained in this case said automobile at the time of the execution and
actual loss, as determined, will represent the total and the Court will respect such factual determination delivery of the said proposal and (c) the ownership of
indemnity due the insured from the insurer except in the absence of proof that it was arrived at said automobile, were false and known to be false by
only that the total indemnity shall not exceed the arbitrarily. There is no such showing. Hence, the said plaintiff at the time of signing and delivering
face value of the policy. applying the open policy clause as expressly agreed the said proposal and were made for the purpose of
- The petitioner argues that since at the time of the upon by the parties in their contract, we hold that misleading and deceiving the defendant, and
fire the building insured was worth P5,800,000.00, the private respondent is entitled to the payment of inducing the defendant, relying upon the warranties,
the private respondent should be considered its own indemnity under the said contract in the total statements, and representations contained in the
insurer for the difference between that amount and amount of P508,867.00. said proposal and believing the same to be true,
the face value of the policy and should share pro rata - The refusal of its vice-president to receive the issued the said policy of insurance.
in the loss sustained. Accordingly, the private private respondent's complaint, as reported in the - The evidence shows that Hermanos, the Manila
respondent is entitled to an indemnity of only sheriff's return, was the first indication of the agents for the Studebaker automobile, sold the
P67,629.31, the rest of the loss to be shouldered by petitioner's intention to prolong this case and automobile to Canson for P3,200 (testimony of Mr.
it alone. The petitioner cites Condition 17 of the postpone the discharge of its obligation to the Diehl); who sold the said automobile to Henry
policy, which provides: private respondent under this agreement. That Harding for the sum of P1,500. Harding sold the said
intention was revealed further in its subsequent acts automobile to J. Brannigan for the sum of P2,000
INSURANCE Page
47

who sold the said automobile Henry Harding for the through its agent Smith, Bell & Company (limited), or any other title not requiring the owner to make a
sum of P2,800; Henry Harding gave the said and sold by it for a small sum, which had never been specific cash outlay for its acquisition.
automobile to his wife as a present; that said tendered to the plaintiff prior to the trial of this case, 2. NO
automobile was repaired and repainted at the Luneta but in open court during the trial the sum of P10 as - It has not been shown by the evidence that the
Garage at a cost of some P900; that while the said the proceeds of such sale was tendered to plaintiff statement was false; on the contrary we believe that
automobile was at the Luneta Garage; the latter and refused. it shows that the automobile had in fact cost more
solicited of Mrs. Harding the insurance of said - Trial judge decided that there was no proof of fraud than the amount mentioned. The court below found,
automobile by the Company; that a proposal was on the part of plaintiff in her statement of the value and the evidence shows, that the automobile was
filled out by the said agent and signed by the plaintiff of the automobile, or with respect to its ownership; bought by plaintiff's husband a few weeks before the
Mrs. Henry E. Harding, and in said proposal under that she had an insurable interest therein; and that issuance of the policy in question for the sum of
the heading "Price paid by proposer," is the amount defendant, having agreed to the estimated value, P2,800, and that between that time and the issuance
of "3,500" and under another heading "Present P3,000, and having insured the automobile for that of the policy some P900 was spent upon it in repairs
value" is the amount of "3,000". amount, upon the basis of which the premium was and repainting.
- After the said proposal was made a representative paid, is bound by it and must pay the loss in - The witness Server, an expert automobile
of the Manila agent of defendant went to the Luneta accordance with the stipulated insured value. mechanic, testified that the automobile was
Garage and examined said automobile and Mr. practically as good as new at the time the insurance
Server, the General Manager of the Luneta Garage, ISSUE was effected. The form of proposal upon which the
an experienced automobile mechanic, testified that 1. WON Mrs. Harding was not the owner of the policy was issued does not call for a statement
at the time this automobile was insured it was worth automobile at the time of the issuance of the policy, regarding the value of the automobile at the time of
about P3,000, and the defendant, by and through its and, therefore, had no insurable interest in it its acquisition by the applicant for the insurance, but
said agent Smith, Bell & Company (limited), 2. WON the statement regarding the cost of the merely a statement of its cost. The amount stated
thereafter issued a policy of insurance upon proposal automobile was a warranty, that the statement was was less than the actual outlay which the automobile
in which policy the said automobile was described as false, and that, therefore, the policy never attached represented to Mr. Harding, including repairs, when
of the "present value" of P3,000 and the said to the risk the insurance policy was issued.
defendant charged the said plaintiff Mrs. Henry E. - The court below found and the evidence shows,
Harding as premium on said policy the sum of P150, HELD without dispute, that the proposal upon which the
or 5 per cent of the then estimated value of P3,000. 1. NO policy in question was issued was made out by
- The "Schedule" in said policy of insurance describes - Article 1334 of the Civil Code which provides that defendant's agent by whom the insurance was
the automobile here in question, and provides in part "All gifts between spouses during the marriage shall solicited, and that appellee simply signed the same.
of follows: be void. Moderate gifts which the spouses bestow on It also appears that an examiner employed by the
"That during the period above set forth and during each other on festive days of the family are not defendant made an inspection of the automobile
any period for which the company may agree to included in this rule." before the acceptance of the risk, and that the sum
renew this policy the company will subject to the - Even assuming that defendant might have invoked after this examination. The trial court found that Mrs.
exception and conditions contained herein or article 1334 as a defense, the burden would be upon Harding, in fixing the value of the automobile at
endorsed hereon indemnify the insured against loss it to show that the gift in question does not fall P3,000, acted upon information given her by her
of or damage to any motor car described in the within the exception therein established. We cannot husband and by Mr. Server, the manager of the
schedule hereto (including accessories) by whatever say, as a matter of law, that the gift of an Luneta Garage. She merely repeated the information
cause such loss or damage may be occasioned and automobile by a husband to his wife is not a which had been given her by her husband, and at
will further indemnify the insured up to the value of moderate one. Whether it is or is not would depend the same time disclosed to defendant's agent the
the car or P3,000 whichever is the greater against upon the circumstances of the parties, as to which source of her information. There is no evidence to
any claim at common law made by any person (not nothing is disclosed by the record. sustain the contention that this communication was
being a person in the said motor car nor in the - We are of the opinion that it would be unfair to hold made in bad faith. We do not think that the facts
insured's service) for loss of life or for accidental the policy void simply because the outlay stated in the proposal can be held as a warranty of
bodily injury or damage to property caused by the represented by the automobile was made by the the insured, even if it should have been shown that
said motor car including law costs payable in plaintiff's husband and not by his wife, to whom he they were incorrect in the absence of proof of willful
connection with such claim when incurred with the had given the automobile. It cannot be assumed that misstatement. Under such circumstance, the
consent of the company." defendant should not have issued the policy unless it proposal is to be regarded as the act of the insurer
- On March 24, 1916, the said automobile was totally were strictly true that the price representing the cost and not of the insured.
destroyed by fire, and that the iron and steel of the machine had been paid by the insured and by Disposition Plaintiff was the owner of the
portions of said automobile which did not burn were no other person ? that it would no event insure an automobile in question and had an insurable interest
taken into the possession of the defendant by and automobile acquired by gift, inheritance, exchange, therein; that there was no fraud on her part in
INSURANCE Page
48

procuring the insurance; that the valuation of the appellate court distinguished between P & I Clubs payments in its behalf. We note that Steamship
automobile, for the purposes of the insurance, is vis-à-vis conventional insurance. The appellate court Mutual even renewed its P & I Club cover until it was
binding upon the defendant corporation, and that the also held that Pioneer merely acted as a collection cancelled due to non-payment of the calls. Thus, to
judgment of the court below is, therefore, correct agent of Steamship Mutual. continue doing business here, Steamship Mutual or
and must be affirmed, with interest, the costs of this through its agent Pioneer, must secure a license
appeal to be paid by the appellant. ISSUES from the Insurance Commission.
1. WON Steamship Mutual, a P & I Club, is engaged - Since a contract of insurance involves public
WHITE GOLD MARINE SERVICES v. PIONEER in the insurance business in the Philippines interest, regulation by the State is necessary. Thus,
INSURANCE 2. WON Pioneer needs a license as an insurance no insurer or insurance company is allowed to
464 SCRA 448 agent/broker for Steamship Mutual engage in the insurance business without a license or
QUISUMBING; July 28, 2005 a certificate of authority from the Insurance
HELD Commission.
NATURE 1. YES 2. YES
This petition for review assails the Decision of the - The test to determine if a contract is an insurance - SEC. 299 . . .
Court of Appeals, affirming the Decision of the contract or not, depends on the nature of the - No person shall act as an insurance agent or as an
Insurance Commission. Both decisions held that promise, the act required to be performed, and the insurance broker in the solicitation or procurement of
there was no violation of the Insurance Code and the exact nature of the agreement in the light of the applications for insurance, or receive for services in
respondents do not need license as insurer and occurrence, contingency, or circumstances under obtaining insurance, any commission or other
insurance agent/broker. which the performance becomes requisite. It is not compensation from any insurance company doing
by what it is called. Basically, an insurance contract business in the Philippines or any agent thereof,
FACTS is a contract of indemnity. In it, one undertakes for without first procuring a license so to act from the
- White Gold procured a protection and indemnity a consideration to indemnify another against loss, Commissioner, which must be renewed annually on
coverage for its vessels from Steamship Mutual damage or liability arising from an unknown or the first day of January, or within six months
through Pioneer Insurance. Subsequently, White contingent event. thereafter.
Gold was issued a Certificate of Entry and - In particular, a marine insurance undertakes to Disposition The petition is PARTIALLY GRANTED.
Acceptance. Pioneer also issued receipts evidencing indemnify the assured against marine losses, such as The Decision dated July 30, 2002 of the Court of
payments for the coverage. When White Gold failed the losses incident to a marine adventure. Section 99 Appeals affirming the Decision dated May 3, 2000 of
to fully pay its accounts, Steamship Mutual refused of the Insurance Code enumerates the coverage of the Insurance Commission is hereby REVERSED AND
to renew the coverage. marine insurance. SET ASIDE. The Steamship Mutual Underwriting
- Steamship Mutual thereafter filed a case against - Relatedly, a mutual insurance company is a Association (Bermuda) Ltd., and Pioneer Insurance
White Gold for collection of sum of money to recover cooperative enterprise where the members are both and Surety Corporation are ORDERED to obtain
the latter’s unpaid balance. White Gold on the other the insurer and insured. In it, the members all licenses and to secure proper authorizations to do
hand, filed a complaint before the Insurance contribute, by a system of premiums or business as insurer and insurance agent,
Commission claiming that Steamship Mutual violated assessments, to the creation of a fund from which all respectively. The petitioner’s prayer for the
Sections 186 and 187 of the Insurance Code, while losses and liabilities are paid, and where the profits revocation of Pioneer’s Certificate of Authority and
Pioneer violated Sections 299, 300 and 301 in are divided among themselves, in proportion to their removal of its directors and officers, is DENIED.
relation to Sections 302 and 303, thereof. interest. Additionally, mutual insurance associations,
- The Insurance Commission dismissed the or clubs, provide three types of coverage, namely, PANDIMAN v. MARINE MANNING MNGT CORP.
complaint. It said that there was no need for protection and indemnity, war risks, and defense 460 SCRA 418
Steamship Mutual to secure a license because it was costs. GARCIA; June 21, 2005
not engaged in the insurance business. It explained - A P & I Club is “a form of insurance against third
that Steamship Mutual was a Protection and party liability, where the third party is anyone other NATURE
Indemnity Club (P & I Club). Likewise, Pioneer need than the P & I Club and the members.” By definition Petition for certiorari to review CA decision
not obtain another license as insurance agent and/or then, Steamship Mutual as a P & I Club is a mutual
a broker for Steamship Mutual because Steamship insurance association engaged in the marine FACTS
Mutual was not engaged in the insurance business. insurance business. - Benito Singhid was hired as chief cook on board the
Moreover, Pioneer was already licensed, hence, a - The records reveal Steamship Mutual is doing vessel MV Sun Richie Five for a term of one year by
separate license solely as agent/broker of Steamship business in the country albeit without the requisite Fullwin Maritime Limited through its Philippine agent,
Mutual was already superfluous. certificate of authority mandated by Section 187 of Marine Manning and Management Corporation. While
- The Court of Appeals affirmed the decision of the the Insurance Code. It maintains a resident agent in the said vessel was on its way to Shanghai from Ho
Insurance Commissioner. In its decision, the the Philippines to solicit insurance and to collect
INSURANCE Page
49

Chih Minh City, Benito suffered a heart attack and insurance contract and hence under Article 1311 of - August 6, 1946 – action filed in CFI Manila to
subsequently died on June 24, 1997. the Civil Code, it is not liable for the obligation recover from the Huenefeld Co the sum of P92,650
- Apparently, the vessel and the crew were insured arising out of the insurance contract. above mentioned. The theory of the Filipinas Cia is
with Ocean Marine Mutual Insurance Association 2. NO that the insured merchandise were burned up after
Limited (OMMIAL), a Protective and Indemnity Club - Fullwin, as Benito’s principal employer is liable the policy issued in 1941 in favor of Huenefeld Co
of which Sun Richie Five Bulkers S.A. is a member. under the employment contract. Marine is also has ceased to be effective because of the outbreak of
Pandiman Philippines, the petitioner, is the local bound by its undertaking pursuant to the Rules and the war between the United States and Germany on
correspondent of OMMIAL. Regulations Governing Overseas Employment that “it December 10, 1941, and that the payment made by
- Benito’s widow, Rosita, filed a claim for death shall assume joint and solidary liability with the the Filipinas Cia to Huenefeld Co during the Japanese
benefits with Marine which referred her to Pandiman. employer for all the claims and liabilities which may military occupation was under pressure.
After her submission of the required documentation, arise in connection with the implementation of the - CFI: dismissed the action without pronouncement
Pandiman recommended payment of the death contract, including but not limited to the payment of as to costs.
benefits amounting to $79,000. However, payment wages, heath and disability compensation and - CA: CFI judgment affirmed, with costs. The case is
has not been made. repatriation”. In other words, both Fullwin and now before us on appeal by certiorari from the
- Rosita filed a complaint with the Labor Arbiter Marine should be held liable for whatever death decision of the Court of Appeals.
naming Marine, Pandiman, OMMIAL, and Fullwin as benefits the widow of Benito may be entitled to.
respondents. The Arbiter ordered all the Disposition The petition is granted and the CA ISSUE
respondents, except Pandiman, to jointly and decision is reversed and set aside. WON the policy in question became null and void
severally pay the widow the death benefits plus legal upon the declaration of war between United States
fees. The NLRC, on appeal by Marine, limited the FILIPINAS COMPANIA DE SEGUROS V and Germany
liable parties to Pandiman and OMMIAL but CHRISTERN, HUENEFELD AND CO INC
maintained the money award. The CA sustained the 89 PHIL 54 HELD
decision of the NLRC. Hence this appeal. PARAS; May 25, 1951 YES
Ratio The Philippine Insurance Law (Act No. 2427,
FACTS as amended,) in section 8, provides that "anyone
ISSUE - October 1, 1941 - Christern Huenefeld, & Co., Inc., except a public enemy may be insured." It stands to
1. WON Pandiman may be held liable for the death after payment of corresponding premium, obtained reason that an insurance policy ceases to be
benefits from the Filipinas Cia. de Seguros a fire policy in the allowable as soon as an insured becomes a public
2. WON Marine and its foreign principal, Fullwin, sum of P1000,000, covering merchandise contained enemy.
should be absolved from the death claim liabilities in No. 711 Roman Street, Binondo Manila. > Effect of war, generally. - All intercourse
- February 27, 1942 or during the Japanese military between citizens of belligerent powers which is
HELD occupation - building and insured merchandise were inconsistent with a state of war is prohibited by
1. NO burned. In due time the Huenefeld Co submitted to the law of nations. Such prohibition includes all
- Pandiman is not an insurance agent as defined by the Filipinas Cia its claim under the policy. The negotiations, commerce, or trading with the
Section 3007 of the Insurance Code. In this case, salvage goods were sold at public auction and, after enemy; all acts which will increase, or tend to
there was no showing that Pndiman in fact deducting their value, the total loss suffered by the increase, its income or resources; all acts of
negotiated the insurance contract between Sun respondent was fixed at P92,650. voluntary submission to it; or receiving its
Richie Five and the insurer OMMIAL. Even, if - Filipinas Cia refused to pay the claim on the ground protection; also all acts concerning the
Pandiman were an agent, payment for claims arising that the policy in favor of the respondent had ceased transmission of money or goods; and all contracts
from peril insured against, to which the insurer is to be in force on the date the United States declared relating thereto are thereby nullified. It further
liable, is definitely not one of the liabilities of an war against Germany, the respondent Corporation prohibits insurance upon trade with or by the
insurance agent. Thus, there is no legal basis (though organized under and by virtue of the laws of enemy, upon the life or lives of aliens engaged in
whatsoever for holding petitioner solidarily liable with the Philippines) being controlled by the German service with the enemy; this for the reason that
insurer OMMIAL for the widow’s claim for death subjects and the Filipinas Cia being a company under the subjects of one country cannot be permitted to
benefits. Also, Pandiman is not a party to the American jurisdiction when said policy was issued on lend their assistance to protect by insurance the
October 1, 1941. Filipinas Cia, however, in pursuance commerce or property of belligerent, alien
7
Section 300. Any person who for compensation solicits or obtains on of the order of the Director of Bureau of Financing, subjects, or to do anything detrimental too their
behalf of any insurance company transmits for a person other than Philippine Executive Commission, dated April 9, country's interest. The purpose of war is to cripple
himself an application for a policy or contract of insurance to or from
such company or offers or assumes to act in the negotiating of such
1943, paid to the Huenefeld Co the sum of P92,650 the power and exhaust the resources of the
insurance shall be an insurance agent within the intent of this section on April 19, 1943. enemy, and it is inconsistent that one country
and shall thereby become liable to all the duties, requirements, should destroy its enemy's property and repay in
liabilities, and penalties to which an insurance agent is subject.
INSURANCE Page
50

insurance the value of what has been so power of seizure and vesting was extended to all The legal wife, Pascuala Vda De Ebrado, also filed
destroyed, or that it should in such manner property of any foreign country or national so that her claim as the widow of the deceased.
increase the resources of the enemy, or render it no innocent appearing device could become a - Insular then filed an interpleader in court (CFI
aid, and the commencement of war determines, Trojan horse." Rizal) to determine to whom the proceeds should be
for like reasons, all trading intercourse with the - The respondent having become an enemy paid. CFI declared that Carponia was disqualified
enemy, which prior thereto may have been lawful. corporation on December 10, 1941, the insurance from becoming beneficiary of the insured and
All individuals therefore, who compose the policy issued in its favor on October 1, 1941, by the directing the Insular to pay the proceeds to the
belligerent powers, exist, as to each other, in a petitioner (a Philippine corporation) had ceased to be estate of Buenaventura.
state of utter exclusion, and are public enemies. (6 valid and enforcible, and since the insured goods
Couch, Cyc. of Ins. Law, pp. 5352-5353.) were burned after December 10, 1941, and during ISSUE
> In the case of an ordinary fire policy, which the war, the respondent was not entitled to any 1. WON a common-law wife named as beneficiary in
grants insurance only from year, or for some other indemnity under said policy from the petitioner. the insurance policy of a legally married man claim
specified term it is plain that when the parties However, elementary rules of justice (in the absence the proceeds of the same
become alien enemies, the contractual tie is of specific provision in the Insurance Law) require
broken and the contractual rights of the parties, so that the premium paid by the respondent for the HELD
far as not vested. lost. (Vance, the Law on period covered by its policy from December 11, 1. NO
Insurance, Sec. 44, p. 112.) 1941, should be returned by the petitioner. Ratio The prohibition that husband and wife cannot
Reasoning Disposition the appealed decision is hereby donate to each other applies to common-law
- The Court of Appeals overruled the contention of reversed and the respondent corporation is ordered relationships. As the appointment of a beneficiary in
the petitioner that the respondent corporation to pay to the petitioner the sum of P77,208.33, insurance may be considered a donation, one cannot
became an enemy when the United States declared Philippine currency, less the amount of the premium, name as beneficiary his common-law wife.
war against Germany, relying on English and in Philippine currency, that should be returned by the Reasoning
American cases which held that a corporation is a petitioner for the unexpired term of the policy in - It is quite unfortunate that the Insurance Code
citizen of the country or state by and under the laws question, beginning December 11, 1941. does not contain any specific provision grossly
of which it was created or organized. It rejected the resolutory of the prime question at hand.
theory that nationality of private corporation is - Rather, general rules of civil law should be applied
determined by the character or citizenship of its to resolve the issue. Art.2011, CC states: “The
controlling stockholders. contract of insurance is governed by special laws.
- There is no question that majority of the Matters not expressly provided for in such special
stockholders of the respondent corporation were laws shall be regulated by this Code.” Thus, when
German subjects. Therefore, Huenefeld Co became not otherwise specifically provided for by the
an enemy corporation upon the outbreak of the war INSULAR LIFE ASSURANCE CO. v. EBRADO Insurance Law, the contract of life insurance is
between the United States and Germany. The English 80 SCRA 181 governed by the general rules of the civil law
and American cases relied upon by the Court of MARTIN; October 28, 1977 regulating contracts.
Appeals have lost their force in view of the latest - Also, Art.2012 “any person who is forbidden from
decision of the Supreme Court of the United States in NATURE receiving any donation under Article 739 cannot be
Clark vs. Uebersee Finanz Korporation, decided on Appeal from judgment of RTC. named beneficiary of a life insurance policy by the
December 8, 1947, in which the controls test has person who cannot make a donation to him.”
been adopted. In "Enemy Corporation" by Martin FACTS Common-law spouses are, definitely, barred from
Domke, a paper presented to the Second - Buenaventura Ebrado obtained a whole-life receiving donations from each other.
International Conference of the Legal Profession held insurance policy from Insular, for P5,882.00 with a - Art.739, CC: The following donations shall be void:
at the Hague (Netherlands) in August. 1948 also rider for accidental death benefits for the same 1. Those made between persons who were guilty of
discussed this dilemma amount. He designated Carponia Ebrado as the adultery or concubinage at the time of donation;
> In Clark vs. Uebersee Finanz Korporation, A. G., revocable beneficiary, referring to her as the wife. - In essence, a life insurance policy is no different
dealing with a Swiss corporation allegedly - Afterwards, he died as a result of an accident when from a civil donation insofar as the beneficiary is
controlled by German interest, the Court: "The he was hit by a falling branch of a tree. Carponia concerned. Both are founded upon the same
property of all foreign interest was placed within filed a claim for the proceeds as the designated consideration: liberality. A beneficiary is like a donee,
the reach of the vesting power (of the Alien beneficiary in the policy, although she admits that because from the premiums of the policy which the
Property Custodian) not to appropriate friendly or she and Buenaventura were merely living as insured pays out of liberality, the beneficiary will
neutral assets but to reach enemy interest which husband and wife without the benefit of marriage. receive the proceeds or profits of said insurance. As
masqueraded under those innocent fronts. . . . The a consequence, the proscription in Art.739 CC should
INSURANCE Page
51

equally operate in life insurance contracts. The - Being a member of the Government Service distinct systems of benefits are paid out from two
mandate of Art.2012 cannot be laid aside: any Insurance System (GSIS, for short) when Consuegra distinct and separate funds that are maintained by
person who cannot receive a donation cannot be died on September 26, 1965, the proceeds of his life the GSIS. Thus, it doesn’t necessarily mean that the
named as beneficiary in the life insurance policy of insurance under policy No. 601801 were paid by the beneficiaries in the life insurance are also the
the person who cannot make the donation. GSIS to petitioner Basilia Berdin and her children beneficiaries in the retirement insurance.
- Policy considerations and dictates of morality who were the beneficiaries named in the policy. - Consuegra started in the government service
rightly justify the institution of a barrier between - However, Consuegra did not designate any sometime during the early part of 1943, or before
common-law spouses in regard to property relations beneficiary who would receive the retirement 1943. In 1943 Com. Act 186 was not yet amended,
since such relationship ultimately encroaches upon insurance benefits due to him. Respondent Rosario and the only benefits then provided for in said Com.
the nuptial and filial rights of the legitimate family. Diaz, the widow by the first marriage, filed a claim Act 186 were those that proceed from a life
There is every reason to hold that the bar in with the GSIS asking that the retirement insurance insurance. Upon entering the government service
donations between legitimate spouses and those benefits be paid to her as the only legal heir of Consuegra became a compulsory member of the
between illegitimate ones should be enforced in life Consuegra, considering that the deceased did not GSIS, being automatically insured on his life,
insurance policies since the same are based on designate any beneficiary with respect to his pursuant to the provisions of Com. Act 186 which
similar consideration. retirement insurance benefits. Petitioner Basilia was in force at the time. During 1943 the operation
- So long as marriage remains the threshold of Berdin and her children, likewise, filed a similar claim of the Government Service Insurance System was
family laws, reason and morality dictate that the with the GSIS, asserting that being the beneficiaries suspended because of the war, and the operation
impediments imposed upon married couple should named in the life insurance policy of Consuegra, they was resumed sometime in 1946. When Consuegra
likewise be imposed upon extra-marital relationship. are the only ones entitled to receive the retirement designated his beneficiaries in his life insurance he
If legitimate relationship is circumscribed by these insurance benefits due the deceased Consuegra. could not have intended those beneficiaries of his life
legal disabilities, with more reason should an illicit Resolving the conflicting claims, the GSIS ruled that insurance as also the beneficiaries of his retirement
relationship be restricted by these disabilities. the legal heirs of the late Jose Consuegra were insurance because the provisions on retirement
Disposition Decision AFFIRMED. Rosario Diaz, his widow by his first marriage who is insurance under the GSIS came about only when
CONSUEGRA v. GSIS entitled to one-half, or 8/16, of the retirement Com. Act 186 was amended by Rep. Act 660 on June
37 SCRA 315 insurance benefits, on the one hand; and Basilia 16, 1951. Hence, it cannot be said that because
ZALDIVAR; January 30, 1971 Berdin, his widow by the second marriage and their herein appellants were designated beneficiaries in
seven children, on the other hand, who are entitled Consuegra's life insurance they automatically
NATURE to the remaining one-half, or 8/16, each of them to became the beneficiaries also of his retirement
Appeal from the decision of the Court of First receive an equal share of 1/16. insurance.
Instance of Surigao del Norte awarding the 8/16 part - Dissatisfied with the foregoing ruling and - The provisions of subsection (b) of Section 11 of
of the proceeds of the deceased Consuegra’s apportionment made by the GSIS, Basilia Berdin and Commonwealth Act 186, as amended by Rep. Act
retirement benefits to Rosario Diaz. her children filed on October 10, 1966 a petition for 660, clearly indicate that there is need for the
mandamus with preliminary injunction in the Court of employee to file an application for retirement
FACTS First Instance of Surigao. insurance benefits when he becomes a member of
- The late Jose Consuegra, at the time of his death, - The CFI of Surigao ruled in favor of respondent the GSIS, and he should state in his application the
was employed as a shop foreman of the office of the Rosario Diaz and upheld the ruling of GSIS in all beneficiary of his retirement insurance. Hence, the
District Engineer in the province of Surigao del Norte. aspect. Thus, Basilia Berdin and her children beneficiary named in the life insurance does not
In his lifetime, Consuegra contracted two marriages, appealed said decision to the Supreme Court. automatically become the beneficiary in the
the first with herein respondent Rosario Diaz, retirement insurance unless the same beneficiary in
solemnized in the parish church of San Nicolas de ISSUE the life insurance is so designated in the application
Tolentino, Surigao, Surigao, on July 15, 1937, out of WON GSIS was correct in awarding half of the for retirement insurance.
which marriage were born two children, namely, Jose retirement benefit of the deceased to Rosario Diaz, - In the case of the proceeds of a life insurance, the
Consuegra, Jr. and Pedro Consuegra, but both the first wife, notwithstanding the fact that the same are paid to whoever is named the beneficiary
predeceased their father; and the second, which was petitioners were named as beneficiaries of the life in the life insurance policy. As in the case of a life
contracted in good faith while the first marriage was insurance insurance provided for in the Insurance Act, the
subsisting, with herein petitioner Basilia Berdin, on beneficiary in a life insurance under the GSIS may
May 1, 1957 in the same parish and municipality, out HELD not necessarily be an heir of the insured. The
of which marriage were born seven children, namely, YES insured in a life insurance may designate any
Juliana, Pacita, Maria Lourdes, Jose, Rodrigo, Lenida - The GSIS offers two separate and distinct systems person as beneficiary unless disqualified to be
and Luz, all surnamed Consuegra. of benefits to its members, one is the life insurance so under the provisions of the Civil Code. And in
and the other is the retirement insurance. These two the absence of any beneficiary named in the life
INSURANCE Page
52

insurance policy, the proceeds of the insurance will made in the person DAVAC as bigamous wife is null donation; (the court did not decide whether this
go to the estate of the insured. and void, because it contravenes the provisions of partakes the nature of a life insurance policy)
- On the other hand, the beneficiary of the the Civil Code 3. NO
retirement insurance can only claim the proceeds of 3. WON the benefits accruing from membership with - The benefit receivable under the Act is in the
the retirement insurance if the employee dies before SSS forms part of the conjugal property thus the nature of a special privilege or an arrangement
retirement. If the employee failed or overlooked to resolution deprives the lawful wife of her share in the secured by the law pursuant to the policy of the
state the beneficiary of his retirement insurance, the conjugal property as well as of her own and her State to provide social security to the workingmen.
retirement benefits will accrue to his estate and will child's legitime in the inheritance The amounts that may thus be received cannot be
be given to his legal heirs in accordance with law, as considered as property earned by the member during
in the case of a life insurance if no beneficiary is HELD his lifetime. His contribution to the fund constitutes
named in the insurance policy. 1. YES only an insignificant portion thereof. Then, the
Disposition Petition Denied. It is Our view, - Section 13, RA1161 provides that the beneficiary benefits are specifically declared not transferable,
therefore, that the respondent GSIS had correctly "as recorded" by the employee's employer is the one and exempted from tax, legal processes, and lien.
acted when it ruled that the proceeds of the entitled to the death benefits. Furthermore, in the settlement of claims thereunder,
retirement insurance of the late Jose Consuegra - Section 13, Republic Act No. 1161, as amended by the procedure to be observed is governed not by the
should be divided equally between his first living wife Republic Act No. 1792, in force at the time of general provisions of law, but by rules and
Rosario Diaz, on the one hand, and his second wife Petronilo Davac's death provides: Upon the covered regulations promulgated by the Commission. Thus, if
Basilia Berdin and his children by her. employee's death or total and permanent disability the money is payable to the estate of a deceased
under such conditions as the Commission may member, it is the Commission, not the probate or
SSS v. DAVAC define, before becoming eligible for retirement and if regular court that determines the person or persons
17 SCRA 863 either such death or disability is not compensable to whom it is payable.
BARRERA: July 30, 1966 under the Workmen's Compensation Act, he or. in - They are disbursed from a public special fund
case of his death, his beneficiaries, as recorded by created by Congress.The sources of this special fund
NATURE his employer shall be entitled to the following are the covered employee's contribution (equal to 2-
APPEAL from a resolution Of the Social Security benefit: 1/2 per cent of the employee's monthly
Commission. - In Tecson vs. Social Security System. Section 13 compensation) ; the employer's 'Contribution
was construed:"it may be true that the purpose of (equivalent to 3-1/2 per cent of the monthly
FACTS the coverage under the Social Security System is compensation of the covered employee) ;and the
- Petronilo Davac, became a member of the Social protection of the employee as well as of his family, Government contribution which consists in yearly
Security System (SSS for short) on September 1, but this purpose or intention of the law cannot be appropriation of public funds to assure the
1957. In the Member's Record he designated enforced to the extent of contradicting the very maintenance of an adequate working balance of the
respondent, Candelaria Davac as his beneficiary and provisions of said law contained in Section 13, funds of the System. Additionally, Section 21 of the
indicated his relationship to her as that of "wife". thereof” Social Security Actprovides that the benefits
- He died on April 5, 1959. It appears that the - When the provisions of a law are clear and explicit, prescribed in this Act shall not be diminished and the
deceased contracted two marriages, the first, with the courts can do nothing but apply its clear and Government of the Republic of the Philippines
Lourdes Tuplano on August 29, 1946, who bore him explicit provisions (Velasco vs. Lopez) accepts general responsibility for the solvency of the
a child, Romeo Davac, and the second, with 2. NO System.
Candelaria Davac on January 18, 1949, with whom - The disqualification mentioned in Article 739 is not - The benefits under the Social Security Act are not
he had a minor daughter, Elizabeth Davac. Both filed applicable to herein appellee Candelaria Davac intended by the lawmaking body to form part of the
their claims for death benefit with the SSS. because she was not guilty of concubinage, there estate of the covered –members.
- Social Security Commission issued the resolution being no proof that she had knowledge of the - Social Security Act is not a law of succession.
declaring respondent Candelaria Davac as the person previous marriage of her husband Petronilo. Disposition Resolution of the Social Security
entitled to receive the death benefits payable for the ART. 2012. Any person who is forbidden from Commission appealed is affirmed
death of Petronilo Davac. receiving any donation under Article 739 cannot be
named beneficiary of a life insurance policy by the FRANCISCO DEL VAL v. ANDRES DEL VAL
ISSUES person who cannot make any donation to him 29 PHIL 534
1. WON the Social Security Commission Candelaria according to said article. MORELAND; February 16, 1915
Davac is entitled to receive the death benefits ART. 739. The following donations shall be void:
2. WON a beneficiary under the Social Security (1) Those made between persons who were guilty NATURE
System partakes of the nature of a beneficiary in a of adultery or concubinage at the time of the
life insurance policy and, therefore the designation
INSURANCE Page
53

Appeal from a judgment of the Court of First plaintiffs account for the use and occupation of the creditors of any kind whatsoever of the person
Instance of the city of Manila dismissing the premises so redeemed since the date of the who effected the insurance in favor of the former."
complaint with costs. redemption. 2. NO
- The trial court refused to give relief to either party - The contract of life insurance is a special contract
FACTS and dismissed the action. In this appeal, it is claimed and the destination of the proceeds thereof is
- Plaintiffs and defendant are brothers and sisters; by the attorney for the plaintiffs that insurance determined by special laws which deal exclusively
that they are the only heirs at law and next of kin of provisions in the Code of Commerce are with that subject. The Civil Code has no provisions
Gregorio Nacianceno del Val, who died in Manila on subordinated to the provisions of the Civil Code as which relate directly and specifically to life-insurance
August 4, 1910, intestate found in article 1035. This article reads: contracts or to the destination of life insurance
- During the lifetime of the deceased he took out "An heir by force of law surviving with others of the proceeds. That subject is regulated exclusively by
insurance on his life for the sum of P40,000 and same character to a succession must bring into the the Code of Commerce which provides for the terms
made it payable to the defendant ANDRES DEL VAL hereditary estate the property or securities he may of the contract, the relations of the parties and the
as sole beneficiary. After his death the defendant have received from the deceased during the life of destination of the proceeds of the policy.
collected the face of the policy. From said policy he the same, by way of dowry, gift, or for any good - Assuming that the proceeds of the life-insurance
paid the sum of P18,365.20 to redeem certain real consideration, in order to compute it in fixing the policy being the exclusive property of the defendant
estate which the decedent had sold to third persons legal portions and in the account of the division." and he having used a portion thereof in the
with a right to repurchase. - Counsel also claims that the proceeds of the repurchase of the real estate sold by the decedent
- The redemption of said premises was made by the insurance policy were a donation or gift made by the prior to his death with right to repurchase, and such
attorney of defendant ANDRES in the name of the father during his lifetime to the defendant and that, repurchase having been made and the conveyance
plaintiffs and the defendant as heirs of the deceased as such, its ultimate destination is determined by taken in the names of all of the heirs instead of the
vendor. It further appears from the pleadings that those provisions of the Civil Code which relate to defendant alone, plaintiffs claim that the property
the defendant, on the death of the deceased, took donations, especially article 819. This article provides belongs to the heirs in common and not to the
possession of most of his personal property, which that "gifts made to children which are not defendant alone.
he still has in his possession, and that he has also betterments shall be considered as part of their legal - The Court rejected this contention unless the fact
the balance on said insurance policy amounting to portion." appear or be shown that the defendant acted as he
P21,634.80. did with the intention that the other heirs should
- Plaintiffs contend that the amount of the insurance ISSUES enjoy with him the ownership of the estate ---- in
policy belonged to the estate of the deceased and 1. WON the insurance belongs to the defendant and other words, that he proposed, in effect, to make a
not to the defendant personally; that, therefore, they not to the decedent’s estate gift of the real estate to the other heirs. If it is
are entitled to a partition not only of the real and 2. WON the Civil code provisions on succession established by the evidence that was his intention
personal property, but also of the P40,000 life prevail over any other law with respect to the and that the real estate was delivered to the
insurance. The complaint prays a partition of all the insurance plaintiffs with that understanding, then it is probable
property, both real and personal, left by the that their contention is correct and that they are
deceased; that the defendant account for HELD entitled to share equally with the defendant therein.
P21,634.80, and that the sum be divided equally 1. YES If, however, it appears from the evidence in the case
among the plaintiffs and defendant along with the - The SC agreed with the finding of the trial court that the conveyances were taken in the name of the
other property of deceased. that the proceeds of the life-insurance policy belong plaintiffs without his knowledge or consent, or that it
- The defendant denies the material allegations of exclusively to the defendant as his individual and was not his intention to make a gift to them of the
the complaint and sets up as special defense and separate property, we agree. That the proceeds of an real estate, then it belongs to him. If the facts are as
counterclaim that the redemption of the real estate insurance policy belong exclusively to the beneficiary stated, he has two remedies. The one is to compel
sold by his father was made in the name of the and not to the estate of the person whose life was the plaintiffs to reconvey to him and the other is to
plaintiffs and himself instead of in his name alone insured, and that such proceeds are the separate let the title stand with them and to recover from
without his knowledge or consent. Andres contends and individual property of the beneficiary, and not of them the sum he paid on their behalf.
that it was not his intention to use the proceeds of the heirs of the person whose life was insured, is the - For the complete and proper determination of the
the insurance policy for the benefit of any person but doctrine in America. We believe that the same questions at issue in this case, the Court was of the
himself, he alleging that he was and is the sole doctrine obtains in these Islands by virtue of section opinion that the cause should be returned to the trial
owner thereof and that it is his individual property. 428 of the Code of Commerce, which reads: court with instructions to permit the parties to frame
He, therefore, asks that he be declared the owner of "The amounts which the underwriter must deliver such issues as will permit the settlement of all the
the real estate redeemed by the payment of the to the person insured, in fulfillment of the questions involved and to introduce such evidence as
P18,365.20, the owner of the remaining P21,634.80, contract, shall be the property of the latter, even may be necessary for the full determination of the
the balance of the insurance policy, and that the against the claims of the legitimate heirs or issues framed. Upon such issues and evidence taken
INSURANCE Page
54

thereunder the court will decide the questions 2. WON the insured, the husband, has the power to premiums paid, and to an endowment policy, as well
involved according to the evidence, subordinating his change the beneficiary, the former wife, and to name as to an ordinary life insurance policy. If the husband
conclusions of law to the rules laid down in this instead his actual wife, where the insured and the wishes to retain to himself the control and ownership
opinion. REMANDED. beneficiary have been divorced and where the policy of the policy he may so provide in the policy. But if
of insurance does not expressly reserve to the the policy contains no provision authorizing a change
GERCIO v. SUN LIFE ASSURANCE OF CANADA insured the right to change the beneficiary of beneficiary without the beneficiary's consent, the
48 PHIL 53 insured cannot make such change. Accordingly, it is
MALCOLM; September 28, 1925 HELD held that a life insurance policy of a husband made
1. Whether the case be considered in the light of the payable to the wife as beneficiary, is the separate
NATURE Code of Commerce, the Civil Code, or the Insurance property of the beneficiary and beyond the control of
Mandamus to compel Sun Life Assurance Co. of Act, the deficiencies in the law will have to be the husband.
Canada to change the beneficiary in the policy issued supplemented by the general principles prevailing on - Unlike the statutes of a few jurisdictions, there is
by the defendant company on the life of the plaintiff the subject. To that end, we have gathered the rules no provision in the Philippine Law permitting the
Hilario Gercio which follow from the best considered American beneficiary in a policy for the benefit of the wife of
authorities. In adopting these rules, we do so with the husband to be changed after a divorce. It must
FACTS the purpose of having the Philippine Law of follow, therefore, in the absence of a statute to the
- On January 29, 1910, the Sun Life Assurance Co. of Insurance conform as nearly as possible to the contrary, that if a policy is taken out upon a
Canada issued an insurance policy on the life of modern Law of Insurance as found in the United husband's life the wife is named as beneficiary
Hilario Gercio. The policy was what is known as a 20- States proper. therein, a subsequent divorce does not destroy her
year endowment policy. By its terms, the insurance - Court’s first duty is to determine what law should rights under the policy.
company agreed to insure the life of Hilario Gercio be applied to the facts. The insurance policy was Reasoning
for the sum of P2,000, to be paid him on February 1, taken out in 1910, that the Insurance Act. No. 2427, - Yore vs. Booth
1930, or if the insured should die before said date, became effective in 1914, and that the effort to “. . . It seems to be the settled doctrine, with but
then to his wife, Mrs. Andrea Zialcita, should she change the beneficiary was made in 1922. slight dissent in the courts of this country, that a
survive him; otherwise to the executors, - Code of Commerce- there can be found in it no person who procures a policy upon his own life,
administrators, or assigns of the insured. The policy provision either permitting or prohibiting the insured payable to a designated beneficiary, although he
did not include any provision reserving to the insured to change the beneficiary. pays the premiums himself, and keeps the policy
the right to change the beneficiary. - Civil Code- it would be most difficult, if indeed it is in his exclusive possession, has no power to
- On the date the policy was issued, Andrea Zialcita practicable, to test a life insurance policy by its change the beneficiary, unless the policy itself, or
was the lawful wife of Hilario Gercio. Towards the provisions. In the case of Del Val vs. Del Val, it the charter of the insurance company, so provides.
end of the year 1919, she was convicted of the crime declined to consider the proceeds of the insurance In policy, although he has parted with nothing,
of adultery. On September 4, 1920, a decree of policy as a donation or gift, saying "the contract of and is simply the object of another's bounty, has
divorce was issued in civil case no. 17955, which had life insurance is a special contract and the acquired a vested and irrevocable interest in the
the effect of completely dissolving their bonds of destination of the proceeds thereof is determined by policy, which he may keep alive for his own benefit
matrimony special laws which deal exclusively with that subject. by paying the premiums or assessments if the
- On March 4, 1922, Hilario Gercio formally notified The Civil Code has no provisions which relate directly person who effected the insurance fails or refuses
the Sun Life that he had revoked his donation in and specifically to life-insurance contracts or to the to do so.”
favor of Andrea Zialcita, and that he had designated destination of life-insurance proceeds. . . ." - Connecticut Mutual Life Insurance Company
in her stead his present wife, Adela Garcia de Gercio, - Insurance Act- there is likewise no provision vs Schaefer
as the beneficiary of the policy. Gercio requested the either permitting or prohibiting the insured to change “We do not hesitate to say, however, that a policy
insurance company to eliminate Andrea Zialcita as the beneficiary. taken out in good faith and valid at its inception, is
beneficiary. This, the insurance company has refused 2. NO not avoided by the cessation of the insurable
and still refuses to do. Ratio The wife has an insurable interest in the life of interest, unless such be the necessary effect of the
her husband. The beneficiary has an absolute vested provisions of the policy itself.. . . .In our judgment
ISSUES interest in the policy from the date of its issuance of life policy, originally valid, does not cease to be
1. (Preliminary) WON the provisions of the Code of and delivery. So when a policy of life insurance is so by the cessation of the assured party's interest
Commerce and the Civil Code shall be in force in taken out by the husband in which the wife is named in the life insured.”
1910, or the provisions of the Insurance Act now in as beneficiary, she has a subsisting interest in the - Central National Bank of Washington City vs.
force, or the general principles of law, guide the policy. And this applies to a policy to which there are Hume
court in its decision attached the incidents of a loan value, cash “It is indeed the general rule that a policy, and the
surrender value, an automatic extension by money to become due under it, belong, the
INSURANCE Page
55

moment it is issued, to the person or persons Disposition The judgment appealed from will be issuance of the questioned Order granting the
named in it as the beneficiary or beneficiaries, and reversed and the complaint ordered dismissed as to petition. Petitioner then filed a MFR which was also
that there is no power in the person procuring the the appellant. denied hence this petition.
insurance, by any act of his, by deed or by will, to
transfer to any other person the interest of the SEPARATE OPINION ISSUE
person named.” 1. WON the designation of the irrevocable
- In re Dreuil & Co. JOHNSON [concur] beneficiaries could be changed or amended without
“In so far as the law of Louisiana is concerned, it - I agree with the majority of the court, that the the consent of all the irrevocable beneficiaries
may also be considered settled that where a policy judgment of the lower court should be revoked, but 2. WON the irrevocable beneficiaries herein, one of
is of the semitontine variety, as in this case, the for a different reason. The purpose of the petition is whom is already deceased while the others are all
beneficiary has a vested right in the policy, of to have declared the rights of certain persons in an minors could validly give consent to the change or
which she cannot be deprived without her insurance policy which is not yet due and payable. It amendment in the designation of the irrevocable
consent” may never become due and payable. The premiums beneficiaries
- Wallace vs Mutual Benefit Life Insurance Co. may not be paid, thereby rendering the contract of
“As soon as the policy was issued Mrs. Wallace insurance of non effect, and many other things may HELD
acquired a vested interest therein, of which she occur, before the policy becomes due, which would 1. NO
could not be deprived without her consent, except render it non effective. The plaintiff and the other - Based on the provision of their contract and the law
under the terms of the contract with the insurance parties who are claiming an interest in said policy applicable, it is only with the consent of all the
company. No right to change the beneficiary was should wait until there is something due them under beneficiaries that any change or amendment in the
reserved. Her interest in the policy was her the same. For the courts to declare now who are the policy concerning the irrevocable beneficiaries may
individual property, subject to be divested only by persons entitled to receive the amounts due, if they be legally and validly effected. Both the law and the
her death, the lapse of time, or by the failure of ever become due and payable, is impossible, for the Policy do not provide for any other exception.
the insured to pay the premiums. She could keep reason that nothing may ever become payable under Reasoning
the policy alive by paying the premiums, if the the contract of insurance, and for many reasons such - Since the policy was procured in 1968, the
insured did not do so. It was contingent upon persons may never have a right to receive anything applicable law in this case is the Insurance Act and
these events, but it was free from the control of when the policy does become due and payable. In under that law, the beneficiary designated in a life
her husband. He had no interest in her property in my judgment, the action is premature and should insurance contract cannot be changed without the
this policy, contingent or otherwise. Her interest have been dismissed. consent of the beneficiary because he has a vested
was free from any claim on the part of the insured interest in the policy.
or his creditors. He could deprive her of her PHIL. AMERICAN LIFE INSURANCE v. PINEDA - The Beneficiary Designation Indorsement in the
interest absolutely in but one way, by living more 175 SCRA 416 policy in the name of Dimayuga states that the
than twenty years.” PARAS; July 19, 1989 designation of the beneficiaries is irrevocable: “no
- Filley vs. Illinois Life Insurance Company right or privilege under the Policy may be exercised,
“The benefit accruing from a policy of life NATURE or agreement made with the Company to any
insurance upon the life of a married man, payable Petition for review on certiorari the orders of CFI change in or amendment to the Policy, without the
upon his death to his wife, naming her, is payable Judge Pineda consent of the said beneficiary/beneficiaries.”
to the surviving beneficiary named, although she - Contracts which are the private laws of the
may have years thereafter secured a divorce from FACTS contracting parties should be fulfilled according to
her husband, and he was thereafter again married - In 1968, Private Respondent Rodolfo Dimayuga the literal sense of their stipulations, if their terms
to one who sustained the relation of wife to him at procured an ordinary life insurance policy from the are clear and leave no room for doubt as to the
the time of his death. petitioner company and designated his wife and intention of the contracting parties, for contracts are
The rights of a beneficiary in an ordinary life children as irrevocable beneficiaries. On Feb. 22, obligatory, no matter in what form they may be,
insurance policy become vested upon the issuance 1980, Dimayuga filed with the CFI a petition to whenever the essential requisites for their validity
of the policy, and can thereafter, during the life of amend the designation of the beneficiaries in his life are present.
the beneficiary, be defeated only as provided by policy from irrevocable to revocable. Petitioner filed - Finally, the fact that the contract of insurance does
the terms of the policy.” an Urgent Motion to reset hearing as well as its not contain a contingency when the change in the
- On the admitted facts and the authorities comment and/or Opposition to the respondent’s designation of beneficiaries could be validly effected
supporting the nearly universally accepted principles petition. means that it was never within the contemplation of
of insurance, we are irresistibly led to the conclusion - Respondent Judge denied petitioner’s Urgent the parties.
that the question at issue must be answered in the Motion, thus allowing private respondent to adduce 2. NO
negative evidence, the consequence of which was the
INSURANCE Page
56

- The parent-insured cannot exercise rights and/or value, at the time of the assured's death, either by subsection 5 the trustee acquires any property of the
privileges pertaining to the insurance contract, for contract or by convention practice of the company in insolvent which the latter could by any means have
otherwise, the vested rights of the irrevocable such cases. assigned to another. The Insolvency Law here in
beneficiaries would be rendered inconsequential. The - Both Ingersoll, as assignee, and Tan Sit, as force, in common with the predecessor laws above-
alleged acquiescence of the 6 children beneficiaries administratix of Dy Poco's estate, asserted claims to mentioned, contains nothing similar to these
cannot be considered an effective ratification to the the proceeds of the policy. The lower court found provisions.
change of the beneficiaries from irrevocable to that Ingersoll had a better right and ordered Sun Life On the applicability of the Insolvency Law
revocable. They were minors at the time, and could to pay the insurance proceeds to him. - Sec 32 of the Insolvency Law among other things,
not validly give consent. Neither could they act declares that the assignment to be made by the clerk
through their father-insured since their interests are ISSUE of the court "shall operate to vest in the assignee all
quite divergent from one another. WON Ingersoll, as assignee, has a right to the of the estate of the insolvent debtor not exempt by
Disposition questioned Orders of respondent judge proceeds of the insurance law from execution." Moreover, by section 24, the
are nullified and set aside. court is required, upon making an order adjudicating
HELD any person insolvent, to stay any civil proceedings
SUN LIFE ASSURANCE v. INGERSOLL NO pending against him; and it is declared in section 60
41 PHIL 331 On the Philippine Insolvency Law (Act No. 1956) that no creditor whose debt is provable under the Act
STREET; November 8, 1921 - The property and interests of the insolvent which shall be allowed, after the commencement of
become vested in the assignee of the insolvent are proceedings in insolvency, to prosecute to final
NATURE specified in section 32 of the Insolvency Law which judgment any action therefor against the debtor. In
Action of interpleader reads as follows: connection with the foregoing may be mentioned
"SEC. 32. As soon as an assignee is elected subsections 1 and 2 of section 36, as well as the
FACTS or appointed and qualified, the clerk of the court opening words of section 33, to the effect that the
- April 16, 1918, Sun Life Assurance Company of shall, by an instrument under his hand and seal of assignee shall have the right and power to recover
Canada (Sun Life), in consideration of the payment the court, assign and convey to the assignee all the and to take into his possession, all of the estate,
of a stipulated annual premium during the period of real and personal property, estate, and effects of the assets, and claims belonging to the insolvent, except
the policy, or until the premiums had been debtor with all his deeds, books, and papers relating such as are exempt by law from execution.
completely paid for twenty years, issued a policy of thereto, and such assignment shall relate back to the - These provisions clearly evince an intention to vest
insurance on the life of Dy Poco for US$12,500, commencement of the proceedings in insolvency, in the assignee, for the benefit of all the creditors of
payable to the said assured or his assigns on the and shall relate back to the acts upon which the the insolvent, such elements of property and
21st day of February, 1938, and if he should die adjudication was founded, and by operation of law property right as could be reached and subjected by
before that date then to his legal representatives. shall vest the title to all such property, estate, and process of law by any single creditor suing alone.
- June 23, 1919, the assured, Dy Poco, was effects in the assignee, although the same is then And this is exactly as it should be: for it cannot be
adjudged an involuntary insolvent by the CFI Manila, attached on mesne process, as the property of the supposed that the Legislature would suppress the
and Frank B. Ingersoll was appointed assignee of his debtor. Such assignment shall operate to vest in the right of action of every individual creditor upon the
estate. assignee all of the estate of the insolvent debtor not adjudication of insolvency, and at the same time
- July 10, 1919, Dy Poco died, and on August 21, exempt by law from execution." allow the insolvent debtor to retain anything subject
1919, Tan Sit, was duly appointed as the - the Insolvency Law is in great part a copy of the to the payment of his debts in a normal state of
administratrix of his intestate estate. Insolvency Act of California, enacted in 1895, though solvency.
- By the terms of the policy it was provided that after it contains a few provisions from the American - "leviable assets" and "assets in insolvency" are
the payment of three full premiums, the assured Bankruptcy Law of 1898 practically coextensive terms. Hence, in determining
could surrender the policy to the company for a - Under each of said laws the assignee acquires all what elements of value constitute assets in
"cash surrender value," indicated in an annexed the real and personal property, estate, and effects of insolvency, SC is at liberty to consider what elements
table; but inasmuch as no more than two premiums the debtor, not exempt by law from execution, with of value are subject to be taken upon execution, and
had been paid upon the policy now in question up to all deeds, books and papers relating thereto; and vice versa.
the time of the death of the assured, this provision while this language is broad, it nevertheless lacks the On whether a policy of insurance having no cash
had not become effective; and it does not appear comprehensiveness of section 70 (a) of the American surrender value, but payable to insured or his legal
that the company would in accordance with its own Bankruptcy Law of 1898 in at least two particulars; representative, is property that may be taken upon
usage or otherwise have made any concession to the for under subsection 3 of section 70 (a) of the last execution against him.
assured in the event he had desired, before his mentioned law, the trustee in bankruptcy acquires - Philippine laws declare no exemption with respect
death, to surrender the policy. It must therefore be the right to exercise any powers which the insolvent to insurance policies; and this species of property is
accepted that this policy had no cash surrender might have exercised for his own benefit, and under not enumerated, in section 48 of the Insolvency Law,
INSURANCE Page
57

among items from the ownership of which the reserve' required by law to be kept by the company assignee in insolvency to wrest from the insolvent a
assignee is excluded. Moreover, all life insurance for the benefit of the assured, and to be maintained policy of insurance which contains in it no present
policies are declared by law to be assignable, to the credit of the policy. So long as the policy realizable assets.
regardless of whether the assignee has an insurable remains in force the company has not practically any On the applicability of the Insolvency Law
interest in the life of the insured or not (Insurance beneficial interest in it, except as its custodian, with - Sec 32 of the Insolvency Law among other things,
Act No. 2427, sec. 166). the obligation to maintain it unimpaired and suitably declares that the assignment to be made by the clerk
- SC has held that insurance policies having a invested for the benefit of the insured. This is the of the court "shall operate to vest in the assignee all
present cash surrender value are subject to be taken practical, though not the legal, relation of the of the estate of the insolvent debtor not exempt by
upon execution. (Misut Garcia vs. West Coast San company to this fund. "Upon the surrender of the law from execution." Moreover, by section 24, the
Francisco Life Ins. Co.) policy before the death of the assured, the company, court is required, upon making an order adjudicating
- a policy devoid of a cash surrender value cannot be to be relieved from all responsibility for the increased any person insolvent, to stay any civil proceedings
either "leviable assets" or "assets in insolvency." risk, which is represented by this accumulating pending against him; and it is declared in section 60
- the assignee in insolvency acquired no beneficial reserve, could well afford to surrender a considerable that no creditor whose debt is provable under the Act
interest in the policy of insurance in question; that part of it to the assured, or his representative. A shall be allowed, after the commencement of
its proceeds are not liable for any of the debts return of a part in some form or other is now Usually proceedings in insolvency, to prosecute to final
provable against the insolvent in the pending made." (In re McKinney) judgment any action therefor against the debtor. In
proceedings, and that said proceeds should therefore - the stipulation providing for a cash surrender value connection with the foregoing may be mentioned
be delivered to his administratrix. is a comparatively recent innovation in life insurance. subsections 1 and 2 of section 36, as well as the
On applicable US case Formerly the contracts provided — as they still opening words of section 33, to the effect that the
- In re McKinney: no beneficial interest in the policy commonly do in the policies issued by fraternal assignee shall have the right and power to recover
had ever passed to the assignee over and beyond organizations and benefit societies — for the and to take into his possession, all of the estate,
what constituted the surrender value, and that the payment of a premium sufficient to keep the assets, and claims belonging to the insolvent, except
legal title to the policy was vested in the assignee estimated risk covered; and in case of a lapse the such as are exempt by law from execution.
merely in order to make the surrender value- policy-holder received nothing. Furthermore, the - These provisions clearly evince an intention to vest
available to him. The assignee should surrender the practice is common among insurance companies in the assignee, for the benefit of all the creditors of
policy upon the payment to him of said value, as he even now to concede nothing in the character of cash the insolvent, such elements of property and
was in fact directed to do. The assignee in surrender value, until three full premiums have been property right as could be reached and subjected by
bankruptcy had no right to keep the estate unsettled paid, as in this case. process of law by any single creditor suing alone.
for an indefinite period, for the mere purpose of - CONLUSION (from this case and other English and And this is exactly as it should be: for it cannot be
speculating upon the chances of the bankrupt's American cases cited following the same opinion): supposed that the Legislature would suppress the
death. As regards everything beyond the surrender the assignee acquires no beneficial interest in right of action of every individual creditor upon the
value, the assignee in bankruptcy would, after the insurance effected on the life of the insolvent, except adjudication of insolvency, and at the same time
discharge of the bankrupt, have no insurable interest to the extent that such insurance contains assets allow the insolvent debtor to retain anything subject
in the life of the bankrupt. which can be realized upon as of the date when the to the payment of his debts in a normal state of
- surrender value of a policy "arises from the fact petition of insolvency is filed. The explanation is to solvency.
that the fixed annual premiums is much in excess of be found in the consideration that the destruction of - "leviable assets" and "assets in insolvency" are
the annual risk during the earlier years of the policy, a contract of life insurance is not only highly practically coextensive terms. Hence, in determining
an excess made necessary in order to balance the prejudicial to the insured and those dependent upon what elements of value constitute assets in
deficiency of the same premium to meet the annual him, but is inimical to the interests of society. insolvency, SC is at liberty to consider what elements
risk during the latter years of the policy. This excess Insurance is a species of property that should be of value are subject to be taken upon execution, and
in the premium paid over the annual cost of conserved and not dissipated. As is well known, life vice versa.
insurance, with accumulations of interest, constitutes insurance is increasingly difficult to obtain with On whether a policy of insurance having no cash
the surrender value. Though this excess of premiums advancing years, and even when procurable after the surrender value, but payable to the insured or his
paid is legally the sole property of the company, still age of fifty, the cost is then so great as to be legal representative, is property that may be taken
in practical effect, though not in law, it is moneys of practically prohibitive to many. Insolvency is a upon execution against him.
the assured deposited with the company in advance disaster likely to overtake men in mature life; and - Philippine laws declare no exemption with respect
to make up the deficiency in later premiums to cover one who has gone through the process of bankruptcy to insurance policies; and this species of property is
the annual cost of insurance, instead of being usually finds himself in his declining years with the not enumerated, in section 48 of the Insolvency Law,
retained by the assured and paid by him to the accumulated savings of years swept away and among items from the ownership of which the
company in the shape of greatly-increased earning power diminished. The courts are therefore assignee is excluded. Moreover, all life insurance
premiums, when the risk is greatest. It is the 'net practically unanimous in refusing to permit the policies are declared by law to be assignable,
INSURANCE Page
58

regardless of whether the assignee has an insurable “operated on for a Tumor [mayoma] of the Court does not clearly and satisfactorily establish
interest in the life of the insured or not (Insurance stomach… associated with ulcer of stomach. Tumor that defense."
Act No. 2427, sec. 166). taken out was hard and of a hen's egg size. -Kwong Nam had informed the appellant's medical
- SC has held that insurance policies having a Operation was two years ago in Chinese General examiner that the tumor for which he was operated
present cash surrender value are subject to be taken Hospital by Dr. Yap. Claims he is completely on was ''associated with ulcer of the stomach." In the
upon execution. (Misut Garcia vs. West Coast San recovered.” Medical report show that insured was absence of evidence that the insured had sufficient
Francisco Life Ins. Co., 41 Phil., 258.) operated on for "peptic ulcer", involving the excision medical knowledge as to enable him to distinguish
- a policy devoid of a cash surrender value cannot be of a portion of the stomach, not tumor. between "peptic ulcer" and "a tumor", his statement
either "leviable assets" or "assets in insolvency." that said tumor was "associated with ulcer of the
- the assignee in insolvency acquired no beneficial ISSUE stomach" should be construed as an expression
interest in the policy of insurance in question; that WON there was concealment (Was appellant, made in good faith of his belief as to the nature of
its proceeds are not liable for any of the debts because of insured's aforesaid representation, misled his ailment and operation. Indeed, such statement
provable against the insolvent in the pending or deceived into entering the contract or in accepting must be presumed to have been made by him
proceedings, and that said proceeds should therefore the risk at the rate of premium agreed upon?) without knowledge of its incorrectness and without
be delivered to his administratrix. any deliberate intent on his part to mislead the
Disposition Judgment reversed. Sun Life is directed HELD appellant.
to pay the proceeds of the policy to Tan Sit. NO 3) Waiver:
-"concealment exists where the assured had While it may be conceded that, from the viewpoint of
CHAPTER VI – RESCISSION OF INSURANCE knowledge of a fact material to the risk, and a medical expert, the information communicated was
CONTRACTS: CONCEALMENT, honesty, good faith, and fair dealing requires that he imperfect, the same was nevertheless sufficient to
MISREPRESENTATION, & BREACH OF should communicate it to the assurer, but he have induced appellant to make further inquiries
WARRANTIES designedly and intentionally withholds the same." about the ailment and operation of the insured.
- It has also been held "that the concealment must, Section 32 of Insurance Law [Act No. 2427]
NG v. ASIAN CRUSADER LIFE ASSURANCE CORP in the absence of inquiries, be not only material, but provides:
122 SCRA 461 fraudulent, or the fact must have been intentionally “The right to information of material facts may be
ESCOLIN; May 30, 1983 withheld." waived either by the terms of insurance or by neglect
Reasoning to make inquiries as to such facts where they are
FACTS 1) The evidence shows that the Insular Life distinctly implied in other facts of which information
- On May 12, 1962, Kwong Nam applied for a 20- Assurance Co., Ltd. approved Kwong Nam's request is communicated.”
year endowment insurance on his life for the sum of for reinstatement and amendment of his lapsed It has been held that where, "upon the face of the
P20,000, with his wife, Ng Gan Zee, as beneficiary. insurance policy on April 24, 1962…. It results, application, a question appears to be not answered
- He died on Dec 1963 of cancer of the liver with therefore, that when on May 12, 1962 Kwong Nam at all or to be imperfectly answered, and the insurers
metastasis. All premiums had been paid at the time answered `No' to the question whether any life issue a policy without any further inquiry, they waive
of his death. insurance company ever refused his application for the imperfection of the answer and render the
- Ng presented a claim for payment of the face value reinstatement of a lapsed policy he did not omission to answer more fully immaterial.
of the policy. Appellant (Asian Crusader) denied the misrepresent any fact. Disposition the judgment appealed from is hereby
claim on the ground that the answers given by the 2) Assuming that the aforesaid answer given by the affirmed, with costs against appellant
insured to the questions appearing in his application insured is false, Sec. 278 of the Insurance Law
for life insurance were untrue. nevertheless requires that fraudulent intent on the CANILANG v. CA (GREAT PACIFIC LIFE
-Appellant: the insured was guilty of part of the insured be established to entitle the ASSURANCE CORP.)
misrepresentation when insurer to rescind the contract. And as correctly 223 SCRA 443
1) he answered "No" to the question (in the observed by the lower court, "misrepresentation as a FELICIANO; June 17, 1993
application) of "Has any life insurance company ever defense of the insurer to avoid liability is an
refused your application for insurance or for `affirmative’ defense. The duty to establish such a NATURE
reinstatement of a lapsed policy or offered you a defense by satisfactory and convincing evidence Petition for review on certiorari of the decision of the
policy different from that applied for?" when in fact, rests upon the defendant. The evidence before the Court of Appeals
Insular Life denied his application for reinstatement
of his lapsed life insurance policy 8 FACTS
"Sec. 27. Such party to a contract of insurance must communicate
2) he gave the appellant's medical examiner false to the other, in good faith, all facts within his knowledge which are
- June 18, 1982 – Jaime Canilang was diagnosed by
and misleading information as to his ailment and material to the contract, and which the other has not the means of Dr. Claudio to have sinus tachycardia. He was
previous operation when he said he was ascertaining, and as to which he makes no warranty."
INSURANCE Page
59

directed by the doctor to take a tranquilizer > It also found that the failure of Jaime to disclose application for insurance; that "probable and
(Trazepam) and a beta-blocker drug (Aptin). previous medical consultation and treatment reasonable influence of the farts" concealed must, of
- August 3, 1982 – Jaime consulted Dr. Claudio again constituted material information which should have course, be determined objectively, by the judge
and was diagnosed to have acute bronchitis. been communicated to Great Pacific to enable the ultimately.
- August 4, 1982 – Jaime applied for a nonmedical latter to make proper inquiries. Reasoning
insurance policy with Great Pacific Life Assurance - The information which Jaime failed to disclose was
Company. He named his wife Thelma as his ISSUES material to the ability of Great Pacific to estimate the
beneficiary. He was issue the policy with a face 1. WON Jaime intentionally withheld information probable risk he presented as a subject of life
value of P19,700 effective August 9, 1982. from Great Pacific insurance.
- August 5, 1983 – Jaime died of congestive heart 2. WON the information withheld would have been - Had Canilang disclosed his visits to his doctor, the
failure, anemia and chronic anemia. Thelma filed her material to Great Pacific’s decision to grant Jaime the diagnosis made and the medicines prescribed by
claim but the insurance company refused to grant it insurance policy such doctor, in the insurance application, it may be
on the ground that Jaime had concealed information. reasonably assumed that Great Pacific would have
- Thelma filed a complaint against Great Pacific to HELD made further inquiries and would have probably
recover the insurance proceeds. She testified that 1. YES refused to issue a non-medical insurance policy or, at
she was not aware of her husband’s ailments and Ratio Section 27 of the Insurance Code of 1978 is the very least, required a higher premium for the
that she thought he had died from a kidney disorder. properly read as referring to "any concealment same coverage.
- Great Pacific presented as witness Dr. Quismorio without regard to whether such concealment is - As held in the case of Saturnino vs. Philippine-
who testified that Jaime’s insurance application was intentional or unintentional. The restoration in 1985 American Life Insurance, “the waiver of medical
the basis of his medical declaration and she by B.P. Blg. 874 of the phrase "whether intentional examination in a non-medical insurance contract
explained that an applicant was required to undergo or unintentional" merely underscored the fact that all renders even more material the information inquired
medical examination only if the applicant had throughout (from 1914 to 1985), the statute did not of the applicant concerning previous condition of
disclosed that he had previously been consulted with require proof that concealment must be "intentional" health and diseases suffered, for such information
a doctor and had been hospitalized. in order to authorize rescission by the injured party. necessarily constitutes an important factor which the
- The Insurance Commissioner ordered Great Pacific Reasoning insurer takes into consideration in deciding whether
to pay Thelma the insurance proceeds, including - Art. 27 of the 1978 Insurance Code reads that “a to issue the policy or not.”
attorney’s fees, holding that Jaime’s illness was not concealment entitles the injured party to rescind a Disposition the Petition for Review is DENIED for
that serious as to Great Pacific’s decision to insure contract of insurance,” which does not include the lack of merit and the Decision of the Court of Appeals
him and that there was no concealment on the part words “whether intentional or unintentional” from the dated 16 October 1989 in C.A.-G.R. SP No. 08696 is
of Jaime with regard to his illness. previous statutes. The Insurance Commissioner hereby AFFIRMED.
Petitioners’ Claim: relied on this deletion in arguing that the statute
> Thelma argues that the non-disclosure of Jaime intended to limit the kinds of concealment which YU PANG CHENG v. CA
did not amount to fraud. generate a right to rescind on the part of the injured 105 PHIL 930
> She also argues that the CA erred in not holding party to "intentional concealments." BAUTISTA ANGELO; May 29, 1959
that the issue in the case agreed upon between the - In the case at bar, the nature of the facts not
parties before the Insurance Commission is whether conveyed to the insurer was such that the failure to FACTS
or not Jaime 'intentionally' made material communicate must have been intentional rather than - September 5, 1950: Yu Pang Eng submitted parts
concealment in stating his state of health; merely inadvertent. II and III of his application for insurance consisting
Respondents’ Comments: > Jaime could not have been unaware that his of the medical declaration made by him to the
> The CA reversed the Insurance Commissioner’s heart beat would at times rise to high and medical examiner of defendant and the medical
decision, holding that the use of the word alarming levels and that he had consulted a doctor examiner's report
'intentionally" by the Insurance Commissioner in twice two months before applying for non-medical - September 7: he submitted part I of his application
defining and resolving the issue agreed upon by the insurance. which is the declaration made by him to an agent of
parties at pre-trial before the Insurance > The last medical consultation took place just the defendant
Commissioner was not supported by the evidence day before the insurance application was filed. - September 8: defendant issued to the insured
and that the issue agreed upon by the parties had 2. YES Policy No. 812858
been whether Jaime made a material concealment as Ratio Materiality relates rather to the "probable and - December 27, 1950: the insured entered St.
to the state of his health at the time of the filing of reasonable influence of the facts" upon the party to Luke's Hospital for medical treatment but he died on
insurance application, justifying the denial of the whom the communication should have been made, in February 27, 1951.
claim. assessing the risk involved in making or omitting to - According to the death certificate, he died of
make further inquiries and in accepting the "infiltrating medullary carcinoma, Grade 4, advanced
INSURANCE Page
60

cardiac and of lesser curvature, stomach metastases connection with the subject of the questions facts been disclosed by the assured, the insurance
spleen." propounded. would never have been granted."
- Plaintiff, brother and beneficiary of the insured, - The negative answers given by the insured Disposition Decision affirmed.
demanded from defendant the payment of the regarding his previous ailment, or his concealment of
proceeds of the insurance policy and when the the fact that he was hospitalized and treated for
demand was refused, he brought the present action. sometime of peptic ulcer and had suffered from
- The insured, in his application for insurance, "dizziness, anemia, abdominal pains and tarry
particularly in his declarations to the examining stools", deprived defendant of the opportunity to
physician, stated the following in answering the make the necessary inquiry as to the nature of his
questions propounded to him: past illness so that it may form its estimate relative GREAT PACIFIC LIFE v. CA (supra p.34)
14. Have you ever had any of the following diseases to the approval of his application.
or symtoms? Each question must be read and - Had defendant been given such opportunity, PACIFIC BANKING CORP v. CA (ORIENTAL
answered "Yes" or "No.". considering the previous illness of the insured as ASSURANCE CORPORATION)
"Gastritis, Ulcer of the Stomach or any disease of disclosed by the records of the Chinese General 168 SCRA 1
that organ? No. Hospital, defendant would probably had never PARAS; November 28, 1988
"Vertigo, Dizziness, Fainting-spells or consented to the issuance of the policy in question.
Unconsciouness? No. In fact, according to the death certificate, the insured NATURE
"Cancer, Tumors or Ulcers of any kind? No. died of "infiltrating medullary carcinoma, Grade, 4, Petition for review on certiorari of the CA decision,
- 15. Have you ever consulted any physician riot advanced cardiac and of lesser curvature, stomach which set aside the decision of CFI Manila, which had
included in any of the above answers? Give names metastases spleen", which may have a direct in turn granted the complaint for a sum of money in
and address or physicians list ailments or accidents connection with his previous illness. civil case filed by Pacific Banking against Oriental
and date. No." - Our Insurance Law provides that "A neglect to Assurance.
- It appears that the insured entered the Chinese communicate that which a party knows and ought to
General Hospital for medical treatment on January communicate, is called concealment" (Section 25, FACTS
29, 1950 having stayed there up to February 11, Act No. 2427). Whether intentional or unintentional, - October 21,1963: an open Fire Policy was issued to
1950. the concealment entitles the insurer to rescind the the Paramount Shirt Manufacturing Co. (insured), by
- An X-ray picture of his stomach was taken and the contract of insurance (Section 26). which Oriental Assurance Corporation bound itself to
diagnosis made of him by his doctors showed that - Our law even requires the insured to communicate indemnify the insured for any loss or damage, not
his illness was "peptic ulcer, bleeding." to the insurer all facts within his knowledge which exceeding P61,000.00, caused by fire to its property
are material to the contract and which the other consisting of stocks, materials and supplies usual to
ISSUE party has not the means of ascertaining (Section a shirt factory, including furniture, fixtures,
WON the insured is guilty of concealment of some 27), and the materiality is to be determined not by machinery and equipment while contained in the
facts material to the risk insured against which has the event but solely by the probable and reasonable ground, second and third floors of the building
the effect of avoiding the policy as found by influence of the facts upon the party to whom the situated at number 256 Jaboneros St., San Nicolas,
respondent court. communication is due (Section 30). Manila, for a period of one year commencing from
- Argente vs. West Coast Life Insurance Co.: "One that date to October 21, 1964.
HELD ground for the rescission of a contract of insurance - Insured was at the time of the issuance of the
- It should be noted that the insured's confinement under the Insurance Act is 'a concealment', which in policy and is up to this time, a debtor of Pacific
in the Chinese General Hospital took place from section 25 is defined 'A neglect to communicate that Banking in the amount of not less P800,000.00 and
January 29, 1950 to February 11, 1950, whereas his which a party knows and ought to communicate.' the goods described in the policy were held in trust
application for insurance wherein he stated his Appellant argues that the concealment was by the insured for the Pacific Banking under thrust
answers to the questions propounded to him by the immaterial and insufficient to avoid the policy. We receipts.
examining physician of defendant was submitted to cannot agree. In an action on a life insurance policy - Said policy was duly endorsed to Pacific Banking as
defendant on September 5, 1950. where the evidence conclusively shows that the mortgagee/trustor of the properties insured, with the
- It is apparent that when the insured gave his answers to questions concerning diseases were knowledge and consent of Oriental Assurance to the
answers regarding his previous ailment, particularly untrue, the truth or falsity of the answers become effect that "loss if any under this policy is payable to
with regard to "Gastritis, Ulcer of the Stomach or any the determining factor. If the policy was procured by the Pacific Banking Corporation".
disease of that organ" and "Vertigo, Dizziness, fraudulent representations, the contract of insurance - While the aforesaid policy was in full force and
Fainting-spells or Unconsciousness", he concealed apparently set forth therein was never legally effect, a fire broke out on the subject premises
the ailment of which he was treated in the Chinese existent. It can fairly be assumed that had the true destroying the goods contained in its ground and
General Hospital which precisely has direct second floors. Counsel for the Pacific Banking sent a
INSURANCE Page
61

letter of demand to Oriental Assurance for indemnity Insurance Clause.' " CFI eventually adjudged Oriental not exist, the superstructure does not arise.
due to the loss of property by fire. Oriental Assurance liable to the Pacific Banking under the said Falsehood in such representations is not shown to
Assurance informed counsel that it was not yet ready contract of insurance. vary or add to the contract, or to terminate a
to accede to the latter's demand as the former is - Court of Appeals reversed. Pacific Banking's MR contract which has once been made, but to show
awaiting the final report of the insurance adjuster, denied. that no contract has ever existed (Tolentino). A void
H.H. Bayne Adjustment Company. or inexistent contract is one which has no force and
- Said insurance adjuster notified counsel for the ISSUES effect from the very beginning, as if it had never
Pacific Banking that the insured under the policy had 1. WON insured is guilty of fraud been entered into, and which cannot be validated
not filed any claim with it, nor submitted proof of 2. WON mortgagee/assignee can still claim from the either by time or by ratification.
loss which is a clear violation of Policy Condition insurance - As the insurance policy against fire expressly
No.11, and for which reason, determination of the required that notice should be given by the insured
liability of Oriental Assurance could not be had. HELD of other insurance upon the same property, the total
Pacific Banking's counsel replied asking the insurance 1. YES absence of such notice nullifies the policy.
adjuster to verify from the records of the Bureau of - The crux of the controversy centers on two points: - Argument that notice of co-insurances may be
Customs the entries of merchandise taken into the (a) unrevealed co-insurances which violated policy made orally is preposterous and negates policy
customs bonded warehouse razed by fire as a conditions No. 3; and (b) failure of the insured to file condition No. 20 which requires every notice and
reliable proof of loss. the required proof of loss prior to court action. other communications to the insurer to be written or
- For failure of the insurance company to pay the - Policy Condition No. 3 explicitly provides: “The printed.
loss as demanded, Pacific Banking field before CFI an Insured shall give notice to the Company of any 2. NO
action for a sum of money against the Oriental insurance already effected, or which may - Subject mortgage clause pecifically provides: “Loss,
Assurance, in the principal sum of P61,000.00 issued subsequently be effected, covering any of the if any, under this policy, shall be payable to the
in favor of Paramount Shirt Manufacturing Co. property hereby insured, and unless such notice be PACIFIC BANKING CORPORATION Manila
Oriental Assurance defenses given and the particulars of such insurance or mortgagee/trustor as its interest may appear, it
(a) lack of formal claim by insured over the loss and insurances be stated in or endorsed on this Policy by being hereby understood and agreed that this
(b) premature filing of the suit as neither plaintiff nor or on behalf of the Company before the occurrence insurance as to the interest of the mortgagee/trustor
insured had submitted any proof of loss on the basis of any loss or damage, all benefit under this policy only herein, shall not be invalidated by any act or
of which defendant would determine its liability and shall be forfeited.” neglect except fraud or misrepresentation, or arson
the amount thereof, either to the Oriental Assurance - It is not disputed that the insured failed to reveal of the mortgagor or owner/trustee of the property
or its adjuster H.H. Bayne Adjustment Co. before the loss three other insurances. By reason of insured; provided, that in case the mortgagor or
Pacific Banking said unrevealed insurances, the insured had been owner/ trustee neglects or refuses to pay any
> presented evidence that insured has undeclared guilty of a false declaration; a clear premium, the mortgagee/ trustor shall, on demand
co-insurances with the following: P30,000.00 with misrepresentation and a vital one because where the pay the same.”
Wellington Insurance; P25,000. 00 with Empire insured had been asked to reveal but did not, that - The paragraph clearly states the exceptions to the
Surety and P250,000.00 with Asian Surety; was deception. Otherwise stated, had the insurer general rule that insurance as to the interest of the
undertaken by insured Paramount on the same known that there were many co-insurances, it could mortgagee, cannot be invalidated; namely: fraud, or
property covered by its policy with Oriental have hesitated or plainly desisted from entering into misrepresentation or arson.
Assurance whereas the only co-insurances declared such contract. Hence, the insured was guilty of clear - Concealment of the aforecited co-insurances can
in the subject policy are those of P30,000.00 with fraud. easily be fraud, or in the very least,
Malayan, P50,000.00 with South Sea, and - Pacific Banking's contention that the allegation of misrepresentation. It is but fair and just that where
P25.000.00 with Victory fraud is but a mere inference or suspicion is the insured who is primarily entitled to receive the
- NOTE: the defense of fraud and/or violation of non- untenable. Concrete evidence of fraud or false proceeds of the policy has by its fraud and/or
declaration of co-insurances was not pleaded in the declaration by the insured was furnished by the misrepresentation, forfeited said right, with more
answer, also not pleaded in the Motion to Dismiss. Pacific Banking itself when the facts alleged in the reason Pacific Banking which is merely claiming as
- CFI denied Oriental Assurance's motion on the policy under clauses "Co-Insurances Declared" and indorsee of said insured, cannot be entitled to such
ground that since the defense was raised for the first "Other Insurance Clause" are materially different proceeds.
time, it must be deemed to have waived the from the actual number of co-insurances taken over - The fact of fraud was tried by express or at least
requirement of proof of loss. Case was submitted for the subject property. Consequently, the whole implied consent of the parties. Pacific Banking did
decision. But upon MR, Oriental Asurance was foundation of the contract fails, the risk does not not only object to the introduction of evidence but on
allowed to present additional evidence, "in order to attach and the policy never becomes a contract the contrary, presented the very evidence that
prove that 'insured has committed a violation of between the parties. Representations of facts are the proved its existence.
condition No. 3 of the policy in relation to the other foundation of the contract and if the foundation does
INSURANCE Page
62

- Be that as it may, SC has ample authority to give such final rejection, there was no real necessity for - April 15, 1986: Robert John B. Bacani procured a
beyond the pleadings where in the interest of justice bringing suit. Pacific Banking should have life insurance contract for himself from SUNLIFE
and the promotion of public policy, there is a need to endeavored to file the formal claim and procure all (petitioner) valued at P100K. The designated
make its own finding to support its conclusion. the documents, papers, inventory needed by Oriental beneficiary was his mother, Bernarda Bacani
Otherwise stated, the Court can consider a fact which Assurance or its adjuster to ascertain the amount of (respondent).
surfaced only after trial proper. loss and after compliance await the final rejection of - June 26, 1987: the insured died in a plane crash.
- Generally, the cause of action on the policy accrues its claim. Indeed, the law does not encourage Bernarda Bacani filed a claim with Sunlife, seeking
when the loss occurs, but when the policy provides unnecessary litigation. the benefits of the insurance policy taken by her son.
that no action shall be brought unless the claim is - Pacific Banking prematurely filed the civil case and Petitioner conducted an investigation and its findings
first presented extrajudicially in the manner provided dismissal thereof was warranted under the prompted it to reject the claim on the ground that
in the policy, the cause of action will accrue from the circumstances. While it is a cardinal principle of the insured did not disclose facts material to the
time the insurer finally rejects the claim for payment. insurance law that a policy or contract of insurance is issuance of the policy. The insured gave false
- In the case at bar, policy condition No. 11 to be construed liberally in favor of the insured and statements in the application when he answered in
specifically provides that the insured shall on the strictly as against the insurer company yet, contracts the negative to the question “have you ever had or
happening of any loss or damage give notice to the of insurance, like other contracts, are to be sought advice for urine, kidney, bladder
company and shall within fifteen (15) days after such construed according to the sense and meaning of the disorder?”
loss or damage deliver to the Oriental Assurance (a) terms which the parties themselves have used. If - Sunlife discovered that two weeks prior to the
a claim in writing giving particular account as to the such terms are clear and unambiguous, they must be issuance, insured was diagnosed with renal failure,
articles or goods destroyed and the amount of the taken and understood in their plain, ordinary and was confined, and underwent tests.
loss or damage and (b) particulars of all other popular sense. - November 17, 1988: Bacani and her husband filed
insurances, if any. Likewise, insured was required "at - Contracts of insurance are contracts of indemnity for specific performance against Sunlife. RTC granted
his own expense to produce, procure and give to the upon the terms and conditions specified in the policy. the plea on the ground that that the facts concealed
company all such further particulars, plans, The parties have a right to impose such reasonable by the insured were made in good faith and under
specifications, books, vouchers, invoices, duplicates conditions at the time of the making of the contract the belief that they need not be disclosed, and that
or copies thereof, documents, proofs and information as they may deem wise and necessary. The the disclosure was not material since the policy was
with respect to the claim". agreement has the force of law between the parties. non-medical.
- Evidence adduced shows that 24 days after the fire, The terms of the policy constitute the measure of the - Sunlife appealed to the CA, but the latter denied
Pacific Banking merely wrote letters to Oriental insurer's liability, and in order to recover, the insured the appeal on the ground that the cause of death
Assurance to serve as a notice of loss, thereafter, the must show himself within those terms. The was unrelated to the facts concealed by the insured.
former did not furnish the latter whatever pertinent compliance of the insured with the terms of the
documents were necessary to prove and estimate its policy is a condition precedent to the light of Petitioner’s Claim
loss. Instead, Pacific Banking shifted upon Oriental recovery. > The insured did not disclose facts relevant to the
Assurance the burden of fishing out the necessary - It appearing that insured has violated or failed to issuance of the policy, thus rescission of the contract
information to ascertain the particular account of the perform the conditions under No. 3 and 11 of the may be invoked by the insurance company.
articles destroyed by fire as well as the amount of contract, and such violation or want of performance Respondents’ Comments
loss. has not been waived by the insurer, the insured > The actual cause of death was not relevant to the
- Oriental Assurance and its adjuster notified Pacific cannot recover, much less the herein Pacific Banking. concealed information, and the policy was entered
Banking that insured had not yet filed a written claim Courts are not permitted to make contracts for the into by the insured in good faith.
nor submitted the supporting documents in parties; the function and duty of the courts is simply
compliance with the requirements set forth in the to enforce and carry out the contracts actually made. ISSUE
policy. Despite the notice, the latter remained Disposition Petition dismissed. CA affirmed. WON the concealment renders the insurance policy
unheedful. Since the required claim by insured, rescissible
together with the preliminary submittal of relevant SUNLIFE ASSURANCE COMPANY v. CA (SPS.
documents had not been complied with, it follows BACANI) HELD
that Oriental Assurance could not be deemed to have 245 SCRA 268 YES
finally rejected Pacific Banking's claim and therefore QUIASON; June 22, 1995 Ratio The terms of the contract are clear. The
the latter's cause of action had not yet arisen. insured is specifically required to disclose to the
Compliance with condition No. 11 is a requirement NATURE insurer matters relating to his health.
sine qua non to the right to maintain an action as A petition for review on certiorari. Reasoning
prior thereto no violation of Pacific Banking's right SEC. 26 (IC)
can be attributable to Oriental Assurance. As before FACTS
INSURANCE Page
63

A neglect to communicate that which a party of his death the plaintiff Francisca Eguaras; that after machinations, wherefore it is plain that the insurance
knows and ought to communicate, is called a compliance with the requisites and the investigation contract between the defendant and Dominador
concealment. carried on by the defendant company, it accepted Albay is null and void because it is false, fraudulent
SEC. 31 (IC) the application for insurance and issued the policy; and illegal.
Materiality is to be determined not by the event, that, said policy being in force, the insured died, and Disposition The judgment appealed from is
but solely by the probable and reasonable influence despite the fact that the beneficiary submitted reversed and the defendant absolved from the
of the facts upon the party to whom communication satisfactory proofs of his death and that the complaint without special finding as to the costs.
is due, in forming his estimate of the disadvantages defendant company investigated the event, still it
of the proposed contract or in making his inquiries refused and continues to refuse to pay to the plaintiff QUA CHEE GAN v. LAW UNION AND ROCK
- The information which the insured failed to disclose the value of the policy. 98 PHIL 85
was material and relevant to the approval and the - Defendant set forth in special defense that the REYES; December 17, 1955
issuance of the insurance policy. The matters insurance policy issued in the name of Dominador
concealed would have definitely affected petitioner's Albay had been obtained through fraud and deceit FACTS
action on his application, either by approving it with known and consented to by the interested parties - Qua Chee Gan insured 4 of his bodegas with Law
the corresponding adjustment for a higher premium and is therefore completely illegal, void, and Union & Rock Insurance Co in 1937. These bodegas
or rejecting the same. ineffective. were used for the storage of stocks of copra and of
- Good faith is no defense in concealment. It appears - A criminal case for frustrated estafa was filed by hemp, baled and loose.
that such concealment was deliberate on the part of defendant against Ponciano Remigio, Castor Garcia - Fire of undetermined origin that broke out in the
the insured. and Francisca Eguaras. They were acquitted, and early morning of July 21, 1940, and lasted almost
- The waiver of a medical examination [in a non- claim that the judgment produces the effect of res one week, gutted and completely destroyed Bodegas
medical insurance contract] renders even more judicata in the present suit. Nos. 1, 2 and 4, with the merchandise stored
material the information required of the applicant therein.
concerning previous condition of health and diseases ISSUE - Qua Chee Gan informed the insurance company of
suffered, for such information necessarily constitutes WON the life insurance obtained by Dominador Albay the fire. Fire adjusters of the company conducted an
an important factor which the insurer takes into was issued through fraud and deceit extensive investigation. Qua Chee Gan submitted the
consideration in deciding whether to issue the policy corresponding fire claims, totaling P398,562.81 (but
or not. HELD reduced to the full amount of the insurance,
- Anent the finding that the facts concealed had no YES P370,000), the Insurance Company resisted
bearing to the cause of death of the insured, it is well Ratio In a contract where one of the contracting payment, claiming violation of warranties and
settled that the insured need not die of the disease parties may have given his consent through error, conditions, filing of fraudulent claims, and that the
he had failed to disclose to the insurer. It is sufficient violence, intimidation, or deceit, and in any of such fire had been deliberately caused by the insured or
that his non-disclosure misled the insurer in forming cases the contract is void, even though, despite this by other persons in connivance with him.
his estimates of the risks of the proposed insurance nullity, no crime was committed. There may not have - Qua Chee Gan, his brother and his employees were
policy or in making inquiries been estafa in the case at bar, but it was tried for arson, where counsel of the insurance
Disposition Petition is granted and the decision of conclusively demonstrated by the trial that deceit company acted as a private prosecutor. They were
CA is reversed and set aside. entered into the insurance contract, fulfillment acquitted.
whereof is claimed, and therefore the conclusions - This civil suit was then instituted to claim against
EGUARAS v. GREAT EASTERN reached by the court in the judgment it rendered in the insurance company. The CFI ruled in favor of
33 PHIL. 263 the criminal proceedings for estafa do not affect this Qua Chee Gan and ordered Law Union Rock Co. to
TORRES.; January 24, 1916 suit, nor can they produce in the present suit the pay.
force of res adjudicata.
NATURE Reasoning ISSUES
Appeal filed through bill of exceptions from the - It is proven that the signatures on the insurance 1. WON there was a breach of the fire hydrant
judgment of the CFI applications reading "Dominado Albay" are false and warranty
forged; that the person who presented himself to Dr. 2. WON the insured violated the Hemp warranty
FACTS Vidal to be examined was not the real Dominador 3. WON Qua Chee Gan is guilty of overvaluation
- Francisca Eguaras filed a written complaint in court, Albay, but Castor Garcia who was positively 4. WON Qua Chee Gan caused the fire
alleging as a cause of action that her son-in-law identified by Dr. Vidal; that at the time of the 5. WON there was an error in the amount of copra
Dominador Albay had applied in writing to the application for insurance and the issuance of the and hemp lost
defendant insurance company to insure his life for policy which is the subject matter of this suit the real 6. WON the claims contained false and fraudulent
the sum of P5,000, naming as the beneficiary in case Dominador Albay was informed of all those statements
INSURANCE Page
64

insurer, the Court referred the controversy to a discrepancies were a result of the insured's
HELD government auditor, Apolonio Ramos; but the latter erroneous interpretation of the provisions of the
1. NO reached a different result from the other two. Not insurance policies and claim forms, caused by his
- It is argued that he should have 11 fire hydrants in only that, but Ramos reported two different imperfect knowledge of English, and that the
the compound, but he only had 2. We are in valuations that could be reached according to the misstatements were innocently made and without
agreement with the trial Court that the appellant is methods employed. Clearly then, the charge of intent to defraud. The trial court’s ruling must be
barred by waiver (or rather estoppel) to claim fraudulent overvaluation cannot be seriously upheld.
violation of the so-called fire hydrants warranty, for entertained. - For example, the occurrence of previous fires in the
the reason that knowing fully all that the number of 4. NO premises insured in 1939, altho omitted in the
hydrants demanded therein never existed from the - This defense is predicted on the assumption that claims, Exhibits EE and FF, were nevertheless
very beginning, the appellant nevertheless issued the the insured was in financial difficulties and set the revealed by the insured in his claims Exhibits Q (filed
policies in question subject to such warranty, and fire to defraud the insurance company, presumably simultaneously with them), KK and WW. Considering
received the corresponding premiums. in order to pay off the Philippine National Bank, to that all these claims were submitted to the smae
2. NO which most of the insured hemp and copra was agent, and that this same agent had paid the loss
- The insurance company avers that the insured pledged. This defense is fatally undermined by the caused by the 1939 fire, we find no error in the trial
violated the hemp warranty when it admitted that it established fact that, notwithstanding the insurer's Court's acceptance of the insured's explanation that
had 36 cans of gasoline in the building. It is well to refusal to pay the value of the policies the extensive the omission in Exhibits EE and FF was due to
note that gasoline is not specifically mentioned resources of the insured enabled him to pay off the inadvertance, for the insured could hardly expect
among the prohibited articles listed in the so-called National Bank in a short time; and if he was able to under such circumstances, that the 1939 would pass
"hemp warranty." The cause relied upon by the do so, no motive appears for attempt to defraud the unnoticed by the insurance agents. Similarly, the 20
insurer speaks of "oils (animal and/or vegetable insurer. While the acquittal of the insured in the per cent overclaim on 70 per cent of the hemo stock,
and/or mineral and/or their liquid products having a arson case is not res judicata on the present civil was explained by the insured as caused by his belief
flash point below 300o Fahrenheit", and is decidedly action, the insurer's evidence, to judge from the that he was entitled to include in the claim his
ambiguous and uncertain; for in ordinary parlance, decision in the criminal case, is practically identical in expected profit on the 70 per cent of the hemp,
"Oils" mean "lubricants" and not gasoline or both cases and must lead to the same result, since because the same was already contracted for and
kerosene. And how many insured, it may well be the proof to establish the defense of connivance at sold to other parties before the fire occurred.
wondered, are in a position to understand or the fire in order to defraud the insurer "cannot be Compared with other cases of over-valuation
determine "flash point below 003o Fahrenheit. Here, materially less convincing than that required in order recorded in our judicial annals, the 20 per cent
again, by reason of the exclusive control of the to convict the insured of the crime of arson. excess in the case of the insured is not by itself
insurance company over the terms and phraseology 5. NO sufficient to establish fraudulent intent. Certainly, the
of the contract, the ambiguity must be held strictly - As to the defense that the burned bodegas could insured's overclaim of 20 per cent in the case at bar,
against the insurer and liberally in favor of the not possibly have contained the quantities of copra duly explained by him to the Court a quo, appears
insured, especially to avoid a forfeiture and hemp stated in the fire claims, the insurer's case puny by comparison (compared to other cases cited
- Another point that is in favor of the insured is that rests almost exclusively on the estimates, inferences by the court), and can not be regarded as "more
the gasoline kept in Bodega No. 2 was only incidental and conclusions of its adjuster investigator, than misstatement, more than inadvertence of
to his business, being no more than a customary 2 Alexander D. Stewart, who examined the premises mistake, more than a mere error in opinion, more
day's supply for the five or six motor vehicles used during and after the fire. His testimony, however, than a slight exaggeration" that would entitle the
for transporting of the stored merchandise). "It is was based on inferences from the photographs and insurer to avoid the policy. It is well to note that the
well settled that the keeping of inflammable oils on traces found after the fire, and must yield to the overcharge of 20 per cent was claimed only on a
the premises though prohibited by the policy does contradictory testimony of engineer Andres Bolinas, part (70 per cent) of the hemp stock; had the
not void it if such keeping is incidental to the and specially of the then Chief of the Loan insured acted with fraudulent intent, nothing
business." (Bachrach vs. British American Ass. Co., Department of the National Bank's Legaspi branch, prevented him from increasing the value of all of his
17 Phil. 555, 560) Porfirio Barrios, and of Bank Appraiser Loreto copra, hemp and buildings in the same proportion.
3. NO Samson, who actually saw the contents of the This also applies to the alleged fraudulent claim for
- The charge that the insured failed or refused to bodegas shortly before the fire, while inspecting burned empty sacks, that was likewise explained to
submit to the examiners of the insurer the books, them for the mortgagee Bank our satisfaction and that of the trial Court. The rule is
vouchers, etc. demanded by them was found 6. NO that to avoid a policy, the false swearing must be
unsubstantiated by the trial Court, and no reason - Appellant insurance company also contends that willful and with intent to defraud which was not the
has been shown to alter this finding. the claims filed by the insured contained false and cause. Of course, the lack of fraudulent intent would
- In view of the discrepancy in the valuations fraudulent statements that avoided the insurance not authorize the collection of the expected profit
between the insured and the adjuster Stewart for the policy. But the trial Court found that the
INSURANCE Page
65

under the terms of the polices, and the trial Court disputed that Vicenta de Ocampo was taken by a diseases were untrue, the truth or falsity of the
correctly deducted the same from its award. patrolman, at the request of her husband, Bernardo answers become the determining factor. If the policy
Disposition Decision affirmed Argente, on May 19, 1924, to the Meisic police was procured by fraudulent representations, the
station, and from there was transferred to the San contract of insurance apparently set forth therein
ARGENTE v. WEST COAST LIFE Lazaro Hospital. In San Lazaro Hospital, her case was never legally existent. It can fairly be assumed
51 PHIL 725 was diagnosed by the admitting physician as that had the true facts been disclosed by the
MALCOLM; March 19, 1928 "alcoholism," but later Doctor Domingo made a assured, the insurance would never have been
diagnosis of probable "manic-depressive psychosis," granted.
FACTS and still, later in Mary Chiles Hospital, made a final - In Joyce, The Law of Insurance, second edition,
- This is an action upon a joint life insurance policy diagnosis of "phycho-neurosis." volume 3, Chapter LV, is found the following:
for P15,000 issued by the West Coast Life Insurance - Bernardo Argente, while readily conceding most of "The basis of the rule vitiating the contract in
Co., on May 15, 1925, in favor of Bernardo Argente, the facts herein narrated, yet alleges that both he cases of concealment is that it misleads or
and his wife, Vicenta de Ocampo, the latter having and his wife revealed to the company's physician, deceives the insurer into accepting the risk, or
died on November 18, 1925. Fraud in obtaining the Doctor Sta. Ana, all the facts concerning their accepting it at the rate of premium agreed upon;
policy was pleaded by way of special defense. On the previous illnesses and medical attendance, but that The insurer, relying upon the belief that the
issue thus suggested, the court adopted the theory Doctor Sta. Ana, presumably acting in collusion with assured will disclose every material fact within his
of the defendant, and held the insurance policy null the insurance agent, Jose Geronimo del Rosario, actual or presumed knowledge, is misled into a
and void, with the result that the complaint was failed to record them in the medical reports. The belief that the circumstance withheld does not
dismissed, with costs. evidence on these points consists of the testimony of exist, and he is thereby induced to estimate the
-Bernardo Argente signed an application for joint the plaintiff and his subordinate clerk, Apolonio risk upon a false basis that it does not exist. The
insurance with his wife in the sum of P2,000. The Espiritu, on the one hand, and of the testimony of principal question, therefore, must be, Was the
wife, Vicenta de Ocampo, signed a like application for Doctor Sta. Ana and Jose Geronimo del Rosario on assurer misled or deceived into entering a contract
the same policy. the other. This was rejected by the Trial Court. Trial obligation or in fixing the premium of insurance by
- Bernardo Argente and his wife was examined by judge found with the insurance company with regard a withholding of material information or facts
Dr. Cesareo Sta. Ana, a medical examiner for the to the question of fact. SC agrees. There appears no within the assured's knowledge or presumed
West Coast Life Insurance Co. which did not show motive whatever on the part of Doctor Sta. Ana to knowledge?
previous and existing health problems. falsify the Medical Examiner's Reports and thereby "It therefore follows that the assurer in assuming a
- A temporary policy for P15,000 was issued to not only jeopardize his career as a physician, but risk is entitled to know every material fact of which
Bernardo Argente and his wife as of May 15, 1925. also gravely implicate himself criminally. the assured has exclusive or peculiar knowledge,
In view of the fact that more than thirty days had as well as all material facts which directly tend to
elapsed since the applicants were examined by the ISSUE increase the hazard or risk which are known by the
company's physician, each of them was required to WON the contract of insurance may be rescinded assured, or which ought to be or are presumed to
file a certificate of health before the policy was be known by him. And a concealment of such facts
delivered to them. vitiates the policy. 'It does not seem to be
- On November 18, 1925, Vicenta de Ocampo died of necessary . . . that the . . . suppression of the
cerebral apoplexy. Thereafter Bernardo Argente HELD truth should have been willful.' If it were but an
presented a claim. Following investigation conducted YES inadvertent omission, yet if it were material to the
by the Manager of the Manila office of the insurance - Bernardo Argente and his wife applications’ were risk and such as the plaintiff should have known to
company, it was apparently disclosed that the false with respect to their state of health during the be so, it would render the policy void. But it is held
answers given by the insured in their medical period of five years preceding the date of such that if untrue or false answers are given in
examinations with regard to their health and applications and that they knew the representations response to inquiries and they relate to material
previous illnesses and medical attendance were made by them in their applications were false. The facts the policy is avoided without regard to the
untrue. West Coast Life Insurance Co. refused to pay question arises as to the state of the law in relation knowledge or fraud of assured, although under the
the claim of Bernardo Argente, and wrote him to the thereto. statute statements are representations which must
effect that the claim was rejected because the - One ground for the rescission of a contract of be fraudulent to avoid the policy. So under certain
insurance was obtained through fraud and insurance under the Insurance Act is "a codes the important inquiries are whether the
misrepresentation. concealment," which in section 25 is defined as "A concealment was willful and related to a matter
- It is admitted that it appears in the Medical neglect to communicate that which a party knows material to the risk.
Examiner's Report that Bernardo Argente gave false and ought to communicate." In an action on a life xxx xxx xxx
responses. As well as with the Medical Examiner's insurance policy where the evidence conclusively "If the assured has exclusive knowledge of
Report that Vicenta de Ocampo. It is, however, not shows that the answers to questions concerning material facts, he should fully and fairly disclose
INSURANCE Page
66

the same, whether he believes them material or the fact whether the assured knew of such ailment policy of P50k on the life of his 1 year old daughter,
not. But notwithstanding this general rule it will or not. . . ." Helen. Ngo Hing supplied the essetntial data which
not infrequently happen, especially in life risks, - Lastly, appellant contends that even if the petitioner Mondragon, branch manager of the Pacific
that the assured may have a knowledge actual or insurance company had a right to rescind the Life in Cebu, wrote on the corresponding form in his
presumed of material facts, and yet entertain an contract, such right cannot now be enforced in view own handwriting, later typing the data on an
honest belief that they are not material. . . . The of the provisions of section 47 of the Insurance Act application form signed by Ngo Hing. The latter paid
determination of the point whether there has or providing "Whenever a right to rescind a contract of the P1077.75 annual premium but retained P1,317
has not been a material concealment must rest insurance is given to the insurer by any provision of as commission as he was also a duly authorized
largely in all cases upon the form of the questions this chapter, such right must be exercised previous agent of Pacific Life. The binding deposit receipt was
propounded and the exact terms of the contract. to the commencement of an action on the contract." then issued to Ngo Hing; Mondragon handwrote his
Thus, where in addition to specifically named This section was derived from section 2583 of the strong recommendation for the approval of the
diseases the insured was asked whether he had California Civil Code, but in contrast thereto, makes application on the back of the form.
had any sickness within ten years, to which he use of the imperative "must" instead of the - On April 30, Mondragon received a letter from
answered 'No,' and it was proven that within that permissive "may." Nevertheless, there are two Pacific Life which stated that the 20 year endowment
period he had had a slight attack of pharyngitis, it answers to the problem as propounded. The first is plan was not available for minors below 7, but that
was held a question properly for the jury whether that the California law as construed by the code Pacific Life could consider the same under the
such an inflammation of the throat was a 'sickness' examiners, at whose recommendation it was Juvenile Triple Action Plan, advising that if the offer
within the intent of the inquiry, and the court adopted, conceded that "A failure to exercise the was acceptable, the Juvenile Non-Medical Declaration
remarked on the appeal decision that if it could be right (of rescission), cannot, of course, prejudice any be sent to the company.
held as a matter of law that the policy was thereby defense to the action which the concealment may -Mondragon allegedly failed to inform Ngo Hing of
avoided, then it was a mere device on the part of furnish." (Codes of California Annotated; Tan Chay the non-acceptance of the insurance plan, instead
insurance companies to obtain money without Heng vs. West Coast Life Insurance Company writing Pacific Life again, recommending the approval
rendering themselves liable under the policy. . . . [1927], p. 80, ante.) The second answer is that the of the endowment plan to children since customers
" . . . The question should be left to the jury insurance company more than one month previous had been asking for such coverage since 1954.
whether the assured truly represented the state of to the commencement of the present action wrote -On May 28, 1957, Helen died of influenza. Ngo Hing
his health so as not to mislead or deceive the the plaintiff and informed him that the insurance sought the payment of the proceeds of the
insurer; and if he did not deal in good faith with contract was void because it had been procured insurance, but having failed to do so, filed an action
the insurer in that matter, then the inquiry should through fraudulent representations, and offered to for recovery with the CFI of Cebu. The Court ordered
be made, Did he know the state of his health so as refund to the plaintiff the premium which the latter Pacific Life to pay P50k with 6% interest, hence this
to be able to furnish a proper answer to such had paid upon the return of the policy for petition.
questions as are propounded? A Massachusetts cancellation. As held in California as to a fire
case, if construed as it is frequently cited, would insurance policy, where any of the material ISSUE
be opposed to the above conclusion; but, on the representations are false, the insurer's tender of the WON the binding deposit receipt constituted a
contrary, it sustains it, for the reason that premium and notice that the policy is canceled, temporary contract of the life insurance in question
symptoms of consumption had so far developed before the commencement of suit thereon, operate
themselves within a few months prior to effecting to rescind the contract of insurance. (Rankin vs. HELD
the insurance as to induce a reasonable belief that Amazon Insurance Co. [1891], 89 Cal., 203.) NO
the applicant had that fatal disease, and we should Disposition Judgment affirmed, with the costs of - The binding deposit receipt is merely a provisional
further construe this case as establishing the rule this instance against the appellant. contract and only upon compliance with the ff
that such a matter cannot rest alone upon the conditions: (1) that the company be satisfied that
assured's belief irrespective of what is a GREAT PACIFIC LIFE v. CA (NGO HING) the applicant was insurable on standard rates (2)
reasonable belief, but that it ought to be judged by 89 SCRA 543 that if the company does not accept the application
the criterion whether the belief is one fairly DE CASTRO, J; April 30, 1979 and offers a different policy, the insurance contract
warranted by the circumstances. A case in Indiana, shall not be binding until the applicant accepts the
however, holds that if the assured has some NATURE new policy (3) that if the applicant is not found to be
affection or ailment of one or more of the organs Petition for certiorari insurable on standard rates and the application is
inquired about so well defined and marked as to disapproved, the insurance shall not be in force at
materially derange for a time the functions of such FACTS any time and the premium be returned to the
organ, as in the case of Bright's disease, the policy - On March 14, 1957, private respondent Ngo Hing applicant.
will be avoided by a nondisclosure, irrespective of filed an application with the Great Pacific Life -This implies the receipt is merely an
Assurance Co. (Pacific Life) for a 20 year endowment acknowledgement, on behalf of the company, that
INSURANCE Page
67

the Cebu branch of Pacific Life had received the - In resisting the claim of the petitioner, the condition contained in the said policy constituted a
premium and had accepted the application subject to respondent insurance corporation relies on a waiver of such condition.
processing by the insurance company, which will provision contained in the Certificate of Insurance, Disposition Judgment appealed from is REVERSED
approve or reject it depending on whether the excluding its liability to pay claims under the policy in and SET ASIDE and respondent insurance
applicant is insurable on standard rates. As such, the behalf of "persons who are under the age of sixteen corporation is ordered to pay to the petitioner the
receipt was never in force—it does not insure (16) years of age or over the age of sixty (60) proceeds of Insurance
outright. No liability attaches until the principal years ..." It is pointed out that the insured being
approves the risk and a receipt is given by the over sixty (60) years of age when she applied for the HARDING v. COMMERCIAL UNION (supra p.36)
agent; because private respondent failed to accept insurance coverage, the policy was null and void, and
Pacific Life’s offer for the Juvenile Triple Action plan, no risk on the part of the respondent insurance TAN v. CA (PHILIPPINE AMERICAN LIFE
there was no meeting of the minds and thus no corporation had arisen therefrom. INSURANCE COMPANY)
contract. Also, being an authorized agent of Pacific - RTC dismissed the complaint. 174 SCRA 403
Life, Ngo Hing must have known the company did GUTIERREZ; June 29, 1989
not offer the insurance applied for and merely took a ISSUE
chance on Mondragon’s recommendation. WON the acceptance by the private respondent NATURE
Disposition the decision appealed from is set aside, insurance corporation of the premium and the Review on certiorari of the decision of the Court of
absolving Pacific Life from their civil liabilities issuance of the corresponding certificate of insurance Appeals affirming the decision of the Insurance
should be deemed a waiver of the exclusionary Commissioner
EDILLON v. MANILA BANKERS LIFE condition of overage stated in the said certificate of
117 SCRA 187 insurance FACTS
VASQUEZ; September 30, 1982 - On September 23,1973, Tan Lee Siong, father of
HELD herein petitioners, applied for life insurance in the
NATURE YES amount of P 80,000.00 with respondent company.
Appeal from a decision of the CFI - The age of the insured Carmen 0. Lapuz was not Said application was approved and was issued
concealed to the insurance company. Her application effective November 6, 1973
FACTS for insurance coverage which was on a printed form - On April 26,1975, Tan Lee Siong died of hepatoma
- Sometime in April 1969, Carmen O, Lapuz applied furnished by private respondent and which contained (Exhibit B). Petitioners then filed with respondent
with respondent insurance corporation for insurance very few items of information clearly indicated her company their claim for the proceeds of the life
coverage against accident and injuries. In the age of the time of filing the same to be almost 65 insurance policy
application form which was dated April 15, 1969, she years of age. Despite such information which could -respondent company denied petitioners' claim and
gave the date of her birth as July 11, 1904. On the hardly be overlooked in the application form, rescinded the policy by reason of the alleged
same date, she paid the sum of P20.00 representing considering its prominence thereon and its misrepresentation and concealment of material facts
the premium for which she was issued the materiality to the coverage applied for, the made by the deceased Tan Lee Siong in his
corresponding receipt signed by an authorized agent respondent insurance corporation received her application for insurance. The premiums paid on the
of the respondent insurance corporation. Upon the payment of premium and issued the corresponding policy were thereupon refunded
filing of said application and the payment of the certificate of insurance without question. The - Petitioners filed on November 27, 1975, a
premium on the policy applied for, the respondent accident which resulted in the death of the insured, a complaint against the former with the Office of the
insurance corporation issued to Carmen O. Lapuz its risk covered by the policy, occurred on May 31, 1969 Insurance Commissioner. Commissioner denied
Certificate of Insurance. The policy was to be or FORTY-FIVE (45) DAYS after the insurance petition. CA affirmed Commissioners decision
effective for a period of 90 days. coverage was applied for. There was sufficient time
- On May 31, 1969 or during the effectivity of the for the private respondent to process the application ISSUE
Insurance, Carmen O. Lapuz died in a vehicular and to notice that the applicant was over 60 years of WON according to Sec. 48 of the Insurance Code,
accident. age and thereby cancel the policy on that ground if it insurance company is barred from rescinding
- On June 7, 1969, petitioner Regina L. Edillon, a was minded to do so. If the private respondent failed contract
sister of the insured and who was the named to act, it is either because it was willing to waive
beneficiary in the policy, filed her claim for the such disqualification; or, through the negligence or HELD
proceeds of the insurance, submitting all the incompetence of its employees for which it has only - Section 48. Whenever a right to rescind a contract
necessary papers and other requisites with the itself to blame, it simply overlooked such fact. Under of insurance is given to the insurer by any provision
private respondent. Her claim having been denied, the circumstances, the insurance corporation is of this chapter, such right must be exercised
Regina L. Edillon instituted this action in the Court of already deemed in estoppel. Its inaction to revoke previous to the commencement of an action on the
First Instance of Rizal. the policy despite a departure from the exclusionary contract.
INSURANCE Page
68

After a policy of life insurance made payable on the consulted any physician; that he had never spit
death of the insured shall have been in force during FACTS blood; and that there was no sign of either
the lifetime of the insured for a period of two years - Plaintiff alleges that defendant accepted and present or past disease of his lungs; whereas in
from the date of its issue or of its last reinstatement, approved a life insurance policy of for the sum of truth and in fact, plaintiff and coconspirators
the insurer cannot prove that the policy is void ab P10,000 in which the plaintiff was the sole well knew, Tan Ceang was addicted to morphine,
initio or is rescindable by reason of the fraudulent beneficiary; that the policy was issued upon the cocaine, and opium and had been convicted and
concealment or misrepresentation of the insured or payment by the said Tan Ceang of the first year's imprisoned therefor, and for about three year
his agent. premium amounting to P936; that in and by its prior thereto had been suffering from pulmonary
- According to the petitioners, the Insurance Law was terms, the defendant agreed to pay the plaintiff as tuberculosis.
amended and the second paragraph of Section 48 beneficiary the amount of the policy upon the receipt 4. Plaintiff caused a confidential report to
added to prevent the insurance company from of the proofs of the death of the insured while the the defendant insurance company to be signed
exercising a right to rescind after the death of the policy was in force; that without any premium being by one V. Sy Yock Kian, who was an employee
insured due or unpaid, Tan Ceang died on May 10, 1925; of Go Chulian, in which it was falsely
- The so-called "incontestability clause" precludes the that in June, 1925, plaintiff submitted the proofs of represented that Tan Ceang was worth about
insurer from raising the defenses of false the death of Tan Ceang with a claim for the payment P40,000, had an annual income of from eight to
representations or concealment of material facts of the policy which the defendant refused to pay, for ten thousand pesos net, had the appearance of
insofar as health and previous diseases are which he prays for a corresponding judgment, with good health, and never had tuberculosis.
concerned if the insurance has been in force for at legal interest from the date of the policy, and costs. 5. After said application for insurance,
least two years during the insured's lifetime. The - Defendant alleges that the insurance policy on the medical certificate and confidential report had
phrase "during the lifetime" found in Section 48 life of Tan Ceang, upon which plaintiff's action is been prepared and falsified, plaintiff and
simply means that the policy is no longer considered based, was obtained by the plaintiff in confabulation coconspirators caused the same to be forwarded
in force after the insured has died. The key phrase in with one Go Chulian, of Bacolod, Negros Occidental; to the defendant at its office in Manila, the
the second paragraph of Section 48 is "for a period Francisco Sanchez of the same place; and Dr. V. S. medical certificate thru the said Dr. V. S. Locsin
of two years." Locsin, of La Carlota, Negros Occidental, thru fraud as medical examiner, and said application for
- The policy was issued on November 6,1973 and the and deceit perpetrated against this defendant in the insurance and confidential report thru the said
insured died on April 26,1975. The policy was thus in following manner, to wit: Francisco Sanchez in his capacity as one of the
force for a period of only one year and five months. 1. Go, Sanchez and Locsin, caused Tan agents of the defendant insurance company in
Considering that the insured died before the two- Caeng to sign an application for insurance with the Province of Occidental Negros; that the
year period had lapsed, respondent company is not, the defendant in the sum of P10,000, in which it defendant, believing that the representations
therefore, barred from proving that the policy is void was said that Tan Ceang was single and was a made in said document were true, and relying
ab initio by reason of the insured's fraudulent merchant, and that the plaintiff Tan Chai Heng, thereon, provisionally accepted the said
concealment or misrepresentation. the beneficiary, was his nephew, whereas in application for insurance on the life of Tan Ceang
- The petitioners contend that there could have been truth and in fact and as the plaintiff and his said in the sum of P10,000 and issued a temporary
no concealment or misrepresentation by their late coconspirators well knew, the said Tan Ceang policy pending the final approval or disapproval
father because Tan Lee Siong did not have to buy was not single but was married and had several of said application by defendant's home-office in
insurance. He was only pressured by insistent children; and was not a merchant but a mere San Francisco, California, where in case of
salesmen to do so employee of Tan Quina from whom he received approval a permanent policy was to be issued;
-The legislative answer to the arguments posed by only a meager salary, and that plaintiff was not that such permanent policy was never delivered
the petitioners is the "incontestability clause" added a nephew of the said Tan Ceang. to the plaintiff because defendant discovered the
by the second paragraph of Section 48. The insurer 2. Tan Ceang was seriously ill, suffering fraud before its delivery.
has two years from the date of issuance of the from pulmonary tuberculosis of about three 6. That the first agreed annual premium
insurance contract or of its last reinstatement within years' duration, which illness was incurable and on the insurance in question of P936.50 not
which to contest the policy, whether or not, the was well known to the plaintiff and his said having been paid within 60 days after medical
insured still lives within such period. After two years, coconspirators. examination of the applicant as required by the
the defenses of concealment or misrepresentation, 3. Locsin, in his capacity as medical regulations of the defendant insurance company,
no matter how patent or well founded, no longer lie examiner for the defendant, prepared and plaintiff and coconspirators caused Tan Ceang to
falsified the necessary medical certificate, in sign a health certificate for reinstatement; that
TAN CHAY HENG v. WEST COAST LIFE which it was made to appear, among other the said temporary policy was delivered by
INSURANCE things, that Tan Ceang had never used defendant to the insured on April 10, 1925, in
51 PHIL 80 morphine, cocaine or any other drug; that he the belief that said statements and
JOHNS; November 21, 1927 was then in good health and had never
INSURANCE Page
69

representations were true and in reliance FERNANDEZ; December 19, 1974


thereon. HELD
7. 2 ½ months after the supposed medical NO NATURE
examination above referred to, and exactly 1 Ratio The word "rescind" has a well defined legal Appeal by certiorari from CA decision affirming a CFI
month after the date of the health certificate for meaning, and as applied to contracts, it presupposes decision which declared plaintiff Yap entitled to
reinstatement above set forth, Tan Ceang died the existence of a contract to rescind. recover from defendant Pioneer Insurance and
in Valladolid, Occidental Negros, of pulmonary Reasoning Surety Corp, the full amount of the damage inquired
tuberculosis, the same illness from which - Plaintiff vigorously contends that section 47 of the in Policy No. 4219
suffering at the time it is supposed he was Insurance Act should be applied, and that when so
examined by Dr. V. S. Locsin, but that the applied, defendant is barred and estopped to plead FACTS
plaintiff coconspirators, pursuant to their and set forth the matters alleged in its special - Yap owned a store in a 2 storey building, where
conspiracy, caused the said Dr. V. S. Locsin to defense. That section is as follows: she sold shopping bags and footwear. Her son-in-law
state falsely in the certificate of death that the Whenever a right to rescind a contract of insurance was in charge of the store
said Tan Ceang had died of cerebral is given to the insurer by any provision of this - April 19, 1962- Yap took out Fire Insurance Policy
hemorrhage. chapter, such right must be exercised previous to No. 4216 from Pioneer with a face value of P25,000
- Defendant also alleges that plaintiff was, like V. Sy the commencement of an action on the contract. covering her stocks, office furniture, fixtures, etc.
Yock Kian, an employee of Go Chulian; that the - It will be noted that defendant does not seek to - among the conditions set forth:
latter was the ringleader of a gang of malefactors, have the alleged insurance contract rescinded. It The Insured shall give notice to the
who, during, and for some years previous to the denies that it ever made any contract of insurance Company of any insurance or insurances already
dates above mentioned, were engaged in the illicit on the life of Tan Ceang or that any such a contract effected, or which may subsequently be
enterprise of procuring fraudulent life insurances ever existed, and that is the question which it seeks effected, covering any of the property hereby
from the present defendant, similar to the one in to have litigated by its special defense. In the very insured, and unless such notice be given and
question, and which enterprise was capitalized by nature of things, if the defendant never made or the particulars of such insurance or insurances
him by furnishing the funds with which to pay the entered into the contract in question, there is no be stated in, or endorsed on this Policy by or on
premium on said fraudulent insurance; that the said contract to rescind, and, hence, section 47 upon behalf of the Company before the occurrence of
Go Chulian was the one who furnished the money which the lower based its decision in sustaining the any loss or damage, all benefits under this
with which to pay the first and only annual premium demurrer does not apply. As stated, an action to Policy shall be forfeited. (emphasis supplied)
on the insurance here in question, amounting to rescind a contract is founded upon and presupposes It is understood that, except as may be
P936.50; that the said Go Chulian, on August 28, the existence of the contract which is sought to be stated on the face of this policy there is no other
1926, was convicted by the Court of First Instance of rescinded. If all of the material matters set forth and insurance on the property hereby covered and
the City of Manila, in criminal case No. 31425 of that alleged in the defendant's special plea are true, there no other insurance is allowed except by the
court, of the crime of falsification of private was no valid contract of insurance, for the simple consent of the Company endorsed hereon. Any
documents in connection with an fraudulent reason that the minds of the parties never met and false declaration or breach or this condition will
insurance, similar to the present, committed against never agreed upon the terms and conditions of the render this policy null and void.
this defendant in the month of September, 1924; contract. We are clearly of the opinion that, if such - At the time of insurance of Policy 4219(April 19,
that in the same case the said Francisco Sanchez matters are known to exist by a preponderance of 1962), an insurance policy for P20,000 issued by the
was one of the coaccused of the said Go Chulian but the evidence, they would constitute a valid defense Great American Insurance Company covering the
was discharged from the complaint, because he to plaintiff's cause of action. Upon the question as to same properties was noted on said policy as co-
offered himself and was utilized as a state's witness; whether or not they or are not true, we do not at this insurance.
that there is another civil action now pending against time have or express any opinion, but we are clear - August 29, 1962 : parties executed an
Go Chulian and Sanchez in the Court of First that section 47 does not apply to the allegations endorsement on Policy 4219 stating:
Instance of Manila (civil case No. 28680), in which made in the answer, and that the trial court erred in It is hereby declared and agreed that the co-
the present defendant is the plaintiff, for the sustaining the demurrer. insurance existing at present under this policy is as
recovery of the amounts of two insurance policies Disposition The judgment of the lower court is follows: P20,000.00 � Northwest Ins., and not as
aggregating P19,000, fraudulently obtained by the reversed and the case is remanded for such other originally stated. (emphasis supplied)
said Go Chulian and Sanchez. and further proceedings as are not inconsistent with Except as varied by this endorsement, all other
- To this, plaintiff filed a demurrer which was this opinion, with costs against the plaintiff. terms and conditions remain unchanged.
granted. - September 26, 1962: Yap took out another fire
PIONEER INSURANCE AND SURETY insurance policy for P20,000 covering the same
ISSUE CORPORATION v. YAP properties, from Federal Insurance Company. This
WON defense is barred by Art. 47 61 SCRA 426 policy was procured without notice to and the written
INSURANCE Page
70

consent of Pioneer, and was therefore not noted as a insured against fire the stocks in trade of New Life - The terms of the contract are clear and
co-insurance in Policy 4219. Enterprises with Western Guaranty Corporation, unambiguous. The insured is specifically required to
- December 19, 1962: Fire burned Yap’s store Reliance Surety and Insurance Co. Inc., and disclose to the insurer any other insurance and its
Equitable Insurance Corporation in the aggregate particulars which he may have effected on the same
ISSUE amount of P1,550,000.00. When the building where subject matter. The knowledge of such insurance by
WON petitioner should be absolved from liability on New Life Enterprises was located, along with the the insurer's agents, even assuming the acquisition
Fire insurance Policy No. 4219 on account of any stocks in trade therein, were gutted by fire, thereof by the former, is not the "notice" that would
violation by respondent Yap of the co-insurance petitioners filed an insurance claim against the three stop the insurers from denying the claim. Besides,
clause therein companies. The insurance companies all denied the so-called theory of imputed knowledge, that is,
Julian Sy’s claim on the ground of “breach of policy knowledge of the agent is knowledge of the principal,
HELD condition,” (i.e., the “other insurance” clause which aside from being of dubious applicability here has
YES required New Life Enterprises to inform each of the likewise been roundly refuted by respondent court
- The petitioner should be absolved. insurance companies in case the former insures with whose factual findings we find acceptable. The mere
Reasoning another company the same property already insured fact that Yap Kam Chuan was an agent for both
- There was a violation by Yap of the co-insurance by each of the insurance companies). Reliance and Equitable does not justify the allegation
clause contained in Policy No. 4219 which resulted in - Because of the denial of their claims for payment that the two are sister companies. Availment of the
the avoidance of the petitioner’s liability. by the 3 insurance companies, petitioners filed services of the same agents and adjusters by
- By the plain terms of the policy, other insurance separate civil actions against the former before the different companies is a common practice in the
without the consent of petitioner would ipso facto Regional Trial Court of Lucena City, which cases were insurance business and such facts do not warrant the
avoid the contract. It required no affirmative act of consolidated for trial. The trial court ruled in favor of speculative conclusion of the trial court.
election on the part of the company to make petitioner. However, the Court of Appeals reversed - Considering the terms of the policy which required
operative the clause avoiding the contract, wherever the trial court’s decision, found petitioner to have the insured to declare other insurances, the
the specified conditions should occur. Its obligations violated Clauses 3 and 27 of the separate insurance statement in question must be deemed to be a
ceased, unless, being informed of the fact, it policies issued by the 3 companies, and exonerated statement (warranty) binding on both insurer and
consented to the additional insurance. the insurance companies from liability. insured, that there were no other insurance on the
- The obvious purpose of the aforesaid requirement property. The annotation then, must be deemed to
in the policy is to prevent over-insurance and thus ISSUE be a warranty that the property was not insured by
avert the perpetration of fraud. The public, as well as WON petitioners violated the “Other Insurance any other policy. Violation thereof entitled the
the insurer, is interested in preventing the situation Clause” of the insurance policies insurer to rescind.
in which a fire would be profitable to the insured. - The obvious purpose of the aforesaid requirement
According to Justice Story: "The insured has no right HELD in the policy is to prevent over-insurance and thus
to complain, for he assents to comply with all the YES avert the perpetration of fraud. The public, as well as
stipulation on his side, in order to entitle himself to - Petitioners admit that the respective insurance the insurer, is interested in preventing the situation
the benefit of the contract, which, upon reason or policies issued by private respondents did not state in which a fire would be profitable to the insured. The
principle, he has no right to ask the court to dispense or endorse thereon the other insurance coverage insured has no right to complain, for he assents to
with the performance of his own part of the obtained or subsequently effected on the same comply with all the stipulations on his side, in order
agreement, and yet to bind the other party to stocks in trade for the loss of which compensation is to entitle himself to the benefit of the contract,
obligations, which, but for those stipulation would claimed by petitioners. It is further admitted by which, upon reason or principle, he has no right to
not have been entered into." petitioners that Equitable's policy stated "nil" in the ask the court to dispense with the performance of his
Disposition the appealed judgment of the Court of space thereon requiring indication of any co- own part of the agreement, and yet to bind the other
Appeals is reversed and set aside, and the petitioner insurance although there were 3 policies subsisting party to obligations, which, but for those stipulations,
absolved from all liability under the policy. on the same stocks in trade at the time of the loss, would not nave been entered into.
namely, that of Western in the amount of - It is not disputed that the insured failed to reveal
NEW LIFE ENTERPRISES v. CA P350,000.00 and two 2 policies of Reliance in the before the loss three other insurances. By reason of
207 SCRA 609 total amount of P1,000,000.00. said unrevealed insurances, the insured had been
REGALADO; March 31, 1992 - The coverage by other insurance or co-insurance guilty of a false declaration; a clear
effected or subsequently arranged by petitioners misrepresentation and a vital one because where the
FACTS were neither stated nor endorsed in the policies of insured had been asked to reveal but did not, that
- Julian Sy and Jose Sy Bang are partners engaged in the 3 private respondents, warranting forfeiture of all was deception. Otherwise stated, had the insurer
the business of selling construction materials under benefits thereunder if we are to follow the express known that there were many co-insurances, it could
the business name “New Life Enterprises.” Julian Sy stipulation in Policy Condition No. 3. have hesitated or plainly desisted from entering into
INSURANCE Page
71

such contract. Hence, the insured was guilty of clear insurance applies, or in any building connected in small quantities, such as oil, paints, etc; and (3)
fraud. therewith." Where such articles or goods were used for lighting
- As the insurance policy against fire expressly - Young placed in the residence and bodega three purposes, and in small quantities.
required that notice should be given by the insured boxes filled with fireworks. Said residence and - In the present case no claim is made that the
of other insurance upon the same property, the total bodega and the contents thereof were partially "hazardous goods" were placed in the bodega for
absence of such notice nullifies the policy. destroyed by fire. present or daily use. It is admitted that they were
- Additionally, insofar as the liability of respondent - The fireworks had been given to Young by the placed in the bodega "for future use," or for future
Reliance is concerned, it is not denied that the former owner of the Luneta Candy Store. He consumption, or for safe keeping. It seems clear to
complaint for recovery was filed in court by intended to use them in the celebration of the us that the "hazardous goods" in question were
petitioners only on January 31, 1984, or after more Chinese New Year. However, the authorities of the "stored" in the bodega, as that word is generally
than one (1) year had elapsed from petitioners' city of Manila had prohibited the use of fireworks on defined. That being true, suppose the defendant had
receipt of the insurers' letter of denial on November said occasion, so Young then placed them in the made an examination of the premises, even in the
29, 1982. bodega where they remained from the 4th or 5th of absence of a fire, and had found the "hazardous
- The condition contained in an insurance policy that February, 1913 until after the fire of March 18, 1913. goods" there, would it not have been justified in
claims must be presented within one year after - Both parties agree that the fireworks come within declaring the policy null and of no effect by reason of
rejection is not merely a procedural requirement but the phrase "hazardous goods," mentioned in a violation of its terms? If it might, then may it not
an important matter essential to a prompt settlement "Warranty B" of the policy; that the fireworks were repudiate its liability, even after the fire? If the
of claims against insurance companies as it demands found in a part of the building not destroyed by the "warranty" is a term of the contract, will not its
that insurance suits be brought by the insured while fire and that they in no way contributed to the fire, violation cause a breach and justify noncompliance
the evidence as to the origin and cause of or to the loss that resulted. or repudiation?
destruction have not yet disappeared. - The lower court rendered a judgment in favor of - Contracts of insurance are contracts of indemnity,
Young for the sum of P2,708.78, and costs. upon the terms and conditions specified therein.
QUA CHEE GAN v. LAW UNION (supra p.48) Parties have a right to impose such reasonable
ISSUE conditions at the time of the making of the contract
1. WON the placing of the fireworks in the building as they deem wise and necessary. The rate of
insured, they being "hazardous goods," was a premium is measured by the character of the risk
violation of the terms of the contract of insurance assumed. The insurer, for a comparatively small
and especially of "Warranty B." consideration, undertakes to guarantee the insured
against loss or damage, upon the terms and
HELD conditions agreed upon, and upon no other. When
YOUNG v. MIDLAND TEXTILE INSURANCE CO. 1. YES. the insurer is called upon to pay, in case of loss, he
30 PHIL 617 Reasoning It is admitted by both parties that the may justly insist upon a fulfillment of the terms of
JOHNSON; March 31, 1915 fireworks are hazardous goods. The defendant the contract. If the insured, cannot bring himself
alleged that they were "stored." The plaintiff within the terms and conditions of the contract, he is
FACTS contends that under all the facts and circumstances not entitled to recover for any loss suffered. The
- K.S. Young had a candy and fruit store on the of the case, they were not “stored” in said building, terms of the contract constitute the measure of the
Escolta, Manila, and occupied a building at 321 Calle and that the placing of them in the building was not insurer's liability. If the contract has been
Claveria, as a residence and bodega. The Midland a violation of the terms of the contract. terminated, by a violation of its terms on the part of
Textile Insurance Co. in consideration of the - Whether a particular article is "stored" or not must, the insured, there can be no recovery. Compliance
payment of a premium of P60, entered into a in some degree, depend upon the intention of the with the terms of the contract is a condition
contract of insurance with Young by the terms of parties. Nearly all of the cases cited by the lower precedent to the right of recovery.
which the company, upon certain conditions, court are cases where the article was being put to - Young argues that since the "storing" of the
promised to pay Young the sum of P3,000 in case some reasonable and actual use, which might easily fireworks on the premises did not contribute in any
said residence and bodega and contents should be have been permitted by the terms of the policy, and way to the damage occasioned by the fire, he should
destroyed by fire. within the intention of the parties, and excepted from be permitted to recover. That argument, however, is
- One of the conditions of the contract is: "Warranty the operation of the warranty, like the present. beside the question, if the "storing" was a violation
B – It is hereby declared and agreed that during the - (1) Where merchants have had or kept the of the terms of the contract. The violation of the
pendency of this policy no hazardous goods be "hazardous" articles in small quantities, and for terms of the contract, by virtue of the provisions of
stored or kept for sale, and no hazardous trade or actual daily use, for sale, such as gasoline, the policy itself, terminated, at the election of either
process be carried on, in the building to which this gunpowder, etc.; (2) Where such articles have been party, the contractual relations.
brought on the premises for actual use thereon, and
INSURANCE Page
72

- Young paid a premium based upon the risk at the current policy-year; provided interest at six per Japanese occupation on account of pre-war policies
time the policy was issued. Certainly, the placing of centum per annum on the whole amount of the for which reason they filed an amended answer
the firecrackers in the building insured increased the loan is paid in advance to the end of the current offering to pay plaintiff the amount of P9,468.29
risk. Young had not paid a premium based upon the policy-year. At the end of the current policy-year which represents the aggregate cash surrender
increased risk, neither had the defendant issued a interest at the same rate for one year in advance values of all the policies in question as of February
policy upon the theory of a different risk. He was will be due and payable, and annually thereafter, 10, 1949, but apparently this offer was refused.
enjoying, if his contention may be allowed, the and if not so paid will be added to the principal - CFI: (1) rescinded the insurance contracts; (2)
benefits of an insurance policy upon one risk, and bear the same rate of interest. Failure to ordered defendant Filipinas Life Assurance Co. to pay
whereas, as a matter of fact, it was issued upon an repay any such loan or interest shall not avoid this plaintiff the amount of P32,072.60; and (3) ordered
entirely different risk. The defendant had neither Policy unless the total indebtedness shall equal or defendant Insular Life Assurance Co., Ltd. to pay
been paid nor had issued a policy to cover the exceed the full amount of the loan value available plaintiff the amount of P2,574.00
increased risk. An increase of risk which is hereunder. - CA affirmed.
substantial and which is continued for a considerable Any indebtedness on this Policy shall first be
period of time, is a direct and certain injury to the deducted from any money payable or in any ISSUES
insurer, and changes the basis upon which the settlement under this Policy. 1. WON CA erred in ruling that as a consequence of
contract of insurance rests. - Nava had so far paid to Insular a total of P2,574; the decision in the Haw Pia case petitioners violated
Disposition Decision of the lower court is and to Filipinas Life, a total of P32,072.60. the loan clause contained in the insurance policies
REVERSED. - April 28, 1948: Nava applied to the companies for a thereby entitling respondent to their rescission
P5k loan in line with the loan clause, but they 2. WON CA erred in ruling that by virtue of Article
TAN v. CA (supra p.51) refused to grant it because certain regulations issued 1295 of the old Civil Code petitioners should refund
by the Insurance Commissioner required the to defendant all the premiums paid on his insurance
AREOLA v. CA (supra p.26) insurance companies to withhold the payments on policies as a consequence of their rescission
premiums made during the Japanese occupation 3. WON CA erred in not ruling that, even if
TAN CHAY v. WEST COAST (supra p.51) because the same shall be subject to future respondent is entitled to the rescission of said
adjustments " as soon as debtor-creditor relationship insurance policies, he can only recover their cash
FILIPINAS LIFE ASSURANCE v. NAVA is established" and because of such process of surrender value at the time the complaint was filed
17 SCRA 210 "withholding" plaintiff was not entitled to borrow any
BAUTISTA ANGELO; May 20, 1966 amount until such adjustment has been made. HELD
- Sept 30, 1948: Nava called the attention of the 1. NO.
NATURE insurance companies to the SC decision (Haw Pia v. - Even assuming the validity of the Insurance
Petition for review of a decision of the Court of China Banking Corporation) establishing and Commissioner’s regulations, the fact however is that
Appeals recognizing the relationship of debtor and creditor such requirement has already lost its legal effect and
with respect to payments in fiat currency made value when our Supreme Court rendered its decision
FACTS during the Japanese occupation on pre-war in the Haw Pia case wherein it was declared, among
- Before the war, Nava entered into a contract of obligations. others, that all payments made in fiat currency
insurance with Insular Life Assurance Co., Ltd. (face - Companies still refused saying that the SC decision during the Japanese occupation in relation with any
value of P5k), and 17 separate contracts of life was not applicable to transactions undertaken during contractual obligation executed before the war were
insurance with Filipinas Life Assurance Co. (total face Japanese occupation when they relate to life valid to all intents and purposes, and yet petitioners
value of P90k). Each and everyone of the 18 policies insurance policies. apparently did not give any importance to such
issued by defendants to plaintiff contains a loan - Feb 4, 1949: Nava was again refused even if the decision for in their opinion it does not have any
clause of the following tenor: total amount of the cash surrender values of the 18 application to transactions which have any relation to
Policy loans. After three full years' policies reached the sum of P9,468.29. payment of premiums on life insurance policies.
premiums have been paid upon this Policy, if no - Feb 10, 1949: Nava brought case to the CFI Manila - It cannot be denied that a life insurance policy
premium payment is in default, the Company, praying for the rescission of the abovementioned 18 involves a contractual obligation wherein the insured
subject to its then existing rules, will advance on policies and for the refund to him of all the premiums becomes duty bound to pay the premiums agreed
proper assignment and delivery of this Policy and so far paid by him to defendants in the amount of upon, lest he runs the risk of having his insurance
on the sole security thereof a sum equal to, or at P31,633.80, plus 6% interest thereon as damages policy lapse if he fails to pay such premiums.
the option of the owner less than, the cash value - Nov 28, 1951: companies passed a resolution - The fact that if the insured had paid in full the
specified in the Schedule of Policy Values, less any which was approved by the Insurance Commissioner, premiums corresponding to the first 3 years of the
existing indebtedness on or secured by this Policy giving full credit to all premium payments made by life of his policy he cannot be considered delinquent
and any unpaid balance of the premium for the their policyholders in fiat currency during the that would cause the lapse of his policy if the same
INSURANCE Page
73

contains an automatic premium payment clause shall supplement its deficiency. And said Article 1295 - Juan Bataclan rode Bus 30 of Medina Transport,
cannot divest such policy of its contractual nature, provides: driven by Saylon, shortly after midnight. The bus
for the result of such failure would only be for him to ART. 1295. Rescission makes necessary the return was running very fast. One of the front tires burst.
pay later the premium plus the corresponding of the things which were the subject-matter of the Bus fell into canal and turned turtle. 4 passengers
interest depending upon the condition of the policy. contract, with their fruits, and of the price paid, couldn’t get out, including Bataclan. Gasoline began
But certainly it does not cease to be a contractual with interest thereon. ...xxx to leak from the overturned bus. 10 men came to
liability insofar as the payment of that premium is - Said the petitioners: "Recovery of the full amount help. 1 carried a torch and when he approached the
concerned for whether he likes it or not that of the premium after the insurer has sustained for bus, fire started, killing the trapped passengers.
premium has to be paid lest he allows the lapse of sometime the risk of the insurance and the insured - TC opined that proximate cause of Bataclan’s
his policy. Consequently, the payment of premiums has enjoyed the benefit of protection is obviously death was not the overturning of bus but the fire. At
on the life insurance policies made by Nava before unjust and is so recognized by the better the time fire started, Bataclan, though injured, was
and during the war up to the time he applied for the authorities." The ruling above quoted merely still alive and damages were awarded, not for his
loan in question with petitioners should be represents the minority rule in the US, the majority death, but for physical injuries suffered.
considered likewise as valid payments upon the rule being that the insured can recover all premiums
theory that such insurance policies are in the nature paid, in some cases with interest in case of wrongful ISSUE/S
of a contractual obligation within the meaning of the cancellation, repudiation, termination or rescission of WON the proximate cause is the overturning of the
civil law. In effect, therefore, those payments were the contract of life insurance. bus or the fire
made by a debtor to a creditor within the meaning of - Contention that because respondent cannot restore
the requirement of the regulations of the Insurance to petitioners the "value of the benefit of protection" HELD
Commissioner and as such they can offer no excuse which he might have received under the 18 life - The proximate cause is the overturning of the bus.
to petitioners for refusing to grant the loan as insurance policies in question he is not entitled to - Ordinarily, when a bus overturns and pins down
contemplated in the loan clause embodied in the rescind them under the provision of Article 1295 of passenger, merely causing him injuries. If through
policies in question. the old Civil Code, is untenable because said article some event, unexpected and extraordinary, the bus
- It is clear from the foregoing that the petitioners only contemplates a transaction whether material is set on fire, and passenger is burned to death, one
violated the loan clause embodied in each of the 18 things are involved, and do not refer to intangible might contend that the proximate cause was the fire
life insurance policies issued to respondent to rescind ones which cannot be the subject of restoration, for and not the overturning of the vehicle.
all said policies under Section 69 of the Insurance to interpret it otherwise would be to defeat the law - But here, the proximate cause of Bataclan’s death
Act, which provides: "The violation of a material itself with the result that rescission can never be had is the overturning of the bus, this for the reason that
warranty, or other material provision of a policy, on under Section 69 of our Insurance Law. when the vehicle turned not only on its side but
the part of either party thereto, entitles the other to - It cannot be denied that petitioners had in turn completely on its back, leaking of gasoline from the
rescind." already derived material benefits from the use of tank was not unnatural or unexpected.
- "The general rule is that a breach of the agreement premiums paid to them by respondent before, during - The coming of the men with the torch was in
to make the loan does not entitle the insured to and after the last war from which they must have response to the call for help, made only not by the
rescind the contract," is not controlling in this realized huge profits, and in this light alone passengers but even the driver and conductor, and
jurisdiction. Firstly, it was not shown that the petitioners cannot claim prejudice or unfairness if because it was very dark, about 2:30 am, rescuers
insurance laws in the states where said ruling they are ordered to refund the premiums paid by had to carry a light with them. Coming as they did
prevails contain a provision identical to Section 69 of respondents. from rural area where lanterns and flashlights were
our Insurance Law we quoted above, and secondly, 3. NO. not available, they had to use a torch. What was
the rule cited by Vance is not a rule uniformly - Issue is corollary to preceding issue. No need to more natural than that said rescuers should
followed by all states in the US, for on this matter refute. innocently approach the overturned vehicle to extend
there is a marked divergence of opinion. Disposition Decision appealed from is AFFIRMED. aid.
2. NO Costs against petitioners - The coming of the men with the torch was to be
- Considering that our Insurance Law does not expected, and was a natural sequence of the
contain an express provision as to what the court CHAPTER VII. RISKS AND COVERAGES overturning of the bus, the trapping of some of its
should do in cases of rescission of an insurance passengers and the call for outside help.
policy under Section 69, the provision that should VDA. DE BATACLAN v. MEDINA - The burning of bus can also in part be attributed to
apply is that embodied in Article 1225 of the old Civil 102 PHIL 181 negligence of carrier, through its driver and
Code, as postulated in Article 16 of the same Code, MONTEMAYOR; October 22, 1957 conductor. They, or at least the driver, should have
which provides that on matters which are not known that in the position in which the overturned
governed by special laws the provisions of said Code FACTS bus was, gasoline could and must have leaked from
the gasoline tank and soaked the area in and around
INSURANCE Page
74

the bus. Gasoline can be smelt and detected even lone stab wound on the insured) [TF they cannot be another thing is therefore applicable in the instant
from a distance, and yet neither the driver nor the made to indemnify the Surposa heirs] case since murder and assault, not having been
conductor would appear to have cautioned or taken expressly included in the enumeration of the
steps to warn rescuers not to bring the lighted torch HELD circumstances that would negate liability in said
too near the bus. NO insurance policy: the failure of the FINMAN to include
- The record is barren of any circumstance showing death resulting from murder or assault among the
FINMAN GENERAL ASSURANCE CORPORATION how the stab wound was inflicted. While the act may prohibited risks leads inevitably to the conclusion
v. CA (SURPOSA) not exempt the unknown perpetrator from criminal that it did not intend to limit or exempt itself from
213 SCRA 493 liability, the fact remains that the happening was a liability for such death.
NOCON; September 2, 1992 pure accident on the part of the victim. The insured - A1377 NCC: The interpretation of obscure words
died from an event that took place without his or stipulations in a contract shall not favor the party
NATURE foresight or expectation, an event that proceeded who caused the obscurity.
Certiorari from an unusual effect of a known cause and, - NPC vs. CA [1986]~ It is well settled that
therefore, not expected. contracts of insurance are to be construed liberally in
FACTS Reasoning favor of the insured and strictly against the insurer.
- Oct. 22, 1986: Carlie Surposa was insured with - De la Cruz vs. Capital Insurance & Surety Co., Inc Thus ambiguity in the words of an insurance contract
Finman General Assurance Corporation under Finman (1966)~ The terms "accident" and "accidental" as should be interpreted in favor of its beneficiary.
General Teachers Protection Plan Master Policy No. used in insurance contracts have not acquired any Disposition DENIED for lack of merit.
2005 and Individual Policy No. 08924 with his technical meaning, and are construed by the courts
parents, spouses Julia and Carlos Surposa, and in their ordinary and common acceptation. Thus, the CALANOC v. CA
brothers Christopher, Charles, Chester and Clifton, terms have been taken to mean that which happen 98 PHIL 79
all surnamed, Surposa, as beneficiaries. by chance or fortuitously, without intention and BAUTISTA; December 16, 1955
- While said insurance policy was in full force and design, and which is unexpected, unusual, and
effect, the insured, Carlie Surposa, died on October unforeseen. An accident is an event that takes place FACTS
18, 1988 as a result of a stab wound inflicted by one without one's foresight or expectation an event that - Basilio was a watchman of the Manila Auto Supply
of the 3 unidentified men without provocation and proceeds from an unknown cause, or is an unusual located at the corner of Avenida Rizal and Zurbaran.
warning on the part of the former as he and his effect of a known cause and, therefore, not He secured a life insurance policy from the Philippine
cousin, Winston Surposa, were waiting for a ride on expected. American Life Insurance Company in the amount of
their way home after attending the celebration of the Ratio The generally accepted rule is that, death or P2,000 to which was attached a supplementary
"Maskarra Annual Festival." injury does not result from accident or accidental contract covering death by accident. On January 25,
- Thereafter, Julia Surposa and the other means within the terms of an accident-policy if it is 1951, he died of a gunshot wound on the occasion of
beneficiaries of said insurance policy filed a written the natural result of the insured's voluntary act, a robbery committed in the house of Atty. Ojeda at
notice of claim with the FINMAN Corp which denied unaccompanied by anything unforeseen except the the corner of Oroquieta and Zurbaran streets.
said claim contending that murder and assault are death or injury. There is no accident when a Calanoc, the widow, was paid the sum of P2,000,
not within the scope of the coverage of the insurance deliberate act is performed unless some additional, face value of the policy, but when she demanded the
policy. unexpected, independent, and unforeseen happening payment of the additional sum of P2,000
- Feb. 24, 1989: Surposa filed a complaint with the occurs which produces or brings about the result of representing the value of the supplemental policy,
Insurance Commission which subsequently ordered injury or death. In other words, where the death or the company refused alleging, as main defense, that
FINMAN to pay Surposa the proceeds of the policy injury is not the natural or probable result of the the deceased died because he was murdered by a
with interest. insured's voluntary act, or if something unforeseen person who took part in the commission of the
- CA affirmed said decision. occurs in the doing of the act which produces the robbery and while making an arrest as an officer of
injury, the resulting death is within the protection of the law which contingencies were expressly excluded
ISSUE the policies insuring against death or injury from in the contract and have the effect of exempting the
WON CA committed GAD in applying the principle of accident. company from liability.
"expresso unius exclusio alterius" in a personal - The personal accident insurance policy involved - It is contended in behalf of the company that
accident insurance policy (since death resulting from herein specifically enumerated only 10 circumstances Basilio was killed which "making an arrest as an
murder and/or assault are impliedly excluded in said wherein no liability attaches to FINMAN for any officer of the law" or as a result of an "assault or
insurance policy considering that the cause of death injury, disability or loss suffered by the insured as a murder" committed in the place and therefore his
of the insured was not accidental but rather a result of any of the stimulated causes. death was caused by one of the risks excluded by
deliberate and intentional act of the assailant in -The principle of " expresso unius exclusio alterius" the supplementary contract which exempts the
killing the former as indicated by the location of the the mention of one thing implies the exclusion of
INSURANCE Page
75

company from liability. This contention was upheld said that the killing was intentional for there is the might offer, it cannot be denied that the act itself of
by the Court of Appeals. Hence, this petition. possibility that the malefactor had fired the shot inflicting the injuries was intentional.
merely to scare away the people around for his own - The exception in the accidental benefit clause
ISSUE protection and not necessarily to kill or hit the invoked by the appellant does not speak of the
WON the death of the victim comes within the victim. In any event, while the act may not exempt purpose — whether homicidal or not — of a third
purview of the exception clause of the the triggerman from liability for the damage done, party in causing the injuries, but only of the fact that
supplementary policy and, hence, exempts the the fact remains that the happening was a pure such injuries have been "intentionally" inflicted —
company from liability accident on the part of the victim. The victim could this obviously to distinguish them from injuries
have been either the policeman or Atty. Ojeda for it which, although received at the hands of a third
HELD cannot be pretended that the malefactor aimed at party, are purely accidental.
NO the deceased precisely because he wanted to take - Examples of unintentional:
- Basilio was a watchman of the Manila Auto Supply his life. >> A gun which discharges while being cleaned and
which was a block away from the house of Atty. Disposition Decision set aside kills a bystander;
Ojeda where something suspicious was happening >> a hunter who shoots at his prey and hits a
which caused the latter to ask for help. While at first BIAGTAN v. THE INSULAR LIFE ASSURANCE person instead;
he declined the invitation of Atty. Ojeda to go with COMPANY, LTD. >> an athlete in a competitive game involving
him to his residence to inquire into what was going 44 SCRA 58 physical effort who collides with an opponent and
on because he was not a regular policeman, he later MAKALINTAL; March 29, 1972 fatally injures him as a result.
agreed to come along when prompted by the traffic - In Calanoc vs. CA: Where a shot was fired and it
policeman, and upon approaching the gate of the NATURE turned out afterwards that the watchman was hit in
residence he was shot and died. The circumstance Appeal from decision of CFI Pangasinan. the abdomen, the wound causing his death, the
that he was a mere watchman and had no duty to Court held that it could not be said that the killing
heed the call of Atty. Ojeda should not be taken as a FACTS was intentional for there was the possibility that the
capricious desire on his part to expose his life to - Juan Biagtan was insured with Insular for P5k and malefactor had fired the shot to scare the people
danger considering the fact that the place he was in a supplementary contract “Accidental Death Benefit” around for his own protection and not necessarily to
duty-bound to guard was only a block away. In clause for another P5k if "the death of the Insured kill or hit the victim. A similar possibility is clearly
volunteering to extend help under the situation, he resulted directly from bodily injury effected solely ruled out by the facts in this case. For while a single
might have thought, rightly or wrongly, that to know through external and violent means sustained in an shot fired from a distance, and by a person who was
the truth was in the interest of his employer it being accident . . . and independently of all other causes." not even seen aiming at the victim, could indeed
a matter that affects the security of the The clause, however, expressly provided that it have been fired without intent to kill or injure, nine
neighborhood. No doubt there was some risk coming would not apply where death resulted from an injury wounds inflicted with bladed weapons at close range
to him in pursuing that errand, but that risk always "intentionally inflicted by a third party." cannot conceivably be considered as innocent insofar
existed it being inherent in the position he was - One night, a band of robbers entered their house. as such intent is concerned.
holding. He cannot therefore be blamed solely for Juan went out of his room and he was met with 9 - In Hucthcraft's Ex'r vs. Travelers' Ins. Co. (US
doing what he believed was in keeping with his duty knife stabs. He died. The robbers were convicted of case): where the insured was waylaid and
as a watchman and as a citizen. And he cannot be robbery with homicide. assassinated for the purpose of robbery, the court
considered as making an arrest as an officer of the - The family was claiming the additional P5k from rendered judgment for the insurance company and
law, as contended, simply because he went with the Insular, under the Accidental Death Benefit clause. held that while the assassination of the insured was
traffic policeman, for certainly he did not go there for Insular refused on the ground that the death resulted as to him an unforeseen event and therefore
that purpose nor was he asked to do so by the from injuries intentionally inflicted by 3rd parties and accidental, "the clause of the proviso that excludes
policeman. was therefore not covered. Biagtans filed against the (insurer's) liability, in case death or injury is
- Much less can it be pretended that Basilio died in Insular. CFI ruled in favor of Biagtans. intentionally inflicted by any other person, applies to
the course of an assault or murder considering the this case."
very nature of these crimes. In the first place, there ISSUE Disposition CFI decision reversed.
is no proof that the death of Basilio is the result of WON the injuries were intentionally inflicted
either crime for the record is barren of any SEPARATE OPINION
circumstance showing how the fatal shot was fired. HELD
Perhaps this may be clarified in the criminal case YES TEEHANKEE [dissent]
now pending in court as regards the incident but - Whether the robbers had the intent to kill or merely - Calanoc v. CA is controlling in this case because
before that is done anything that might be said on to scare the victim or to ward off any defense he the insurance company wasn’t able to prove that the
the point would be a mere conjecture. Nor can it be
INSURANCE Page
76

killing was intentional. (Burden of proof is with the FACTS - The petitioner, however, cites one of the four
insurance company) - The petitioner issued Personal Accident Policy to exceptions provided for in the insurance contract and
- Insurance, being contracts of adhesion, must be Felix Lim, Jr. with a face value of P200,000.00. Two contends that the private petitioner's claim is barred
construed strictly against insurance company in months later, he was dead with a bullet wound in his by such provision. It is there stated:
cases of ambiguity. head. As beneficiary, his wife Nerissa Lim sought Exceptions —The company shall not be liable in
- The supplementary contract enumerated payment on the policy but her claim was rejected. respect of.
exceptions. The only exception which is not The petitioner agreed that there was no suicide. It 1. Bodily injury.
susceptible of classification is that provided in argued, however, that there was no accident either. xxx xxx xxx
paragraph 5(e), the very exception herein involved, Pilar Nalagon, Lim's secretary, was the only b. consequent upon.
which would also except injuries "inflicted eyewitness to his death. According to Nalagon, Lim i) The insured persons attempting to commit
intentionally by a third party, either with or without was in a happy mood (but not drunk) and was suicide or wilfully exposing himself to needless peril
provocation on the part of the insured, and whether playing with his handgun, from which he had except in an attempt to save human life.
or not the attack or the defense by the third party previously removed the magazine. As she watched - To repeat, the parties agree that Lim did not
was caused by a violation of the law by the insured." the television, he stood in front of her and pointed commit suicide. Nevertheless, the petitioner
- This ambiguous clause conflicts with all the other the gun at her. She pushed it aside and said it might contends that the insured willfully exposed himself to
four exceptions in the same paragraph 5 particularly be loaded. He assured her it was not and then needless peril and thus removed himself from the
that immediately preceding it in item (d) which pointed it to his temple. The next moment there was coverage of the insurance policy. That posture is
excepts injuries received where the insured has an explosion and Lim slumped to the floor. He was arguable. But what is not is that, as the secretary
violated the law or provoked the injury, while this dead before he fell. testified, Lim had removed the magazine from the
clause, construed as the insurance company now - The term "accident" has been defined as follows: gun and believed it was no longer dangerous. He
claims, would seemingly except also all other The words "accident" and "accidental" have never expressed assured her that the gun was not loaded.
injuries, intentionally inflicted by a third party, acquired any technical signification in law, and It is submitted that Lim did not willfully expose
regardless of any violation of law or provocation by when used in an insurance contract are to be himself to needless peril when he pointed the gun to
the insured, and defeat the very purpose of the construed and considered according to the his temple because the fact is that he thought it was
policy of giving the insured double indemnity in case ordinary understanding and common usage and not unsafe to do so. The act was precisely intended
of accidental death by "external and violent means" speech of people generally. In substance, the to assure Nalagon that the gun was indeed harmless.
— in the very language of the policy.' courts are practically agreed that the words Disposition CA Affirmed
- It is obvious from the very classification of the "accident" and "accidental" mean that which
exceptions and applying the rule of noscitus a sociis, happens by change or fortuitously, without DE LA CRUZ v. CAPITAL INSURANCE
that the double-indemnity policy covers the insured intention or design, and which is unexpected, 17 SCRA 554
against accidental death, whether caused by fault, unusual, and unforeseen. The definition that has BARRERA; June 30, 1966
negligence or intent of a third party which is usually been adopted by the courts is that an
unforeseen and unexpected by the insured. All the accident is an event that takes place without one's NATURE
associated words and concepts in the policy plainly foresight or expectation — an event that proceeds Appeal from the decision of the CFI of Pangasinan
exclude the accidental death from the coverage of from an unknown cause, or is an unusual effect of
the policy only where the injuries are self-inflicted or a known case, and therefore not expected. FACTS
attended by some proscribed act of the insured or - An accident is an event which happens without any - Eduardo de la Cruz, employed in the Itogon-Suyoc
are incurred in some expressly excluded calamity human agency or, if happening through human Mines, Inc., was the holder of an accident insurance
such as riot, war or atomic explosion. agency, an event which, under the circumstances, is policy underwritten by the Capital Insurance &
- The untenability of insurer's claim that the unusual to and not expected by the person to whom Surety Co., Inc., for the period beginning November
insured's death fell within the exception is further it happens. It has also been defined as an injury 13, 1956 to November 12, 1957.
heightened by the stipulated fact that two other which happens by reason of some violence or - On January 1, 1957, the Itogon-Suyoc Mines, Inc.
insurance companies which likewise covered the casualty to the insured without his design, consent, sponsored a boxing contest wherein the insured
insured for much larger sums under similar or voluntary co-operation. Eduardo de la Cruz participated.
accidental death benefit clauses promptly paid the - In the course of his bout, Eduardo slipped and was
benefits thereof to plaintiffs beneficiaries. ISSUE hit by his opponent on the left part of the back of the
WON what happened was an accident head, causing Eduardo to fall, with his head hitting
SUN INSURANCE v. CA (LIM) the rope of the ring.
211 SCRA 554 HELD - He was brought to the Baguio General Hospital, but
CRUZ; July 17, 1992 YES he died as a result of hemorrhage, intracranial, left.
INSURANCE Page
77

- Simon de la Cruz, the father and named beneficiary - The fact that boxing is attended with some risks of - SS Eastern Explorer was then found to be a
of the insured, filed a claim with the insurance external injuries does not make any injuries received constructive total loss and its voyage was declared
company for payment of the indemnity, but it was in the course of the game not accidental. In boxing abandoned.
denied. as in other equally physically rigorous sports, such as - After the fire was extinguished, the cargoes which
- He instituted the action in the CFI of Pangasinan for basketball or baseball, death is not ordinarily were saved were loaded to another vessel for
specific performance. anticipated to result. If, therefore, it ever does, the delivery to their original ports of destination. ESLI
- Defendant insurer set up the defense that the injury or death can only be accidental or produced by charged the consignees several amounts
death of the insured, caused by his participation in a some unforeseen happening or event as what corresponding to additional freight and salvage
boxing contest, was not accidental and, therefore, occurred in this case. charges.
not covered by insurance - Furthermore, the policy involved herein specifically - The charges were all paid by Philippine Home
- The court rendered the decision in favor of the excluded from its coverage: Assurance Corporation (PHAC) under protest for and
plaintiff, hence, the present appeal. (e) Death or disablement consequent upon the in behalf of the consignees. PHAC, as subrogee of
Insured engaging in football, hunting, pigsticking, the consignees, then filed a complaint before the RTC
ISSUE steeplechasing, polo-playing, racing of any kind, of Manila, against ESLI to recover the sum paid
WON the death of the insured was not accidental mountaineering, or motorcycling. under protest on the ground that the same were
and, therefore, not covered by insurance - Death or disablement resulting from engagement in actually damages directly brought about by the fault,
HELD boxing contests was not declared outside of the negligence, illegal act and/or breach of contract of
NO protection of the insurance contract. Failure of the ESLI.
- The terms "accident" and "accidental", as used in defendant insurance company to include death - ESLI contended that it exercised the diligence
insurance contracts, have not acquired any technical resulting from a boxing match or other sports among required by law in the handling, custody and carriage
meaning, and are construed by the courts in their the prohibitive risks leads inevitably to the of the shipment; that the fire was caused by an
ordinary and common acceptation. Thus, the terms conclusion that it did not intend to limit or exempt unforeseen event; that the additional freight charges
have been taken to mean that which happen by itself from liability for such death. are due and demandable pursuant to the Bill of
chance or fortuitously, without intention and design, Disposition The decision appealed from is affirmed Lading; and that salvage charges are properly
and which is unexpected, unusual, and unforeseen. collectible under Act No. 2616, known as the Salvage
An accident is an event that takes place without FORTUNE INSURANCE v. CA (supra p.7) Law.
one's foresight or expectation, an event that - RTC: dismissed PHAC's complaint and ruled in
proceeds from an unknown cause, or is an unusual PHIL HOME ASSURANCE CORP v. CA (EASTERN favor of ESLI.
effect of a known cause and, therefore, not SHIPPING) - The burning of the vessel was not the fault or
expected. 257 SCRA 468 negligence of defendant but a natural disaster or
- The generally accepted rule is that, death or injury KAPUNAN; June 20, 1996 calamity. Salvage operations conducted by Fukuda
does not result from accident or accidental means Salvage Company was perfectly a legal operation
within the terms of an accident-policy if it is the NATURE and charges made on the goods recovered were
natural result of the insured's voluntary act, - Eastern Shipping Lines, Inc. loaded on board SS legitimate charges. Section 19 of Act No. 2616, the
unaccompanied by anything unforeseen except the Eastern Explorer in Kobe, Japan, the following Salvage Law is applicable. With respect to the
death or injury. There is no accident when a shipment for carriage to Manila and Cebu, freight additional freight charged by defendant from the
deliberate act is performed unless some additional, pre-paid and in good order and condition: (a) 2 consignees of the goods, the same are also validly
unexpected, independent, and unforeseen happening boxes internal combustion engine parts, consigned to demandable.
occurs which produces or brings about the result of William Lines, Inc.; (b) 10 metric tons (334 bags) - The burning of "EASTERN EXPLORER" while off
injury or death. In other words, where the death or ammonium chloride, consigned to Orca's Company; Okinawa rendered it physically impossible for
injury is not the natural or probable result of the (c) 200 bags Glue 300, consigned to Pan Oriental defendant to comply with its obligation of delivering
insured's voluntary act, or if something unforeseen Match Company; and (d) garments, consigned to the goods to their port of destination pursuant to the
occurs in the doing of the act which produces the Ding Velayo. All consignations were made by virtue contract of carriage. Under Article 1266 of the Civil
injury, the resulting death is within the protection of of a Bill of Lading. Code, the physical impossibility of the prestation
policies insuring against death or injury from - While the vessel was off Okinawa, a small flame extinguished defendant's obligation.
accident. was detected on the acetylene cylinder located in the
9
- In the present case, while the participation of the accommodation area near the engine room. As the Section 1. When in case of shipwreck, the vessel or its cargo shall
insured in the boxing contest is voluntary, the injury crew was trying to extinguish the fire, the cylinder be beyond the control of the crew, or shall have been abandoned by
them, and picked up and conveyed to a safe place by other persons,
was sustained when he slid, giving occasion to the suddenly exploded, thus causing death and severe the latter shall be entitled to a reward for the salvage.
infliction by his opponent of the blow that threw him injuries to the crew and instantly setting fire to the Those who, not being included in the above paragraph, assist in saving
to the ropes of the ring. whole vessel. a vessel or its cargo from shipwreck, shall be entitled to like reward.
INSURANCE Page
78

- Note: The goods subject of the present tested and examined and subsequently certified as $610,998.63 and made subsequent to July 16, 1959.
controversy were neither lost nor damaged in transit having complied with the safety measures and Philamlife filed a claim for refund on the ground that
by the fire that razed the carrier. In fact, these were standards by qualified experts before it was loaded in the reinsurance premiums remitted were paid in
all delivered to the consignees, even if the the vessel only shows to a great extent that pursuant to the January 1950 reinsurance treaty,
transshipment took longer than necessary. What is negligence was present in the handling of the and therefore exempted.
at issue therefore is not whether or not the carrier is acetylene cylinder after it was loaded and while it - Monetary Board exempted Philamlife from payment
liable for the loss, damage, or deterioration of the was on board the ship. of margin fee. However, Auditor of CB refused to
goods transported by them but who, among the - From the foregoing premises, it indubitably follows pass in audit Philamlife’s claim for refund. Philamlife
carrier, consignee or insurer of the goods, is liable that the cargo consignees cannot be made liable to sought reconsideration but was denied, saying
for the additional charges or expenses incurred by respondent carrier for additional freight and salvage reinsurance treaty NOT EXEMPTED.
the owner of the ship in the salvage operations and charges.
in the transshipment of the goods via a different Disposition Judgment appealed from is REVERSED ISSUES
carrier. In absolving respondent carrier of any and SET ASIDE. Respondent Eastern Shipping Lines, 1. WON the premia remitted were in pursuance of
liability, CA sustained the trial court's finding that the Inc. is ORDERED to return to petitioner Philippine the reinsurance treaty between Philamlife and Airco
fire that gutted the ship was a natural disaster or Home Assurance Corporation the amount it paid of January 1959, a contract antedating the Margin
calamity. under protest in behalf of the consignees herein. Law, and therefore, Philamlife exempted from paying
margin fee
ISSUE 2. WON Margin Law impairs the obligation of contract
WON the burning of the SS Eastern Explorer 3. WON reinsurance contracts abroad would be made
rendering it a constructive total loss was a natural impractical by the imposition of the 25% margin fee
disaster or calamity
HELD
HELD 1. NO
NO - For an exemption to come into play, there must be
Ratio In our jurisprudence, fire may not be PHILIPPINE AMERICAN LIFE INSURANCE a reinsurance policy or, as in the reinsurance treaty
considered a natural disaster or calamity since it COMPANY v. THE AUDITOR GENERAL provided, a "reinsurance cession" which may be
almost always arises from some act of man or by 22 SCRA 135 automatic or facultative.
human means. It cannot be an act of God unless SANCHEZ, JANUARY 18, 1968 Ratio A reinsurance policy is thus a contract of
caused by lightning or a natural disaster or casualty indemnity one insurer makes with another to protect
not attributable to human agency. NATURE the first insurer from a risk it has already assumed. .
Reasoning PETITION FOR REVIEW of a ruling of the Auditor . . In contradistinction, a reinsurance treaty is
- There was no showing, and none was alleged by General. merely an agreement between two insurance
the parties, that the fire was caused by a natural companies whereby one agrees to cede and the
disaster. On the contrary, there is strong evidence FACTS other to accept reinsurance business pursuant to
indicating that the acetylene cylinder caught fire - Philamlife, a domestic life insurance corp., and provisions specified in the treaty. The practice of
because of the fault and negligence of respondent American International Reinsurance Company issuing policies by insurance companies includes,
ESLI, its captain and its crew: (Airco), a corporation organized under the laws of among other things, the issuance of reinsurance
(1) The acetylene cylinder which was fully loaded the Republic of Panama, entered into a policies on standard risks and also on substandard
should not have been stored near the engine room REINSURANCE TREATY wherein Philamlife agrees to risks under special arrangements. The lumping of the
where the heat generated therefrom could cause the reinsure with Airco on January 1950. Philamlife different agreements under a contract has resulted in
acetylene cylinder to explode by reason of agreed to pay premiums for all reinsurances on an the term known to the insurance world as 'treaties.'
spontaneous combustion. ESLI should have easily annual premium basis. Such a treaty is, in fact, an agreement between
foreseen that the acetylene cylinder, containing - In July 16, 1959, the Margin Law was approved and insurance companies to cover the different situations
highly inflammable material, was in a real danger of became effective, which exempts certain “obligations described. Reinsurance treaties and reinsurance
exploding. from payment of margin fees, particularly contractual policies are not synonymous. Treaties are
(2) ESLI should have known that by storing the obligations calling for payment of foreign exchange contracts for insurance; reinsurance policies or
acetylene cylinder in the accommodation area issued, approved and outstanding as of the date this cessions are contracts of insurance.
supposed to be reserved for passengers, it Act takes place”. Reasoning
unnecessarily exposed its passengers to grave - Central Bank of the Philippines collected - Even if reinsurance treaty preceded the Margin Law
danger and injury. P268,747.48 as foreign exchange margin on by over nine years, nothing in the treaty obligates
(3) The fact that the acetylene cylinder was checked, Philamlife remittances to Airco purportedly totalling Philamlife to remit to Airco a fixed, certain, and
INSURANCE Page
79

obligatory sum by way of reinsurance premiums. The General of October 24, 1961 denying refund is liability when the insured property was burned on
reinsurance treaty per se cannot give rise to a hereby affirmed. February 16, 1962. Since the policy was issued on
contractual obligation for the payment of foreign Costs against petitioner. So ordered. July 1, 1961, it was supposed to expire on July 1,
exchange. Philamlife’s obligation to remit reinsurance 1962. 2 The next day, Feb. 17, ASIAN immediately
premiums becomes fixed and definite upon the FIELDMEN'S INSURANCE CO INC v. ASIAN notified FIELDMEN'S of said fire loss.
execution of the reinsurance cession. It is only SURETY & INSURANCE CO INC - FIELDMEN'S, relying on the sufficiency of its notice
after a reinsurance cession is made that payment of 34 SCRA 36 of termination dated September 19, 1961 and
reinsurance premium may be exacted, as it is only MAKALINTAL; July 31, 1970 obviously bent on avoiding its liability under the
after Philamlife seeks to remit that reinsurance reinsurance agreements with ASIAN, filed a petition
premium that the obligation to pay the margin fee FACTS for declaratory relief with the CFI of Manila to seek a
arises. - On various dates between April 11, 1960 and Jan. declaration that all the reinsurance contracts entered
2. NO 9, 1961 the Asian Surety & Insurance Company, into between them had terminated as of December
Ratio. Existing laws form part of the contract "as the Inc. and the Fieldmen's Insurance Company, Inc. 31, 1961 and to obtain an order directing ASIAN to
measure of the obligation to perform them by the entered into 7 reinsurance agreements under which render final accounting of the transactions between
one party and the right acquired by the other. If the the former, as the ceding company undertook to them with respect to said reinsurance treaties as of
obligation does not inhere and subsist in the cede to the latter, as the reinsuring company, a the cut-off date.
contract itself, propio vigore, but in the law specified portion of the amount of insurance - In its answer below ASIAN denied having received
applicable to the contract. underwritten by ASIAN upon payment to FIELDMEN'S FIELDMEN'S letter dated Sep 19, 1961, and argued
of a proportionate share of the gross rate of the that even assuming it did, FIELDMEN'S could not
premium applicable with respect to each cession have terminated the reinsurance treaties as of Dec
after deducting a commission. Said agreements were 31, 1961 because the letter was merely an
Reasoning to take effect from certain specific dates and were to expression of FIELDMEN'S desire to cancel the
- . When petitioner entered into the reinsurance be in force until cancelled by either party upon treaties and not a formal notice of cancellation as
treaty of January 1, 1950 with Airco, it did so with previous notice of at least 3 months by registered contemplated in their reinsurance agreements. By
the understanding that the municipal laws of the mail to the other party, the cancellation to take way of special defense Asian contended that even if
Philippines at the time said treaty was executed, effect as of Dec. 31 of the year in which the notice the Sep. 19 letter were considered sufficient notice of
became an unwritten condition thereof. Such was given. cancellation — thereby rendering the reinsurance
municipal laws constitute part of the obligation of - On Sep. 19, 1961 FIELDMEN'S, by means of agreements terminated as of December 31, 1961 —
contract. registered mail, served notice to ASIAN of the the liability of FIELDMEN'S with respect to policies or
-Rationale of Margin Law: to reduce the excessive former's desire to be relieved from all participation in cessions issued under two of the said agreements
demand on and prevent further decline of our its various agreements with the latter effective Dec. prior to their cancellation continued to have full force
international reserves; to provide the Central Bank 31, 1961. This communication, although admittedly and effect until the stated expiry dates of such
with an additional instrument for effectively coping received by ASIAN on Sep. 25, 1961, did not elicit policies or cessions.
with the problem and achieving domestic and any reply from ASIAN. - On Dec. 4, 1962, the trial court declared 6 of the 7
international stability of our currency; to reduce the - On Dec. 7, 1961 FIELDMEN'S sent another letter to reinsurance agreements in question cancelled as of
excessive demand-for foreign exchange. ASIAN expressing regrets at alleged violations Dec 31, 1961. At the same time, it upheld ASIAN'S
- implementation of Margin Law in accordance with committed by the latter with respect to the various position that all cessions of reinsurance made by it to
police power agreements between them and reiterated its position FIELDMEN'S prior to the cancellation of the
3. NO that it would consider itself "no longer at risk for any reinsurance treaties continued in full force and effect
Reasoning reinsurance and/or cession" given by ASIAN which until expiry dates and ordered FIELDMEN'S to make
- First, there is no concrete evidence that such might be in force on Dec. 31, 1961. Not having an accounting of its business transactions with
imposition of the 25% margin fee is unreasonable, received any formal reply from ASIAN, FIELDMEN'S ASIAN within 30 days.
Second, if really continuance of the existing sent a new a letter on Feb. 17, 1962 reminding - On appeal to the CA, the decision of the trial court
reinsurance treaty becomes unbearable, that ASIAN of the cancellation of all the reinsurance was substantially affirmed, with the slight
contract itself provides that petitioner may treaties and cessions as of Dec. 31, 1961 and modification that the order for accounting was
potestatively write finis thereto on ninety days' requested ASIAN to submit its final accounting of all eliminated, without prejudice to the filing of a proper
written notice. Petitioner is not forced to continue cessions made to the former for the preceding action between the parties for that purpose.
its reinsurance treaty indefinitely with Airco. months when the reinsurance agreements were in
Disposition For the reasons given, the petition for force. ISSUE
review is hereby denied, and the ruling of the Auditor - Meanwhile one of the risks reinsured with WON the cancellation as of Dec. 31, 1961 of the
FIELDMEN'S issued in favor of the GSIS became a reinsurance treaties had the effect of terminating
INSURANCE Page
80

also the liability of FIELDMEN'S as reinsurer with definite terminology in the reinsurance agreements
respect to policies or cessions issued prior to the which the parties may enter into henceforth. ISSUES
termination of the principal reinsurance contracts or 1. WON Equitable had no cause of action as the
treaties EQUITABLE INSURANCE v. RURAL INSURANCE matter was not referred to the decision of arbitrators
4 SCRA 343 2. WON in a facultative obligation the right to choose
HELD BARRERA; January 31, 1962 an alternative remedy lies only with the debtor (here
NO to the 2 reinsurance contracts the defendant) under Art 1206
- Of the 6 reinsurance contracts, 2 contain FACTS
provisions, which clearly and expressly recognize the - Plaintiff Equitable Insurance file a complaint with HELD
continuing effectivity of policies ceded under them the CFI of Manila against defendant Rural Insurance 1. NO
for reinsurance notwithstanding the cancellation of alleging, as first cause of action, that they entered - The requirement of submission for decision to 2
the contracts themselves. The said treaties provide into a reciprocal facultative reinsurance agreement, arbitrators or an umpire the matter of losses by fire
"that in the event of termination of this wherein they agreed to cede to each other. Pursuant or the liability of the parties thereto under Art VIII of
Agreement . . ., the liability of the Fieldmen's under to said agreement, plaintiff reinsured for P2k with the agreement arises only if the same is disputed by
current cessions shall continue in full force and effect defendant the stock covered by fire insurance Policy one of the parties. In the instant case, there is no
until their natural expiry . . .;" and the 4th paragraph No. 5880 issued by plaintiff which was later burned; dispute between the parties; in the stipulation of
of Article VI of the Personal Accident Reinsurance the share of the loss of defendant as per insurance facts defendant admitted that plaintiff had paid its
Treaty states: agreement was computed at P2,024 for which liability and defendant likewise admitted that it
"4. On the termination of this Agreement from plaintiff sent to defendant a statement of account for ignored plaintiff’s demands for reimbursement for
any cause whatever, the liability of the REINSURER payment by the latter. Despite repeated demands defendant’s failure to pay its share as reinsurer. As
(Fieldmen's) under any current cession including by plaintiff, defendant refused to pay. held in Maligad v United Assurance Co., if in the
any amounts due to be ceded under the terms of - On the second cause of action, plaintiff reinsured course of the settlement of a loss, the action of the
this Agreement and which are not cancelled in the for P2k with defendant stock covered by fire company or its agents amounts to refusal to pay, the
ordinary course of business shall continue in full insurance Policy No. 6062 which also burned. Again, company will be deemed to have waived the
force until their expiry unless the COMPANY defendant refused to pay its share of the loss of condition precedent with reference to arbitration and
(Asian) shall, prior to the thirty-first December P1,334 hence said complaint. a suit upon the policy will lie.
next following such notice, elect to withdraw the - Defendant filed a motion to dismiss on the ground 2. NO
existing cessions . . ." that it states no cause of action, as pursuant to Art - There is no connection between Art 1206 NCC and
- Thus, insofar as the two reinsurance agreements VIII of the Reinsurance Agreement between the the agreement of this action. The term “facultative”
are concerned, there is clearly no merit in parties, before a court action can be brought, the is used in reinsurance contracts, and it is so used in
FIELDMEN'S claim that their cancellation carried with parties agreed to submit all disputes to a board of this particular case, merely to define the right of the
it ipso facto the termination of all reinsurance arbitrators. The Court denied the motion and reinsurer to accept or not to accept participation in
cessions thereunder. Such cessions continued to be required defendant to answer. the risk insured. But once the share is accepted, as it
in force until their respective dates of expiration. - Defendant filed its answer, alleging that the nature was in the case at bar, the obligation is absolute and
Since it was under one of said agreements that the of the agreement is “self-liquidating between the the liability assumed thereunder can be discharged
reinsurance cession corresponding to the GSIS policy parties”, the reinsurer becoming the reinsured and by only one way—the payment of the share of the
had been made, FIELDMEN'S cannot avoid liability vice versa; and that said agreement has not yet losses.
which arose by reason of the burning of the insured been abrogated so the liability of either to the other Disposition judgment appealed from the TC is
property. is not yet known. Defendant prayed that the affirmed
- With respect to the other 4 agreements, it would complaint be dismissed and plaintiff filed a motion
seem that the petition for declaratory relief is moot, for judgment on the pleadings which the court ARTEX DEVELOPMENT CO INC v. WELLINGTON
and that no useful purpose would be served by denied. INSURANCE CO INC
defining the respective rights and obligations of the - Instead of going into a formal hearing, the parties 51 SCRA 352
parties thereunder. The said agreements have been submitted their case for decision stipulating the ff TEEHANKEE; June 27, 1973
cancelled, and it does not appear that any claim by facts: defendant admits the allegations of the
or liability in favor of the insured has actually arisen complaint and that plaintiff admits that the issues of FACTS
under any of the reinsurance cessions made prior to the complaint were not submitted to a Board of - Wellington Insurance Co. Inc. insured for
such cancellation. Future conflicts of the same nature Arbitrators as provided in par VIII of the complaint, P24,346,509.00 the buildings, stocks and machinery
as those which have motivated the present action but instead referred it to the Insurance of plaintiff Artex Development Co. Inc. against loss
can of course be obviated by using more precise and Commissioner. The CFI rendered judgment in favor or damage by fire or lighting upon payment of the
of plaintiff. Hence this appeal. plaintiff of the corresponding premiums; that said
INSURANCE Page
81

properties were insured for an additional sum of PERLA COMPANIA DE SEGUROS v. CA(LIM) other laws or regulations, to drive the Scheduled
P883,034.00; that defendant insured plaintiff against 208 SCRA 487 Vehicle, or has been permitted and is not
business interruption (use and occupancy) for NOCON; May 7, 1992 disqualified by order of a Court of Law or by
P5,200,000.00; Wellington entered into a contract reason of any enactment or regulation in that
of reinsurance with Alexander and Alexander, Inc. of NATURE behalf."
New York. USA. Petition for certiorari by Perla Compania de Seguros - On Nov17, 1982, private respondents requested
- The buildings, stocks and machineries of plaintiffs and FOC Credit Corporation seeking to annul and set from petitioner FCP for a suspension of payment on
spinning department were burned. aside CA decision revering the RTC decision for the monthly amortization agreed upon due to the
- Notice of the loss and damage was given the replevin and damages. loss of the vehicle and, since the carnapped vehicle
defendant; that as per report of the adjusters, the was insured with petitioner Perla, said insurance
total property loss of the plaintiff was the sum of FACTS company should be made to pay the remaining
P10,106,554.40 and the total business interruption - Private respondents spouses Herminio and Evelyn balance of the promissory note and the chattel
loss was P3,000,000.00; Lim executed a promissory note in favor of mortgage contract.
- That defendant has paid to the plaintiff the sum of Supercars, Inc. in the sum of P77,940.00, payable in - Perla, however, denied private respondents' claim.
P6,481,870.07 of the property loss suffered by monthly installments according to the schedule of Consequently, petitioner FCP demanded that private
plaintiff and P1,864,134.08 on its business payment indicated in said note, and secured by a respondents pay the whole balance of the promissory
interruption loss, leaving a balance of P3,624,683.43 chattel mortgage over a brand new red Ford Laser, note or to return the vehicle but the latter refused.
and P1,748,460.00, respectively. which is registered under the name of private - On July25, 1983, petitioner FCP filed a complaint
- The counsel for Artex filed a Manifestation saying respondent Herminio Lim and insured with the against private respondents, who in turn filed an
that in view of the Deeds of Discharge and Collateral petitioner Perla Compania de Seguros, Inc. (Perla for amended third party complaint against petitioner
Agreement, the only remaining liability subject of brevity) for comprehensive coverage. Perla on Dec8, 1983. After trial on the merits, TC
litigation shall be the proportion of the loss reinsured - On the same date, Supercars, Inc., with notice to ordered sps Lim to pay jointly and severally, plaintiff
with or through Alexander and Alexander, Inc. of private respondents spouses, assigned to petitioner the sum of P55,055.93 plus interest thereon at the
New York, USA, namely, P397,813.00. FCP Credit Corporation (FCP for brevity) its rights, rate of 24% per annum from July 2, 1983 until fully
- The document recited further that Artex title and interest on said promissory note and chattel paid; as well as the cost of suit. It also ordered the
acknowledges receipt of the sum of P3.6M paid by mortgage as shown by the Deed of Assignment. dismissal of the Third party complaint against Third-
the insurer in full and final settlement of all or any - At around 2:30pm Nov9, 1982, said vehicle was Party Defendant.
claims of Artex against its insurer. It discharges its carnapped while parked at the back of Broadway - Upon appeal, CA reversed said decision
insurer from all actions, proceedings, claims, Centrum. Evelyn Lim, who was driving said car - After petitioners' separate MFRs were denied by
demands, costs and expenses in respect thereof. before it was carnapped, immediately called up the CA, petitioners filed these separate petitions for
- With regard the balance unpaid, Wellington Anti-Carnapping Unit of the Philippine Constabulary review on certiorari.
contends that Artex should have been directed to report said incident and thereafter, went to the ISSUE
against the reinsurers to cover the liability and not nearest police substation to make a police report 1. WON there was grave abuse of discretion on the
against Wellington. regarding said incident. part of the appellate court in holding that private
- On Nov10, 1982, Evelyn Lim reported said incident respondents did not violate the insurance contract
ISSUE to the LTO in compliance with the insurance because the authorized driver clause is not applicable
WON the insured (Artex) has a cause of action requirement. She also filed a complaint with the to the "Theft" clause of said Contract
against the reinsurer Headquarters. Constabulary Highway Patrol Group. 2. WON the loss of the collateral exempted the
- On Nov11, 1982, private respondent filed a claim debtor from his admitted obligations under the
HELD for loss with the petitioner Perla but said claim was promissory note particularly the payment of interest,
NO denied on Nov18, 1982 on the ground that Evelyn litigation expenses and attorney's fees
- Unless there is a specific grant in, or assignment Lim, who was using the vehicle before it was
of, the reinsurance contract in favor of the insured or carnapped, was in possession of an expired driver's HELD
a manifest intention of the contracting parties to the license at the time of the loss of said vehicle which is 1. NO
reinsurance contract to grant such benefit or favor to in violation of the authorized driver clause of the - The comprehensive insurance policy issued by
the insured, the insured, not being privy to the insurance policy, which states, to wit: petitioner Perla undertook to indemnify the private
reinsurance contract, has no cause of action against "AUTHORIZED DRIVER: respondents against loss or damages to the car (a)
the reinsurer. It is expressly provided in Section 91 Any of the following: (a) The Insured (b) Any by accidental collision or overturning, or collision or
the Insurance Act 1 that "(T)he original insured has person driving on the Insured's order, or with his overturning consequent upon mechanical breakdown
no interest in a contract of insurance." permission. Provided that the person driving is or consequent upon wear and tear; (b) by fire,
permitted, in accordance with the licensing or external explosion, self-ignition or lightning or
INSURANCE Page
82

burglary, housebreaking or theft; and (c) by between the three contracts in this case, i. e., the petitioner Perla to indemnify private respondents for
malicious act. promissory note, the chattel mortgage contract and the loss of their insured vehicle. However, the latter
- Where a car is unlawfully and wrongfully taken the insurance policy, the Court is compelled to should be ordered to pay petitioner FCP the amount
without the owner's consent or knowledge, such construe all three contracts as intimately interrelated of P55,055.93, representing the unpaid installments
taking constitutes theft, and, therefore, it is the to each other, despite the fact that at first glance from December 30, 1982 up to July 1, 1983, as
"THEFT" clause, and not the "AUTHORIZED DRIVER" there is no relationship whatsoever between the shown in the statement of account prepared by
clause, that should apply.The risk against accident is parties thereto. petitioner FCP, 18 plus legal interest from July 2,
distinct from the risk against theft. The 'authorized - Under the promissory note, Lim spouses are 1983 until fully paid.
driver clause' in a typical insurance policy as in obliged to pay Supercars, Inc. the amount stated - As to the award of moral damages, exemplary
contemplation or anticipation of accident in the legal therein in accordance with the schedule provided for. damages and attorney's fees, private respondents
sense in which it should be understood, and not in To secure said promissory note, private respondents are legally entitled to the same since Perla had acted
contemplation or anticipation of an event such as constituted a chattel mortgage in favor of Supercars, in bad faith by unreasonably refusing to honor the
theft. The distinction often seized upon by insurance Inc. over the automobile the former purchased from insurance claim of the private respondents. Besides,
companies in resisting claims from their assureds the latter. The chattel mortgage, in turn, required awards for moral and exemplary damages, as well as
between death occurring as a result of accident and private respondents to insure the automobile and to attorney's fees are left to the sound discretion of the
death occurring as a result of intent may apply to the make the proceeds thereof payable to Supercars, Court. Such discretion, if well exercised, will not be
case at bar. Inc. The promissory note and chattel mortgage were disturbed on appeal.
- If the insured vehicle had figured in an accident at assigned by Supercars, Inc. to petitioner FCP, with Disposition the assailed decision of the CA is
the time she drove it with an expired license, then, the knowledge of private respondents. Private hereby MODIFIED to require private respondents to
appellee Perla Compania could properly resist respondents were able to secure an insurance policy pay petitioner FCP the amount of P55,055.93, with
appellants' claim for indemnification for the loss or from petitioner Perla, and the same was made legal interest from July 2, 1983 until fully paid. The
destruction of the vehicle resulting from the accident. specifically payable to petitioner FCP. decision appealed from is hereby affirmed as to all
But in the present case, the loss of the insured - From the abovementioned provision that upon the other respects. No pronouncement as to costs.
vehicle did not result from an accident where intent loss of the insured vehicle, the insurance company
was involved; the loss in the present case was Perla undertakes to pay directly to the mortgagor or SHAFER v. JUDGE
caused by theft, the commission of which was to their assignee, FCP, the outstanding balance of 167 SCRA 386
attended by intent." the mortgage at the time of said loss under the PADILLA; November 14, 1988
- There is no causal connection between the mortgage contract. If the claim on the insurance
possession of a valid driver's license and the loss of a policy had been approved by petitioner Perla, it NATURE
vehicle. To rule otherwise would render car insurance would have paid the proceeds thereof directly to Petition for review on certiorari
practically a sham since an insurance company can petitioner FCP, and this would have had the effect of
easily escape liability by citing restrictions which are extinguishing private respondents' obligation to FACTS
not applicable or germane to the claim, thereby petitioner FCP. Therefore, private respondents were - Sherman Shafer obtained a private car policy over
reducing indemnity to a shadow. justified in asking petitioner FCP to demand the his Ford Laser from Makati Insurance Company, Inc.,
2. The court agrees with FCP that Lim spouses are unpaid installments from petitioner Perla. for third party liability. During the effectivity of the
not relieved of their obligation to pay the former the - Because petitioner Perla had unreasonably denied policy, an information for reckless imprudence
installments due on the promissory note on account their valid claim, private respondents should not be resulting in damage to property and serious physical
of the loss of the automobile. The chattel mortgage made to pay the interest, liquidated damages and injuries was filed against shafer. The information
constituted over the automobile is merely an attorney's fees as stipulated in the promissory note. said that on or about the 17th day of May 1985, in
accessory contract to the promissory note. Being the As mentioned above, the contract of indemnity was the City of Olongapo. Shafer hit and bumped a
principal contract, the promissory note is unaffected procured to insure the return of the money loaned Volkswagen car owned and driven by Felino llano y
by whatever befalls the subject matter of the from petitioner FCP, and the unjustified refusal of Legaspi, thereby causing damage in the total amount
accessory contract. petitioner Perla to recognize the valid claim of the of P12,345.00 and as a result thereof one Jovencio
- The unpaid balance on the promissory note should private respondents should not in any way prejudice Poblete, Sr. who was on board of the said
be paid, and not just the installments due and the latter. Volkswagen car sustained physical injuries which
payable before the automobile was carnapped, as - Private respondents can not be said to have unduly injuries causing deformity on the face. The owner of
erronously held by the CA enriched themselves at the expense of FCP since the damaged Volkswagen car filed a separate civil
- However, this does not mean that private they will be required to pay the latter the unpaid action against petitioner for damages, while Jovencio
respondents are bound to pay the interest, litigation balance of its obligation under the promissory note. Poblete, Sr., who was a passenger in the Volkswagen
expenses and attorney's fees stipulated in the - In view of the foregoing discussion, We hold that car, did not reserve his right to file a separate civil
promissory note. Because of the peculiar relationship the Court of Appeals did not err in requiring action for damages. Instead, in the course of the trial
INSURANCE Page
83

in the criminal case, Poblete, Sr. testified on his - A third party complaint is a device allowed by the action" clause under the policy-which requires that a
claim for damages for the serious physical injuries rules of procedure by which the defendant can bring final judgment be first obtained against the insured
which he claimed to have sustained as a result of the into the original suit a party against whom he will and that only thereafter can the person insured
accident. have a claim for indemnity or remuneration as a recover on the policy can prevail over the Rules of
- The court issued an order dismissing the third party result of a liability established against him in the Court provisions aimed at avoiding multiplicity of
complaint on the ground that it was premature, original suit. 13 Third party complaints are allowed to suits.
based on the premise that unless the accused minimize the number of lawsuits and avoid the Disposition instant petition is GRANTED. The
(herein petitioner) is found guilty and sentenced to necessity of bringing two (2) or more actions questioned order dated 24 April 1987 is SET ASIDE
pay the offended party (Poblete Sr.) indemnity or involving the same subject matter. They are and a new one entered admitting petitioner's third
damages, the third party complaint is without cause predicated on the need for expediency and the party complaint against the private respondent
of action. The court further stated that the better avoidance of unnecessary lawsuits. If it appears Makati Insurance Company, Inc.
procedure is for the accused (petitioner) to wait for probable that a second action will result if the
the outcome of the criminal aspect of the case to plaintiff prevails, and that this result can be avoided VDA DE MAGLANA v. CONSOLACION
determine whether or not the accused, also the third by allowing the third party complaint to remain, then 212 SCRA 268
party plaintiff, has a cause of action against the third the motion to dismiss the third party complaint ROMERO; August 6, 1992
party defendant for the enforcement of its third party should be denied.
liability (TPL) under the insurance contract. 6 - Compulsory Motor Vehicle Liability Insurance (third NATURE
Petitioner moved for reconsideration of said order, party liability, or TPL) is primarily intended to provide Petition for certiorari
but the motion was denied; hence, this petition. compensation for the death or bodily injuries
suffered by innocent third parties or passengers as a FACTS
ISSUE result of a negligent operation and use of motor - Lope Maglana was an employee of the Bureau of
WON the court a quo erred in dismissing petitioner's vehicles. The victims and/or their dependents are Customs whose work station was at Lasa, here in
third party complaint on the ground that petitioner assured of immediate financial assistance, regardless Davao City. One day, when he was on his way to his
had no cause of action yet against the insurance of the financial capacity of motor vehicle owners. work, he met an accident that resulted in his death.
company - The liability of the insurance company under the He died on the spot.
Compulsory Motor Vehicle Liability Insurance is for - The PUJ jeep that bumped the deceased was driven
HELD loss or damage. Where an insurance policy insures by Pepito Into, operated and owned by defendant
YES directly against liability, the insurer's liability accrues Destrajo. From the investigation conducted by the
- There is no need on the part of the insured to wait immediately upon the occurrence of the injury or traffic investigator, the PUJ jeep was overtaking
for the decision of the trial court finding him guilty of event upon which the liability depends, and does not another passenger jeep that was going towards the
reckless imprudence. The occurrence of the injury to depend on the recovery of judgment by the injured city poblacion. While overtaking, the PUJ jeep of
the third party immediately gave rise to the liability party against the insured. defendant Destrajo running abreast with the
of the insurer under its policy. Respondent insurance - The injured for whom the contract of insurance is overtaken jeep, bumped the motorcycle driven by
company's contention that the third party complaint intended can sue directly the insurer. The general the deceased. The point of impact was on the lane of
involves extraneous matter which will only clutter, purpose of statutes enabling an injured person to the motorcycle and the deceased was thrown from
complicate and delay the criminal case is without proceed directly against the insurer is to protect the road and met his untimely death.
merit. The civil aspect of the offense charged, i.e., injured persons against the insolvency of the insured - Heirs of Lope Maglana, Sr. filed an action for
serious physical injuries allegedly suffered by who causes such injury, and to give such injured damages and attorney's fees against operator
Jovencio Poblete, Sr., was impliedly instituted with person a certain beneficial interest in the proceeds of Patricio Destrajo and the Afisco Insurance
the criminal case. Petitioner may thus raise all the policy, and statutes are to be liberally construed Corporation (AFISCO). An information for homicide
defenses available to him insofar as the criminal and so that their intended purpose may be accomplished. thru reckless imprudence was also filed against
civil aspects of the case are concerned. The claim of It has even been held that such a provision creates a Pepito Into.
petitioner for payment of indemnity to the injured contractual relation which inures to the benefit of - During the pendency of the civil case, Into was
third party, under the insurance policy, for the any and every person who may be negligently sentenced to suffer an indeterminate penalty, with
alleged bodily injuries caused to said third party, injured by the named insured as if such injured all the accessory penalties provided by law, and to
arose from the offense charged in the criminal case, person were specifically named in the policy. indemnify the heirs of Lope Maglana, Sr. in the
from which the injured (Jovencio Poblete, Sr.) has - In the event that the injured fails or refuses to amount of twelve thousand pesos with subsidiary
sought to recover civil damages. Hence, such claim include the insurer as party defendant in his claim for imprisonment in case of insolvency, plus five
of petitioner against the insurance company cannot indemnity against the insured, the latter is not thousand pesos in the concept of moral and
be regarded as not related to the criminal action. prevented by law to avail of the procedural rules exemplary damages with costs. No appeal was
intended to avoid multiplicity of suits. Not even a "no
INSURANCE Page
84

interposed by accused who later applied for - The underlying reason behind the third party confusion among the petitioners and their counsel.
probation. liability (TPL) of the Compulsory Motor Vehicle What should have been clearly stressed as to leave
- The lower court rendered a decision finding that Liability Insurance is "to protect injured persons no room for doubt was the liability of AFISCO under
Destrajo had not exercised sufficient diligence as the against the insolvency of the insured who causes the explicit terms of the insurance contract.
operator of the jeepney ordering him to pay plaintiffs such injury, and to give such injured person a certain Disposition present petition is hereby GRANTED.
the sum for loss of income; funeral and burial beneficial interest in the proceeds of the policy . . ." The award of P28,800.00 representing loss of income
expenses of the deceased; moral damages, and Since petitioners had received from AFISCO the sum is INCREASED to P192,000.00 and the death
attorney's fees and costs of suit. The defendant of P5,000.00 under the no-fault clause, AFISCO's indemnity of P12,000.00 to P50,000.00.
insurance company is ordered to reimburse liability is now limited to P15,000.00.
defendant Destrajo whatever amounts the latter - However, we cannot agree that AFISCO is likewise
shall have paid only up to the extent of its insurance solidarily liable with Destrajo. In Malayan Insurance
coverage. Co., Inc. v. Court of Appeals, this Court had the
- Petitioners filed a motion for the reconsideration of opportunity to resolve the issue as to the nature of
the second paragraph of the decision contending that the liability of the insurer and the insured vis-a-vis
AFISCO should not merely be held secondarily liable the third party injured in an accident. We FAR EASTERN SURETY v. MISA
because the Insurance Code provides that the categorically ruled thus: While it is true that where 25 SCRA 663
insurer's liability is "direct and primary and/or jointly the insurance contract provides for indemnity against REYES; October 26, 1968
and severally with the operator of the vehicle, liability to third persons, such third persons can
although only up to the extent of the insurance directly sue the insurer, however, the direct liability NATURE
coverage." Hence, they argued that the P20,000.00 of the insurer under indemnity contracts against Appeal by petition for review from a CA judgment
coverage of the insurance policy issued by AFISCO, third party liability does not mean that the insurer
should have been awarded in their favor. can be held solidarily liable with the insured and/or FACTS
- AFISCO argued that since the Insurance Code does the other parties found at fault. The liability of the - Socorro Dancel Vda.de Misa and Araceli Pinto hired
not expressly provide for a solidary obligation, the insurer is based on contract; that of the insured is a taxi cab operated by La Mallorca on September 3,
presumption is that the obligation is joint. based on tort. In the case at bar, petitioner as 1957. The taxi they were riding in collided with a
- The lower court denied the motion for insurer of Sio Choy, is liable to respondent Vallejos gravel and sand truck resulting to injuries to both
reconsideration ruling that since the insurance (the injured third party), but it cannot, as incorrectly Misa and Pinto.
contract "is in the nature of suretyship, then the held by the trial court, be made "solidarily" liable - The two passengers instituted a suit for damages
liability of the insurer is secondary only up to the with the two principal tortfeasors, namely against La Mallorca who, while denying
extent of the insurance coverage." respondents Sio Choy and San Leon Rice Mill, Inc. responsibility, instituted a third party complaint
- Petitioners filed a second motion for For if petitioner-insurer were solidarily liable with against Far Eastern Surety to recoup from the latter
reconsideration reiterating that the liability of the said, two (2) respondents by reason of the indemnity any award for damages that might be recovered by
insurer is direct, primary and solidary with the contract against third party liability under which an the passengers.
jeepney operator because the petitioners became insurer can be directly sued by a third party this will - It would appear from the case that a sticker was
direct beneficiaries under the provision of the policy result in a violation of the principles underlying placed in all the taxis of La Mallorca stating that
which, in effect, is a stipulation pour autrui. This solidary obligation and insurance contracts. passengers of the taxis were insured against
motion was likewise denied for lack of merit. - While in solidary obligations, the creditor may accidents. This was done to entice the public into
enforce the entire obligation against one of the patronizing La Mallorca.
ISSUE solidary debtors, in an insurance contract, the - The trial court awarded to Misa and Pinto actual,
WON AFISCO can be held directly liable insurer undertakes for a consideration to indemnify moral and exemplary damages, and attorney’s fees
the insured against loss, damage or liability arising payable by La Mallorca and sentenced Far Eastern to
HELD from an unknown or contingent event. pay La Mallorca P10,000. on its third party liability
YES - Similarly, petitioners herein cannot validly claim insurance.
- As this Court ruled in Shafer vs. Judge, RTC of that AFISCO, whose liability under the insurance - On appeal, the CA, while holding that the collision
Olongapo City, Br. 75, "[w]here an insurance policy policy is also P20,000.00, can be held solidarily liable was due to the fault of the driver of the gravel and
insures directly against liability, the insurer's liability with Destrajo for the total amount of P53,901.70 in sand truck, found the taxi company liable for
accrues immediately upon the occurrence of the accordance with the decision of the lower court. damages to the passengers on the strength of its
injury or even upon which the liability depends, and Since under both the law and the insurance policy, representation contained in the sticker above noted
does not depend on the recovery of judgment by the AFISCO's liability is only up to P20,000.00, the that the passengers were insured against accidents.
injured party against the insured." second paragraph of the dispositive portion of the In so ruling, the CA overruled the defense of the
decision in question may have unwittingly sown insurance company to the effect that it was
INSURANCE Page
85

responsible only if the insured, La Mallorca, was FACTS proposition that there was no reason for confiscation
involved in accidents caused by, or arising out of, the - vehicular accident with 2 children running across of Amar's license (2) Amar's license had not expired,
use of the motor vehicle. A motion for the path of a Chevrolet "Carry-All", belonging to a but had been renewed.
reconsideration was filed in and dismissed by the CA. partnership known as Diman & Company driven by - Judge Alikpala did not admit such evidence
its driver, Perfecto Amar, as it was passing a
ISSUE national highway at barrio Makiling Calamba, ISSUES
WON Far Eastern Surety is liable to the insured on its Laguna. They were killed. It was insured with the 1. WON Judge Alikapala committed grave abuse of
insurance policy Empire Insurance Co., Inc. under a so-called discretion in not admitting evidence
'comprehensive coverage" policy, loss by theft 2. WON confiscation of license and expiration of TVR
HELD excluded. The policy was in force at the time of the of the driver would serve as bar for Peza in
NO accident. recovering from Empire
- The award for damages made to the passengers - Placida Peza, the managing partner of Diman & Co.
was exclusively predicated on the representation filed a claim with Empire, for payment of HELD
made by La Mallorca that its passengers were compensation to the family of the 2 children who 1. NO
insured against accidents and not because it was at died as a result of the accident. Empire refused to - Even positing error in the Judge's analysis of the
fault in causing the accident. pay on the ground that the driver had no authority to evidence attempted to be introduced and his
Reasoning operate the vehicle, a fact which it expressly rejection thereof, it is clear that it was at most an
- In this case, the findings of the CA and the trial excepted from liability under the policy. What Peza error of judgment, not such an error as may be
court that the causative factor of the mishap was the did was to negotiate directly with the deceased branded a grave abuse of discretion, i.e., such
negligence of the gravel and truck driver would have children father for an out-of-court settlement. The capricious and whimsical exercise of judgment as is
been sufficient to relieve the taxi company of any father agreed to accept P6,200.00 in fun settlement equivalent to lack of jurisdiction, against which the
liability arising from the accident. However, in view of the liability of the vehicles owner and driver, and writ of certiorari will lie. In any event, the
of the sticker in all of its taxicabs, La Mallorca has Peza paid him this sum. established principle is "that ruling of the trial court
insured its passengers against accidents, whether it - Peza thereafter sued Empire to recover this sum of on procedural questions and on admissibility of
was at fault or not. In other words, La Mallorca P6,200.00 as actual damages, as well as P20,000.00 evidence during the course of the trial are
accepted the responsibility for damages or injuries to as moral damages, P10,000.00 as exemplary interlocutory in nature and may not be the subject of
passengers even if it had no fault at all. damages, and P10,000.00 as attorney's fees. She separate appeal or review on certiorari, but are to be
- In the case of the insurance company, the SC ruled amended her complaint shortly thereafter to include assigned as errors and reviewed in the appeal
that it neither authorized nor consented to the Diman & Co. as alternative party plaintiff. properly taken from the decision rendered by the
representations made by the taxi company to its - Empire's basic defense to the suit was anchored trial court on the merits of the case.
passengers. As such, the liability of the said on the explicit requirement in the policy limiting the - In the meantime, Judge Alikpala rendered
insurance company based on its insurance contract operation of the insured vehicle to the "authorized judgment on the merits, since the case was then
is limited to the recovery by the insured of all sums, driver" therein defined, namely, (a) the insured, or already ripe for adjudication. The judgment ordered
cost and expenses which the insured shall become (b) any person driving on the insured order or with dismissal of the case for failure on the part of the
legally liable. The insurance company therefore his permission, provided that- plaintiff to prove their cause of action against
cannot be held liable for the award. ... that the person driving is permited in Empire. Notice of the judgment was served on the
- The taxi company is adjudged to be the sole party accordance with the licensing or other laws or parties in due course.
responsible for the award. regulations to drive the Motor vehicle or has been 2. YES
Disposition The decision of the CA is modified by so permitted and is not disqualified by order of the - It would seem fairly obvious that whether the LTC
eliminating the award against Far Eastern. Court of Law of by reason of any enactment or agent was correct or not in his opinion that driver
regulation in that behalf from driving such Motor Amar had violated some traffic regulation warranting
PEZA v. ALIKPALA Vehicle.- confiscation of his license and issuance of a TVR in
160 SCRA 31 - driver Perfecto Amar, only having a temporary lieu thereof, this would not alter the undisputed fact
NARVASA; April 15, 1988 operator's permit (TVR) [already expired] his driver’s that Amar's licence had indeed been confiscated and
license having earlier been confiscated by an agent a TVR issued to him, and the TVR had already
NATURE of the Land Transportation Commission for an expired at the time that the vehicle being operated
Motion praying that Judge Alikpala be declared guilty alleged violation of Land Transportation and Traffic by him killed two children by accident. Neither would
of contempt of court for having decided the case on Rules, was not permitted by law and was in truth proof of the renewal of Amar's license change the
the merits despite the pendency in this Court of the disqualified to operate any motor vehicle; Peza fact that it had really been earlier confiscated by the
certiorari action instituted by the plaintiffs attempted to neutralize that fact by(1) the issuance LTC agent.
of the TVR by the LTC officer to Amar; in proof of the Disposition petition is DISMISSED for lack of merit
INSURANCE Page
86

Perla denied its liability under the above provision should be read together with the requirement for
and said that the insurer of the vehicle that the compulsory passenger and/or 3rd party liability
respondents were riding (Malayan Insurance in this insurance (Sec. 377) which was mandated in order
case) should be liable. Its 2 MFRs denied, Perla filed to ensure ready compensation for victims of
this action vehicular accidents.
-Irrespective of whether or not fault or negligence
ISSUE lies with the driver of the Superlines bus, as
WON Perla is the insurer liable to indemnify under respondents were not occupants of the bus, they
PERLA COMPANIA DE SEGUROS v. ANCHETA Sec. 378 cannot claim the “no fault indemnity” provided in
164 SCRA 144 Sec. 378 from Perla. The claim should be made
CORTES; August 8, 1988 HELD against the insurer of the vehicle they were riding.
NO Disposition Petition GRANTED. Orders of CFI
NATURE Ratio The law is very clear – the claim shall lie ordering Perla to pay respondents immediately
Petition for certiorari and prohibition with prelim against the insurer of the vehicle in which the P5000 ANNULLED and SET ASIDE
injunction to review orders of CFI Camarines Norte “occupant xxx is riding,” and no other. The claimant
is not free to choose from which insurer he will claim WESTERN GUARANTY CORPORATION v. CA
FACTS the “no fault indemnity,” as the law, by using the (RODRIGUEZ & DE DIOS TRANS)
- Perla was the insurer of a Superlines bus which word “shall”, makes it mandatory that the claim be 185 SCRA 652
figured in a collision with a III Scout (it’s a kind of made against the insurer of the vehicle in which the FELICIANO; July 20, 1990
vehicle). Injured passengers of the latter (and occupant is riding, mounting or dismounting from.
respondents in this case) filed a complaint for Reasoning NATURE
damages against Superlines, the bus driver, and - the rules on claims under the “no fault indemnity” Petition for review the decision of CA affirming in
Perla (as insurer of the bus). CFI Judge Ancheta provision, where proof of fault or negligence is not toto the damages awarded to private respondent by
ordered that Perla should pay the respondents necessary for payment of any claim for death or the trial court.
immediately the P5000 under the “no fault clause” as injury to a passenger or to a 3rd party, are
provided in Sec. 378. established: FACTS
Sec. 378: Any claim for death or injury to any 1. A claim may be made against one motor vehicle - Respondent Priscilla E. Rodriguez was struck by a
passenger or 3rd party pursuant to the provisions of only. De Dios passenger bus owned by respondent De Dios
this chapter shall be paid without the necessity of 2. If the victim is an occupant of a vehicle, the claim Transportation Co., Inc. Priscilla was thrown to the
proving fault or negligence of any kind. Provided, shall lie against the insurer of the vehicle in which he ground, hitting her forehead. She was treated at the
That for purposes of this section is riding, mounting or dismounting from. Protacio Emergency Hospital and later on
(i) The indemnity in respect of any one person shall 3. In any other case (i.e. if the victim is not an hospitalized at the San Juan De Dios Hospital. Her
not exceed P5,000; occupant of a vehicle), the claim shall lie against the face was permanently disfigured, causing her serious
(ii) The following proofs of loss, when submitted insurer of the directly offending vehicle. anxiety and moral distress. Respondent bus company
under oath, shall be sufficient evidence to 4. In all cases, the right of the party paying the claim was insured with petitioner Western Guaranty
substantiate the claim: to recover against the owner of the vehicle Corporation ("Western") under its Master Policy
(a) Police report of accident, and responsible for the accident shall be maintained. which provided, among other things, for protection
(b) Death certificate and evidence sufficient to -That the vehicle ridden might not be the one that against third party liability, the relevant section
establish, the proper payee, or caused the accident is of no moment since the law reading as follows:
(c) Medical report and evidence of medical or itself provides that the party paying the claim under Section 1. Liability to the Public ? Company will,
hospital disbursement in respect of which refund is Sec. 378 may recover against the owner of the subject to the Limits of Liability, pay all sums
claimed; vehicle responsible for the accident. This is precisely necessary to discharge liability of the insured in
(iii) Claim may be made against one motor vehicle the essence of “no fault indemnity” insurance which respect of ?
only. In the case of an occupant of a vehicle, claim was introduced to and made part of our laws in order (a) death of or bodily injury to or damage to
shall lie against the insurer of the vehicle in which to provide victims of vehicular accidents or their property of any passenger as defined herein.
the occupant is riding, mounting or dismounting heirs immediate compensation, although in a limited (b) death of or bodily injury or damage to property
from. In any other case, claim shall lie against the amount, pending final determination of who is of any THIRD PARTY as defined herein in any
insurer of the directly offending vehicle. In all cases, responsible for the accident and liable for the victims' accident caused by or arising out of the use of the
the right of the party paying the claim to recover injuries or death. In turn, the “no fault indemnity” Schedule Vehicle, provided that the liability shall
against the owner of the vehicle responsible for the provision is part and parcel of the Code provisions on have first been determined. In no case, however,
accident shall be maintained. compulsory motor vehicle liability insurance and shall the Company's total payment under both
INSURANCE Page
87

Section I and Section 11 combined exceed the Western. A car accident may, for instance, result in jurisprudence that the terms of such contract are to
Limits of Liability set forth herein. With respect to injury to internal organs of a passenger or third be construed strictly against the party which
death of or bodily injury to any third party or party, without any accompanying amputation or loss prepared the contract, which in this case happens to
passenger, the company's payment per victim in of an external member (e.g., a foot or an arm or an be petitioner Western.
any one accident shall not exceed the limits eye). But such internal injuries are surely covered by Disposition Petition denied.
indicated in the Schedule of indemnities provided Section I of the Master Policy, since they certainly
for in this policy excluding the cost of additional constitute bodily injuries. SUMMIT GUARANTY & INSURANCE COMPANY v.
medicines, and such other burial and funeral - The Schedule of Indemnities does not purport to ARNALDO
expenses that might have been incurred. restrict the kinds of damages that may be awarded 158 SCRA 332
- Respondent Priscilla Rodriguez filed a complaint for against Western once liability has arisen. Section 1, GANCAYCO; February 29, 1988
damages before the Regional Trial Court of Makati quoted above, does refer to certain "Limits of
against De Dios Transportation Co. and Walter A. Liability" which in the case of the third party liability NATURE
Saga Respondent De Dios Transportation Co., in section of the Master Policy, is apparently PETITION to review the order of the Insurance
turn, filed a third-party complaint against its P50,000.00 per person per accident. Within this Commissioner.
insurance carrier, petitioner Western. On 6 August over-all quantitative limit, all kinds of damages
1985, the trial court rendered a decision in favor of allowable by law" actual or compensatory FACTS
respondent Priscilla E. Rodriguez, awarding moral damages"; "moral damages'; "nominal damages"; - On Nov. 26, 1976, a Ford Pick-up truck owned by
damages, lossof earning and attorney's fees among "temperate or moderate damages"; "liquidated Marcos Olasco was bumped by a cargo truck owned
others. damages"; and "exemplary damages" ? may be by Floralde.
- On appeal, the Court of Appeals affirmed in toto awarded by a competent court against the insurer FGU Insurance Corporation (FG U) by reason of
the decision of the trial court. once liability is shown to have arisen, and the Motor Vehicle Insurance Policy No. IC-VF-07185 paid
- Petitioner contends that it cannot be held liable for essential requisites or conditions for grant of each Olaso the sum of P2,817.50 as its share in the repair
loss of earnings, moral damages and attorney's fees species of damages are present. It appears to us cost of the said Ford Pick-up. Having been
because these items are not among those included in self-evident that the Schedule of Indemnities was not subrogated to the rights and causes of action of
the Schedule of Indemnities set forth in the intended to be an enumeration, much less a closed Olaso in the said amount FGU formally demanded
insurance policy. enumeration, of the specific kinds of damages which payment of said amount from Floralde and
may be awarded under the Master Policy Western attempted to verify Floralde's insurance carrier but
ISSUE has issued. failed to do so. In 1978 FGU was able to ascertain
WON petitioner can be held liable for loss of - The reading urged by Western of the Schedule of the identity of Floralde's insurance carrier to be the
earnings, moral damages and attorney's fees Indemnities comes too close to working fraud upon Summit Guaranty and Insurance Company, Inc.
both the insured and the third party beneficiary of (Summit) and thus requested the insurance
HELD Section 1, quoted above. For Western's reading commissioner for a conference with Summit and
YES would drastically and without warning limit the demanded from Summit through counsel on
- The Schedule of Indemnities does not purport to otherwise unlimited and comprehensive scope of February 28, 1978 the payment of the damages
restrict the kinds of damages that may be awarded liability assumed by the insurer Western under sustained by the car of Olaso but to no avail.
against Western once liability has arisen. It was Section 1: "all sums necessary to discharge liability - Hence on May 22, 1978 FGU filed a case in the
merely meant to set limits to the amounts the of the insured in respect of [bodily injury to a third Insurance Commissioner's Office against Summit for
movant would be liable for in cases of claims for party]". This result- which is not essentially different recovery of said amount.
death, bodily injuries of, professional services and from taking away with the left hand what had been - Summit filed a motion to dismiss on the ground of
hospital charges, for services rendered to traffic given with the right hand we must avoid as obviously prescription under Section 384 of PD No. 612.
accident victims,' and not necessarily exclude claims repugnant to public policy. If what Western now Averring that the accident happened on November
against the insurance policy for other kinds of urges is what Western intended to achieve by its 26, 1976 while the complaint was filed on May 22,
damages, such as those in question. Schedule of Indemnities, it was incumbent upon 1978 beyond the one-year period from the time of
- It will be seen that the above quoted Schedule of Western to use language far more specific and the accident provided for by the said provision.
Indemnities establishes monetary limits which precise than that used in fact by Western, so that the
Western may invoke in case of occurrence of the insured, and potential purchasers of its Master ISSUE
particular kinds of physical injury there listed. Policy, and the Office of the Insurance WON the action must be dismissed on the ground of
- It must be stressed, however, that the Schedule of Commissioner, may be properly informed and act prescription under Section 384 of PD No. 612
Indemnities does not purport to limit, or to accordingly.
enumerate exhaustively, the species of bodily injury - Moreover, an insurance contract is a contract of HELD
occurrence of which generate liability for petitioner adhesion. The rule is well entrenched in our NO
INSURANCE Page
88

- The case do not fall within the meaning of proper claims and to effectuate prompt, fair and equitable allegedly taken by six (6) persons and driven out to
cases' as contemplated in Section 384 of the settlement of claims, and with manifest bad faith, Montalban, Rizal. While travelling along Mabini St.,
Insurance Code. petitioner company devised means and ways of Sitio Palyasan, Barrio Burgos, going North at
stalling the settlement proceedings. In G.R. No. L- Montalban, Rizal, the car figured in an accident,
50997, no steps were taken to process the claim and hitting and bumping a gravel and sand truck parked
Reasoning no rejection of said claim was ever made even if at the right side of the road going south. As a
- Section 384 of PD 612 (Insurance Code) private respondent had already complied with all the consequence, the gravel and sand truck veered to
Any person having any claim upon the policy requirements. the right side of the pavement going south and the
issued pursuant to this chapter shall, without any - In G.R. No. L-48758-petitioner company even car veered to the right side of the pavement going
unnecessary delay, present to the insurance provided legal assistance to one of the private north. The driver, Benito Mabasa, and one of the
company concerned a written notice of claim respondents in the criminal case filed against him passengers died and the other four sustained
setting forth the amount of his loss, and/or the leading Private respondents to believe that it was physical injuries. The car, as well, suffered extensive
nature, extent and duration of the injuries ready to pay. In the same case, petitioner company damage. Complainant, thereafter, filed a claim for
sustained as certified by a duly licensed physician. admits that it took no final action or adjudication of total loss with the respondent company but claim
Notice of claim must be filed within six months the claim. Worse still, in G.R. No. L-48679, was denied. Hence, complainant was compelled to
from date of the accident, otherwise, the claim assurances of payment were constantly given and institute the present action."
shall be deemed waived. Action or suit for petitioner company even said that a check was ready - The comprehensive motor car insurance policy for
recovery of damage due to loss or injury must be for release. This Court has made the observation that P35,000.00 issued by respondent Empire Insurance
brought, in proper cases, with the Commissioner some insurance companies have been inventing Company admittedly undertook to indemnify the
or the Courts within one year from date of excuses to avoid their just obligations and it is only petitioner-insured against loss or damage to the car
accident, otherwise, the claimant's right of action the State that can give the protection which the (a) by accidental collision or overturning, or collision
shall prescribe. insuring public needs from possible abuses of the or overturning consequent upon mechanical
- It is very clear that the one-year period is only insurers. In view of the foregoing, breakdown or consequent upon wear and tear; (b)
required In proper cases. Had the lawmakers - It is not denied that an extrajudicial demand for by fire, external explosion, self-ignition or lightning
intended it to be the way Petitioner Company payment was made by respondent FGU on petitioner or burglary, housebreaking or theft; and (c) by
assumes it to be, then the phrase 'in proper cases' but petitioner failed to respond to the same. malicious act.
would not have been inserted. Nevertheless the complaint was filed even before a - Respondent insurance commission, however,
- in Aisporna. vs. Court of Appeals: denial of the claim was made by petitioner. For all dismissed petitioner's complaint for recovery of the
'Legislative intent must be ascertained from a legal purposes, the one-year prescriptive period total loss of the vehicle against private respondent,
consideration of the statute as a whole. The provided for in Section 384 of the Insurance Code sustaining respondent insurer's contention that the
particular words, clauses and phrases should not has not begun to run.The cause of action arises only accident did not fall within the provisions of the
be studied as detached and isolated expressions, and starts to run upon the denial of the claim by the policy either for the Own Damage or Theft coverage,
but the whole and every part of the statute must insurance company.The court takes note of the invoking the policy provision on "Authorized Driver"
be considered in fixing the meaning of any of its dilatory tactics employed by petitioner in this as in clause, which clause limits the use of the insured
parts and in order to produce a harmonious whole. the several cases aforecited to avoid payment of its vehicle to two (2) persons only, namely: the insured
A statute must be so construed as to harmonize liabilities. himself or any person on his (insured's) permission.
and give effect to all its provisions whenever Apparently, the Insurance commission sees the
possible.' VILLACORTA v. THE INSURANCE COMMISSION unauthorized taking of the vehicle for a joyride as a
- Petitioner company is trying to use Section 384 of 100 SCRA 467 violation of the 'Authorized Driver' clause of the
the Insurance Code as a cloak to hide itself from its TEEHANKEE; October 30, 1980 policy."
liabilities. The facts of these cases evidently reflect - Respondent commission likewise upheld private
the deliberate efforts of petitioner company to FACTS respondent's assertion that the car was not stolen
prevent the filing of a formal action against it. - JEWEL VILLACORTA was the owner of a Colt and therefore not covered by the Theft clause, ruling
Bearing in mind that if it succeeds in doing so until Lancer, Model 1976, insured with respondent that "(T)he element of 'taking' in Article 308 of the
one year lapses from the date of the accident it could company for P35,000.00 - Own Damage; P30,000.00 Revised Penal Code means that the act of depriving
set up the defense of prescription, petitioner - Theft; and P30,000.00 - Third Party Liability, another of the possession and dominion of a movable
company made private respondents believe that their effective May 16, 1977 to May 16, 1978. thing is coupled . . . with the intention, at the time of
claims would be settled in order that the latter will - On May 9, 1978, the vehicle was brought to the the 'taking', of withholding it with the character of
not find it necessary to immediately bring suit. In Sunday Machine Works, Inc., for general check-up permanency
violation of its duties to adopt and implement and repairs. On May 11, 1978, while it was in the
reasonable standards for the prompt investigation of custody of the Sunday Machine Works, the car was ISSUE
INSURANCE Page
89

WON the Insurance commission’s findings are in the nature of theft as defined in Article 308 of the - Defendant denies liability to his amount, alleging,
accord with law Revised Penal Code. among other things, that the stranding of the vessel
- The Court rejects respondent commission's was due to the fault, negligence and lack of skill of
HELD premise that there must be an intent on the part of its master, that the expenses incurred in putting it
NO the taker of the car "permanently to deprive the afloat did not constitute general average, and that
- First, respondent commission's ruling that the insured of his car" and that since the taking here was the liquidation of the average was not made in
person who drove the vehicle in the person of Benito for a "joy ride" and "merely temporary in nature," a accordance with law.
Mabasa, who, according to its own finding, was one "temporary taking is held not a taking insured - The lower court found for plaintiff
of the residents of the Sunday Machine Works, Inc. against."
to whom the car had been entrusted for general - The insurer must therefore indemnify the petitioner ISSUE
check-up and repairs was not an "authorized driver" owner for the total loss of the insured car in the sum WON the expenses incurred in floating a vessel so
of petitioner-complainant is too restrictive and of P35,000.00 under the theft clause of the policy, stranded should be considered general average and
contrary to the established principle that insurance subject to the filing of such claim for reimbursement shared by the cargo owners
contracts, being contracts of adhesion where the or payment as it may have as subrogee against the
only participation of the other party is the signing of Sunday Machine Works, Inc. HELD
his signature or his "adhesion" thereto, "obviously NO
call for greater strictness and vigilance on the part of Ratio The law on averages is contained in the Code
courts of justice with a view of protecting the weaker of Commerce. Under that law, averages are classified
party from abuse and imposition, and prevent their into simple or particular and general or gross.
becoming traps for the unwary." Generally speaking, simple or particular averages
- The main purpose of the "authorized driver" clause, include all expenses and damages caused to the
as may be seen from its text, supra, is that a person vessel or cargo which have not inured to the
other than the insured owner, who drives the car on CHAPTER VIII. MARINE INSURANCE common benefit (Art. 809), and are, therefore, to be
the insured's order, such as his regular driver, or borne only by the owner of the property gave rise to
with his permission, such as a friend or member of MAGSAYSAY INC v. AGAN same (Art. 810); while general or gross averages
the family or the employees of a car service or repair 96 PHIL 504 include "all the damages and expenses which are
shop must be duly licensed drivers and have no REYES; January 31, 1955 deliberately caused in order to save the vessel, its
disqualification to drive a motor vehicle. A car owner cargo, or both at the same time, from a real and
who entrusts his car to an established car service FACTS known risk" (Art. 811). Being for the common
and repair shop necessarily entrusts his car key to - The S S "San Antonio", vessel owned and operated benefit, gross averages are to be borne by the
the shop owner and employees who are presumed to by plaintiff, left Manila on October 6, 1949, bound for owners of the articles saved (Art. 812).
have the insured's permission to drive the car for Basco, Batanes, vis Aparri, Cagayan, with general Reasoning
legitimate purposes of checking or road-testing the cargo belonging to different shippers, among them - the stranding of plaintiff's vessel was due to the
car. The mere happenstance that the employee(s) of the defendant. The vessel reached Aparri, but while sudden shifting of the sandbars at the mouth of the
the shop owner diverts the use of the car to his own still in the port, it ran aground at the mouth of the river which the port pilot did not anticipate. The
illicit or unauthorized purpose in violation of the trust Cagayan river, and, attempts to refloat it under its standing may, therefore, be regarded as accidental.
reposed in the shop by the insured car owner does own power having failed, plaintiff had it refloated by - Tolentino, in his commentaries on the Code of
not mean that the "authorized driver" clause has the Luzon Stevedoring Co. at an agreed Commerce, gives the following requisites for
been violated such as to bar recovery, provided that compensation. Once afloat the vessel returned to general average:
such employee is duly qualified to drive under a valid Manila to refuel and then proceeded to Basco, the First, there must be a common danger. This means,
driver's license. port of destination. There the cargoes were delivered that both the ship and the cargo, after has been
- Secondly, and independently of the foregoing to their respective owners or consignees, who, with loaded, are subject to the same danger, whether
(since when a car is unlawfully taken, it is the theft the exception of defendant, made a deposit or signed during the voyage, or in the port of loading or
clause, not the "authorized driver" clause, that a bond to answer for their contribution to the unloading; that the danger arises from the accidents
applies), where a car is admittedly as in this case average. of the sea, dispositions of the authority, or faults of
unlawfully and wrongfully taken by some people, be - On the theory that the expenses incurred in floating men, provided that the circumstances producing the
they employees of the car shop or not to whom it the vessel constitute general average to which both peril should be ascertained and imminent or may
had been entrusted, and taken on a long trip to ship and cargo should contribute, plaintiff brought rationally be said to be certain and imminent. This
Montalban without the owner's consent or the present action in the CFI of Manila to make last requirement exclude measures undertaken
knowledge, such taking constitutes or partakes of defendant pay his contribution, which, as determined against a distant peril.
by the average adjuster, amounts to P841.40.
INSURANCE Page
90

Second, that for the common safety part of the - The insurance company, insisting that its obligation contribute to the general average resulting from the
vessel or of the cargo or both is sacrificed did not extend beyond the insurance of the “absolute jettison of a part of said vessel's cargo
deliberately. total loss of the vessel only, and to pay proportionate
Third, that from the expenses or damages caused salvage of the declared value,” refused to contribute HELD
follows the successful saving of the vessel and cargo. to the settlement of the gen. ave. The present action 1. NO
Fourth, that the expenses or damages should have was thereupon instituted, and after trial the court Ratio In case repugnance exists between written
been incurred or inflicted after taking proper legal below rendered judgment in favor of the plaintiff and and printed portions of a policy, the written portion
steps and authority. ordered the defendant to pay the plaintiff P2,610.86 prevails.
- With respect to the first requisite, the evidence as its part of the indemnity for the gen. ave. brought Reasoning
does not disclose that the expenses sought to be about by the jettison of cargo. The insurance - Section 291 of the Code of Civil Procedure provides
recovered from defendant were incurred to save company then appealed to the SC. that “when an instrument consists partly of written
vessel and cargo from a common danger...it is the - The insurance contract is printed in the English words and partly of a printed form and the two are
safety of the property, and not of the voyage, which common form of marine policies. One of the clauses inconsistent, the former controls the latter.”
constitutes the true foundation of the general of the document originally read as follows:
average. “Touching the Adventures and Perils which the
- As to the second requisite, we need only repeat said NUFIC is content to bear, and to take upon 2. NO
that the expenses in question were not incurred for them in this Voyage; they are of the Seas, Men-of- Ratio The liability for contribution in general average
the common safety of vessel and cargo, since they, War, Fire, Pirates, Thieves, Jettison, Letters of is not based on the express terms of the policy, but
or at least the cargo, were not in imminent peril. Mart and Countermart, Surprisals, and Takings at rests upon the theory that from the relation of the
- With respect to the third requisite, the salvage Sea. Arrests, Restraints and Detainments, of all parties and for their benefit, a quasi contract is
operation, it is true, was a success. But as the Kings, Princes and People of what Nation, implied by law.
sacrifice was for the benefit of the vessel to enable it Condition or Quality soever; Barratry of the Master Reasoning
to proceed to destination and not for the purpose of and Marines, and of all other Perils, Losses and - In the absence of positive legislation to the
saving the cargo, the cargo owners are not in law Misfortunes, that have or shall come to the Hurt, contrary, the liability of the defendant insurance
bound to contribute to the expenses. Detriment, or Damage of the said Vessel or any company on its policy would, perhaps, be limited to
- The final requisite has not been proved, for it does part thereof; and in case of any Loss or “absolute loss of the vessel only, and to pay
not appear that the expenses here in question were Misfortunes, it shall be lawful for the Assured, his proportionate salvage of the declared value.” But the
incurred after following the procedure laid down in or their Factors, Servants, or assigns, to sue, policy was executed in this jurisdiction and
article 813. labour and travel for, in and about the Defence. “warranted to trade within the waters of the
Disposition Wherefore, the decision appealed from Safeguard, and recovery of the said Vessel or any Philippine Archipelago only.” Here, Art. 859 of the
is reversed. part thereof, without Prejudice to this Insurance; Code of Commerce is still in force:
to the Charges whereof the said Company, will “ART. 859. The underwriters of the vessel, of the
JARQUE v. SMITH, BELL & CO. contribute, according to the rate and quantity of freight, and of the cargo shall be obliged to pay for
56 PHIL 758 the sum herein assured...” the indemnity of the gross average in so far as is
OSTRAND; November11, 1932 - Attached to the policy over and above the said required of each one of these objects
clause is a “rider” containing typewritten provisions, respectively.”
NATURE among which appears in capitalized type the - The article is mandatory in its terms, and the
Appeal from judgment of the lower court following clause: insurers (whether for the vessel or for the freight or
“AGAINST THE ABSOLUTE TOTAL LOSS OF THE for the cargo) are bound to contribute to the
FACTS VESSEL ONLY, AND TO PAY PROPORTIONATE indemnity of the general average. The provision
- Plaintiff’s motorboat, “Pandan” was insured on a SALVAGE CHARGES OF THE DECLARED VALUE.” simply places the insurer on the same footing as
marine insurance policy with National Union Fire other persons who have an interest in the vessel, or
Insurance Company (NUFIC) for P45K. According to ISSUES the cargo therein, at the time of the occurrence of
the provisions of a “rider” attached to the policy, the 1. WON the lower court erred in disregarding the the general average and who are compelled to
insurance was against the “absolute total loss of the typewritten clause endorsed upon the policy, contribute (Art. 812, Code of Commerce).
vessel only.” On Oct. 31, 1928, the ship ran into very expressly limiting insurer's liability thereunder of the - In the present case it is not disputed that the ship
heavy sea and it became necessary to jettison a total loss of the wooden vessel Pandan and to was in grave peril and that the jettison of part of the
portion of the cargo. As a result of the jettison, the proportionate salvage charges cargo was necessary. If the cargo was in peril to the
NUFIC was assessed P2,610.86 as its contribution to 2. WON lower court erred in concluding that extent of call for general average, the ship must also
the general average. defendant and appellant, NUFIC is liable to have been in great danger, possibly sufficient to
cause its absolute loss. The jettison was therefore as
INSURANCE Page
91

much to the benefit of the underwriter as to the permitting the continued flow of the salt water provide the vessel with proper equipment to convey
owner of the cargo. The latter was compelled to into the compartment of rice. the cargo under ordinary conditions, is not a peril of
contribute to the indemnity; why should not the - The court found in effect that the opening above the sea. Such a loss is rather due to what has been
insurer be required to do likewise? If no jettison had described had resulted in course of time from aptly called the "peril of the ship." The insurer
taken place and if the ship by reason thereof had ordinary wear and tear and not from the straining of undertakes to insure against perils of the sea and
foundered, the underwriter's loss would have been the ship in rough weather on that voyage. The court similar perils, not against perils of the ship. There
many times as large as the contribution now also found that the repairs that had been made on must, in order to make the insurer liable, be "some
demanded. the pipe were slovenly and defective and that, by casualty, something which could not be foreseen as
Disposition Appealed judgment is affirmed reason of the condition of this pipe, the ship was not one of the necessary incidents of the adventure. The
properly equipped to receive the rice at the time the purpose of the policy is to secure an indemnity
GO TIACO v. UNION INSURANCE voyage was begun. For this reason the court held against accidents which may happen, not against
40 PHIL 40 that the ship was unseaworthy. events which must happen." (Wilson, Sons & Co. vs.
STREET; September 1, 1919 - The policy purports to insure the cargo from the Owners of Cargo per the Xantho)
following among other risks: "Perils . . . of the seas, - In the present case the entrance of the sea water
FACTS men, of war, fire, enemies, pirates, rovers, into the ship's hold through the defective pipe
- Union Insurance Society of Canton, Ltd., issued a thieves, .jettisons, . . . barratry of the master and already described was not due to any accident which
marine insurance policy upon a cargo of rice mariners, and of all other perils, losses, and happened during the voyage, but to the failure of the
belonging to the Go Tiaoco Brothers, which was misfortunes that have or shall come to the hurt, ship's owner properly to repair a defect of the
transported in the early days of May, 1915, on the detriment, or damage of the said goods and existence of which he was apprised. The loss was
steamship Hondagua from the port of Saigon to merchandise or any part thereof." therefore more analogous to that which directly
Cebu. results from simple unseaworthiness than to that
- On discharging the rice from one of the which results from perils of the sea.
compartments in the after hold, upon arrival at ISSUE - there is no room to doubt the liability of the
Cebu, it was discovered that 1473 sacks had been WON Union Insurance is liable for the loss of the Go shipowner for such a loss as occurred in this case. By
damaged by sea water. The loss was P3,875.25. Tiaco Brothers parity of reasoning the insurer is not liable; for,
- The trial court found that the inflow of the sea generally speaking, the shipowner excepts the perils
water during the voyage was due to a defect in one HELD of the sea from his engagement under the bill of
of the drain pipes of the ship and concluded that the NO lading, while this is the very peril against which the
loss was not covered by the policy of insurance. The - the words "all other perils, losses, and misfortunes" insurer intends to give protection. As applied to the
trial court made the ff findings: are to be interpreted as covering risks which are of present case it results that the owners of the
The drain pipe which served as a discharge like kind (ejusdem generis) with the particular risks damaged rice must look to the shipowner for redress
from the water closet passed down through which are enumerated in the preceding part of the and not to the insurer.
the compartment where the rice in question same clause of the contract. ''According to the The same conclusion must be reached if the question
was stowed and thence out to sea through the ordinary rules of construction, these words must be be discussed with reference to the seaworthiness of
wall of the compartment, which was a part of interpreted with reference to the words which the ship. It is universally accepted that in every
the wall of the ship. The joint or elbow where immediately precede them. They were no doubt contract of insurance upon anything which is the
the pipe changed its direction was of cast inserted in order to prevent disputes founded on nice subject of marine insurance, a warranty is implied
iron; and in course of time it had become distinctions. X x x For example, if the expression that the ship shall be seaworthy at the time of the
corroded and abraded until a longitudinal 'perils of the seas' is given its widest sense the inception of the voyage. This rule is accepted in our
opening had appeared in the pipe about one general words have little or no effect as applied to own Insurance Law (Act No. 2427, sec. 106). It is
inch in length. This hole had been in existence that case. If on the other hand that expression is to also well settled that a ship which is seaworthy for
before the voyage was begun, and an attempt receive a limited construction, as apparently it did in the purpose of insurance upon the ship may yet be
had been made to repair it by filling with Cullen vs. Butler (5 M. & S., 461), and loss by perils unseaworthy for the purpose of insurance upon the
cement and bolting over it a strip of iron. The of the seas is to be confined to loss ex marine cargo (Act No. 2427, sec. 106).
effect of loading the boat was to submerge tempestatis discrimine, the general words become Disposition Decision of trial court is affirmed
the vent, or orifice, of the pipe until it was most important. X x x" (Thames and Mersey Marine
about 18 inches or 2 feet below the level of Insurance Co. vs. Hamilton, Fraser & Co.) CATHAY INSURANCE CO. v. CA (REMINGTON
the sea. As a consequence the sea water rose - a loss which, in the ordinary course of events, INDUSTRIAL SALES CORP.)
in the pipe. Navigation under these conditions results from the natural and inevitable action of the 151 SCRA 710
resulted in the washing out of the cement- sea, from the ordinary wear and tear of the ship, or PARAS; June 30 1987
filling from the action of the sea water, thus from the negligent failure of the ship's owner to
INSURANCE Page
92

FACTS accountable therefor, We would fail to observe a - The IAC found that one of the hatches was left
- Remington Industrial Sales Corp insured its cardinal rule in the interpretation of contracts, open, causing water to enter the barge and because
shipment of seamless steel pipes. It incurred losses namely, that any ambiguity therein should be the barge was not provided with the necessary cover
and damages (I gather the steel pipes rusted during construed against the maker/issuer/drafter thereof, or tarpaulin, the splash of sea waves brought more
the voyage from Japan to the Phils. on board vessel namely, the insurer. Besides the precise purpose of water inside the barge.
SS "Eastern Mariner”) and filed complaint against insuring cargo during a voyage would be rendered - Petitioners contend that the implied warranty of
Cathay Insurance Co seeking collection of the sum of fruitless. seaworthiness provided for in the Insurance Code
P868,339.15 Disposition WHEREFORE, this petition is hereby refers only to the responsibility of the shipowner who
- TC decided for Remington. Cathay filed MR, which DENIED, and the assailed decision of the Court of must see to it that his ship is reasonably fit to make
was denied. CA affirmed. Appeals is hereby AFFIRMED. in safety the contemplated voyage.
- CA said (among other things): 1. Coverage of - The petitioners state that a mere shipper of cargo,
private respondent's loss under the insurance policy ROQUE v. IAC (PIONEER INSURANCE AND having no control over the ship, has nothing to do
issued by petitioner is unmistakable; SURETY CORP.) with its seaworthiness. They argue that a cargo
2. Alleged contractual limitations contained in 139 SCRA 596 owner has no control over the structure of the ship,
insurance policies are regarded with extreme caution GUTIERREZ; November 11, 1985 its cables, anchors, fuel and provisions, the manner
by courts and are to be strictly construed against the of loading his cargo and the cargo of other shippers,
insurer; obscure phrases and exceptions should not NATURE and the hiring of a sufficient number of competent
be allowed to defeat the very purpose for which the Petition for certiorari to review the decision of the officers and seamen.
policy was procured; IAC ISSUE
3. Rust is not an inherent vice of the seamless steel WON the loss should have been covered by the
pipes without interference of external factors FACTS marine insurance policy
- Cathay contend (among other things): 1. private - February 19, 1972 – Common carrier Manila Bay
respondent has admitted that the questioned Lighterage Corp. entered into a contract with Roque HELD
shipment is not covered by a "square provision of the Timber Enterprises and Chiong. The contract stated NO
contract," but private respondent claims implied that Manila Bay would carry 422.18 cu. meters of Ratio It is universally accepted that in every
coverage from the phrase "perils of the sea" logs on its vessel Mable 10 from Malampaya Sound, contract of insurance upon anything which is the
mentioned in the opening sentence of the policy; 2. Palawan to Manila North Harbor. Roque insured the subject of marine insurance, a warranty is implied
The insistence of private respondent that rusting is a logs with Pioneer Insurance for P100,000. that the ship shall be seaworthy at the time of the
peril of the sea is erroneous; 3. Rusting is not a - February 29, 1972 – 811 logs were loaded in inception of the voyage. In marine insurance, the
risk insured against, since a risk to be insured Malampaya but en route to Manila, Mable 10 sank. risks insured against are classified as 'perils of the
against should be a casualty or some casualty, - March 8,1972 – Roque and Chiong wrote a letter to sea,’ which includes such losses that are of
something which could not be foreseen as one of the Manila Bay, demanding payment of P150,000.00 for extraordinary nature, or arise from some
necessary incidents of adventure; 4. A fact capable the loss of the shipment plus P100,000.00 as overwhelming power, which cannot be guarded
of unquestionable demonstration or of public unrealized profits but the latter ignored the demand. against by the ordinary exertion of human skill and
knowledge needs no evidence. This fact of - A letter was also sent to Pioneer, claiming the full prudence.
unquestionable demonstration or of public knowledge amount of P100,000.00 under the insurance policy Reasoning
is that heavy rusting of steel or iron pipes cannot but Pioneer refused to pay on the ground that its - Based on Sec. 113 and Sec. 99 of the Insurance
occur within a period of a seven (7) day voyage. liability depended upon the "Total Loss by Total Loss Code, the term "cargo" can be the subject of marine
Besides, petitioner had introduced the clear cargo of Vessel only". insurance and that once it is so made, the implied
receipts or tally sheets indicating that there was no - After hearing, the trial court favored Roque. warranty of seaworthiness immediately attaches to
damage on the steel pipes during the voyage. Pioneer and Manila Bay were ordered to pay Roque whoever is insuring the cargo whether he be the
P100,000. Pioneer appealed the decision. shipowner or not.
ISSUE - January 30, 1984 – Pioneer was absolved from - The fact that the un-seaworthiness of the ship was
WON rusting is a “peril of the sea” liability after finding that there was a breach of unknown to the insured is immaterial in ordinary
implied warranty of seaworthiness on the part of the marine insurance and may not be used by him as a
HELD petitioners and that the loss of the insured cargo was defense in order to recover on the marine insurance
YES caused by the "perils of the ship" and not by the policy.
- There is no question that the rusting of steel pipes "perils of the sea". It ruled that the loss is not - Since the law provides for an implied warranty of
in the course of a voyage is a "peril of the sea" in covered by the marine insurance policy. seaworthiness in every contract of ordinary marine
view of the toll on the cargo of wind, water, and salt - It was alleged that Mable 10 was not seaworthy insurance, it becomes the obligation of a cargo
conditions. At any rate if the insurer cannot be held and that it developed a leak owner to look for a reliable common carrier which
INSURANCE Page
93

keeps its vessels in seaworthy condition. The shipper the pipe were slovenly and defective and that, by already described was not due to any accident which
of cargo my have no control over the vessel but he reason of the condition of this pipe, the ship was not happened during the voyage, but to the failure of the
has full control in the choice of the common carrier properly equipped to receive the rice at the time the ship's owner properly to repair a defect of the
that will transport his goods. voyage was begun. For this reason the court held existence of which, he was apprised. The loss was
- In marine cases, the risks insured against are that the ship was unseaworthy. therefore more analogous to that which directly
'perils of the sea.’ The term extends only to losses results from simple unseaworthiness than to that
caused by sea damage, or by the violence of the ISSUE which results from perils of the sea.
elements, and does not embrace all losses happening WON the insurer is liable - It is universally accepted that in every contract of
at sea. insurance upon anything which is the subject of
- It is quite unmistakable that the loss of the cargo HELD marine insurance, a warranty is implied that the ship
was due to the perils of the ship rather than the - The question whether the insurer is liable on this shall be seaworthy at the time of the inception of the
perils of the sea. policy for the loss caused in the manner above stated voyage. This rule is accepted in our own Insurance
- Loss which, in the ordinary course of events, presents two phases which are in a manner involved Law (Act No. 2427, see. 106).
results from the natural and inevitable action of the with each other. One has reference to the meaning - It is also well settled that a ship which is
sea, from the ordinary wear and tear of the ship, or of the expression "perils of the seas and all other seaworthy for the purpose of insurance upon the ship
from the negligent failure of the ship's owner to perils, losses, and misfortunes," as used in the may yet be unseaworthy for the purpose of
provide the vessel with proper equipment to convey policy; the other has reference to the implied insurance upon the cargo (Act No. 2427, see. 106).
the cargo under ordinary conditions, is not a ‘peril of warranty, on the part of the insured, as to the Disposition Jjudgment affirmed.
the sea’ but is called ‘peril of the ship.’ seaworthiness of the ship.
Disposition Decision appealed from is affirmed. - The meaning of the expression "perils * * * of the MALAYAN INSURANCE v. CA (supra p.10)
seas * * * and all other perils, losses, and
LA RAZON v. UNION INSURANCE SOCIETY OF misfortunes," used in describing the risks covered by FILIPINO MERCHANTS INS. CO. v. CA (supra
CANTON, LTD. policies of marine insurance, has been the subject of p.19)
40 PHIL 40 frequent discussion; and certain propositions relative
STREET; September 1, 1919 thereto are now so generally accepted as to be COASTWISE LIGHTERAGE CORP v. CA
considered definitely settled. (PHILIPPINE GENERAL INSURANCE COMPANY)
FACTS - The words "all other perils, losses, and 245 SCRA 796
- This is an action on a policy of marine insurance misfortunes" are to be interpreted as covering risks FRANCISCO; July 12, 1995
issued by the Union Insurance Society of Canton, which are of like kind (ejusdem generis) with the
Ltd., upon a cargo of rice belonging to the plaintiffs, particular risks which are enumerated in the NATURE
Go Tiaoco Brothers, which was transported on the preceding part of the same clause of the contract. Petition for review of CA Decision affirming decision
steamship Hondagua from the port of Saigon to - A loss which, in the ordinary course of events, of RTC Manila holding that Coastwise is liable to pay
Cebu. results from the natural and inevitable action of the PhilGen Insurance the amount of P700thou plus legal
- On discharging the rice from one of the sea, from the ordinary wear and tear of the ship, or interest thereon, another sum of P100thou as
compartments in the after hold, upon arrival at from the negligent failure of the ship's owner to attorney's fees and the cost of the suit.
Cebu, it was discovered that 1,473 sacks had been provide the vessel with proper equipment to convey
damaged by sea water. the cargo under ordinary conditions, is not a peril of FACTS
- The loss so resulting to the owners of rice, after the sea. Such a loss is rather due to what has been - Pag-asa Sales, Inc. entered into a contract to
proper deduction had been made for the portion aptly called the "peril of the ship." The insurer transport molasses from the province of Negros to
saved, was P3,875.25. undertakes to insure against perils of the sea and Manila with Coastwise, using the latter's dumb
- The trial court found that the inflow of the sea similar perils, not against perils of the ship. barges. The barges were towed in tandem by the
water during the voyage was due to a defect in one - As was said by Lord Herschell in Wilson, Sons & Co. tugboat MT Marica, also owned by Coastwise. Upon
of the drain pipes of the ship and concluded that the vs. Owners of Cargo per the Xantho, there must, in reaching Manila Bay, while approaching Pier 18, one
loss was not covered by the policy of insurance. order to make the insurer liable, be "some casualty, of the barges struck an unknown sunken object. The
Judgment was accordingly entered in favor of the something which could not be foreseen as one of the forward buoyancy compartment was damaged, and
defendant and the plaintiffs appealed. necessary incidents of the adventure. The purpose of water gushed in through a hole "two inches wide and
- The court found in effect that the opening above the policy is to secure an indemnity against accidents twenty-two inches long."
described had resulted in course of time from which may happen, not against events which must - As a consequence, the molasses at the cargo tanks
ordinary wear and tear and not from the straining of happen." were contaminated and rendered unfit for the use it
the ship in rough weather on that voyage. The court - In the present case the entrance of the sea water was intended. This prompted consignee Pag-asa
also found that the repairs that had been made on into the ship's hold through the defective pipe Sales to reject the shipment of molasses as a total
INSURANCE Page
94

loss. Thereafter, Pag-asa Sales filed a formal claim or all of its space to haul goods for others. It is a Captains, masters, or patrons of vessels must be
with the insurer of its lost cargo (PhilGen) and contract for special service to be rendered by Filipinos, have legal capacity to contract in
against the carrier (Coastwise). Coastwise denied the the owner of the vessel and under such contract accordance with this code, and prove the skill
claim and it was PhilGen which paid Pag-asa Sales the general owner retains the possession, capacity and qualifications necessary to command
the amount of P700k representing the value of the command and navigation of the ship, the and direct the vessel, as established by marine and
damaged cargo of molasses. charterer or freighter merely having use of the navigation laws, ordinances or regulations, and must
- PhilGen then filed an action against Coastwise space in the vessel in return for his payment of not be disqualified according to the same for the
before the RTC Manila, seeking to recover the P700k the charter hire... discharge of the duties of the position.”
which it paid to Pag-asa Sales for the latter's lost An owner who retains possession of the ship - Clearly, Coastwise Lighterage's embarking on a
cargo. PhilGen now claims to be subrogated to all the though the hold is the property of the charterer, voyage with an unlicensed patron violates this rule.
contractual rights and claims which the consignee remains liable as carrier and must answer for It cannot safely claim to have exercised
may have against the carrier, which is presumed to any breach of duty as to the care, loading and extraordinary diligence, by placing a person whose
have violated the contract of carriage. unloading of the cargo.” navigational skills are questionable, at the helm of
- RTC awarded the amount prayed for by PhilGen. CA - Although a charter party may transform a common the vessel which eventually met the fateful accident.
affirmed. Hence, this petition. carrier into a private one, the same however is not It may also logically, follow that a person without
true in a contract of affreightment on account of the license to navigate, lacks not just the skill to do so,
ISSUES aforementioned distinctions between the two. but also the utmost familiarity with the usual and
1. WON Coastwise Lighterage was transformed into a - SC agrees with Coastwise's admission that the safe routes taken by seasoned and legally authorized
private carrier, by virtue of the contract of contract it entered into with the consignee was one ones. Had the patron been licensed, he could be
affreightment which it entered into with the of affreightment. Pag-asa Sales, Inc. only leased presumed to have both the skill and the knowledge
consignee, Pag-asa Sales, Inc. (Corollarily, if it were three of petitioner's vessels, in order to carry cargo that would have prevented the vessel's hitting the
in fact transformed into a private carrier, did it from one point to another, but the possession, sunken derelict ship that lay on their way to Pier 18.
exercise the ordinary diligence to which a private command and navigation of the vessels remained - As a common carrier, Coastwise is liable for breach
carrier is in turn bound?) with Coastwise. As such, Coastwise, by the contract of the contract of carriage, having failed to overcome
2. WON the insurer was subrogated into the rights of of affreightment, was not converted into a private the presumption of negligence with the loss and
the consignee against the carrier, upon payment by carrier, but remained a common carrier and was still destruction of goods it transported, by proof of its
the insurer of the value of the consignee's goods lost liable as such. exercise of extraordinary diligence.
while on board one of the carrier's vessels - Therefore, the mere proof of delivery of goods in 2. YES
good order to a carrier and the subsequent arrival of - Coastwise is liable for breach of the contract of
HELD the same goods at the place of destination in bad carriage it entered into with Pag-asa Sales, Inc.
1. NO order makes for a prima facie case against the However, for the damage sustained by the loss of
- The distinction between the two kinds of charter carrier. The presumption of negligence that attaches the cargo which petitioner-carrier was transporting,
parties (i.e. bareboat or demise and contract of to common carriers, once the goods it transports are it was not the carrier which paid the value thereof to
affreightment) is more clearly set out in the case of lost, destroyed or deteriorated, applies to Coastwise. Pag-asa Sales, Inc. but the latter's insurer, herein
Puromines, Inc. vs. Court of Appeals, wherein SC This presumption, which is overcome only by proof of private respondent PhilGen.
ruled: the exercise of extraordinary diligence, remained - Article 2207 of the Civil Code: If the plaintiffs
“Under the demise or bareboat charter of the unrebutted in this case. property has been insured, and he has received
vessel, the charterer will generally be regarded - The damage to the barge which carried the cargo of indemnity from the insurance company for the injury
as the owner for the voyage or service molasses was caused by its hitting an unknown or loss arising out of the wrong or breach of contract
stipulated. The charterer mans the vessel with sunken object as it was heading for Pier 18. The complained of, the insurance company shall be
his own people and becomes the owner pro hac object turned out to be a submerged derelict vessel. subrogated to the rights of the insured against the
vice, subject to liability to others for damages The evidence on record appeared that far from wrongdoer or the person who violated the contract.”
caused by negligence. To create a demise, the having rendered service with the greatest skill and - This legal provision is founded on the well-settled
owner of a vessel must completely and utmost foresight, and being free from fault, the principle of subrogation. If the insured property is
exclusively relinquish possession, command and carrier was culpably remiss in the observance of its destroyed or damaged through the fault or
navigation thereof to the charterer, anything duties. negligence of a party other than the assured, then
short of such a complete transfer is a contract of - Jesus R. Constantino, the patron of the vessel the insurer, upon payment to the assured will be
affreightment (time or voyage charter party) or "Coastwise 9" admitted that he was not licensed. The subrogated to the rights of the assured to recover
not a charter party at all. Code of Commerce, which subsidiarily governs from the wrongdoer to the extent that the insurer
On the other hand a contract of affreightment is common carriers (which are primarily governed by has been obligated to pay. Payment by the insurer to
one in which the owner of the vessel leases part the provisions of the Civil Code) provides: “Art. 609. the assured operated as an equitable assignment to
INSURANCE Page
95

the former of all remedies which the latter may have unstable condition. It further alleged that the against FELMAN, the shipowner
against the third party whose negligence or wrongful vessel was improperly manned and that its
act caused the loss. The right of subrogation is not officers were grossly negligent in failing to take HELD
dependent upon, nor does it grow out of, any privity appropriate measures to proceed to a nearby port or 1. YES
of contract or upon written assignment of claim. It beach after the vessel started to list. - “MV Asilda” was unseaworthy when it left the port
accrues simply upon payment of the insurance claim - FELMAN filed a motion to dismiss based on the of Zamboanga. We subscribe to the findings of the
by the insurer. affirmative defense that no right of subrogation in Elite Adjusters, Inc., and the Court of Appeals that
- Undoubtedly, upon payment by respondent insurer favor of PHILAMGEN was transmitted by the shipper, the proximate cause of the sinking of “MV Asilda”
PhilGen of the amount of P700,000.00 to Pag-asa and that, in any event, FELMAN had abandoned all was its being top-heavy. Contrary to the ship
Sales, Inc., the consignee of the cargo of molasses its rights, interests and ownership over “MV Asilda” captain’s allegations, evidence shows that
totally damaged while being transported by together with her freight and appurtenances for the approximately 2,500 cases of softdrink bottles were
petitioner Coastwise Lighterage, the former was purpose of limiting and extinguishing its liability stowed on deck. Several days after “MV Asilda”
subrogated into all the rights which Pag-asa Sales, under Art. 587 of the Code of Commerce. sank, an estimated 2,500 empty Coca-Cola plastic
Inc. may have had against the carrier, herein - Trial court dismissed the complaint of PHILAMGEN. cases were recovered near the vicinity of the sinking.
petitioner Coastwise Lighterage. On appeal the Court of Appeals set aside the Considering that the ship’s hatches were properly
Disposition Petition denied. CA affrimed. dismissal and remanded the case to the lower court secured, the empty Coca-Cola cases recovered could
for trial on the merits. FELMAN filed a petition for have come only from the vessel’s deck cargo. It is
THE PHILIPPINE AMERICAN GENERAL certiorari with this Court but it was subsequently settled that carrying a deck cargo raises the
INSURANCE COMPANY INC v. CA (FELMAN denied on 13 February 1989. presumption of unseaworthiness unless it can be
SHIPPING LINES) - Trial court rendered judgment in favor of FELMAN. shown that the deck cargo will not interfere with the
273 SCRA 226 It ruled that “MV Asilda” was seaworthy when it left proper management of the ship. However, in this
BELLOSILLO; June 11, 1997 the port of Zamboanga as confirmed by certificates case it was established that “MV Asilda” was not
issued by the Philippine Coast Guard and the designed to carry substantial amount of cargo on
FACTS shipowner’s surveyor attesting to its seaworthiness. deck. The inordinate loading of cargo deck resulted
- Coca-Cola Bottlers Philippines, Inc., loaded on Thus the loss of the vessel and its entire shipment in the decrease of the vessel’s metacentric height
board “MV Asilda,” a vessel owned and operated by could only be attributed to either a fortuitous event, thus making it unstable. The strong winds and
Felman 7,500 cases of 1-liter Coca-Cola softdrink in which case, no liability should attach unless there waves encountered by the vessel are but the
bottles to be transported from Zamboanga City to was a stipulation to the contrary, or to the ordinary vicissitudes of a sea voyage and as such
Cebu for consignee Coca-Cola Bottlers Philippines, negligence of the captain and his crew, in which merely contributed to its already unstable and
Inc., Cebu. The shipment was insured with petitioner case, Art. 587 of the Code of Commerce should unseaworthy condition.
Philippine American General under Marine Open apply. 2. NO
Policy. - CA ruled that “MV Asilda” was unseaworthy for - The ship agent is liable for the negligent acts of the
- The vessel sank in the waters of Zamboanga del being top- heavy as 2,500 cases of Coca-Cola captain in the care of goods loaded on the vessel.
Norte bringing down her entire cargo with her softdrink bottles were improperly stowed on deck. This liability however can be limited through
including the subject 7,500 cases of 1-liter Coca-Cola Nonetheless, the appellate court denied the claim of abandonment of the vessel, its equipment and
softdrink bottles. PHILAMGEN on the ground that the assured’s implied freightage as provided in Art. 587. Nonetheless,
- The consignee filed a claim with respondent warranty of seaworthiness was not complied with. there are exceptional circumstances wherein the
FELMAN for recovery of damages it sustained as a Perfunctorily, PHILAMGEN was not properly ship agent could still be held answerable despite the
result of the loss of its softdrink bottles that sank subrogated to the rights and interests of the shipper. abandonment, as where the loss or injury was due to
with “MV Asilda.” Respondent denied the claim Furthermore, respondent court held that the filing of the fault of the shipowner and the captain. The
thus prompting the consignee to file an insurance notice of abandonment had absolved the international rule is to the effect that the right of
claim with PHILAMGEN which paid its claim of shipowner/agent from liability under the limited abandonment of vessels, as a legal limitation of a
P755,250.00. liability rule. shipowner’s liability, does not apply to cases where
- Claiming its right of subrogation PHILAMGEN the injury or average was occasioned by the
sought recourse against respondent FELMAN which ISSUES shipowner’s own fault.
disclaimed any liability for the loss. Consequently, 1. WON “MV Asilda” was seaworthy when it left the 3. YES
PHILAMGEN sued the shipowner for sum of money port of Zamboanga - The doctrine of subrogation has its roots in equity.
and damages. 2. WON the limited liability under Art. 587 of the It is designed to promote and to accomplish justice
- PHILAMGEN alleged that the sinking and total loss Code of Commerce should apply and is the mode which equity adopts to compel the
of “MV Asilda” and its cargo were due to the vessel’s 3. WON PHILAMGEN was properly subrogated to the ultimate payment of a debt by one who in justice,
unseaworthiness as she was put to sea in an rights and legal actions which the shipper had equity and good conscience ought to pay. Therefore,
INSURANCE Page
96

the payment made by PHILAMGEN to Coca-Cola the date of judicial demand, pursuant to Arts. 2212 and as applied to the conditions existing in the
Bottlers Philippines, Inc., gave the former the right and 2213 of the Civil Code. Manila Bay, this kind of policy would be worthless,
to bring an action as subrogee against FELMAN. and there would not be any consideration for the
Having failed to rebut the presumption of fault, the PHILIPPINE MFTG. CO. v. UNION INSURANCE premium.
liability of FELMAN for the loss of the 7,500 cases of SOCIETY OF CANTON 2. NO
1-liter Coca-Cola softdrink bottles is inevitable. 42 PHIL 378 - The defendant argues that the policy contains the
- Sec. 113 of the Insurance Code provides that “(i)n JOHNS; November 22, 1921 provision that it “shall be of as force and effect as
every marine insurance upon a ship or freight, or the surest writing or policy of insurance made in
freightage, or upon anything which is the subject of FACTS London”. However, for such law to apply to our
marine insurance, a warranty is implied that the ship - The plaintiff’s steel tank lighter was insured by courts the existence of such law must be proven. It
is seaworthy.” Under Sec. 114, a ship is “seaworthy defendant company for absolute total loss. As a cannot apply when such proof is lacking.
when reasonably fit to perform the service, and to result of a typhoon, the lighter sunk in Manila Bay. Nevertheless, in the English practice, a ship is a total
encounter the ordinary perils of the voyage, The plaintiff demanded payment from the defendant loss when she has sustained such extensive damages
contemplated by the parties to the policy.” Thus it insurance company but the latter refused. The that it would not be reasonably practical to repair
becomes the obligation of the cargo owner to look company asked the plaintiff to salvage the ship, her.
for a reliable common carrier which keeps its vessels which it was able to do so. Disposition Decision reversed
in seaworthy condition. He may have no control - With the plaintiff able to raise the lighter,
over the vessel but he has full control in the reconstruct it and placed it in commission, the
selection of the common carrier that will transport defendant insurance company claims that it was only
his goods. He also has full discretion in the choice of liable for a total absolute loss and that there was no
assurer that will underwrite a particular venture. total destruction of the lighter.
- In policies where the law will generally imply a - The trial court decided in favor of the defendant,
warranty of seaworthiness, it can only be excluded saying that the policy only covered an actual total
by terms in writing in the policy in the clearest loss, not a constructive total loss.
language. And where the policy stipulates that the
seaworthiness of the vessel as between the assured ISSUES
and the assurer is admitted, the question of 1. WON there was an absolute total loss that can be CHOA TIEK SENG v. CA (FILIPINO MERCHANTS
seaworthiness cannot be raised by the assurer covered by the policy INSURANCE)
without showing concealment or misrepresentation 2. WON the Marine Law of Great Britain applies 183 SCRA 223
by the assured. GANCAYO; March 15, 1990
- PHILAMGEN’s action against FELMAN is squarely HELD
sanctioned by Art. 2207 of the Civil Code which 1. YES NATURE
provides: - At the time that the lighter was at the bottom of Appeal from a decision of the Court of Appeals
Art. 2207. If the plaintiff’s property has been the bay, it was of no value to the owner, thus there
insured, and he has received indemnity from was an actual total loss. FACTS
the insurance company for the injury or loss - The ship was sunk in July 1, 1918. After several - Petitioner imported some lactose crystals from
arising out of the wrong or breach of contract futile attempts, it was finally raised on Sept. 20, Holland.
complained of, the insurance company shall 1918. It is faitr to assume that in its then condition - The importation involved fifteen (15) metric tons
be subrogated to the rights of the insured much further time would be required to make the packed in 600 6-ply paper bags with polythelene
against the wrongdoer or the person who has necessary repairs and install the new machinery inner bags, each bag at 25 kilos net. The goods were
violated the contract. If the amount paid by before it could again be placed in commission. loaded at the port at Rotterdam in sea vans on board
the insurance company does not fully cover During that time the owner would be deprived of the the vessel "MS Benalder' as the mother vessel, and
the injury or loss, the aggrieved party shall use of its vessel or the interest on its investment. thereafter aboard the feeder vessel "Wesser Broker
be entitled to recover the deficiency from the When those questions are considered the testimony V-25" of respondent Ben Lines Container, Ltd. (Ben
person causing the loss or injury. is conclusive that the cost of salvage, repair and Lines for short). The goods were insured by the
Disposition Petition is GRANTED. Respondent reconstruction was more than the original cost of the respondent Filipino Merchants' Insurance Co., Inc.
FELMAN SHIPPING LINES is ordered to pay petitioner ship at the time the policy was issued. As found by (insurance company for short) for the sum of
PHILIPPINE AMERICAN GENERAL INSURANCE CO., the trial court, “t is difficult to see how there could P98,882.35, the equivalent of US$8,765.00 plus 50%
INC., Seven Hundred Fifty-five Thousand Two have been a more complete loss of the vessel than mark-up or US $13,147.50, against all risks under
Hundred and Fifty Pesos (P755,250.00) plus legal that which actually occurred”. Upon the facts shown the terms of the insurance cargo policy. Upon arrival
interest thereon counted from 29 November 1983, here, any other construction would nullify the statute at the port of Manila, the cargo was discharged into
INSURANCE Page
97

the custody of the arrastre operator respondent E. "5. This insurance is against all risks of loss or from Boston, U.S.A. a shipment of 1 skid carton
Razon, Inc. (broker for short), prior to the delivery to damage to the subject matter insured but shall in parts for valves. The shipment was ordered from
petitioner through his broker. Of the 600 bags no case be deemed to extend to cover loss, Jamesbury, Singapore PTE, LTD., which issued the
delivered to petitioner, 403 were in bad order. The damage, or expense proximately caused by delay cargo's packing list and Invoice number showing the
surveys showed that the bad order bags suffered or inherent vice or nature of the subject matter contents of the carton. The Philippine Consulate in
spillage and loss later valued at P33,117.63. insured. Claims recoverable hereunder shall be Singapore issued invoice for the shipment showing
Petitioner filed a claim for said loss dated February payable irrespective of percentage." the contents and its total price of $39,419.60 and
16, 1977 against respondent insurance company in - The terms of the policy are so clear and require no the freight and other charges of $2,791.73. When
the amount of P33,117.63 as the insured value of interpretation. The insurance policy covers all loss or the cargo arrived in Manila, it was received and
the loss. damage to the cargo except those caused by delay deposited in the office of Aboitiz Shipping
- Respondent insurance company rejected the claim or inherent vice or nature of the cargo insured. It is Corporation (Aboitiz) for transhipment to Nonoc
alleging that assuming that spillage took place while the duty of the respondent insurance company to Island.
the goods were in transit, petitioner and his agent establish that said loss or damage falls within the - In July 1980, Marinduque, as consignee of the
failed to avert or minimize the loss by failing to exceptions provided for by law, otherwise it is liable cargo, made a report that said cargo was pilfered on
recover spillage from the sea van, thus violating the therefor. July 3, 1980 due to heavy rain at the Aboitiz
terms of the insurance policy sued upon; and that - An "all risks" provision of a marine policy creates a terminal and that of the total value of the cargo of
assuming that the spillage did not occur while the special type of insurance which extends coverage to $42,209.33, only $7,412.00 worth remains of the
cargo was in transit, the said 400 bags were loaded risks not usually contemplated and avoids putting cargo with the recommendation that the claim be
in bad order, and that in any case, the van did not upon the insured the burden of establishing that the made against Aboitiz.
carry any evidence of spillage. loss was due to peril falling within the policy's - The services of the Manila Adjusters and Surveyors
- Petitioner filed a complaint in the RTC against the coverage. The insurer can avoid coverage upon Co. (Manila Adjusters) were engaged by the Phil-
insurance company seeking payment of the sum of demonstrating that a specific provision expressly American General Insurance Co., Inc. (Phil Am)
P33,117.63 as damages plus attorney's fees and excludes the loss from coverage. which came out with the report that the cargo in
expenses of litigation. Insurance company denied all - In this case, the damage caused to the cargo has question, when inspected, showed that it was
the material allegations of the complaint and raised not been attributed to any of the exceptions provided pilfered. A confirmatory report was submitted by the
several special defenses as well as a compulsory for nor is there any pretension to this effect. Thus, Manila Adjusters.
counterclaim. Insurance company filed a third-party the liability of respondent insurance company is - On August 11, 1980 Marinduque then filed a claim
complaint against respondents Ben Lines and broker. clear. against Aboitiz in the amount of P246,430.80
- RTC dismissed the complaint, the counterclaim and Disposition the decision appealed from is hereby representing the value of the pilfered cargo. On the
the third-party complaint with costs against the REVERSED AND SET ASIDE and another judgment is same day Marinduque filed a claim for the same
petitioner. Appealed in CA but denied. MFR was hereby rendered ordering the respondent Filipinas amount against the Phil-Am on the latter's policy.
denied as well. Merchants Insurance Company, Inc. to pay the sum Phil-Am paid Marinduque the sum of P246,430.80 as
of P33,117.63 as damages to petitioner with legal insurer of the cargo.
ISSUE interest from the filing of the complaint, plus - Phil-Am then filed a complaint in RTC Manila
WON insurance company should be held liable even attorney's fees and expenses of litigation in the against Aboitiz for recovery of same amount alleging
if the technical meaning in marine insurance of an amount of P10,000.00 as well as the costs of the that it has been subrogated to the rights of
“insurance against all risk" is applied suit. Marinduque. Complaint dismissed and MFR denied.
CA reversed. MFR thereof was denied. Hence, this
HELD FILIPINO MERCHANTS INS. CO. v. CA (supra petition.
YES p.19)
- In Gloren Inc. vs. Filipinas Cia. de Seguros, 12 it ISSUE
was held that an all risk insurance policy insures ABOITIZ SHIPPING v. PHILAMGEN INSURANCE WON petitioner Aboitiz was properly held liable to the
against all causes of conceivable loss or damage, 179 SCRA 357 private respondent Phil-Am by the appellate court
except as otherwise excluded in the policy or due to GANCAYCO; October 5, 1989
fraud or intentional misconduct on the part of the HELD
insured. It covers all losses during the voyage NATURE YES
whether arising from a marine peril or not, including Petition for review on certiorari - The questioned shipment is covered by a
pilferage losses during the war. continuing open insurance coverage (which took
- In the present case, the "all risks" clause of the FACTS effect after Sept. 1, 1975, as contained in Marine
policy sued upon reads as follows: - Marinduque Mining Industrial Corporation Open Policy No. 100184) from the time it was loaded
(Marinduque) shipped on board SS Arthur Maersk aboard the SS Arthur Maersk in Boston, U.S.A. to the
INSURANCE Page
98

time it was delivered to the possession of petitioner and insured it against loss for P1-M with petitioner (b) The irretrievable loss of the thing by sinking, or
at its offices at Pier 4 in Manila until it was pilfered Oriental Assurance Corporation (Oriental Assurance). by being broken up;
when the great majority of the cargo was lost on July - While the logs were being transported, rough seas (c) Any damage to the thing which renders it
3, 1980. Hence, petitioner Aboitiz was properly held and strong winds caused damage to one of the two valueless to the owner for the purpose for which
liable to Phil-Am. barges resulting in the loss of 497 pieces of logs out he held it; or
Reasoning of the 598 pieces loaded thereon. (d) Any other event which effectively deprives the
[a] Records of the case show that Phil-Am executed - Panama demanded payment for the loss but owner of the possession, at the port of destination,
a continuous and open insurance coverage covering Oriental Assurance refuse on the ground that its of the thing insured. (Section 130, Insurance
goods of Marinduque imported into and exported contracted liability was for "TOTAL LOSS ONLY." Code).
from the Philippines which took effect after Sept. 1, - Unable to convince Oriental Assurance to pay its - A constructive total loss is one which gives to a
1975, as contained in Marine Open Policy No. claim, Panama filed a Complaint for Damages against person insured a right to abandon, under Section
100184. A similar insurance coverage was also Oriental Assurance before the Regional Trial Court. 139 of the Insurance Code. This provision reads:
executed by petitioner in favor of Marinduque for all - RTC ordered Oriental Assurance to pay Panama SECTION 139. A person insured by a contract of
its goods shipped or moved within the territorial with the view that the insurance contract should be marine insurance may abandon the thing insured,
limits of the Philippines also effective after Sept. 1, liberally construed in order to avoid a denial of or any particular portion thereof separately valued
1975 and contained in Marine Open Policy No. substantial justice; and that the logs loaded in the by the policy, or otherwise separately insured, and
100185. two barges should be treated separately such that recover for a total loss thereof, when the cause of
[b] TC in dismissing the complaint apparently relied the loss sustained by the shipment in one of them the loss is a peril injured against,
on Marine Risk Note No. 017545 issued by private may be considered as "constructive total loss" and (a) If more than three-fourths thereof in value is
respondent Phil-Am only on July 28, 1980 after the correspondingly compensable. CA affirmed in toto. actually lost, or would have to be expended to
shipment in question was already pilfered. Obviously recover it from the peril;
TC mistook said Marine Risk Note as an insurance ISSUE (b) If it is injured to such an extent as to reduce
policy when it is NOT. It is only an acknowledgment WON Oriental Assurance can be held liable under its its value more than three-fourths;
or declaration of the private respondent confirming marine insurance policy based on the theory of a xxx xxx xxx
the specific shipment covered by its Marine Open divisible contract of insurance and, consequently, a - The requirements for the application of Section 139
Policy, the evaluation of the cargo and the constructive total loss of the Insurance Code, quoted above, have not been
chargeable premium. met. The logs involved, although placed in two
[c] The contention of the Aboitiz that it could not be HELD barges, were not separately valued by the policy, nor
liable for the pilferage of the cargo as it was stolen NO separately insured. Resultantly, the logs lost in the
even before it was loaded on its vessel is untenable. - The terms of the contract constitute the measure of damaged barge in relation to the total number of
Aboitiz received cargo when it arrived in Manila at its the insurer liability and compliance therewith is a logs loaded on the same barge cannot be made the
offices, and it was while in its possession and before condition precedent to the insured's right to recovery basis for determining constructive total loss. The logs
loading it in its vessel that the cargo was pilfered. Its from the insurer. Whether a contract is entire or having been insured as one inseparable unit, the
liability is clear. severable is a question of intention to be determined correct basis for determining the existence of
Disposition Petition DISMISSED. by the language employed by the parties. The policy constructive total loss is the totality of the shipment
in question shows that the subject matter insured of logs. Of the entirety of 1,208, pieces of logs, only
ORIENTAL ASSURANCE v. CA (PANAMA SAW was the entire shipment of 2,000 cubic meters of 497 pieces thereof were lost or 41.45% of the entire
MILL) apitong logs. The fact that the logs were loaded on shipment. Since the cost of those 497 pieces does
200 SCRA 459 two different barges did not make the contract not exceed 75% of the value of all 1,208 pieces of
MELENCIO-HERRERA; August 9, 1991 several and divisible as to the items insured. The logs, the shipment cannot be said to have sustained
logs on the two barges were not separately valued or a constructive total loss under Section 139(a) of the
NATURE separately insured. Only one premium was paid for Insurance Code.
Petition for review on certiorari the entire shipment, making for only one cause or Disposition judgment under review is SET ASIDE
consideration. The insurance contract must,
FACTS therefore, be considered indivisible. PAN MALAYAN INSURANCE v. CA (THE FOOD
- Sometime in January 1986, private respondent - More importantly, the insurer's liability was for AND AGRICULTURAL ORGANIZATION OF THE
Panama Sawmill Co., Inc. (Panama) bought, in "total loss only." A total loss may be either actual or UNITED NATIONS)
Palawan, 1,208 pieces of apitong logs, with a total constructive (Sec. 129, Insurance Code). An actual 201 SCRA 382
volume of 2,000 cubic meters. It hired Transpacific total loss is caused by: REGALADO; September 5, 1991
Towage, Inc., to transport the logs by sea to Manila (a) A total destruction of the thing insured;
FACTS
INSURANCE Page
99

- The Food and Agricultural Organization of the 1. NO - It is thus clear that FAO suffered actual total loss
United Nations (hereinafter referred to as FAO), - The law classifies loss into either total or partial. under Section 130 of the Insurance Code, specifically
ntended and made arrangements to send to Total loss may be actual or absolute, or it may under paragraphs (c) and (d) thereof, recompense
Kampuchea 1,500 metric petitions of IR-36 certified otherwise be constructive or technical. Petitioner for which it has been denied up to the present
rice seeds to be distributed to the people for seedling submits that respondent court erred in ruling that -Section 135 of the Insurance Code explicitly
purposes there was total loss of the shipment despite the fact provides that "(u)pon an actual total loss, a person
- LUZTEVECO was to ship the cargo amounting to that only 27,922 bags of rice seeds out of 34,122 insured is entitled to payment without notice of
US$83,325.92 in respect of one lot of 1,500 metric bags were rendered valueless to FAO and the abandonment." This is a statutory adoption of a long
petitions winch is the subject of the present action. shipment sustained only a loss of 78%. - FAO, standing doctrine in maritime insurance law that in
The cargo was loaded on board LUZTEVECO Barge however, claims that, for all intents and purposes, it case of actual total loss, the right of the insured to
No. LC-3000 and consisted of 34,122 bags of IR-36 has practically lost its total or entire shipment in this claim the whole insurance is absolute, without need
certified rice seeds purchased by FAO from the case, inclusive of expenses, premium fees, and so of a notice of abandonment
Bureau of Plant Industry for P4,602,270.00 forth, despite the alleged recovery by defendant
- FAO secured insurance coverage in the amount of LUZTEVECO. As found by the court below and PHILIPPINE AMERICAN LIFE INSURANCE
P5,250,000.00 from petitioner, Pan Malayan reproduced with approval by respondent court, FAO COMPANY v. CA (ELIZA PULIDO)
Insurance Corporation "has never been compensated for this total loss or 344 SCRA 260
- On June 16, 1980, FAO gave instructions to damage, a fact which is not denied nor controverted GONZAGA-REYES; November 15, 2000
LUZTEVECO to leave for Vaung Tau, Vietnam to - If there were some cargoes saved, by LUZTEVECO,
deliver the cargo which, by its nature, could not private respondent abandoned it and the same was NATURE
withstand delay because of the inherent risks of sold or used for the benefit of LUZTEVECO or Pan This petition for review on certiorari seeks to reverse
termination and/or spoilage. On the same date, the Malayan Corporation. Under Sections 129 and 130 of the Decision of the Special Second Division of the
insurance premiums on the shipment was paid by the New Insurance Code, a total loss may either be Court of Appeals
FAO petitioner actual or constructive. In case of total loss in Marine
- On June 26, 1980, FAO was advised of the sinking Insurance, the assured is entitled to recover from FACTS
of the barge in the China Sea, hence it informed the underwriter the whole amount of his subscription - On January 9, 1989, petitioner received from one
petitioner thereof and, later, formally filed its claim - SEC. 130. An actual total loss is caused by: (c) Any Florence Pulido an application for life insurance,
under the marine insurance policy. On July 29, 1980, damage to the thing which renders it valueless to the dated December 16, 1988, in the amount of
FAO was informed by LUSTEVECO of the recovery of owner for the purpose for which he held it; or P100,000.00 which designated her sister, herein
the lost shipment, for which reason FAO formally (d) Any other event which effectively deprives the private respondent, as its principal beneficiary.
filed its claim with LUZTEVECO for compensation of owner of the possession, at the port of destination of Because the insurance applied for was non-medical,
damage to its cargo the thing insured. petitioner did not require a medical examination and
- LUZTEVECO failed and refused to pay. Pan Malayan -as said and proven, the seeds were of fragile issued a policy on the sole basis of the application on
likewise failed to pay for the losses and damages nature. And the wetting of said seeds affected the February 11, 1989. On April 1992, petitioner
sustained by FAO by reason of its inability to recover state of seeds. Thus rendering them useless for FAO. received private respondent’s claim, which declared
the value of the shipment from LUZTEVECO Although there were bags which were recovered, that the insured, Florence Pulido, died of acute
- Pan Malayan claims that part of the cargo was these were “stained” and not in the same condition it pneumonia on September 10, 1991.
recovered and thus the claim by FAO was was brought in. in addition to this, FAO did not - Petitioner withheld payment on the ground that the
unwarranted. This is evidenced by two surveys upon receive any compensation for said recovered bags as policy claimed under was void from the start for
the cargo wherein it was found that only around 78% the same were distributed by LUZVETECO without having been procured in fraud. It is petitioner’s
was lost. authorization of FAO contention that even before they received private
- FAO filed a civil case against both LUZTEVECO and - the complete physical destruction of the subject respondent’s claim for death benefits, their
Pan Malayan. Trial court found in favor of FAO and matter is not essential to constitute an actual total investigation concerning the subject policy yielded
ordered both to pay jointly and severally the full loss. Such a loss may exist where the form and the information that the insured, Florence Pulido,
amount of the claim. This was affirmed by CA specie of the thing is destroyed, although the died in 1988, before the application for insurance on
materials of which it consisted still exist (Great her life was made. While this was communicated to
ISSUE Western Ins. Co. vs. Fogarty, N.Y., 19 Wall 640, 22 private respondent in a letter dated April 29, 1992,
1. WON respondent court committed a reversible L. Ed. 216), as where the cargo by the process of private respondent had already filed her claim earlier
error in holding that the trial court is correct in decomposition or other chemical agency no longer that month. In another letter dated July 27, 1992,
holding that there is a total loss of the shipment remains the same kind of thing as before (Williams however, petitioner confirmed to private respondent
vs. Cole, 16 Me. 207). receipt of the claim papers and assured her that her
HELD case was “being given preferential attention and
INSURANCE Page
100

prompt action”. duties, are prima facie evidence of facts therein Company of the Philippines issued a policy on the life
- Following the filing by private respondent of her stated. A duly-registered death certificate is of Jose C. Londres whereby it undertook to pay its
claim, petitioner caused another investigation considered a public document and the entries found beneficiary upon his death the sum of P3,000. All the
respecting the subject policy. Pursuant to the therein are presumed correct, unless the party who premiums due under the policy were actually paid on
findings of this second investigation, petitioner stood contests its accuracy can produce positive evidence their dates of maturity and the policy was in force
by its initial decision to treat the policy as void and establishing otherwise. Petitioner’s contention that when the insured died on February 7, 1945.
not to honor the claim. On November 9, 1992, the death certificate is suspect because Dr. Gutierrez Salvacion V. Londres, as beneficiary, demanded from
private respondent enlisted the services of counsel in was not present when Florence Pulido died, and the company the payment of the proceeds of the
reiterating her claim for death benefits. Petitioner knew of Florence’s death only through Ramon policy, and her demand having been refused, she
still refused to make payment and thus, this action. Piganto, does not merit a conclusion of fraud. No instituted the present action against the company in
- Petitioner: the results of its investigations having motive was imputed to Dr. Gutierrez for seeking to the Court of First Instance of Manila.
indicated that the insured was already dead at the perpetuate a falsity in public records. Petitioner was - Defendant in its answer denied, for lack of
time the policy was applied for. It also likewise unable to make out any clear motive as to sufficient proof, the allegation that the insured died
counterclaimed for attorney’s fees. The first report, why Ramon Piganto would purposely lie. Mere on February 7, 1945, and set up the following special
prepared by one Dr. Benedicto Briones, was dated allegations of fraud could not substitute for the full defenses: (a) that plaintiff's claim is covered by the
April 1, 1992, and had attached to it a questionnaire, and convincing evidence that is required to prove it. Moratorium Law; (b) that the policy having been
responded to by one Ramon Piganto, who A failure to do so would leave intact the presumption issued during the Japanese occupation, it is
represented to be the brother-in-law of the insured of good faith and regularity in the performance of presumed that its face value should be paid in
and the barangay chairman of Cardiz, Bagulin, La public duties, which was the basis of both respondent Japanese currency, there being no provision in the
Union. To the question “Where does [Florence court and the trial court in finding the date of policy from which can be inferred that the parties
Pulido] reside now?”, Piganto had replied that Florence Pulido’s death to be as plaintiff-private contemplated payment in any other currency; (c)
Florence Pulido used to live in Cardiz, but was dead respondent maintained. that the money paid by the insured as premiums,
since 1988. Piganto’s statement was signed by him, - We cannot likewise give credence to petitioner’s together with the money received from other policy-
and witnessed by his wife, Nenita Piganto. This submission that the inconsistencies in the holders, was all deposited by the defendant in the
report was petitioner’s basis for treating the disputed testimonies of the witnesses for plaintiff-private Philippine National Bank and said deposit was
policy as void since April 1992, even before receipt of respondent are in themselves evidence of fraud. declared without value by Executive Order No. 49 of
private respondent’s claim. Such alleged inconsistencies are matters of credibility the President of the Philippines; and (d) that the
which had been ably passed upon by the lower court. policy having been issued under abnormal
ISSUE - The absence of fraud, as a factual finding of the circumstances, it should be considered in the light of
WON there was fraud (whether the insured, Florence lower court adopted by the Court of Appeals, entirely equity which does not permit anyone to enrich
Pulido, was in fact dead before the application for consistent with the evidence on record, will not be himself at the expense of another. Defendant,
insurance on her life was made) reversed and, hence, is final and conclusive upon this however, as a proof of good faith, offered to pay the
Court. value of the policy in accordance with the Ballantyne
HELD Disposition The instant petition is DENIED scale of values, or the sum of P2,400, Philippine
NO currency.
- This the lower courts had effected ruled on, upon a CHAPTER IX. CLAIMS SETTLEMENT & - It appears that the deceased took up the policy
preponderance of the evidence duly received from SUBROGATION under consideration on April 15, 1943 for the sum of
both parties. We see no reversible error in the P3,000. All the premiums due under the policy were
finding of both respondent court and the trial court in LONDRES v. NATIONAL LIFE INSURANCE actually paid on their dates of maturity and the
favor of the correctness of the entries in Certificate 94 PHIL 627 policy was in force when the insured died on
of Death, duly registered with the Local Civil BAUTISTA ANGELO; March 29, 1954 February 7, 1945. On said date, the battle of the
Registrar of Bagulin, La Union, which declared that liberation of the City of Manila was still raging. While
Florence Pulido died of acute pneumonia on NATURE the northern part may have been liberated, not so
September 10, 1991. Dr. Irineo Gutierrez, the Appeal from a decision of the Court of First Instance the southern part, as shown from the very affidavits
Municipal Health Officer of Bagulin, La Union whose of Manila ordering defendant to pay to plaintiff the submitted by appellee wherein it was stated that on
signature appeared in the death certificate, testified sum of P3,000, Philippine currency, plus legal the aforesaid date, the insured, Jose Londres, and
in addition that he ministered to the ailing Florence interest thereon from the time of the filing of the his two sons were taken by the Japanese soldiers
Pulido for two days immediately prior to her death. complaint until its full payment. from their house at Singalong Street and were
This fact is likewise noted in the death certificate. massacred by their captors. It may therefore be said
- Death certificates, and notes by a municipal health FACTS that the policy became due when the City of Manila
officer prepared in the regular performance of his - On April 14, 1943, the National Life Insurance was still under the yoke of the enemy and became
INSURANCE Page
101

payable only after liberation which took place on suffered by it under Article 307 of the Code of Japanese occupation was still pending consideration
March 10, 1945 when President Osmena issued Commerce which provides: "When the deposits are before the courts. NLIC said that because the policy
Proclamation No. 6 following the restoration of the of cash, with a specification of the coins constituting matured upon the insured’s death in November,
civil government by General Douglas Mac Arthur. them, . . . the increase or reduction which their value 1944, they should compute the value of their claim
And we say that the policy became payable only may suffer shall be for the account of the depositor." under the Ballantyne scale of values (which would
after liberation even if it matured sometime before, Appellant, by entering into an insurance amount only to P500)
because before that eventuality the insurance contract, cannot claim, if it suffers loss, that - beneficiaries commenced suit, and the lower court
company, appellant herein, was not yet in a position the beneficiary cannot enrich herself at its sustained the stand of the company, dismissed the
to pay the value of the policy for the simple reason expense. This is a risk attendant to any wagering complaint.
that it had not yet reopened. contract. One who gambles and loses cannot be - beneficiaries maintain that the obligation of the
heard to complain of his loss. To appellant, we can company to pay accrued not upon the death of
ISSUE only repeat the following admonition: Fernandez, but only upon the receipt and approval
WON the amount of P3,000 which appellant bound "The parties herein gambled and speculated on the by the company, on proof of death of the insured,
itself to pay to the insured under the policy upon his date of the termination of the war and the which was in 1954. The policy reads:
death should be paid in accordance with the present liberation of the Philippines by the Americans. This National Life Insurance Company of the Philippine
currency or should be adjusted under the Ballantyne can be gleaned from the stipulation about hereby agrees to pay at its Home Office, Manila,
scale of values redemption, particularly that portion to the effect Ten Thousand Pesos to Juan D. Fernandez
that redemption could be effected not before the (hereinafter called the insured) on the 15th day of
HELD expiration of one year from June 24, 1944. This July, 1964, if the Insured is living and this Policy is
YES, present currency. kind of agreement is permitted by law. We find in force, or upon receipt and approved at its Office
Reasoning nothing immoral or unlawful in it." (Gomez vs. of due proofs of the title of the claimant and of the
- In the case of Rutter vs. Esteban, 93 Phil., 68, the Tabia) prior death of the Insured while this Policy is in
Moratorium Law was declared invalid and Disposition Wherefore, the decision appealed from force to Teresa Duat Vda. De Fernandez, Maria T.
unconstitutional. is affirmed, with costs against appellant. and Manuela Fernandez, mother and sisters
- During those days of liberation, while the people respectively of the Insured (Hereinafter called the
were rejoicing because of the happy event, the Beneficiary) subject to the right of the Insured to
banks, the insurance companies, and for that matter change the beneficiary as stated on the second
other commercial and business firms, were still page of this Policy.
feeling the adverse effects of the sudden fall of - The above stipulation is apparently based on Sec.
values and were uncertain and apprehensive as to VDA. DE FERNANDEZ v. NATIONAL LIFE 91-A of the Insurance Law which provides as follows:
the manner the readjustment would be made by the INSURANCE CO OF THE PHILS The proceeds of a life insurance policy shall be paid
new Government. It is for this reason that the 105 PHIL 59 immediately upon maturity of the policy, unless such
beneficiary, after realizing the truth about the death ENDENCIA; January 27, 1959 proceeds are made payable in installments or a as an
of her husband, and after gathering evidence to annuity, in which case the installments or annuities
substantiate his death, had difficulty in effecting the NATURE shall be paid as they become due: Provided,
collection of her claim from the insurance company Appeal from CFI decision applying the Ballantyne however, That in case of a policy maturing by the
because at that time it had not yet reopened for scale of values upon the proceeds of life insurance death of the insured, the proceeds thereof shall be
business purposes. Although the record does not taken and maturing during the Japanese occupation paid within sixty days after presentation of the claim
disclose the exact date on which the insurance but claimed after liberation and filing of the proof of the death of the insured.
company reopened for this purpose, this Court can Refused to pay the claim within the time prescribed
take judicial notice that it only did so after liberation. FACTS herein will entitle the beneficiary to collect interest
At that time the legal tender was already the - National Life Insurance Company (NLIC) insured J. on the proceeds of the policy for the duration of the
present currency. Fernandez’s life for P10,000 upon his payment of delay at the rate of six per centum per annum,
- As final plea, appellant invokes equity in its favor in P444 from July 15, 1944 to July 14, 1945 unless such failure or refusal to pay is based on the
view of the nullification of the deposits made by it - The insured died on November 2, 1944, while the ground that the claim is fraudulent . . . .
with the Philippine National Bank of all fiat money policy was in force - Based on the foregoing provision of law and the
received from its policyholders, which money was - After more than 7 years, in 1952, Atty de la Torre, aforequoted stipulation as well as on the allegation
declared without value by Executive Order No. 49 of representing the benficiaries of the policy, informed that the filing of proof of death by the beneficiaries is
the President of the Philippines. Appellant claims the company that Fernandez had died in 1944, and a condition precedent of the demandability of the
that, considering the unexpected circumstances that claimed the proceeds of the policy. The company obligation of the insurer to pay the proceeds,
developed, the indemnity to be paid by it should be said that the status of the policies issued during the appellants claim that they should be paid P10,000 in
INSURANCE Page
102

Philippine currency and not under the Ballantyne were found to have been damaged by sea water - And in the light of the fact that the contending
scale of values. which rendered the fishmeal useless. Petitioner filed parties did not allege the rate of interest stipulated in
a claim with EASCO and Far Eastern Shipping. Both the insurance contract, the legal interest was
ISSUE refused to pay. Whereupon, petitioner sued them properly pegged by the Appellate Court at 6% per
WON the policy matured upon the death of the before the then Court of First Instance of Cebu for cent.
insured damages. EASCO, as the insurer, filed a counterclaim
against the petitioner for the recovery of the unpaid CATHAY INSURANCE v. CA(LUGAY)
HELD insurance premiums. 174 SCRA 11
YES - The trial court rendered judgment in favor of GRINO-AQUINO; June 5, 1989
Ratio In life insurance, the policy matures either petitioner. The judgment became final as to EASCO
upon the expiration of the term set forth therein, or but the shipping company appealed to the Court of FACTS
upon his death occuring at any time prior to the Appeals and was absolved from liability by the said - Petitioners are 6 insurance companies that issued
expiration of such stipulated term, in which case, the court. fire insurance policies for the total sum of
proceeds are payable to his beneficiaries within sixty - The trial court, upon motion by petitioner, issued a P4,000,000 to the Cebu Filipina Press owned by
days after their filing of proof of death. writ of execution against EASCO. The sheriff Emilia Chan Lugay. The fire policies described the
Reasoning enforcing the writ reportedly fixed the legal rate of insured property as "stocks of Printing materials,
- The sixty day period fixed by law within which to interest at 12%. Respondent EASCO moved to quash papers and general merchandise usual to the
pay the proceeds after presentation of proof of death the writ alleging that the legal interest to be Assured's trade" stored in a one-storey building of
Is merely procedural in nature, evidently to computed should be 6% per cent per annum in strong materials housing the Cebu Filipina Press
determine the exact amount to be paid and the accordance with Article 2209 of the Civil Code. The located at UNNO Pres. Quirino cor. Don V. Sotto
interest thereon to which the beneficiaries may be trial court denied EASCO's motion. On appeal, the Sts., Mabolo, Cebu City. The co-insurers were
entitled to collect in case of unwarranted refusal of Court of Appeals reversed the trial court’s denial of indicated in each of the policies. All, except one
the company to pay, and also to enable the insurer EASCO’s motion and ruled that the applicable policy (Paramount's), were renewals of earlier
to verify or check on the fact of death which it may interest is 6% per annum. Hence, this petition. policies issued for the same property.
even validly waive. It is the happening of the - On December 18, 1981, the Cebu Filipina Press was
suspensive condition of death that renders a life ISSUE razed by electrical fire together with all the stocks
policy matured, and not ht efiling of proof of death WON the applicable rate of interest is 12% per and merchandise stored in the premises. On January
which, as above stated, is merely procedural. The annum 15, 1982, Lugay submitted sworn Statements of Loss
insured having died during the Japanese occupation, and Formal Claims to the insurers, through their
the proceeds of his policy should be adjusted HELD adjusters. She claimed a total loss of P4,595,000.
accordingly, for “The rule is already settled that NO - After nearly 10 months of waiting, she sued to
where a debtor could have paid his obligation at any - Sections 243 and 244 of the Insurance Code apply collect on December 15, 1982. The insurance
time during the Japanese occupation, payment after only when there is an unjustified refusal or companies denied liability, alleging violation of
liberation must be adjusted in accordance with the withholding of payment on the insured’s claim. In certain conditions of the policy, misdeclaration, and
Ballantyne schedule (De Asis vs. Agdamag, among this case, EASCO's refusal to settle the claim to Tio even arson which was not seriously pressed for,
other cases). (Collaboration is defined as the acts Khe Chio was based on some ground which, while come the pre-trial, the petitioners offered to pay
of working together in a joint project. not sufficient to free it from liability under its policy, 50% of her claim, but she insisted on full recovery.
Disposition Judgment affirmed nevertheless is sufficient to negate any assertion - Trial court rendered judgment in her favor ordering
that in refusing to pay, it acted unjustifiably. Simply the insurers to pay her a total of P4,000,000 as
TIO KHE CHIO v. CA (EASTERN ASSURANCE & put, the said provisions of the Insurance Code are indemnity, P48,000 representing expenses of the
SURETY) not pertinent to the instant case. They apply only plaintiff, a separate amount of 20% of the
202 SCRA 119 when the court finds an unreasonable delay or P4,000,000 representing fees of counsel, interests at
FERNAN; September 30, 1991 refusal in the payment of the claims. the rate of twice the ceiling being prescribed by the
- Circular No. 416 of the Central Bank, which raised Monetary Board starting from the time when the
FACTS the legal rate of interest from 6% to 12% per annum case was filed, and finally, with costs. CA affirmed.
- Petitioner Tio Khe Chio imported 1,000 bags of refers only to loans or forbearances of money, goods
fishmeal valued at $36,000.30 from Agro Impex, or credits and court judgments thereon but not to ISSUES
S.A. Dallas, Texas, U.S.A. The goods were insured court judgments for damages arising from injury to 1. WON the insured's cause of action had already
with respondent EASCO and shipped on board the persons and loss of property which does not involve accrued before she filed her complaint
M/V Peskov, a vessel owned by Far Eastern Shipping a loan. Clearly, the applicable law is Article 2209 of 2. WON sufficient proofs of loss had been presented
Company. When the goods reached Manila, they the Civil Code. by the insured
INSURANCE Page
103

3. WON the private respondents claim for loss was whether the payment of the claim of the insured other personal effects, and Item 2 for P60k on stocks
inflated has been unreasonably denied or withheld; and in in trade usual to petitioner's retail business situated
4. WON lower court erred in awarding damages to the affirmative case, the insurance company shall in a two-storey building at 039 Barreda St.,
the private respondent in the form of interest be adjudged to pay damages which shall consist of Mangagoy, Bislig, Surigao del Sur.
equivalent to double the interest ceiling set by the attorney's fees and other expenses incurred by the - While both policies were in force, fire destroyed
Monetary Board insured person by reason of such unreasonable petitioner's insured properties at the market site on
5. WON attorney's fees awarded were exorbitant denial or withholding of payment plus interest of September 5, 1977 and at Barreda St. on November
twice the ceiling prescribed by the Monetary Board 9, 1977.
HELD of the amount of claim due the insured. - When petitioner failed to obtain indemnity on his
1. YES - The petitioners' contention that the charging of claims from Zenith, he filed a complaint with the
- As the fire which destroyed the Cebu Filipina Press double interest was improper because no Insurance Commission praying that Zenith be
occurred on December 19, 1981 and the proofs of unreasonable delay in the processing of the fire claim ordered to pay him P130kj representing the value of
loss were submitted from January 15, 1982 through was proven is refuted by the trial court's explicit the 2 policies insured by respondent with interest at
June 21, 1982 in compliance with the adjusters' finding that "there was a delay that was not 12% per annum, plus damages, attorney's fees and
numerous requests for various documents, payment reasonable in processing the claim and doing other expenses of litigation. ...
should have been made within 90 days thereafter payments". Under Section 244, a prima facie - Zenith interposed that petitioner had no cause of
(Sec 243), or on or before September 21, 1982. evidence of unreasonable delay in payment of the action; that Policy No. F-03724 was not in full force
Hence, when the assured filed her complaint on claim is created by the failure of the insurer to pay and effect at the time of the fire because the
December 15, 1982, her cause of action had already the claim within the time fixed in both Sec. 242 and premium on the policy was not paid; that Zenith's
accrued. 243 of the IC. liability under Policy No. F-03734, if any, was limited
2. YES - In view of the not insubstantial value of the private to P15,472.50 in view of the co-insurance; and that
- There is no merit in the petitioners' contention that respondent's claims and the considerable time and petitioner failed to substantiate his claim as to the
the proofs of loss were insufficient because Lugay effort expended by them and their counsel in value of the goods reputedly destroyed by fire.
failed to comply with the adjuster's request for the prosecuting these claims for the past 8 years, - While the case was pending, Zenith settled
submission of her bank statements. Condition No. 13 attorney's fees were properly awarded to the private petitioner’s fire loss claim under Item 1 of Policy No.
of the policy does not require the insured to produce respondents. 03734 in the amount of P15,472.50.
her bank statements. Therefore, the insured was not - Insurance Commissioner allowed petitioner to
obligated to produce them and the insurers had no 5. YES recover under said policy and ordered Zenith to pay
right to ask for them. Condition No. 13 was prepared - An award equivalent to 10% of the proceeds of the him the amount of P20k with legal interest from the
by the insurers themselves, hence, it should be policies would be more reasonable than the 20% date the complaint was filed, including P1k as
taken most strongly against them. awarded by the trial court and the CA. attorney's fees but excluding the actual, moral and
3. NO Disposition Decision of the CA AFFIRMED with exemplary damages prayed for. As for petitioner's
- Both the trial court and the CA noted that the MODIFICATION. claim under Policy No. F-03734, she held that in view
proofs were ample and more than enough for of the payment of P15,472.50 to petitioner, Zenith
defendant insurers to do a just assessment NODA v. CRUZ had fully discharged its liability under said policy
supporting the 1981 fire claim for an amount 151 SCRA 227 which covered furniture, fixtures, fittings and other
exceeding four million pesos. FERNAN; June 22, 1987 personal belongings of petitioner.
4. NO - In allowing recovery under Policy No. F-03734,
- The award of double interest on the claim is lawful NATURE Commissioner placed much weight on the final report
and justified under Sections 243 and 244 of the Petition to review decision of the Insurance prepared by Dela Merced Adjustment Corporation, an
Insurance Code which provide: Commissioner independent fire, marine and casualty adjuster
Sec. 243 Refusal or failure to pay the loss or contracted by Zenith to investigate the claims of its
damage within the time prescribed herein will FACTS various policyholders. Said report concluded that
entitle the assured to collect interest on the - In 1977, Noda obtained from Zenith Insurance "the sound value of P26,666.67 represented the
proceeds of the policy for the duration of the delay Corporation 2 fire insurance policies: [1] No. F- whole loss and damage" incurred by petitioner, but
at the rate of twice the ceiling prescribed by the 03724 with a face value of P30k covering the goods with the application of the three-fourths loss clause,
Monetary Board. and stocks in trade in his business establishment at Zenith's liability was reduced to P20k.
Sec. 244 In case of any litigation for the the market site in Mangagoy, Bislig, Surigao del Sur
enforcement of any policy or contract of insurance, and [2] No. F-03734 with a face value in the ISSUES
it shall be the duty of the Commissioner or the aggregate amount of P100k and consisting of Item 1
Court, as the case may be, to make a finding as to for P40k on household furniture, fixtures, fittings and
INSURANCE Page
104

1. WON Insurance Commissioner erred in denying admission of its liability up to the amount to Caltex. Delsan refused to pay, forcing American
petitioner's demand for P60k under Item 2 of Policy recommended. Being in the nature of an admission home to file a case for collection in the RTC.
No. F-03734 against interest, it is the best evidence which affords - RTC found that the vessel, MT Maysun, was
2. WON Insurance Commissioner erred in not the greatest certainty of the facts in dispute. seaworthy to undertake the voyage, and that the
awarding in favor of petitioner exemplary damages Respondent Commissioner should not have incident was caused by an unexpected inclement
for Zenith's unjustified and wanton refusal to pay perfunctorily dismissed that particular evidence as a weather condition or force majeure, thus exempting
petitioner's claim under the said two insurance worthless piece of paper. the common carrier from liability for the loss of its
contracts 2. NO cargo.
- There is no showing that Zenith, in contesting - CA reversed RTC decision on the basis of evidence
HELD payment, had acted in a wanton, oppressive or from PAG-ASA that there were no 20 ft. waves in the
1. YES malevolent manner to warrant the imposition of area. CA ruled that the petitioner is liable on its
- To prove the existence of the stocks in trade corrective damages. obligation as common carrier to respondent
covered by Policy No. F-03734, petitioner offered his Disposition Zenith Insurance Corporation ordered to insurance company as subrogee of Caltex.
testimony and that of his wife as well as pay petitioner Norman R. Noda the sum of Petitioner’s Claim
documentary exhibits. The foregoing evidence for P60,592.10 with legal interest from the filing of the > In every marine insurance upon a ship or freight,
petitioner preponderantly showed the presence of complaint until full payment, but deducting or freightage, or upon any thing which is the subject
some P590k worth of goods in his retail store during therefrom the amount of P15,472.50 which it had of marine insurance there is an implied warranty by
the fire of November 9, 1977. earlier paid to petitioner. the shipper that the ship is seaworthy.10 When
- While the insurer, and the Insurance Commissioner private respondent paid Caltex the value of its lost
for that matter, have the right to reject proofs of loss DELSAN TRANSPORT, INC. v. CA (AMERICAN cargo, the act of the private respondent is equivalent
if they are unsatisfactory, they may not set up for HOME ASSURANCE) to a tacit recognition that the ill-fated vessel was
themselves an arbitrary standard of satisfaction. 369 SCRA 24 seaworthy.
Substantial compliance with the requirements will DE LEON, JR; November 15, 2001
always be deemed sufficient. Respondent’s Comment
- Zenith introduced in evidence the final report on NATURE > American Home Assurance is entitled to payment
Policy No. F-03734 submitted by its own adjuster, A petition for review on certiorari of the decision of by its right of subrogation.
Dela Merced Adjustment Corporation. Respondent CA.
Commissioner however ignored such report, ISSUES
reasoning that with regard to Item 2 of Policy No. F- FACTS 1. WON payment made by American Home to Caltex
03734 the claim for loss of the stocks in trade was - Caltex entered into a contract of affreightment with for the insured value of the lost cargo amounted to
not successfully proven in view of petitioner's failure the petitioner, Delsan Transport Lines, Inc. an admission that the vessel was seaworthy, thus
to present evidence; that the adjuster's report (petitioner), for a period of one year whereby the precluding any action for recovery against the
deserved scant consideration since the allegations said common carrier agreed to transport Caltex’s petitioner
therein were not substantiated, and that said report industrial fuel oil from the Batangas-Bataan Refinery 2. WON MT Maysun was seaworthy at the time of the
did not even make a recommendation for payment. to different parts of the country. Delsan took on voyage (outline topic)
- A scrutiny of the abovementioned adjuster's report board its vessel, MT Maysun, 2,277.314 kiloliters of 3. WON non-presentation of the marine insurance
reveals that together with the formal demand for full industrial fuel oil of Caltex to be delivered to the policy bars the complaint for recovery of sum of
indemnity, petitioner submitted his income tax return Caltex Oil Terminal in Zamboanga City. The money for lack of cause of action
for 1978, purchase invoices, certification from his shipment was insured by American Home Assurance
suppliers as to his purchases, and other supporting Corporation (respondent). HELD
papers. The report even took into account the - August 14, 1986: MT Maysun set sail from 1. NO
appraisals of the other adjusters and concluded that Batangas for Zamboanga City. The vessel sank in Ratio The fact of payment grants American Home
the total loss sustained by petitioner in his household the early morning of August 16, 1986 near Panay the subrogatory right which enables it to exercise
effects and stocks in trade reached P379,302.12. But Gulf in the Visayas taking with it the entire cargo of legal remedies that would otherwise be available to
after apportioning said amount among petitioner's fuel oil. Caltex as owner of the lost cargo against the
six different insurers [the co-insurance being known - Respondent paid Caltex P5,096,635.57 petitioner common carrier.
to Zenith], the liability of Zenith was placed at representing the insured value of the lost cargo. Reasoning
P60,592.10. It therefore recommended that Zenith Exercising its right of subrogation under Article 2207 Art. 2207. (Civil Code)
pay the petitioner the amount of P60, 592.10. of the New Civil Code, the private respondent
- Said document was offered as evidence by Zenith demanded of the petitioner the same amount it paid
itself and could very well be considered as an 10
Sec. 113 Insurance Code
INSURANCE Page
105

If the plaintiff’s property has been insured, and he - Authorities are clear that diligence in securing - Complainants have no cause of action against
has received indemnity from the insurance certificates of seaworthiness does not satisfy the Finman
company for the injury or loss arising out of the vessel owner’s obligation. Also securing the approval - Amounts claimed were paid as deposits and not as
wrong or breach of contract complained of, the of the shipper of the cargo, or his surveyor, of the placement fees.
insurance company shall be subrogated to the condition of the vessel or her stowage does not - POEA Administrator issued Order that respondents
rights of the insured against the wrongdoer or the establish due diligence if the vessel was in fact should pay. Finman appealed to Secretary of Labor.
person who has violated the contract. If the unseaworthy, for the cargo owner has no obligation Secretary upheld the POEA order.
amount paid by the insurance company does not in relation to seaworthiness.
fully cover the injury or loss, the aggrieved party 3. NO ISSUE
shall be entitled to recover the deficiency from the Ratio The presentation in evidence of the marine WON Finman can be held liable for complainants’
person causing the loss or injury. insurance policy is not indispensable in this case claims against Pan Pacific
- The right of subrogation is designed to promote before the insurer may recover from the common
and to accomplish justice and is the mode which carrier the insured value of the lost cargo in the HELD
equity adopts to compel the ultimate payment of a exercise of its subrogatory right. The subrogation YES
debt by one who in justice and good conscience receipt, by itself, is sufficient to establish not only - Under Insurance Code, liability of surety in a
ought to pay. It is not dependent upon, nor does it the relationship of respondent as insurer and Caltex, surety bond is joint and several with the principal
grow out of, any privity of contract or upon written as the assured shipper of the lost cargo of industrial obligor.
assignment of claim. It accrues simply upon fuel oil, but also the amount paid to settle the - Conditions of a bond specified and required in the
payment by the insurance company of the insurance insurance claim. The right of subrogation accrues provisions of a statute providing for submission of
claim. simply upon payment by the insurance company of the bond, are incorporated into all bonds tendered
2. NO the insurance claim. under that statute even though not set out in
Ratio Seaworthiness relates to a vessel’s actual Disposition Petition is denied, and the decision of printer’s ink.
condition. Neither the granting of classification or the CA is affirmed. - POEA held and Secretary of Labor affirmed that
the issuance of certificates establishes Pan Pacific had violated Labor Code, and at least one
seaworthiness. of the conditions for the grant and continued use of
Reasoning the recruitment license. POEA and Secretary of
- Common carriers are bound to observe Labor can require Pan Pacific to refund the placement
extraordinary diligence in the vigilance over the FINMAN GENERAL ASSURANCE CORP v. fees and to impose the fine.
goods and for the safety of passengers transported INOCENCIO - If Pan Pacific is liable, and if Finman is solidarily
by them, according to all the circumstances of each 179 SCRA 480 liable with Pan Pacific, then Finman is liable both to
case. There is no liability if the loss, destruction or FELICIANO; November 15, 1989 private respondents and to POEA.
deterioration is by force majeure. - Cash and surety bonds are required from
- The tale of strong winds and big waves by the said FACTS recruitment companies as means of ensuring prompt
officers of the petitioner however, was effectively - Pan Pacific is a recruitment and employment and effective recourse against such companies when
rebutted and belied by the weather report from PAG- agency. It posted surety bond issued by Finman held liable. Public policy will be effectively negated if
ASA. MT Maysun sank with its entire cargo for the General Assurance and was granted license to POEA and the DoLE were held powerless to compel a
reason that it was not seaworthy. There was no operate by POEA. surety company to make good on its solidary
squall or bad weather or extremely poor sea - Inocencio, Palero, Cardones, Hernandez filed with undertaking.
condition in the vicinity when the said vessel sank. POEA complaints against Pan Pacific for violation of
- Petitioner may not escape liability by presenting in Labor Code and for refund of placement fees. POEA EAGLE STAR INSURANCE CO LTD v. CHIA YU
evidence certificates that tend to show that at the Administrator motu propio impleaded Finman as 96 PHIL 696
time of dry-docking and inspection by the Philippine surety for Pan Pacific. REYES; March 31, 1955
Coast Guard MT Maysun, was fit for voyage. These - Pan Pacific moved out and no notice of transfer was
pieces of evidence do not necessarily take into furnished to POEA as required. POEA considered NATURE
account the actual condition of the vessel at the time that constructive service of complaints had been Certiorari
of the commencement of the voyage. At the time of effected.
dry-docking and inspection, the ship may have - Finman denied liability and said that FACTS
appeared fit. The certificates issued, however, do - POEA had no jurisdiction over surety bonds; - Atkin, Kroll & Co., loaded on the S. S. Roeph
not negate the presumption of unseaworthiness jurisdiction is vested in Insurance Commission Silverlight owned and operated by Leigh Hoegh &
triggered by an unexplained sinking. - Finman had not violated Labor Code Co., A/S, of San Francisco California, 14 bales of
INSURANCE Page
106

assorted underwear valued at P8,085.23 consigned - Being contrary to the law of the forum, the insurer and should not be permitted to prevent a
to Chia Yu in the City of Manila. stipulation in the policy cannot be given effect as it recovery when their just and honest application
- The shipment was insured against all risks by Eagle would reduce the period allowed the insured for would not produce that result. (46 C. J. S. 273.)
Star Ins. Co. of San Francisco, California, under a bringing his action to less than one year (because Disposition Judgment appealed from is REVERSED
policy issued to the shipper and by the latter the prescription period begins from the “happening with respect to the carrier and its agents but
assigned to the consignee. of the loss” and that before any suit could be AFFIRMED with respect to the insurance company
- The vessel arrived in Manila but of the 14 bales sustained the insured shall have to comply with the and its agents.
(a.k.a. freights =p) consigned to Chia Yu only 10 terms and conditions of the policy first TF lessening
were delivered to him as the remaining 3 could not the period to less than a year. )
be found.3 of those delivered were also found - Insular Government vs. Frank(13 Phil. 236)~
damaged to the extent of 50 per cent. "matters respecting a remedy, such as the bringing
-Chia Yu claimed indemnity for the missing and of suit, admissibility of evidence, and statute of
damaged bales. But the claim was declined, first, by limitations, depend upon the law of the place where ACCFA v. ALPHA INSURANCE
the carrier and afterward by the insurer, whereupon the suit is brought" TF any policy clause repugnant to 24 SCRA 151
Chia Yu brought the present action against both, this amendment to the Insurance Act cannot be REYES; July 29, 1968
including their respective agents in the Philippines. given effect in an action in our courts.
- An action was filed at the CFI after more than 2 SEC. 61-A. (Insurance Code) ~ Any condition, FACTS
years after delivery of the damaged bales and the stipulation or agreement in any policy of - In order to guarantee the Asingan Farmers'
date when the missing bales should have been insurance, limiting the time for commencing an Cooperative Marketing Association, Inc. (FACOMA)
delivered, the action was resisted by the Atkins and action thereunder to a period of less than one year against loss on account of "personal dishonesty,
Eagle Star principally on the ground of prescription. from the time when the cause of action accrues, is amounting to larceny or estafa of its Secretary-
-TC favored Chia Yu and CA affirmed. void. Treasurer, Ladines, the appellee, Alpha Insurance &
*** CARRIER’s defense of prescription is made to - The prescription clause could be harmonized with Surety Company had issued, on 14 February 1958,
rest on the following stipulation of the bill of lading: section 61-A of the Insurance Act by taking it to its bond, No. P-FID-15-58, for the sum of P5,000
In any event the carrier and the ship shall be mean that the time given the insured for bringing his with said Ladines as principal and the appellee as
discharged from all liability in respect of loss or suit is twelve months after the cause of action solidary surety. On the same date, the Asingan
damage unless suit is brought within one year accrues. FACOMA assigned its rights to the appellant,
after the delivery of the goods or the date when - If so, when did the cause of action accrue? Chia Agricultural Credit Cooperative and Financing
the goods should have been delivered. (This Yu’s action did not accrue until his claim was finally Administration (ACCFA for short), with approval of
stipulation is but a repetition of a provision in the rejected by the insurance company. This is because, the principal and the surety.
CA 65 which says that bills of lading covering before such final rejection, there was no real - During the effectivity of the bond, Ladines
shipments from the US to the Phils should be necessity for bringing suit. converted and misappropriated, to his personal
brought w/in one year after the delivery of the - As the policy provides that the insured should file benefit, some P11,513.22 of the FACOMA funds, of
goods or the date when the goods should have his claim, first, with the carrier and then with the which P6,307.33 belonged to the ACCFA. Upon
been delivered to hold the carrier liable.) insurer, he had a right to wait for his claim to be discovery of the loss, ACCFA immediately notified in
*** INSURER’s claim of prescription is founded upon finally decided before going to court. writing the survey company on 10 October 1958, and
the terms of the policy and not upon the bill of - Furthermore, there is nothing in the record to show presented the proof of loss within the period fixed in
lading. (But in our jurisdiction, as per A1144, that the claim was rejected in the year 1947, either the bond; but despite repeated demands the surety
prescription is 10 years after action accrues.) by the insurance company in London or its settling company refused and failed to pay. Whereupon,
No suit action on this Policy, for the recovery of agents in the Philippines. ACCFA filed suit against appellee on 30 May 1960.
any claim, shall be sustainable in any Court of law - For the purpose of this action, Chia Yu's claim was - Defendant Alpha Insurance & Surety Co., Inc.,
or equity unless the insured shall have fully considered to have been finally rejected by the (now appellee) moved to dismiss the complaint for
complied with all the terms and conditions of this insurer on April 22, 1948. Having been filed within failure to state a cause of action, giving as reason
Policy nor unless commenced with twelve (12) twelve months form that date, the action cannot be that (1) the same was filed more than one year after
months next after the happening of the loss . . . deemed to have prescribed even on the supposition plaintiff made claim for loss, contrary to the eighth
that the period given the insured for bringing suit condition of the bond, providing as follows:
ISSUE under the prescriptive clause of the policy is twelve EIGHT LIMITATION OF ACTION: No action, suit or
WON ATKIN’ s action has prescribed months after the accrual of the cause of action. proceeding shall be had or maintained upon this
- Contractual limitations contained in insurance Bond unless the same be commenced within one
HELD policies are regarded with extreme jealousy by year from the time of making claim for the loss upon
NO courts and will be strictly construed against the
INSURANCE Page
107

which such action, suit or proceeding, is based, in loss, for that does not import that the surety arbitrators or umpire shall have made their award
accordance with the fourth section hereof. company will refuse to pay. In so far, therefore, as all benefit under this Policy shall be forfeited."
(2) the complaint failed to show that plaintiff had condition eight of the bond requires action to be filed
filed civil or criminal action against Ladines, as within one year from the filing of the claim for loss, ISSUE
required by conditions 4 and 11 of the bond; and (3) such stipulation contradicts the public policy WON the suit against the agent tolled the
that Ladines was a necessary and indispensable expressed in Section 61-A of the Philippine Insurance prescription period, such that the filing against Fulton
party but had not been joined as such. Act. was only 9 months after the claim was rejected
- At first, the Court of First Instance denied - Condition eight of the bond, therefore, is null and
dismissal; but, upon reconsideration, the court void, and the appellant is not bound to comply with HELD
reversed its original stand, and dismissed the its provisions. The discouraging of unnecessary NO
complaint on the ground that the action was filed litigation must be deemed a rule of public policy, - The bringing of the action against the Paramount
beyond the contractual limitation period. Hence, this considering the unrelieved congestion in the courts. Surety & Insurance Company, the agent of the
appeal. As a consequence of the foregoing, action may be defendant company, cannot have any legal effect
brought within the statutory period of limitation for except that of notifying the agent of the claim.
ISSUE written contracts (New Civil Code, Article 1144). Beyond such notification, the filing of the action can
WON the provision of a fidelity bond that no action serve no other purpose. There is no law giving any
shall be had or maintained thereon unless ANG v. FULTON FIRE INSURANCE CO. effect to such action upon the principal. Besides,
commenced within one year from the making of a 2 SCRA 945 there is no condition in the policy that the action
claim for the loss upon which the action is based, is LABRADOR; July 31, 1961 must be filed against the agent, and the Court can
valid, in view of Section 61-A of the Insurance Act not by interpretation extend the clear scope of the
invalidating stipulations limiting the time for NATURE agreement beyond what is agreed upon by the
commencing an action thereon to less than one year Appeal from judgment of the CFI ordering the parties.
from the time the cause of action accrues defendant Fulton Fire Insurance Co. to pay the - Their contract is the law between the parties, and
plaintiffs the sum of P10,000.00, with interest, and their agreement that an action on a claim denied by
HELD an additional sum of P2,000.00 as attorney's fees, the insurer must be brought within one year from the
NO and costs. denial, governs, not the rules on the prescription of
- A fidelity bond is, in effect, in the nature of a actions.
contract of insurance against loss from misconduct, FACTS Disposition The judgment appealed from is hereby
and is governed by the same principles of - The stocks of general merchandise in the store of set aside and the case dismissed, with costs against
interpretation. Consequently, the condition of the the Ang spouses are insured with Fulton. While the plaintiffs-appellees.
bond in question, limiting the period for bringing insurance was in force, fire destroyed the goods. The
action thereon, is subject to the provisions of Section Angs filed their first claim immediately after the fire. TRAVELLERS INSURANCE & SURETY CORP. v.
61-A of the Insurance Act (No. 2427), as amended - Their claim was denied on April 6, 1956. They CA (MENDOZA)
by Act 4101 of the pre-Commonwealth Philippine received notice on April 19, 1956. 272 SCRA 536
Legislature, prescribing that: - The Angs brought an action against the agent on HERMOSISIMA, JR; May 22, 1997
SEC. 61-A: A condition, stipulation or agreement May 11, 1956. The court denied the suit and the mfr
in any policy of insurance, limiting the time for on Sept. 3 and 12, 1957. NATURE
commencing an action thereunder to a period of - The Angs filed against Fulton on May 26, 1958. The petition herein seeks the review and reversal of
less than one year from the time when the cause - There was a clause in the policy: the decision of respondent Court of Appeals affirming
of action accrues is void. 13.If the claim be in any respect fraudulent, or if in toto the judgment of the Regional Trial Court in an
- Since a "cause of action" requires, as essential any false declaration is made or used in support action for damages filed by private respondent
elements, not only a legal right of the plaintiff and a thereof, or if any fraudulent means or devices are Vicente Mendoza, Jr. as heir of his mother who was
correlative obligation of the defendant but also "an used by the Insured or any one acting on his killed in a vehicular accident.
act or omission of the defendant in violation of said behalf to obtain any benefit under this Policy, or, if
legal right," the cause of action does not accrue until the loss or damage be occasioned by the wilful act FACTS
the party obligated refuses, expressly or impliedly, to or with the connivance of the Insured, or, if the -an old lady was hit by a taxicab. The taxicab was
comply with its duty (in this case, to pay the amount claim be made and rejected and an action or suit later identified and a case was filed against the
of the bond). The year for instituting action in court be not commenced within twelve months after driver and owner. Later, an amendment was filed to
must be reckoned, therefore, from the time of such rejection or (in case of arbitration taking include the insurance company. RTC and CA ordered
appellee's refusal to comply with its bond; it can not place in pursuance of the 18th condition of this that the owner, driver as well as the insurance
be counted from the creditor's filing of the claim of Policy) within twelve months after the arbitrator or company be held solidarily liable.
INSURANCE Page
108

NATURE > It cites Art. 220711 and Art. 130412 of the Civil
ISSUE Petition to review CA decision ordering Manila Code, and claims a preferred right to retain the
WON RTC and CA erred Mahogany Manufacturing Corporation to pay Zenith amount coming from San Miguel, despite the
HELD Insurance Corporation P5,000 with 6% annual subrogation in favor of ZENITH.
YES interest, attorney's fees, and costs of suit Respondent’s Arguments
- Where the contract provides for indemnity against > There was no qualification to its right of
liability to third persons, then third persons to whom FACTS subrogation under the Release of Claim executed by
the insured is liable can sue the insurer. Where the - From 6 March 1970 to 6 March 1971, MLA petitioner, the contents having expressed all intents
contract is for indemnity against actual loss or MAHOGANY insured its Mercedes Benz 4-door sedan and purposes of the parties.
payment, then third persons cannot proceed against with ZENITH.
the insurer, the contract being solely to reimburse - On 4 May 1970, the insured vehicle was bumped ISSUE
the insured for liability actually discharged by him and damaged by a truck owned by San Miguel WON the insurer may recover the sum of P5,000
thru payment to third persons, said third persons' Corporation. For the damage caused, ZENITH paid
recourse being thus limited to the insured alone. But MLA MAHOGANY P5,000 in amicable settlement. MLA HELD
in the case at bar, there was no contract shown. MAHOGANY's general manager executed a Release of YES
What then was the basis of the RTC and the CA Claim, subrogating respondent company to all its Ratio Since the insurer can be subrogated to only
to say that the insurance contract was a third- right to action against San Miguel Corporation. such rights as the insured may have, should the
party liability insurance policy? Consequently, - On 11 Dec 1972, ZENITH wrote Insurance insured, after receiving payment from the insurer,
the trial court was confused as it did not distinguish Adjusters, Inc. to demand reimbursement from San release the wrongdoer who caused the loss, the
between the private respondent's cause of action Miguel. Insurance Adjusters, Inc. refused insurer loses his rights against the latter. But in such
against the owner and the driver of the Lady Love reimbursement, alleging that San Miguel had already a case, the insurer will be entitled to recover from
taxicab and his cause of action against petitioner. paid petitioner P4,500, as evidenced by a cash the insured whatever it has paid to the latter, unless
The former is based on torts and quasi-delicts while voucher and a Release of Claim executed by the the release was made with the consent of the
the latter is based on contract. General Manager of petitioner. insurer.
- Even assuming arguendo that there was such a - ZENITH thus demanded from petitioner Reasoning
contract, private respondent's cause of action can reimbursement of the sum of P4,500 paid by San - Although petitioner’s right to file a deficiency claim
not prevail because he failed to file the written Miguel. against San Miguel is with legal basis, without
claim mandated by the Insurance Code (before - City Court ordered petitioner to pay respondent prejudice to the insurer's right of subrogation,
it was amended-action must be brought within P4,500. nevertheless, when Manila Mahogany executed
six months from date of the accident (this is - CFI affirmed the City Court's decision in toto. another release claim discharging San Miguel from
what’s applicable here) ; after amendment- - CA affirned CFI, with the modification that "all actions, claims, demands and rights of action
"action or suit for recovery of damage due to loss or petitioner was to pay the total amount of P5,000 it that now exist or hereafter arising out of or as a
injury must be brought in proper cases, with the had earlier received from ZENITH. consequence of the accident" after the insurer had
Commissioner or the Courts within one year from Petitioner’s Claims paid the proceeds of the policy - the compromise
denial of the claim, otherwise the claimant's right of > It is not bound to pay P4,500, and much more, agreement of P5,000 being based on the insurance
action shall prescribe" ). He is deemed, under this P5,000 to ZENITH as the subrogation in the Release policy - the insurer is entitled to recover from the
legal provision, to have waived his rights as against of Claim it executed in favor of respondent was insured the amount of insurance money paid. Since
petitioner-insurer. conditioned on recovery of the total amount of petitioner by its own acts released San Miguel,
Disposition petition granted damages petitioner had sustained. Since total thereby defeating private respondent’s right of
damages were valued by petitioner at P9,486.43 and subrogation, the right of action of petitioner against
SUN INSURANCE v. CA (supra p.57) only P5,000 was received by petitioner, MLA the insurer was also nullified.
MAHOGANY argues that it was entitled to go after
11
COASTWISE v. CA (supra p.70) San Miguel to claim the additional P4,500. Article 2207: “If the plaintiff's property has been insured, and he
has received indemnity from the insurance company for the injury or
loss arising out of the wrong or breach of contract complained of the
CEBU SHIPYARD v. WILLIAM LINES (supra p.3) insurance company shall be subrogated to the rights of the insured
against the wrongdoer or the person who has violated the contract. If
the amount paid by the insurance company does not fully cover the
MANILA MAHOGANY MANUFACTURING CORP v. injury or loss the aggrieved party shall be entitled to recover the
CA (ZENITH INSURANCE CORP) deficiency from the person causing the loss or injury.”
154 SCRA 652 12
Article 1305: “A creditor, to whom partial payment has been made,
PADILLA; October 12, 1987 may exercise his right for the remainder, and he shall be preferred to
the person who has been subrogated in his place in virtue of the partial
payment of the same credit.”
INSURANCE Page
109

- As held in Phil. Air Lines v. Heald Lumber Co., latter's suretyship in favor of the former. It was This is the amount for which Pioneer may sue
under Art. 2207, the real party in interest with stipulated therein that Lim transfer and convey to defendants, assuming that the indemnity agreement
regard to the portion of the indemnity paid is the the surety the two aircrafts. is still valid and effective. But since the amount
insurer and not the insured. - Lim defaulted on his subsequent installment realized from the sale of the mortgaged chattels are
SUBROGATION: The right of subrogation can only payments prompting JDA to request payments from P35,000.00 for one of the airplanes and P2,050.00
exist after the insurer has paid the insured, the surety. for a spare engine, or a total of P37,050.00, Pioneer
otherwise the insured will be deprived of his right to - Pioneer paid a total sum of P298,626.12. is still overpaid by P33,383.72. Therefore, Pioneer
full indemnity. If the insurance proceeds are not - Pioneer then filed a petition for the extrajudicial has no more claim against defendants.
sufficient to cover the damages suffered by the foreclosure of the said chattel mortgage before the - The payment to the petitioner made by the
insured, then he may sue the party responsible for Sheriff of Davao City. reinsurers was not disputed. Considering this
the damage for the remainder. To the extent of the - The Cervanteses and Maglana, however, filed a admitted payment, the only question was the effect
amount he has already received from the insurer third party claim alleging that they are co-owners of of payment made by the reinsurers to the petitioner
enjoys the right of subrogation. the aircrafts, - In general a reinsurer, on payment of a loss
Disposition Petition DENIED. Judgment appealed - On July 19, 1966, Pioneer filed an action for judicial acquires the same rights by subrogation as are
from is AFFIRMED with costs against petitioner. foreclosure with an application for a writ of acquired in similar cases where the original insurer
preliminary attachment against Lim and respondents, pays a loss (Universal Ins. Co. v. Old Time Molasses
PIONEER INSURANCE v. CA (BORDER the Cervanteses, Bormaheco and Maglana. Co.).
MACHINERY & HEAVY EQUIPMENT INC) **Maglana, Bormaheco and the Cervanteses filed - The rules of practice in actions on original
175 SCRA 668 cross-claims against Lim alleging that they were not insurance policies are in general applicable to actions
GUTIERREZ, JR.; July 28, 1989 privies to the contracts signed by Lim and, by way of or contracts of reinsurance (Delaware, Ins. Co. v.
counterclaim, sought for damages for being exposed Pennsylvania Fire Ins. Co.).
NATURE to litigation and for recovery of the sums of money - Hence the applicable law is Article 2207 of the new
Petitions for review on certiorari of a decision of the they advanced to Lim for the purchase of the Civil Code, to wit: Art. 2207. If the plaintiff’s
CA aircrafts in question. (this constitutes the second property has been insured, and he has received
petition but will no longer be discussed because it is indemnity from the insurance company for the injury
FACTS not relevant to the topic) or loss arising out of the wrong or breach of contract
- In 1965, Jacob S. Lim was engaged in the airline - After trial on the merits, a decision was rendered complained of, the insurance company shall be
business as owner-operator of Southern Air Lines holding Lim liable to pay Pioneer but dismissed subrogated to the rights of the insured against the
(SAL), a single proprietorship. Pioneer's complaint against all other wrongdoer or the person who has violated the
-On May 17, 1965, at Tokyo, Japan, Japan Domestic defendants. contract. If the amount paid by the insurance
Airlines (JDA) and Lim entered into and executed a - CA modified the trial court's decision in that the company does not fully cover the injury or loss, the
sales contract for the sale and purchase of two (2) plaintiff’s complaint against all the defendants aggrieved party shall be entitled to recover the
DC-3A Type aircrafts and one (1) set of necessary (including Lim) was dismissed. deficiency from the person causing the loss or injury
spare parts for the total agreed price of US - If a property is insured and the owner receives the
$109,000.00 to be paid in installments. ISSUE indemnity from the insurer, it is provided in said
- On May 22, 1965, Pioneer Insurance and Surety WON the petition of Pioneer Insurance and Surety article that the insurer is deemed subrogated to the
Corporation, as surety, executed and issued its Corporation against all defendants was rightly rights of the insured against the wrongdoer and if
Surety Bond No. 6639 in favor of JDA, in behalf of its dismissed the amount paid by the insurer does not fully cover
principal, Lim, for the balance price of the aircrafts the loss, then the aggrieved party is the one entitled
and spare parts. HELD to recover the deficiency. Evidently, under this legal
-Border Machinery and Heavy Equipment Company, YES provision, the real party in interest with regard to
Inc. (Bormaheco), Francisco and Modesto Cervantes - Both the TC and CA made the finding that Pioneer the portion of the indemnity paid is the insurer and
(Cervanteses) and Constancio Maglana contributed reinsured its risk of liability under the surety bond it not the insured (. Air Lines, Inc. v. Heald Lumber
some funds used in the purchase of the above had executed in favor of JDA, collected the proceeds Co., and Manila Mahogany Manufacturing
aircrafts and spare parts. They executed two (2) of such reinsurance in the sum of P295,000, and paid Corporation v. Court of Appeals)
separate indemnity agreements in favor of Pioneer, with the said amount the bulk of its alleged liability - It is clear from the records that Pioneer sued in its
one signed by Maglana and the other jointly signed to JDA under the said surety bond. The total amount own name and not as an attorney-in-fact of the
by Lim for SAL, Bormaheco and the Cervanteses. paid by Pioneer to JDA is P299,666.29. Since Pioneer reinsurer. Accordingly, the appellate court did not
- On June 10, 1965, Lim doing business under the has collected P295,000.00 from the reinsurers, the commit a reversible error in dismissing the
name and style of SAL executed in favor of Pioneer uninsured portion of what it paid to JDA is the petitioner's complaint as against the respondents for
as deed of chattel mortgage as security for the difference between the two amounts, or P3,666.28. the reason that the petitioner was not the real party
INSURANCE Page
110

in interest in the complaint and, therefore, has no ISSUE “Property Damage” coverage (liabilities from damage
cause of action against the respondents. WON the insurer PANMALAY may institute an action caused by insured vehicle to properties of 3rd parties)
Disposition Petitions dismissed. Questioned decision to recover the amount it had paid its assured in ON CA: the terms of a contract are to be construed
of CA affirmed. settlement of an insurance claim against private according to the sense and meaning of the terms
respondents as the parties allegedly responsible for which the parties thereto have used. In the case of
the damage caused to the insured vehicle, in property insurance policies, the evident intention of
PAN MALAYAN INSURANCE CORPORATION v. accordance with A2207, NCC the contracting parties, i.e., the insurer and the
CA (FABIE, HER UNKNOWN DRIVER) assured, determine the import of the various terms
184 SCRA 54 HELD and provisions embodied in the policy. It is only
CORTES, April 3, 1990 YES when the terms of the policy are ambiguous,
Ratio Article 2207 of the Civil Code is founded on equivocal or uncertain, such that the parties
NATURE the well-settled principle of subrogation. If the themselves disagree about the meaning of particular
PETITION to review the decision of the Court of insured property is destroyed or damaged through provisions, that the courts will intervene. In such an
Appeals the fault or negligence of a party other than the event, the policy will be construed by the courts
assured, then the insurer, upon payment to the liberally in favor of the assured and strictly against
FACTS assured, will be subrogated to the rights of the the insurer
- Pan Malayan Insurance Company (Panmalay) assured to recover from the wrongdoer to the extent - Both Panmalay and Canlubang had the same
insured the Mitsubishi Colt Lancer car registered in that the insurer has been obligated to pay. Payment interpretation regarding the coverage of insured risk
the name of Canlubang Automotive Resources by the insurer to the assured operates as an regarding “accidental collision or overturning…” to
Corporation (Canlubang) under its motor vehicle equitable assignment to the former of all remedies include damages caused by 3rd party to Canlubang so
insurance policy. Among the provisions of the policy which the latter may have against the third party it was improper for CA to ascribe meaning contrary
was a “own-damage” clause whereby Panmalay whose negligence or wrongful act caused the loss. to the clear intention and understanding of the
agrees to indemnify Canlubang in cases of damage The right of subrogation is not dependent upon, nor parties.
caused by “accidental collision or overturning, or does it grow out of, any privity of contract or upon - Court on several occasions defined “accident” or
collision or overturning consequent upon mechanical written assignment of claim. It accrues simply upon “accidental” as taking place “without one’s foresight
breakdown or consequent upon wear and tear”. payment of the insurance claim by the insurer or expectation, an event that proceeds from an
- On 1985, the insured car was sideswept and Exceptions unknown cause, or is an unusual effect of a known
damaged by a car owned by Erlinda Fabie, driven by (1) if the assured by his own act releases the cause and, therefore, not expected” [Dela Cruz v.
an unknown driver who fled the scene. Panmalay, in wrongdoer or third party liable for the loss or Capital Insurance & Surety Co.] The concept
accordance with the policy, defrayed the cost of damage, from liability, the insurer's right of "accident" is not necessarily synonymous with the
repair of the insured car and was subrogated to the subrogation is defeated; concept of "no fault". It may be utilized simply to
rights of Canlubang against the driver and owner of (2) where the insurer pays the assured the value of distinguish intentional or malicious acts from
the pick-up. Panmalay then filed a complaint for the lost goods without notifying the carrier who has negligent or careless acts of man.
damages with RTC Makati against Erlinda Fabie and in good faith settled the assured's claim for loss, the - damage/loss to insured vehicle due to negligence of
her driver on the grounds of subrogation, with the settlement is binding on both the assured and the 3rd parties not listed as exceptions to coverage in the
latter failing and refusing to pay their claim. Fabie insurer, and the latter cannot bring an action against insurance policy
filed a Motion for Bill of Particulars. the carrier on his right of subrogation; - Interpretation given by Panmalay is more in
- RTC: dismissed complaint for lack of cause of (3) where the insurer pays the assured for a loss keeping with rationale behind rules on interpretation
action (payment by PANMALAY of CANLUBANG's which is not a risk covered by the policy, thereby of insurance contracts in favor of assured or
claim under the "own damage" clause of the effecting "voluntary payment", the former has no beneficiary: indemnity or payment
insurance policy was an admission by the insurer right of subrogation against the third party liable for - EVEN if voluntarily indemnified Canlubang, as
that the damage was caused by the assured and/or the loss interpreted by TC: the insurer who may have no
its representatives) – Panmalay appealed Reasoning rights of subrogation due to "voluntary" payment
- CA: dismissed appeal, affirmed RTC (applying the - Both TC and CA are incorrect. may never. theless recover from the third party
ejusdem generis rule held that Section III-1 of the ON TC: “Own damage” (not found in the insurance responsible for the damage to the insured property
policy, which was the basis for settlement of policy) simply meant that Panmalay had assumed to under Article 1236 of the Civil Code. [Sveriges
CANLUBANG's claim, did not cover damage arising reimburse the cost for repairing the damage to the Angfartygs Assurans Forening v. Qua Chee Gan]
from collision or overturning due to the negligence of insured vehicle. It’s different from “Third Party Disposition the present petition is GRANTED.
third parties as one of the insurable risk) Liability” coverage (liabilities arising from the death Petitioner's complaint for damages against private
of or bodily injuries suffered by 3rd parties) and from respondents is hereby REINSTATED. Let the case be
remanded to the lower court for trial on the merits.
INSURANCE Page
111

- Jamila in its MR invoked the first ground which - Article 2207 is a restatement of a settled principle
FIREMAN'S FUND INSURANCE COMPANY v. had never been passed upon by the lower court. But of American jurisprudence. Subrogation has been
JAMILA & COMPANY, INC. the lower court in its order granting Jamila's motion referred to as the doctrine of substitution. It is an
70 SCRA 323 for reconsideration, completely ignored that first arm of equity that may guide or even force one to
AQUINO; April 1976 ground. It reverted to the second ground which was pay a debt for which an obligation was incurred but
relied upon in its order previous order. The lower which was in whole or in part paid by another.
FACTS court reiterated its order, stating that Fireman's Fund - Subrogation is founded on principles of justice and
- Jamila & Co., Inc. or the Veterans Philippine had no cause of action against Jamila because Jamila equity, and its operation is governed by principles of
Scouts Security Agency contracted to supply security did not consent to the subrogation. The court did not equity. It rests on the principle that substantial
guards to Firestone. Jamila assumed responsibility mention Firestone, the co-plaintiff of Fireman's Fund. justice should be attained regardless of form, that is,
for the acts of its security guards. The First Quezon - Firestone and Fireman's Fund filed an MR on the its basis is the doing of complete, essential, and
City Insurance Co., Inc. executed a bond in the sum ground that Fireman's Fund was suing on the basis perfect justice between all the parties without regard
of P20,000 to guarantee Jamila's obligations under of legal subrogation whereas the lower court to form.
that contract. erroneously predicated its dismissal order on the - Subrogation is a normal incident of indemnity
- On May 18, 1963 properties of Firestone valued at theory that there was no conventional subrogation insurance. Upon payment of the loss, the insurer is
P11,925 were lost allegedly due to the acts of its because the debtor's consent was lacking. entitled to be subrogated pro tanto to any right of
employees who connived with Jamila's security - The plaintiffs cited article 2207 of the Civil Code action which the insured may have against the third
guard. Fireman's Fund, as insurer, paid to Firestone which provides that "if the plaintiff's property has person whose negligence or wrongful act caused the
the amount of the loss. Fireman's Fund was been insured, and he has received indemnity from loss. The right of subrogation is of the highest
subrogated to Firestone's right to get reimbursement the insurance company for the injury or loss arising equity. The loss in the first instance is that of the
from Jamila. Jamila and its surety, First Quezon City out of the wrong or breach of contract complained of, insured but after reimbursement or compensation, it
failed to pay the amount of the loss in spite of the insurance company shall be subrogated to the becomes the loss of the insurer.
repeated demands. rights of the insured against the wrongdoer or the - Although many policies including policies in the
- Upon defendant's motions, the lower court person who has violated the contract". standard form, now provide for subrogation, and
dismissed the complaint as to Jamila on the ground - The lower court denied plaintiff's motion. They thus determine the rights of the insurer in this
that there was no allegation that it had consented to filed a second MR, calling the lower court's attention respect, the equitable right of subrogation as the
the subrogation and, therefore, Fireman's Fund had to the fact that the issue of subrogation was of no legal effect of payment inures to the insurer without
no cause of action against it. It also dismissed the moment because Firestone, the subrogor, is a party- any formal assignment or any express stipulation to
complaint as to First Quezon City on the ground of plaintiff and could sue directly Jamila in its own right. that effect in the policy. Stated otherwise, when the
res judicata. It appears that the same action was Without resolving that contention, the lower court insurance company pays for the loss, such payment
previously filed in a civil case which was dismissed denied plaintiffs' second MR. operates as an equitable assignment to the insurer of
because of the failure of the same plaintiffs and their the property and all remedies which the insured may
counsel to appear at the pre-trial. ISSUE have for the recovery thereof. That right is not
- Upon an MR, the lower court set aside its order of WON the complaint of Firestone and Fireman's Fund dependent upon, nor does it grow out of, any privity
dismissal and sustained plaintiff's contention that states a cause of action against Jamila of contract, or upon written assignment of claim, and
there was no res judicata as to First Quezon City payment to the insured makes the insurer an
because the civil case was dismissed without HELD assignee in equity.
prejudice. However, due to inadvertence, the lower YES - On the other hand, Firestone is really a nominal
court did not state in its order why it set aside its - Fireman's Fund's action against Jamila is squarely party in this case. It had already been indemnified
prior order dismissing the complaint with respect to sanctioned by article 2207. As the insurer, Fireman's for the loss which it had sustained. Obviously, it
Jamila. Jamilla had originally moved for the dismissal Fund is entitled to go after the person or entity that joined as a party-plaintiff in order to help Fireman's
of the complaint on the ground of lack of cause of violated its contractual commitment to answer for Fund to recover the amount of the loss from Jamila
action. Its basis for its contention were: (1) that the the loss insured against. and First Quezon City. Firestone had tacitly assigned
complaint did not allege that Firestone, pursuant to - The trial court erred in applying to this case the to Fireman's Fund its cause of action against Jamila
the contractual stipulation quoted in the complaint, rules on novation. The plaintiffs in alleging in their for breach of contract. Sufficient ultimate facts are
had investigated the loss and that Jamila was complaint that Fireman's Fund "became a party in alleged in the complaint to sustain that cause of
represented in the investigation and (2) that Jamila interest in this case by virtue of a subrogation right action.
did not consent to the subrogation of Fireman's Fund given in its favor by" Firestone, were not relying on
to Firestone's right to get reimbursement from Jamila the novation by change of creditors as contemplated TABACALERA v. NORTH FRONT SHIPPING
and its surety. The lower court in its order of in articles 1291 and 1300 to 1303 of the Civil Code 272 SCRA 527
dismissal had sustained the second ground. but rather on article 2207. BELLOSILLO; May 16, 1997
INSURANCE Page
112

- As a corporation engaged in the business of issued a policy on the sole basis of the application on
FACTS transporting cargo offering its services February 11, 1989. On April 1992, petitioner
- 20,234 sacks of corn grains valued at P3.5M were indiscriminately to the public, it is without a doubt a received private respondent’s claim, which declared
shipped on board North Front 777, defendant’s common carrier. As such, it has the burden of that the insured, Florence Pulido, died of acute
vessel. The cargo was consigned to Republic Flour proving that it observed extraordinary diligence to pneumonia on September 10, 1991.
Mills Corp. under Bill of Lading No. 001 and insured avoid responsibility for the lost cargo. The clean bill - Petitioner withheld payment on the ground that the
with Tabacalera, Prudential Guarantee & Assurance, of lading it issued disprove the master of the vessel’s policy claimed under was void from the start for
and New Zealand Insurance. claim that the grains were farm wet when loaded. If having been procured in fraud. It is petitioner’s
- Republic Flour was advised of the vessel’s arrival in they were wet, the master of the vessel should have contention that even before they received private
Manila, but did not immediately commence the known that the grains would eventually deteriorate respondent’s claim for death benefits, their
unloading operations. Unloading was sometimes when sealed in hot compartments in hatches of a investigation concerning the subject policy yielded
stopped due to varying weather and sometimes for ship and should have undertaken precautionary the information that the insured, Florence Pulido,
no apparent reason. Unloading was only completed measures to avoid this. The arrival of the goods at died in 1988, before the application for insurance on
20 days after the arrival of the barge; by then, the the place of destination in bad order makes a prima her life was made. While this was communicated to
cargo was short 26.333 metric tons and the rest was facie case against the common carrier, which must private respondent in a letter, private respondent
already moldy and deteriorating. prove its non-liability. had already filed her claim earlier that month. In
- Analyses showed that the deterioration was caused - While petitioners presented evidence of the vessel’s another letter, however, petitioner confirmed to
by moisture content from salt water, which could be bad shape and a laboratory analysis revealing that private respondent receipt of the claim papers and
arrested by drying. However, Republic Flour rejected the grains were contaminated with salt water, assured her that her case was “being given
the entire cargo and demanded that defendant North defendants failed to rebut said arguments or even preferential attention and prompt action”.
Front Shipping pay the damages suffered by it. The endeavor to establish that the loss, destruction or - Petitioner caused another investigation respecting
demands were unheeded and the insurance deterioration was due to a fortuitous event; an the subject policy. Pursuant to the findings of this
companies were obliged to pay Republic Flour act/omission of the owner of the goods; the second investigation, petitioner stood by its initial
P2,189,433 character of the goods or defects in their packing; or decision to treat the policy as void and not to honor
- By virtue of the insurance companies’ payment, an order or act of a competent public authority. the claim. On November 9, 1992, private
they were subrogated to the rights of Republic Flour. - However, Republic Flour is also found to be guilty respondent enlisted the services of counsel in
Petitioners filed a complaint against North Front of contributory negligence for not immediately reiterating her claim for death benefitsPetitioner still
Shipping, claiming the loss was exclusively staring the unloading operations and for providing no refused to make payment and thus, this action.
attributable to the latter’s fault and negligence. explanation for the delay. As such, it should share at
Having surveyed the vessel, it was found that the least 40% of the loss. ISSUE
barge had cracks in its bodega. The hatches on the Disposition The decision of the CA is REVERSED WON lower court erred in holding that there was no
crates of grain were not sealed and the tarpaulins and SET ASIDE fraud
used in covering them were not new, contrary to
North Front Shipping’s claims. North Front Shipping PHILIPPINE AMERICAN LIFE INSURANCE HELD
reiterated that the barge was inspected prior to COMPANY v. CA (ELIZA PULIDO) - The records bear out that since the onset of this
loading and found seaworthy and were issued a 344 SCRA 360 case, the main issue has always been whether there
permit to sail by the Coast Guard. They further GONZAGA-REYES; November 15, 2000 was fraud in the obtainment of the disputed policy,
averred that the grains were farm wet and not or put differently, whether the insured, Florence
properly dried before loading. NATURE Pulido, was in fact dead before the application for
- The court dismissed the complaint, ruling that the This petition for review on certiorari seeks to reverse insurance on her life was made. This the lower
contract entered into was a charter-party the Decision of the Special Second Division of the courts had effected ruled on, upon a preponderance
agreement; as such, only ordinary diligence in the Court of Appeals of the evidence duly received from both parties. We
care of the goods was required of North Front see no reversible error in the finding of both
Shipping. FACTS respondent court and the trial court in favor of the
- On January 9, 1989, petitioner received from one correctness of the entries in Certificate of Death,
ISSUE Florence Pulido an application for life insurance, duly registered with the Local Civil Registrar of
WON defendant is required to observe extraordinary dated December 16, 1988, in the amount of Bagulin, La Union, which declared that Florence
diligence in its vigilance over the goods it transports P100,000.00 which designated her sister, herein Pulido died of acute pneumonia on September 10,
private respondent, as its principal beneficiary. 1991. Dr. Irineo Gutierrez, the Municipal Health
HELD Because the insurance applied for was non-medical, Officer of Bagulin, La Union whose signature
YES petitioner did not require a medical examination and appeared in the death certificate, testified in addition
INSURANCE Page
113

that he ministered to the ailing Florence Pulido for and/or damage with the St. Paul Fire & Marine Ratio The purpose of the bill of lading is to provide
two days immediately prior to her death. This fact is Insurance Company. for the rights and liabilities of the parties in reference
likewise noted in the death certificate. - The SS "Tai Ping" arrived at the Port of Manila and to the contract to carry. The stipulation in the bill of
- Death certificates, and notes by a municipal health discharged its aforesaid shipment into the custody of lading limiting the common carrier's liability to the
officer prepared in the regular performance of his Manila Port Service, the arrastre contractor for the value of the goods appearing in the bill, unless the
duties, are prima facie evidence of facts therein Port of Manila. The said shipment was discharged shipper or owner declares a greater value, is valid
stated. A duly-registered death certificate is complete and in good order with the exception of one and binding. This limitation of the carrier's liability is
considered a public document and the entries found (1) drum and several cartons which were in bad sanctioned by the freedom of the contracting parties
therein are presumed correct, unless the party who order condition. Because consignee failed to receive to establish such stipulations, clauses, terms, or
contests its accuracy can produce positive evidence the whole shipment and as several cartons of conditions as they may deem convenient, provided
establishing otherwise. Petitioner’s contention that medicine were received in bad order condition, the they are not contrary to law, morals, good customs
the death certificate is suspect because Dr. Gutierrez consignee filed the corresponding claim in the and public policy. A stipulation fixing or limiting the
was not present when Florence Pulido died, and amount of P1,109.67 representing the C.I.F. value of sum that may be recovered from the carrier on the
knew of Florence’s death only through Ramon the damaged drum and cartons of medicine with the loss or deterioration of the goods is valid, provided it
Piganto, does not merit a conclusion of fraud. No carrier and the Manila Port Service. However, both is (a) reasonable and just under the circumstances,
motive was imputed to Dr. Gutierrez for seeking to refused to pay such claim. Consequently, the and (b) has been fairly and freely agreed upon. In
perpetuate a falsity in public records. Petitioner was consignee filed its claim with the insurer, St. Paul the case at bar, the liabilities of the defendants-
likewise unable to make out any clear motive as to Fire & Marine Insurance Co., the insurance company, appellees with respect to the lost or damaged
why Ramon Piganto would purposely lie. Mere on the basis of such claim, paid to the consignee the shipments are expressly limited to the C.I.F. value of
allegations of fraud could not substitute for the full insured value of the lost and damagcd goods, the goods as per contract of sea carriage embodied
and convincing evidence that is required to prove it. including other expenses in connection therewith, in in the bill of lading.
A failure to do so would leave intact the presumption the total amount of $1,134.46 U.S. currency. - The plaintiff-appellant, as insurer, after paying the
of good faith and regularity in the performance of - As subrogee of the rights of' the shipper and/or claim of the insured for damages under the
public duties, which was the basis of both respondent consignee, the insurer, St. Paul Fire & Marine insurance, is subrogated merely to the rights of the
court and the trial court in finding the date of Insurance Co., instituted an action against the assured. As subrogee, it can recover only the
Florence Pulido’s death to be as plaintiff-private defendants for the recovery of said amount of amount that is recoverable by the latter. Since the
respondent maintained. $1,134.46, plus costs. right of the assured, in case of loss or damage to the
- We cannot likewise give credence to petitioner’s - The defendants resisted the action. However, for goods, is limited or restricted by the provisions in the
submission that the inconsistencies in the the purpose only of avoiding litigation without bill of lading, a suit by the insurer as subrogee
testimonies of the witnesses for plaintiff-private admitting liability to the consignee, the defendants necessarily is subject to like limitations and
respondent are in themselves evidence of fraud. offered to settle the latter’s claim in full by paying restrictions.
Such alleged inconsistencies are matters of credibility the C.I.F. value of the damaged cargo, but this offer 2. On the date of the discharge of the cargo. The
which had been ably passed upon by the lower court. was declined by the plaintiff. peso equivalent was based by the consignee on the
Disposition the instant petition is DENIED - The LC rendered judgment ordering the exchange rate of P2.015 to $1.00 which was the rate
defendants to pay the plaintiff the sum of P300.00. existing at that time.
ST.PAUL FIRE & MARINE INSURANCE CO v. The plaintiff filed a MFR contending that it should
MACONDRAY & CO INC recover the amount of $1,134.46 or its equivalent in PHILAM v. AUDITOR (supra p.59)
70 SCRA 122 pesos at the rate of P3.90, instead of P2.00, but this
ANTONIO; March 25, 1976 was denied. Hence, this appeal. FIELDMEN’S v. ASIAN SURETY (supra p.60)

FACTS ISSUES EQUITABLE v. RURAL INSURANCE (supra p.60)


- Winthrop Products, Inc. shipped aboard the SS 1. WON in case of loss or damage, the liability of the
"Tai Ping", owned and operated by Wilhelm carrier to the consignee is limited to the C.I.F. value COQUIA v. FIELDMEN'S INSURANCE CO. INC.
Wilhelmsen, 218 cartons and drums of drugs and of the goods which were lost or damaged 26 SCRA 178
medicine, with the freight prepaid, which were 2. WON the insurer who has paid the claim in dollars CONCEPCION; November 29, 1968
consigned to Winthrop-Stearns, Inc. Barber to the consignee should be reimbursed in its peso
Steamship Lines, Inc., agent of Wilhelm Wilhelmsen equivalent on the date of discharge of the cargo or NATURE
issued Bill of Lading No. 34, in the name of Winthrop on the date of the decision Appeal from the decision of the CFI certified by CA
Products, Inc. as shipper, with arrival notice in-
Manila to consignee Winthrop-Stearns, Inc. The HELD FACTS
shipment was insured by the shipper against loss 1. YES
INSURANCE Page
114

- December 1, 1961, appellant Fieldmen's Insurance fulfillment provided he communicated his acceptance and did properly join the latter in filing the complaint
Company, Inc. issued, in favor of the Manila Yellow of the obligor before its revocation. A mere incidental herein.
Taxicab Co., Inc. a common carrier accident benefit or interest of a person is not sufficient. The 2. Based upon Section 17 of the policy:
insurance policy, covering the period from December contracting parties must have clearly and "If any difference or dispute shall arise with
1, 1961 to December ,1962. It was stipulated in said deliberately conferred a favor upon a third person." respect to the amount of the Company's liability
policy that: - Does the policy in question belong to such class of under this Policy, the same shall be referred to the
"The Company will, subject to the Limits of Liability contracts pour autrui? decision of a single arbitrator to be agreed upon by
and under the Terms of this Policy, indemnify the In this connection, said policy provides, inter alia: both parties or failing such agreement of a single
Insured in the event of accident caused by or arising "Section I — Liability to Passengers. 1. The arbitrator, to the decision of two arbitrators, one to
out of the use of Motor Vehicle against all sums Company will, subject to the Limits of Liability and be appointed in writing by each of the parties
which the Insured will become legally liable to pay in under the Terms of this Policy, indemnify the Insured within one calendar month after having been
respect of: Death or bodily injury to any fare-paying in the event of accident caused by or arising out of required in writing so to do by either of the parties
passenger including the Driver, Conductor and/or the use of Motor Vehicle against all sums which the and in case of disagreement between the
Inspector who is riding in the Motor Vehicle insured Insured will become legally liable to pay in respect arbitrators, to the decision of an umpire who shall
at the time of accident or injury." of: Death or bodily injury to any fare-paying have been appointed in writing by the arbitrators
- While the policy was in force, or on February 10, passenger including the Driver. . . who is riding in before entering on the reference and the costs of
1962, a taxicab of the Insured, driven by Carlito the Motor Vehicle insured at the time of accident or and incidental to the reference shall be dealt with
Coquia, met a vehicular accident to which he died. injury. in the Award. And it is hereby expressly stipulated
The Insured filed therefor a claim for P5,000.00 to "Section II. — Liability to the Public and declared that it shall be a condition precedent
which the Company replied with an offer to pay "3. In terms of and subject to the limitations of to any right of action or suit upon this Policy that
P2,000.00, by way of compromise. The Insured and for the purposes of this Section, the Company the award by such arbitrator, arbitrators or umpire
rejected it and made a counter-offer for P4,000.00, will indemnify any authorized Driver who is driving of the amount of the Company's liability hereunder
but the Company did not accept it. the Motor Vehicle . . . " if disputed shall be first obtained."
- On September 18, 1962, the Insured and Carlito's "Conditions - The record shows that none of the parties to the
parents filed a complaint against the Company to "7. In the event of death of any person entitled contract invoked this section, or made any reference
collect the proceeds of the policy. In its answer, the to indemnify under this Policy, the Company will, in to arbitration, during the negotiations preceding the
Company admitted the existence thereof, but respect of the liability incurred by such person, institution of the present case. In fact, counsel for
pleaded lack of cause of action on the part of the indemnify his personal representatives in terms of both parties stipulated, in the trial court, that none
plaintiffs. and subject to the limitations of this Policy, provided, of them had, at any time during said negotiations,
- TC rendered a decision sentencing the Company to that such representatives shall, as though they were even suggested the settlement of the issue between
pay to the plaintiffs the sum of P4,000.00 and the the Insured, observe, fulfill and be subject to the them by arbitration, as provided in said section.
costs. Hence, this appeal by the Company, which Terms of this Policy insofar as they can apply. Their aforementioned acts or omissions had the
contends that plaintiffs have no cause of action "8. The Company may, at its option, make effect of a waiver of their respective right to demand
because: 1) the Coquias have no contractual relation indemnity payable directly to the claimants or heirs an arbitration.
with the Company; and 2) the Insured has not of claimants, with or without securing the consent of Disposition The decision appealed from should be
complied with the provisions of the policy concerning or prior notification to the Insured, it being the true as it is hereby affirmed in toto, with costs against the
arbitration. intention of this Policy to protect, to the extent herein defendant-appellant, Fieldmen's Insurance
herein specified and subject always to the Terms of Co., Inc.
ISSUES this Policy, the liabilities of the Insured towards the
1. WON there was contractual relations between the passengers of the Motor Vehicle and the Public." COUNTRY BANKERS INSURANCE CORP v.
Coquias and the Company - Thus, the policy under consideration is typical of LIANGA BAY
2. WON the insured has not complied with the contracts pour autrui, this character being made DE LEON; January 25, 2002
provisions of the policy concerning arbitration more manifest by the fact that the deceased driver
paid fifty percent (50%) of the corresponding NATURE
HELD premiums, which were deducted from his weekly Petition for review on certiorari
1. Although, in general, only parties to a contract commissions. Under these conditions, it is clear that
may bring an action based thereon, this rule is the Coquias — who, admittedly, are the sole heirs of FACTS
subject to exceptions, one of which is found in the the deceased — have a direct cause of action against - Lianga Bay is a duly registered cooperative
Art 1311 CC, reading: the Company, and, since they could have maintained judicially declared insolvent and is here represented
"If a contract should contain some stipulation in this action by themselves, without the assistance of by, Cornelio Jamero. Country Bankers Insurance
favor of a third person, he may demand its the Insured, it goes without saying that they could and Lianga Bay entered into a contract of fire
INSURANCE Page
115

insurance. Country Bankers insured the HELD Petition for certiorari


respondent’s stocks-in-trade against fire loss, - Where a risk is excepted by the terms of a policy
damage or liability during the period starting from which insures against other perils or hazards, loss FACTS
June 20, 1989 at 4:00 p.m. to June 20, 1990 at 4:00 from such a risk constitutes a defense which the - In the evening of July 27, 1988, the radio station of
p.m., for the sum of P200,000.00. insurer may urge, since it has not assumed that risk, Radio Mindanao Network located at the SSS Building
- On July 1, 1989, at or about 12:40 a.m., the and from this it follows that an insurer seeking to in Bacolod City was burned down causing damage in
respondent’s building at Barangay Diatagon, Lianga, defeat a claim because of an exception or limitation the amount of over one million pesos. Respondent
Surigao del Sur was gutted by fire, resulting in the in the policy has the burden of proving that the loss sought to recover under two insurance policies but
total loss of the respondent’s stocks-in-trade, pieces comes within the purview of the exception or the claims were denied on the basis that the case of
of furnitures and fixtures, equipments and records. limitation set up. If a proof is made of a loss the loss was an excepted risk under condition no. 6
- Due to the loss, the respondent filed an insurance apparently within a contract of insurance, the burden (c) and (d), to wit:
claim with the petitioner under its Fire Insurance is upon the insurer to prove that the loss arose from 6. This insurance does not cover any loss or damage
Policy, submitting: (a) the Spot Report of Pfc. Arturo a cause of loss which is excepted or for which it is occasioned by or through or in consequence, directly
V. Juarbal, INP Investigator, dated July 1, 1989; (b) not liable, or from a cause which limits its liability. or indirectly, of any of the following consequences,
the Sworn Statement of Jose Lomocso; and (c) the Stated else wise, since Country bank here is namely:
Sworn Statement of Ernesto Urbiztondo. defending on the ground of non-coverage and relying (c) War, invasion, act of foreign enemies, hostilities,
- The petitioner, however, denied the insurance claim upon an exemption or exception clause in the fire or warlike operations (whether war be declared or
on the ground that, based on the submitted insurance policy it has the burden of proving the not), civic war.
documents, the building was set on fire by 2 NPA facts upon which such excepted risk is based, by a (d) Mutiny, riot, military or popular uprising,
rebels who wanted to obtain canned goods, rice and preponderance of evidence. But petitioner failed to insurrection, rebellion, revolution, military or usurped
medicines as provisions for their comrades in the do so. power.
forest, and that such loss was an excepted risk under - The petitioner relies on the Sworn Statements of - The insurers maintained that based on witnesses
paragraph No. 6 of the policy conditions of Fire Jose Lomocso and Ernesto Urbiztondo as well as on and evidence gathered at the site, the fire was
Insurance Policy No. F-1397, which provides: the Spot Report of Pfc. Arturo V. Juarbal. A witness caused by the members of the Communist Party of
This insurance does not cover any loss or damage can testify only to those facts which he knows of his the Philippines/New People’s Army. Hence the refusal
occasioned by or through or in consequence, personal knowledge, which means those facts which to honor their obligations.
directly or indirectly, of any of the following are derived from his perception. Consequently, a - The trial court and the CA found in favor of the
occurrences, namely: witness may not testify as to what he merely learned respondent. In its findings, both courts mentioned
(d) Mutiny, riot, military or popular uprising, from others either because he was told or read or the fact that there was no credible evidence
insurrection, rebellion, revolution, military or heard the same. Such testimony is considered presented that the CCP/NPA did in fact cause the fire
usurped power. hearsay and may not be received as proof of the that gutted the radio station in Bacolod.
Any loss or damage happening during the truth of what he has learned.
existence of abnormal conditions (whether Disposition the appealed Decision is MODIFIED. ISSUE
physical or otherwise) which are occasioned by or The rate of interest on the adjudged principal WON the insurance companies are liable to pay Radio
through or in consequence, directly or indirectly, amount of Two Hundred Thousand Pesos Mindanao Network under the insurance policies
of any of said occurrences shall be deemed to be (P200,000.00) shall be six percent (6%) per annum
loss or damage which is not covered by this computed from the date of filing of the Complaint in HELD
insurance, except to the extent that the Insured the trial court. The awards in the amounts of Fifty YES
shall prove that such loss or damage happened Thousand Pesos (P50,000.00) as actual damages, - The Court will not disturb the factual findings of the
independently of the existence of such abnormal Fifty Thousand Pesos (P50,000.00) as exemplary appellant and trial courts absent compelling reason.
conditions. damages, Five Thousand Pesos (P5,000.00) as Under this mode of review, the jurisdiction of the
- Finding the denial of its claim unacceptable, Lianga litigation expenses, and Ten Thousand Pesos court is limited to reviewing only errors of law.
Bay then instituted in the trial court the complaint for (P10,000.00) as attorney?s fees are hereby - Particularly in cases of insurance disputes with
recovery of "loss, damage or liability" against DELETED. regard to excepted risks, it is the insurance
Country Bankers. companies which have the burden to prove that the
- RTC ruled in favor of the cooperative. CA affirmed. DBP POOL OF ACCREDITED INSURANCE v. loss comes within the purview of the exception or
RADIO MINDANAO NETWORK limitation set up. It is sufficient for the insured to
ISSUE 480 SCRA 314 prove the fact of damage or loss. Once the insured
WON the cause of the loss was an excepted risk MARTINEZ; January 27, 2006 makes out a prima facie case in its favor, the duty or
under the terms of the fire insurance policy burden of evidence shifts to the insurer to controvert
NATURE said prima facie case.
INSURANCE Page
116

Disposition Petition dismissed. Decision of the CA is and only cause of the loss. It should have exercised gone due to exposure to sea water. LOASTAR thus
affirmed. due diligence to prevent or minimize the loss before, failed to deliver the goods to MARKET in Manila.
during and after the occurrence of the event. - MARKET demanded from LOADSTAR full
LEA MER INDUSTRIES v. MALAYAN INSURANCE - Petitioner bore the burden of proving that it had reimbursement of the cost of the lost shipment.
471 SCRA 698 exercised extraordinary diligence to avoid the loss, LOADSTAR refused to reimburse the MARKET despite
PANGANIBAN; September 30, 2005 or that the loss had been occasioned by a fortuitous repeated demands.
event -- an exempting circumstance. - March 11, 1985 – PIONEER paid the MARKET
NATURE - The evidence presented by petitioner in support of P1,400,000 plus an additional amount of P500,000,
Petition for Review its defense of fortuitous event was sorely the value of the lost shipment of cement. In return,
insufficient. It was not enough for the common the MARKET executed a Loss and Subrogation
FACTS carrier to show that there was an unforeseen or Receipt in favor of PIONEER concerning the latter’s
- Ilian Silica Mining entered into a contract of unexpected occurrence. subrogation rights against LOADSTAR.
carriage with Lea Mer Industries for the shipment of Disposition Petition is DENIED and the assailed - October 15, 1986 – PIONEER filed a complaint
900 metric tons of silica sand valued at P565,000. against LOADSTAR with the RTC Manila alleging
Consigned to Vulcan Industrial and Mining Decision and Resolution are AFFIRMED. Costs that: (1) the M/V Weasel was not seaworthy at the
Corporation, the cargo was to be transported from commencement of the voyage; (2) the weather and
Palawan to Manila. The silica sand was placed on against petitioner. sea conditions then prevailing were usual and
board Judy VII, a barge leased by Lea Mer, the expected for that time of the year and as such, was
vessel sank, resulting in the loss of the cargo. LOADSTAR SHIPPING CO INC v. PIONEER ASIA an ordinary peril of the voyage for which the M/V
INSURANCE CORP Weasel should have been normally able to cope with;
ISSUE GR No. 157481 and (3) LOADSTAR was negligent in the selection
WON Lea Mer is liable for the loss of the cargo QUISUMBING; January 24, 2006 and supervision of its agents and employees then
manning the M/V Weasel.
HELD NATURE - LOADSTAR alleged that no fault nor negligence
YES Review on certiorari (1) the Decision dated October could be attributed to it because it exercised due
- Common carriers are bound to observe 15, 2002 and (2) the Resolution dated February 27, diligence to make the ship seaworthy, as well as
extraordinary diligence in their vigilance over the 2003 of CA properly manned and equipped and failure to deliver
goods and the safety of the passengers they was due to force majeure.
transport, as required by the nature of their business FACTS - February 15, 1993 - RTC decided in favor of
and for reasons of public policy. Extraordinary - June 6, 1984 - Petitioner Loadstar Shipping Co., PIONEER and that LOADSTAR , as a common carrier,
diligence requires rendering service with the greatest Inc. (LOADSTAR), registered owner and operator of bears the burden of proving that it exercised
skill and foresight to avoid damage and destruction the vessel M/V Weasel, entered into a voyage- extraordinary diligence in its vigilance over the goods
to the goods entrusted for carriage and delivery. charter with Northern Mindanao Transport Company, it transported. The trial court explained that in case
- Common carriers are presumed to have been at Inc. for the carriage of 65,000 bags of cement from of loss or destruction of the goods, a statutory
fault or to have acted negligently for loss or damage Iligan City to Manila. The shipper was Iligan Cement presumption arises that the common carrier was
to the goods that they have transported. This Corporation, while the consignee in Manila was negligent unless it could prove that it had observed
presumption can be rebutted only by proof that they Market Developers, Inc. (MARKET) extraordinary diligence. LOADSTAR’S defense of
observed extraordinary diligence, or that the loss or - June 24, 1984 - 67,500 bags of cement were force majeure was found bereft of factual basis as a
damage was occasioned by any of the following loaded on board M/V Weasel and stowed in the cargo PAG-ASA report that at the time of the incident,
causes: holds for delivery to the consignee. The shipment tropical storm “Asiang” had moved away from the
“(1) Flood, storm, earthquake, lightning, or other was covered by petitioner’s Bill of Lading dated June Philippines was presented.
natural disaster or calamity; 23, 1984. - October 15, 2002 – CA affirmed RTC Decision with
“(2) Act of the public enemy in war, whether - Prior to the voyage, the consignee insured the modification
international or civil; shipment of cement with respondent Pioneer Asia
“(3) Act or omission of the shipper or owner of the Insurance Corporation (PIONEER) for P1,400,000, for ISSUES
goods; which it issued Marine Open Policy No. MOP-006 1. WON LOADSTAR is a common carrier under
“(4) The character of the goods or defects in the dated September 17, 1980, covering all shipments Article 1732 CC
packing or in the containers; made on or after September 30, 1980 2. Assuming it is a common carrier, WON proximate
“(5) Order or act of competent public authority.” - June 25, 1984 - Captain Montera of M/V Weasel cause of the loss of cargo was not a fortuitous event
- To excuse the common carrier fully of any liability, ordered the vessel to be forced aground which but was allegedly due to the failure of petitioner to
the fortuitous event must have been the proximate rendered the entire shipment of cement as good as exercise extraordinary diligence
INSURANCE Page
117

(3) Act or omission of the shipper or


HELD owner of the goods;
1. YES (4) The character of the goods or defects
- A1732 CC defines a “common carrier” as in the packing or in the containers; and
follows: (5) Order or act of competent public
Common carriers are persons, corporations, firms authority
or associations engaged in the business of carrying - LOADSTAR claims that the loss of the goods was
or transporting passengers or goods or both, by due to a fortuitous event under paragraph 1. Yet, its
land, water, or air, for compensation, offering their claim is not substantiated. It is supported by
services to the public. evidence that the loss of the entire shipment of
- LOADSTAR is a corporation engaged in the business cement was due to the gross negligence of
of transporting cargo by water and for compensation, LOADSTAR
offering its services indiscriminately to the public. - Records show that in the evening of June 24, 1984,
Thus, without doubt, it is a common carrier. Even if the sea and weather conditions in the vicinity of
it entered into a voyage-charter agreement with Negros Occidental were calm. The records reveal
Northern Mindanao Transport Company, Inc, it did that LOADSTAR took a shortcut route, instead of the
not in any way convert the common carrier into a usual route, which exposed the voyage to
private carrier. unexpected hazard. LOADSTAR has only itself to
> Planters Products, Inc. v. CA - public carrier blame for its misjudgment.
shall remain as such, notwithstanding the charter of Disposition petition is DENIED
the whole or portion of a vessel by one or more
persons, provided the charter is limited to the ship
only, as in the case of a time-charter or voyage-
charter. It is only when the charter includes both
the vessel and its crew, as in a bareboat or demise
that a common carrier becomes private, at least
insofar as the particular voyage covering the charter-
party is concerned.
2. YES
- As a common carrier, LOADSTAR is required to
observe extraordinary diligence in the vigilance over
the goods it transports. When the goods placed in its
care are lost, LOADSTAR is presumed to have been
at fault or to have acted negligently. LOADSTAR has
the burden of proving that it observed extraordinary
diligence in order to avoid responsibility for the lost
cargo.
- Compania Maritima V CA - It requires common
carriers to render service with the greatest skill and
foresight and “to use all reasonable means to
ascertain the nature and characteristics of goods
tendered for shipment, and to exercise due care in
the handling and stowage, including such methods as
their nature requires.
- A1734 CC enumerates the instances when a carrier
might be exempt from liability for the loss of the
goods.
(1) Flood, storm, earthquake, lightning,
or other natural disaster or calamity;
(2) Act of the public enemy in war,
whether international or civil;
i

Das könnte Ihnen auch gefallen